Mathematics: New Syllabus

You might also like

Download as pdf or txt
Download as pdf or txt
You are on page 1of 348

7th NEW SYLLABUS

EDITION
MATHEMATICS
TEACHER’S RESOURCE BOOK

3
Revision Exercise C1 . . . . . . . . . . . . . . . . . . . . . . . . . . . . . . . . . . . . . . . . . . . . . . . . 282
Revision Exercise C2 . . . . . . . . . . . . . . . . . . . . . . . . . . . . . . . . . . . . . . . . . . . . . . . . 284
Chapter 11: Congruence and Similarity Tests
Teaching Notes . . . . . . . . . . . . . . . . . . . . . . . . . . . . . . . . . . . . . . . . . . . . . . . . . . . 286
Worked Solutions . . . . . . . . . . . . . . . . . . . . . . . . . . . . . . . . . . . . . . . . . . . . . . . . . 287
Chapter 12: Area and Volume of Similar Figures and Solids
Teaching Notes . . . . . . . . . . . . . . . . . . . . . . . . . . . . . . . . . . . . . . . . . . . . . . . . . . . 303
Worked Solutions . . . . . . . . . . . . . . . . . . . . . . . . . . . . . . . . . . . . . . . . . . . . . . . . . 304
Chapter 13: Geometrical Properties of Circles
Teaching Notes . . . . . . . . . . . . . . . . . . . . . . . . . . . . . . . . . . . . . . . . . . . . . . . . . . . 316
Worked Solutions . . . . . . . . . . . . . . . . . . . . . . . . . . . . . . . . . . . . . . . . . . . . . . . . . 317
Revision Exercise D1 . . . . . . . . . . . . . . . . . . . . . . . . . . . . . . . . . . . . . . . . . . . . . . . . 335
Revision Exercise D2 . . . . . . . . . . . . . . . . . . . . . . . . . . . . . . . . . . . . . . . . . . . . . . . . 337
Problems in Real-World Contexts . . . . . . . . . . . . . . . . . . . . . . . . . . . . . . . . . . . . . . 339

3
CONTENTS
Syllabus Matching Grid . . . . . . . . . . . . . . . . . . . . . . . . . . . . . . . . . . . . . . . . . . . . . . . . 1
Scheme of Work . . . . . . . . . . . . . . . . . . . . . . . . . . . . . . . . . . . . . . . . . . . . . . . . . . . . . . 7
Chapter 1: Quadratic Equations and Functions
Teaching Notes . . . . . . . . . . . . . . . . . . . . . . . . . . . . . . . . . . . . . . . . . . . . . . . . . . . . 28
Worked Solutions . . . . . . . . . . . . . . . . . . . . . . . . . . . . . . . . . . . . . . . . . . . . . . . . . . 30
Chapter 2: Further Functions
Teaching Notes . . . . . . . . . . . . . . . . . . . . . . . . . . . . . . . . . . . . . . . . . . . . . . . . . . . . 78
Worked Solutions . . . . . . . . . . . . . . . . . . . . . . . . . . . . . . . . . . . . . . . . . . . . . . . . . . 79
Chapter 3: Linear Inequalities
Teaching Notes . . . . . . . . . . . . . . . . . . . . . . . . . . . . . . . . . . . . . . . . . . . . . . . . . . . . 88
Worked Solutions . . . . . . . . . . . . . . . . . . . . . . . . . . . . . . . . . . . . . . . . . . . . . . . . . . 89
Chapter 4: Indices and Standard Form
Teaching Notes . . . . . . . . . . . . . . . . . . . . . . . . . . . . . . . . . . . . . . . . . . . . . . . . . . . 106
Worked Solutions . . . . . . . . . . . . . . . . . . . . . . . . . . . . . . . . . . . . . . . . . . . . . . . . . 108
Revision Exercise A1 . . . . . . . . . . . . . . . . . . . . . . . . . . . . . . . . . . . . . . . . . . . . . . . . 127
Revision Exercise A2 . . . . . . . . . . . . . . . . . . . . . . . . . . . . . . . . . . . . . . . . . . . . . . . . 130
Chapter 5: Graphs of Functions and Graphical Solution
Teaching Notes . . . . . . . . . . . . . . . . . . . . . . . . . . . . . . . . . . . . . . . . . . . . . . . . . . . 132
Worked Solutions . . . . . . . . . . . . . . . . . . . . . . . . . . . . . . . . . . . . . . . . . . . . . . . . . 134
Chapter 6: Coordinate Geometry
Teaching Notes . . . . . . . . . . . . . . . . . . . . . . . . . . . . . . . . . . . . . . . . . . . . . . . . . . . 149
Worked Solutions . . . . . . . . . . . . . . . . . . . . . . . . . . . . . . . . . . . . . . . . . . . . . . . . . 150
Chapter 7: Graphs of Functions and Graphical Solution
Teaching Notes . . . . . . . . . . . . . . . . . . . . . . . . . . . . . . . . . . . . . . . . . . . . . . . . . . . 169
Worked Solutions . . . . . . . . . . . . . . . . . . . . . . . . . . . . . . . . . . . . . . . . . . . . . . . . . 171
Revision Exercise B1 . . . . . . . . . . . . . . . . . . . . . . . . . . . . . . . . . . . . . . . . . . . . . . . . 212
Revision Exercise B2 . . . . . . . . . . . . . . . . . . . . . . . . . . . . . . . . . . . . . . . . . . . . . . . . 214
Chapter 8: Further Trigonometry
Teaching Notes . . . . . . . . . . . . . . . . . . . . . . . . . . . . . . . . . . . . . . . . . . . . . . . . . . . 216
Worked Solutions . . . . . . . . . . . . . . . . . . . . . . . . . . . . . . . . . . . . . . . . . . . . . . . . . 217
Chapter 9: Applications of Trigonometry
Teaching Notes . . . . . . . . . . . . . . . . . . . . . . . . . . . . . . . . . . . . . . . . . . . . . . . . . . . 238
Worked Solutions . . . . . . . . . . . . . . . . . . . . . . . . . . . . . . . . . . . . . . . . . . . . . . . . . 239
Chapter 10: Arc Length, Area of Sector and Radian Measure
Teaching Notes . . . . . . . . . . . . . . . . . . . . . . . . . . . . . . . . . . . . . . . . . . . . . . . . . . . 261
Worked Solutions . . . . . . . . . . . . . . . . . . . . . . . . . . . . . . . . . . . . . . . . . . . . . . . . . 262

2
Revision Exercise C1 . . . . . . . . . . . . . . . . . . . . . . . . . . . . . . . . . . . . . . . . . . . . . . . . 282
Revision Exercise C2 . . . . . . . . . . . . . . . . . . . . . . . . . . . . . . . . . . . . . . . . . . . . . . . . 284
Chapter 11: Congruence and Similarity Tests
Teaching Notes . . . . . . . . . . . . . . . . . . . . . . . . . . . . . . . . . . . . . . . . . . . . . . . . . . . 286
Worked Solutions . . . . . . . . . . . . . . . . . . . . . . . . . . . . . . . . . . . . . . . . . . . . . . . . . 287
Chapter 12: Area and Volume of Similar Figures and Solids
Teaching Notes . . . . . . . . . . . . . . . . . . . . . . . . . . . . . . . . . . . . . . . . . . . . . . . . . . . 303
Worked Solutions . . . . . . . . . . . . . . . . . . . . . . . . . . . . . . . . . . . . . . . . . . . . . . . . . 304
Chapter 13: Geometrical Properties of Circles
Teaching Notes . . . . . . . . . . . . . . . . . . . . . . . . . . . . . . . . . . . . . . . . . . . . . . . . . . . 316
Worked Solutions . . . . . . . . . . . . . . . . . . . . . . . . . . . . . . . . . . . . . . . . . . . . . . . . . 317
Revision Exercise D1 . . . . . . . . . . . . . . . . . . . . . . . . . . . . . . . . . . . . . . . . . . . . . . . . 335
Revision Exercise D2 . . . . . . . . . . . . . . . . . . . . . . . . . . . . . . . . . . . . . . . . . . . . . . . . 337
Problems in Real-World Contexts . . . . . . . . . . . . . . . . . . . . . . . . . . . . . . . . . . . . . . 339

3
Syllabus Matching Grid

Cambridge O Level Mathematics (Syllabus D) 4024/4029. Syllabus for examination in 2018, 2019 and 2020.

Theme or Topic Subject Content Reference

1. Number Identify and use: Book 1:

, 2)

2. Set language and notation Book 2:

e.g. A = {x : x is a natural number}, Book 4:


B = {(x, y): y = mx + c},
C = {x : a ! x ! b},
D = {a, b, c, …}

Calculate Book 1:
cube roots

4. Directed numbers Book 1:

Book 1:

6. Ordering Book 1:
!, ".

7. Standard form A 10n n Book 3:


1!A

Book 1:

9. Estimation Book 1:

10 Limits of accuracy Book 3:


accuracy

1
Book 1:

Book 2:

Book 1:

Book 3:

Book 1:

Book 2:

Book 3:

Book 4:

14. Time Book 1:

15. Money Book 3:

Book 3:

Book 1:
formulae algebraically

Book 2:

Book 3:

Book 1:

Book 2:
ax + bx + kay + kby
a2x2 – b2y2
a2 + 2ab + b2
ax2 + bx + c

19. Indices Book 3:

2
Book 1:

Book 2:
the formula

Book 3:

Book 4:

Book 1:

Book 2:

Book 1:

Book 2:

Book 3:

axn Book 1:
n
more than three of these and for functions of the form kax a
integer Book 2:

functions

Book 3:

x) = 3x – 5; f : x ! 3x Book 2:
functions
–1
(x)
Book 3:

27. Coordinate geometry Book 1:

it Book 2:

y = mx + c Book 3:

3
Book 1:

and exterior angles; similarity and congruence

Book 2:

tangent, arc, sector and segment


Book 3:

Book 1:

Book 2:

Book 4:

30. Similarity and congruence Book 2:


congruence

Book 3:

similar solids

31. Symmetry Book 2:

Book 3:

Book 1:

Book 3:

33. Loci Book 4:

(b) at a given distance from a given straight line

34. Measures Book 1:

4
35. Mensuration Book 1:

(c) the circumference and area of a circle


(d) arc length and sector area as fractions of the circumference and area of a Book 2:
circle

and cone Book 3:

36. Trigonometry Book 2:

Book 3:

1
formula area of triangle = ab sin C
2

Book 4:
x
y , AB or a

x
y as x 2 + y 2

38. Matrices Book 4:

2
matrices
Calculate the determinant A and inverse A–1 A

39. Transformations Book 2:


translation (T), enlargement (E) and their combinations

Book 4:

Book 2:

the event not occurring


Book 4:

5
41. Categorical, numerical and Book 1:

diagrams
Book 2:

Book 4:

42. Statistical diagrams Book 1:

intervals and scatter diagrams


Book 2:

Book 4:

reference to a scatter diagram

6
Secondary 3 Mathematics Scheme of Work

Week Reasoning,
Additional
(5 classes Chapter Section Activity
Content
× 45 min) and Connection
1 1 1.1 Solving

Completing the

(pp. 3 – 9) x2 + px + q = 0 x2 + px x2 + px
(p. 6) (p. 6)

1 1.2 Solving

(p. 11)
(pp. 10 – 12) (p. 11)

7
Week Reasoning,
Additional
(5 classes Chapter Section Activity
Content
× 45 min) and Connection
2 1.3 Solving
(p. 15) (p. 15)
axn a
by Graphical n=2
Method (p. 15)
(pp. 12 – 17)
2 1.4 Solving
Fractional (p. 20) (p. 20)

(pp. 17 – 20)
3 1.5 Applications
(p. 27)

Real-World
Contexts

8
(pp. 20 – 27)
4 1.6 Graphs of
(p. 28) (p. 28)

(pp. 27 – 35) y = (x – h)(x – k


y = –(x – h)(x – k
y = (x – p)2 + q
y = –(x – p)2 + q
Week Reasoning,
Additional
(5 classes Chapter Section Activity
Content
× 45 min) and Connection

y = (x – h)(x – k) y = (x – h)(x – k) y = (x – h)(x – k)

y = –(x – h)(x – k) y = –(x – h)(x – k) y = –(x – h)(x – k)


(p. 29) (p. 29) (p. 29)

y = (x – p)2 + q y = (x – p)2 + q y = (x – p)2 + q


y = –(x – p)2 + q y = –(x – p)2 + q y = –(x – p)2 + q
(p. 31) (p. 31) (p. 31)

(p. 33) (p. 33)

9
(p. 34)
(p. 34)
4

4 2

Higher Order
Expressions
(pp. 43 – 44)
5
f–1(x) (p. 46)
(pp. 45 – 49)
5
Week Reasoning,
Additional
(5 classes Chapter Section Activity
Content
× 45 min) and Connection
6 3
Linear (pp. 53 – 59) (p. 56)

(p. 53)
(p. 57)

(p. 55)

(p. 56)

(p. 57)
6 3.2 Problem
Solving
involving

10
(pp. 60 – 61)
7 3.3 Solving
(p. 63) (p. 63)
Linear

(pp. 62 – 65)
7 3.4 Limits of
(p. 69)
(pp. 65 – 69)

7
Week Reasoning,
Additional
(5 classes Chapter Section Activity
Content
× 45 min) and Connection
8 4
(pp. 75 – 76)
Standard Form (p. 75)

(p. 76)

(p. 76)
8
(pp. 76 – 85)
(pp. 76 – 77)
(p. 81)

(p. 78)

11
(p. 79)
Week Reasoning,
Additional
(5 classes Chapter Section Activity
Content
× 45 min) and Connection

(p. 83)
(p. 80)

a + b)n
= an + bn?
a – b)n
= an – bn?
(p. 81) (p. 84)

(p. 82)

(p. 83)

a + b)n
= an + bn?

12
a – b)n
= an – bn?
(p. 84)
8 4.3 Zero and
Negative
(pp. 85 – 86) (pp. 85 – 86)
(pp. 85 – 90)

(p. 87) (p. 87)


Week Reasoning,
Additional
(5 classes Chapter Section Activity
Content
× 45 min) and Connection

(p. 87) (p. 87)

(p. 89) (p. 89)


7 4.4 Rational

(pp. 90 – 97) (p. 92) (p. 92)

(p. 92) (p. 92)

(p. 94)
(p. 93)

13
(p. 94)
9 4.5 Standard Form
(pp. 98 – 105) A × 10 n (p. 100)
(p. 98) (pp. 98)
A < 10.

(p. 102) (p. 100) (p. 102)

(p. 104) (p. 102)


9
Week Reasoning,
Additional
(5 classes Chapter Section Activity
Content
× 45 min) and Connection
10 5
Application of (pp. 113 – 115)
Mathematics
in Practical

10

Commission
(pp. 115 – 124) (p. 118) (p. 118)

14
(p. 120) (p. 120)
11
(p. 127)
(p. 125)
(pp. 125 – 131)

(p. 127)
(pp. 127 – 128)

(pp. 127 – 128)


11
(pp. 132 – 134)
12 5.5 Money
Exchange (p. 136) (p. 136)
(pp. 135 – 137)

12
Week Reasoning,
Additional
(5 classes Chapter Section Activity
Content
× 45 min) and Connection
12 6 6.1 Gradient of a
Coordinate Straight Line
Geometry (pp. 143 – 149)

(p. 144) (p. 144)

(p. 145) (p. 145)

(pp. 145 – 146) (pp. 146 – 146) (pp. 145 – 146)


13 6.2 Length of a
Line Segment (p. 150) (p. 150)
(pp. 149 – 155)

15
13
Straight Line (p. 158) (p. 158)
(pp. 155 – 160)
y = mx + c y = mx + c

14 6.4 Parallel and


(p. 161) (p. 161)
Lines
(pp. 161 – 167)

(pp. 161 – 162)


(pp. 161 – 162)

(p. 164)
14
Week Reasoning,
Additional
(5 classes Chapter Section Activity
Content
× 45 min) and Connection
15 7 7.1 Graphs
Graphs of
y = axn
Graphical (pp. 173 – 176) n axn (p. 173) (p. 173)

15 7.2 Graphs of
Reciprocal y= a y= a y= a
x x x
(p. 176) (p. 176) (p. 176)
(pp. 176 – 182) axn
n=
(p. 177) (p. 177)
y = a2

16
x
(p. 179) (p. 177)
y = a2
x
(p. 179)

y = a2
x
(p. 179)

(p. 179)
Week Reasoning,
Additional
(5 classes Chapter Section Activity
Content
× 45 min) and Connection
15 7.3 Graphs of
Exponential y = ax y = ax y = ax
x x x x
y = ka a y = ka y = ka y = ka
(pp. 183 – 186) kax a (p. 183) (p. 135) (p. 183)

(p. 186) (p. 186)

(p. 184) (p. 184) (p. 184)


16 7.4 Gradient of a
(p. 192)
(pp. 187 – 192)

17
Week Reasoning,
Additional
(5 classes Chapter Section Activity
Content
× 45 min) and Connection
16 7.5 Applications of
Graphs in Real-
World Contexts
(pp. 192 – 209) (p. 193) (p. 193)

(p. 194) (p. 194)

(p. 197) (p. 197)

18
16

17 8 8.1 Sine and Cosine


(pp. 221 – 222)
Angles
(pp. 221 – 229)

(p. 223)

(p. 223)
Week Reasoning,
Additional
(5 classes Chapter Section Activity
Content
× 45 min) and Connection
17 8.2 Area of
1 ab C (p. 232) (p. 232)
(pp. 229 – 234) 2

= 1 ab C
2
18
(pp. 235 – 245)
(pp. 235 – 236) (pp. 235 – 236) (pp. 235 – 236)

(p. 242) (p. 242)

18
(pp. 245 – 253) (p. 245)

19
(pp. 246 – 247) (pp. 246 – 247)

(p. 248) (pp. 246 – 247)

(p. 248)
18
Week Reasoning,
Additional
(5 classes Chapter Section Activity
Content
× 45 min) and Connection
19 9 9.1 Angles of
Applications of Elevation and
Depression
(pp. 261 – 266)

19 9.2 Bearings
(pp. 267 – 275) (p. 219)

20
Dimensional
Problems
(pp. 275 – 288) (pp. 275 – 277) (pp. 275 – 277)

(p. 278)

20
(p. 280)
20
Week Reasoning,
Additional
(5 classes Chapter Section Activity
Content
× 45 min) and Connection
21 10 10.1 Length of Arc
Arc Length, (pp. 295 – 305)
Area of Sector (pp. 296 – 297) (pp. 296 – 297) (pp. 296 – 297)
and Radian

21 10.2 Area of Sector


(pp. 306 – 312)
(pp. 306 – 307) (pp. 306 – 307) (pp. 306 – 307)

21
22 10.3 Radian

(pp. 313 – 321)


(p. 314)
(p. 314)

(p. 315)

(p. 315)
Week Reasoning,
Additional
(5 classes Chapter Section Activity
Content
× 45 min) and Connection
22 10.4 Arc Length and
Area of Sector s=r A = 1 r2
2

(pp. 322 – 332)

22

23 11

and Similarity (pp. 343 – 359) (p. 344)


(p. 344)

(pp. 346 – 347)

22
(pp. 346 – 347)

(pp. 350 – 351)

(pp. 350 – 351)


(p. 352)

(p. 352)
(p. 354)

(p. 354)
Week Reasoning,
Additional
(5 classes Chapter Section Activity
Content
× 45 min) and Connection
23
(pp. 359 – 373)
(p. 360) (p. 360)

(p. 360) (p. 360)

(p. 363)
(p. 363)

(p. 364)
(p. 364)

(pp. 365 – 366)

(pp. 365 – 366)

23
(p. 366)

(p. 366)
24 11.3 Applications

and Similar

(pp. 374 – 378)

24
Week Reasoning,
Additional
(5 classes Chapter Section Activity
Content
× 45 min) and Connection
24 12 12.1 Area of
Area and
(pp. 387 – 395)
(pp. 387 – 389) (pp. 387 – 389)
and Solids
25
Similar Solids (p. 402)
(pp. 396 – 404)
(pp. 396 – 397) (pp. 396 – 397)

(p. 402) (p. 402)

(p. 402) (p. 402)

24
Week Reasoning,
Additional
(5 classes Chapter Section Activity
Content
× 45 min) and Connection
25 13 13.1 Symmetric
Geometrical Properties of
Properties of Circles
Circles (pp. 411 – 429) (pp. 411 – 413) (pp. 411 – 413) (pp. 411 – 413)

(p. 414) (p. 414)

(pp. 417 – 418)

(pp. 417 – 418)


(p. 416)
(p. 422)

(p. 425)
(pp. 417 – 418)
(p. 425)

25
(p. 425)
(p. 422)

(p. 425)
Week Reasoning,
Additional
(5 classes Chapter Section Activity
Content
× 45 min) and Connection
26 13.2 Angle
Properties of
Circles
(pp. 430 – 454) (p. 432)
(p. 431)
(p. 431)

(p. 436)
(p. 432)
(p. 432)

(p. 434)
(p. 434)

(p. 436)
(p. 436)

26
(p. 438)
(p. 438)
Week Reasoning,
Additional
(5 classes Chapter Section Activity
Content
× 45 min) and Connection

(p. 440) (p. 439)


(p. 439)

(pp. 442 – 443) (p. 440)


(p. 440)

(p. 445) (pp. 442 – 443)


(pp. 442 – 443)

(p. 445)

27
(p. 445)

(p. 446)
(p. 446)

(p. 446)
26
Chapter 1 Quadratic Equations and Functions
TEACHING NOTES
Suggested Approach
In Book 2, students have learnt how to solve quadratic equations by factorisation. Teachers may want to begin this chapter by
doing a recap and building up on what students have learnt so far. Once students have learnt the methods of solving quadratic
equations, teachers may get the students to do a reflection on the methods of solving quadratic equations (see Journal Writing on
page 15 of the textbook). Teachers should give students the opportunity to use a graphing software to explore the characteristics
of the graphs of the form y = (x – h)(x – k) or y = –(x – h)(x – k) and y = (x – p)2 + q or y = –(x – p)2 + q.

Section 1.1: Solving Quadratic Equations by Completing the Square


Teachers can first get students to discuss with one another on what they have learnt about quadratic equations
in Book 2. Students should be reminded that the right hand side of the equation should be zero before they
factorise the equation. Teachers can ask students to solve a quadratic equation that cannot be easily solved by
factorisation and highlight to them that such an equation can be rearranged into the form (x + a)2 = b, where a and
b are constants. The equation can then be solved easily by taking the square roots on both sides of the equation
to obtain the solution.

Since students are familiar with algebra discs which they have come across in Book 2, teachers can first teach
students how to complete the square for a quadratic expression with the use of the discs. When the students have
understood how to complete the square, teachers can then teach them how to use a multiplication frame instead
(see Investigation: Completing the Square for Quadratic Expressions of the Form x2 + px).

Section 1.2: Solving Quadratic Equations Using Formula


To give students a better understanding of how the general formula is derived, teachers can get the students to
work in small groups and use the method of completing the square for the general form of the quadratic equation.
Teachers may guide the students along when they have difficulty deriving the formula.

Teachers can raise the question to the class on how they can tell if a quadratic equation has no real solutions,
one real solution or two real solutions (see Class Discussion: Solutions to Quadratic Equations).

Section 1.3: Solving Quadratic Equations by Graphical Method


The graphical method is another method of solving quadratic equations in which the solutions of the quadratic
equation ax2 + bx + c = 0 are the x-coordinates of the points of intersection of the graph y = ax2 + bx + c with
the x-axis. However, students need to note that the solutions obtained by the graphical method can only be an
approximation. Teachers should highlight to students that the answers obtained by the graphical method can
only be accurate up to half of a small square grid. Teachers should give students more examples of the different
types of quadratic graphs cutting the x-axis so that the students can better understand and identify the number
of points of intersection between the graph and the x-axis.

Section 1.4: Solving Fractional Equations that can be reduced to Quadratic Equations
In Book 2, students have learnt how to solve equations involving algebraic fractions using the method of changing
the subject of a formula. In this section, they will learn to solve equations which have one or more algebraic
fractions, known as fractional equations which can be reduced to quadratic equations. Also, teachers should
highlight the common mistakes that students tend to make when solving fractional equations, especially when
students cancel the common factors out (see Thinking Time on page 20 of the textbook).

Section 1.5: Applications of Quadratic Equations and Functions in Real-World Contexts


Teachers should go through the problems given in the textbook and take a look at mathematics and real-life
problems that involve the graphs of quadratic equations and functions. Students should use Polya’s 4-step Problem
Solving Model to help them understand the problem and form an equation to solve the problem (see pages 21
and 22 of the textbook).

28
Section 1.6: Graphs of Quadratic Functions
In Book 2, students have learnt the properties of a quadratic graph of the form y = ax2 + bx + c. Teachers
should revise with them on what they have learnt (see Thinking Time on page 28 of the textbook) before
teaching them to sketch the graphs of the form y = (x – h)(x – k) or y = –(x – h)(x – k) and y = (x – p)2 + q or
y = –(x – p)2 + q.

For graphs of the form y = (x – h)(x – k) or y = –(x – h)(x – k), teachers should guide the students along and help
them to recognise that the graphs cut the x-axis at (h, 0) and (k, 0) and are symmetrical about the vertical line that
passes through the minimum or maximum point (see Investigation: Graphs of y = (x – h)(x – k) or y = –(x – h)
(x – k)). Teachers should also highlight to students that the line of symmetry is halfway between the x-intercepts
and how this can be used to find the coordinates of the minimum/maximum point.

For graphs of the form y = (x – p)2 + q or y = –(x – p)2 + q, teachers should guide the students along and help them
to recognise that the minimum or maximum point is (p, q) and the line of symmetry is x = p (see Investigation:
Graphs of y = (x – p)2 + q or y = –(x – p)2 + q).

Teachers can check on students’ level of understanding of the graphs of the form y = (x – h)(x – k) or y = –(x – h)
(x – k) and y = (x – p)2 + q or y = –(x – p)2 + q by getting them to match graphs with their respective functions
and justify their answers (see Class Discussion: Matching Quadratic Graphs with the Corresponding Functions).

Challenge Yourself
For Question 1, guide the students to use the information on ‘sum of its digits is 6’ to represent the other digit
in terms of x. Also, teachers may go through numerical examples on the meaning of 2-digit numbers which are
represented by the tens and ones digits.

For Question 2, advise the students to use the quadratic formula to attempt this question.

29
WORKED SOLUTIONS
Investigation (Completing the Square for Quadratic Expressions of the Form x2 + px)

Number that must be 1


Quadratic Expression added to complete the 2 p Quadratic expression of the
of x,
x2 + px square, 2 form (x + a)2 – b
b

(a) x2 + 4x 22 = 4 4 x2 + 4x
=2
2 = x2 + 4x + 22 – 22
× x 2 = (x + 2)2 – 4
2
x x 2x
× x 2
2 2x
2
x x 2x
2 2x 4 –4

(b) x2 + 6x 32 = 9 6 x2 + 6x
=3
2 = x2 + 6x + 32 – 32
× x 3 = (x + 3)2 – 9
2
x x 3x
× x 3
3 3x
2
x x 3x
3 3x 9 –9

(c) x2 + 8x 42 = 16 8 x2 + 8x
=4
2 = x2 + 8x + 42 – 42
× x 4 = (x + 4)2 – 16
x x2 4x
× x 4
4 4x
2
x x 4x
4 4x 16 –16

(d) x2 + 10x 52 = 25 10 x2 + 10x


=5
2 = x2 + 10x + 52 – 52
× x 5 = (x + 5)2 – 25
2
x x 5x
× x 5
5 5x
2
x x 5x
5 5x 25 –25

1. The number that must be added to each of the quadratic expressions Class Discussion (Solutions to Quadratic Equations)
x2 + px x of
(a) 4x2 – 12x + 9 = 0
the quadratic expression.
2 2 Comparing 4x2 – 12x + 9 = 0 with ax2 + bx + c = 0,
p p
2. x2 + px = x2 + px + – we have a = 4, b = –12 and c = 9.
2 2
2 2 1. b2 – 4ac = (–12)2 – 4(4)(9) = 0
p p
= x+ –
2 2 –b ± b 2 – 4ac –(–12) ± 0
2 2. x = = = 1.5
p p 2a 2(4)
= (x + a)2 – b where a = and b =
2 2 3. When b2 – 4ac = 0, the equation has one real solution.

30
(b) 2x2 + 5x + 8 = 0 (i) From the graph, there are no points of intersection between the graph
Comparing 2x2 + 5x + 8 = 0 with ax2 + bx + c = 0, and the x-axis.
we have a = 2, b = 5 and c = 8. (ii) There are no real solutions to the equation 2x2 + 4x + 3 = 0. There
1. b2 – 4ac = 52 – 4(2)(8) = –39 are no points of intersection between the graph and the x-axis.
–b ± b 2 – 4ac –5 ± –39 (iii) Comparing 2x2 + 4x + 3 = 0 with ax2 + bx + c = 0,
2. x = = (N.A.) we have a = 2, b = 4 and c = 3.
2a 2(5)
3. When b2 – 4ac < 0, the equation has no real solutions. b2 – 4ac = 42 – 4(2)(3) = –8
(c) 3x2 + 5x – 4 = 0 (iv) When b2 – 4ac < 0, the equation has no real solutions. As such, there
Comparing 3x2 + 5x – 4 = 0 with ax2 + bx + c = 0, are no points of intersection between the graph and the x-axis.
we have a = 3, b = 5 and c = –4.
1. b2 – 4ac = 52 – 4(3)(–4) = 73 Journal Writing (Page 15)
–b ± b 2 – 4ac –5 ± 73 (i) Factorisation
2. x = =
2a 2(3) Advantage: Can be solved easily by factorising the quadratic
= –2.26 (to 3 s.f.) or 0.591 (to 3 s.f.) expression.
3. When b2 – 4ac > 0, the equation has two real solutions. Disadvantage: Can only be used when the quadratic expression
can be factorised.
Thinking Time (Page 15) (ii) Completing the square
y = 2x2 + 4x + 3 Disadvantages: This method is complex and hence the chances of
errors are higher.
x –2 –1 0 1 2 4 This method always works and it gives some insight
y 3 1 3 9 19 51 into how algebra works more generally.
(iii) Use of the quadratic formula
y Advantage: This method always works and is straightforward.
Disadvantage: This formula provides no insight and can become
50 a rote technique.
y = 2x2 + 4x + 3
(iv) Graphical method
45 Advantage: The answer can be observed from the graph, given
that the graph has been drawn accurately.
40 Disadvantage: The exact value cannot be obtained and it is a more
tedious method.

35
Thinking Time (Page 20)
7 21
30 No, the solution x = 3 is not valid since the terms and
x–3 x(x – 3)
of the original equation will be invalid.

25
Thinking Time (Page 28)

20 y = x2 + 4x – 5
x2 is 1, the graph opens upwards.
When y = 0,
15
x2 + 4x – 5 = 0
(x + 5)(x – 1) = 0
10 Scale: x = –5 or x = 1
x-axis: 2 cm represent 1 unit
x-intercepts = –5, 1
y-axis: 2 cm represent 5 units
5 When x = 0,
y = (0)2 + 4(0) – 5
x = –5
–2 –1 0 1 2 3 4 y-intercept = –5

31
y
y

6
x
O
4
Graph 4
2
Graph 4 corresponds to the quadratic function y = x2 + 4x – 5.
x
y = –x2 – 4x + 5 –5 –4 –3 –2 –1 0 1 2 3 4 5
x2 is –1, the graph opens downwards. –2
When y = 0, –4
–x2 – 4x + 5 = 0
x2 + 4x – 5 = 0 –6
(x + 5)(x – 1) = 0
x = –5 or x = 1 For k = 0, y = x(x – 3)
x-intercepts = –5, 1
y
When x = 0,
y = (0)2 – 4(0) + 5
10
=5
y-intercept = 5 8
y
6

x 4
O

x
–3 –2 –1 0 1 2 3 4 5
Graph 8
Graph 8 corresponds to the quadratic function y = –x2 – 4x + 5. –2

–4
Investigation (Graphs of y = (x – h)(x – k) or
y = –(x – h)(x – k))
1. y = (x – 3)(x – k) For k = 1, y = (x – 3)(x – 1)
For k = –2, y = (x – 3)(x + 2) y

y
10

4 8

2 6

x 4
–4 –3 –2 –1 0 1 2 3 4
–2 2

–4 x
–2 –1 0 1 2 3 4 5 6
–6 –2

–8

For k = 2, y = (x – 3)(x – 2)

For k = –1, y = (x – 3)(x + 1)

32
y
For k = 2, y = (x – 3)(x – 2)
(a) The graph opens upwards.
10 (b) (2, 0) and (3, 0)
(c) (0, 6)
8 (d) The line of symmetry is halfway between the x-intercepts.
1
6 (e) x = 2
2
1 1
4 (f) minimum point 2 , –
2 4
2 3. Step 1:
y = –(x – 3)(x – k)
x
–1 0 1 2 3 4 5 6 For k = –2, y = –(x – 3)(x + 2)
y
–2

8
2. For k = –2, y = (x – 3)(x + 2)
6
(a) The graph opens upwards.
(b) (–2, 0) and (3, 0) 4
(c) (0, –6)
(d) The line of symmetry is halfway between the x-intercepts. 2
1
(e) x = x
2 –4 –3 –2 –1 0 1 2 3 4 5
1 1
(f) minimum point , –6 –2
2 4

For k = –1, y = (x – 3)(x + 1) –4


(a) The graph opens upwards.
–6
(b) (–1, 0) and (3, 0)
(c) (0, –3)
(d) The line of symmetry is halfway between the x-intercepts. For k = –1, y = –(x – 3)(x + 1)
(e) x = 1 y
(f) minimum point (1, – 4)

For k = 0, y = x(x – 3) 6
(a) The graph opens upwards.
4
(b) (0, 0) and (3, 0)
(c) (0, 0)
2
(d) The line of symmetry is halfway between the x-intercepts.
1 x
(e) x = 1 0
2 –3 –2 –1 1 2 3 4 5
1 1 –2
(f) minimum point 1 , –2
2 4
–2
For k = 1, y = (x – 3)(x – 1)
(a) The graph opens upwards. –6
(b) (1, 0) and (3, 0)
(c) (0, 3)
(d) The line of symmetry is halfway between the x-intercepts.
(e) x = 2
(f) minimum point (2, –1)

33
For k = 0, y = –x(x – 3) y = (x – 5)(x – k)
y For k = –2, y = (x – 5)(x + 2)
y
4

2 6
4
x 2
–2 –1 0 1 2 3 4 5
x
–4 –2 0 2 4 6 8
–2 –2
–4
–4
–6
–8
–6
–10
–12
–8
–14

For k = 1, y = –(x – 3)(x – 1)


For k = –1, y = (x – 5)(x + 1)
y
y

2
10

x 8
–2 –1 0 1 2 3 4 5 6 6
–2 4
2
–4 x
–4 –2 0 2 4 6 8
–2
–6
–4
–6
–8
–8
–10

For k = 2, y = –(x – 3)(x – 2)


y For k = 0, y = x(x – 5)
y

2
10
8
x
–1 0 1 2 3 4 5 6 6
4
–2 2
x
–2 0 2 4 6 8
–4 –2
–4
–6 –6
–8

–8

34
For k = 1, y = (x – 5)(x – 1) For k = –1, y = –(x – 5)(x + 1)
y y

12 8

10 6
8
4
6

4 2

2 x
–3 –2 –1 0 1 2 3 4 5 6 7
x
–2 0 2 4 6 8 –2
–2

–4 –4

–6
For k = 0, y = –x(x – 5)
y
For k = 2, y = (x – 5)(x – 2)
y
6

16
4
14
12 2
10
8 x
–1 0 1 2 3 4 5 6
6
–2
4
2
–4
x
–2 0 2 4 6 8
–2
–4 For k = 1, y = –(x – 5)(x – 1)
y

y = –(x – 5)(x – k)
For k = –2, y = –(x – 5)(x + 2) 4

y
2

14 x
–2 –1 0 1 2 3 4 5 6 7
12
–2
10
–4
8

6 –6

x
–4 –3 –2 –1 0 1 2 3 4 5 6 7
–2

35
For k = 2, y = –(x – 5)(x – 2) (e) x = 2
(f) maximum point (2, 1)
y
For k = 2, y = –(x – 3)(x – 2)
(a) The graph opens downwards.
4 (b) (2, 0) and (3, 0)
(c) (0, –6)
2
(d) The line of symmetry is halfway between the x-intercepts.
x 1
–1 0 1 2 3 4 5 6 7 8 (e) x = 2
2
–2
1 1
(f) maximum point 2 ,
–4 2 4

y = (x – 5)(x – k)
–6
For k = –2, y = (x – 5)(x + 2)
–8 (a) The graph open upwards.
(b) (–2, 0) and (5, 0)
–10
(c) (0, –10)
–12 (d) The line of symmetry is halfway between the x-intercepts.
1
(e) x = 1
Step 2: 2
1 1
y = –(x – 3)(x – k) (f) minimum point 1 , –12
2 4
For k = –2, y = –(x – 3)(x + 2)
(a) The graph opens downwards. For k = –1, y = (x – 5)(x + 1)
(b) (–2, 0) and (3, 0) (a) The graph opens upwards.
(c) (0, 6) (b) (–1, 0) and (5, 0)
(d) The line of symmetry is halfway between the x-intercepts. (c) (0, –5)
1 (d) The line of symmetry is halfway between the x-intercepts.
(e) x =
2 (e) x = 2
1 1 (f) minimum point (2, –9)
(f) maximum point ,6
2 4
For k = 0, y = x(x – 5)
For k = –1, y = –(x – 3)(x + 1) (a) The graph opens upwards.
(a) The graph opens downwards. (b) (0, 0) and (5, 0)
(b) (–1, 0) and (3, 0) (c) (0, 0)
(c) (0, 3) (d) The line of symmetry is halfway between the x-intercepts.
(d) The line of symmetry is halfway between the x-intercepts. 1
(e) x = 1 (e) x = 2
2
(f) maximum point (1, 4) 1 1
(f) minimum point 2 , –6
2 4
For k = 0, y = –x(x – 3)
(a) The graph opens downwards. For k = 1, y = (x – 5)(x – 1)
(b) (0, 0) and (3, 0) (a) The graph opens upwards.
(c) (0, 0) (b) (1, 0) and (5, 0)
(d) The line of symmetry is halfway between the x-intercepts. (c) (0, 5)
1 (d) The line of symmetry is halfway between the x-intercepts.
(e) x = 1
2 (e) x = 3
1 1 (f) minimum point (3, – 4)
(f) maximum point 1 , 2
2 4
For k = 2, y = (x – 5)(x – 2)
For k = 1, y = –(x – 3)(x – 1) (a) The graph opens upwards.
(a) The graph opens downwards. (b) (2, 0) and (5, 0)
(b) (1, 0) and (3, 0) (c) (0, 10)
(c) (0, –3) (d) The line of symmetry is halfway between the x-intercepts.
(d) The line of symmetry is halfway between the x-intercepts.

36
1 6. For the equation y = (x – 3)(x – k), the graph is symmetrical about the
(e) x = 3
2 vertical line that passes through the minimum point. For the equation
1 1 y = –(x – 3)(x – k), the graph is symmetrical about the vertical line
(f) minimum point 3 , –2
2 4 that passes through the maximum point.
y = –(x – 5)(x – k)
For k = –2, y = –(x – 5)(x + 2) Investigation (Graphs of y = (x – p)2 + q or y = –(x – p)2 + q)
(a) The graph opens downwards. 1. y = (x – 2)2 + q
(b) (–2, 0) and (5, 0) For q = –4, y = (x – 2)2 – 4
(c) (0, 10) y
(d) The line of symmetry is halfway between the x-intercepts.
1 6
(e) x = 1
2
4
1 1
(f) maximum point 1 , 12
2 4
2
For k = –1, y = –(x – 5)(x + 1)
(a) The graph opens downwards. x
–2 –1 0 1 2 3 4 5 6
(b) (–1, 0) and (5, 0)
–2
(c) (0, 5)
(d) The line of symmetry is halfway between the x-intercepts. –4
(e) x = 2
(f) maximum point (2, 9) –6

For k = 0, y = –x(x – 5)
For q = –1, y = (x – 2)2 – 1
(a) The graph opens downwards.
y
(b) (0, 0) and (5, 0)
(c) (0, 0)
(d) The line of symmetry is halfway between the x-intercepts. 8
1
(e) x = 2 6
2
1 1
(f) maximum point 2 , 6 4
2 4

For k = 1, y = –(x – 5)(x – 1) 2


(a) The graph opens downwards.
(b) (1, 0) and (5, 0) x
0
(c) (0, –5) –2 –1 1 2 3 4 5 6
(d) The line of symmetry is halfway between the x-intercepts. –2
(e) x = 3
(f) maximum point (3, 4) For q = 0, y = (x – 2)2
For k = 2, y = –(x – 5)(x – 2) y
(a) The graph opens downwards.
(b) (2, 0) and (5, 0) 10
(c) (0, –10)
8
(d) The line of symmetry is halfway between the x-intercepts.
1
(e) x = 3 6
2
1 1 4
(f) maximum point 3 , 2
2 4
4. For the equation y = (x – 3)(x – k), the graph opens upwards while 2
for the equation y = –(x – 3)(x – k), the graph opens downwards.
5. For the equation y = (x – 3)(x – k), the graph cuts the x-axis at x
–2 –1 0 1 2 3 4 5 6
(3, 0) and (k, 0). For the equation y = –(x – 3)(x – k), the graph cuts
the x-axis at (3, 0) and (k, 0).

37
For q = 1, y = (x – 2)2 + 1 For q = 1, y = (x – 2)2 + 1
y
(a) The graph opens upwards.
(b) No x-intercepts
(c) (0, 5)
10 (d) x = 2
(e) minimum point (2, 1)
8
For q = 4, y = (x – 2)2 + 4
6 (a) The graph opens upwards.
(b) No x-intercepts
4 (c) (0, 8)
(d) x = 2
2 (e) minimum point (2, 4)
3. Step 1:
x y = –(x – 2)2 + q
–2 –1 0 1 2 3 4 5 6
For q = –4, y = –(x – 2)2 – 4
y
For q = 4, y = (x – 2)2 + 4
y
2

x
12 –2 –1 0 1 2 3 4 5 6
–2
10
–4

8 –6

6 –8

–10
4
–12
2

x For q = –1, y = –(x – 2)2 – 1


–2 –1 0 1 2 3 4 5 6
y
2
2. y = (x – 2) + q
For q = – 4, y = (x – 2)2 – 4 2
(a) The graph opens upwards.
(b) (0, 0) and (4, 0) x
–2 –1 0 1 2 3 4 5 6
(c) (0, 0)
(d) x = 2 –2

(e) minimum point (2, – 4)


–4
2
For q = –1, y = (x – 2) – 1
(a) The graph opens upwards. –6

(b) (1, 0) and (3, 0)


–8
(c) (0, 3)
(d) x = 2 –10
(e) minimum point (2, –1)
–12
For q = 0, y = (x – 2)2
(a) The graph opens upwards.
(b) (2, 0)
(c) (0, 4)
(d) x = 2
(e) minimum point (2, 0)

38
For q = 0, y = –(x – 2)2 y = (x + 3)2 + q
y For q = –4, y = (x + 3)2 – 4
y
2
8
x
–2 –1 0 1 2 3 4 5 6
6
–2
4
–4
2
–6
x
–7 –6 –5 –4 –3 –2 –1 0 1
–8
–2
–10 –4

–12 –6

For q = –1, y = (x + 3)2 – 1


For q = 1, y = –(x – 2)2 + 1
y y

2 12

x 10
–3 –2 –1 0 1 2 3 4 5 6 7
–2 8

–4 6

–6 4
–8 2
–10
x
–7 –6 –5 –4 –3 –2 –1 0 1
–12
–2

For q = 4, y = –(x – 2)2 + 4


For q = 0, y = (x + 3)2
y y

4
12
2
10
x
–2 –1 0 1 2 3 4 5 6 8
–2
6
–4
4
–6
2

–8 x
–7 –6 –5 –4 –3 –2 –1 0 1
–10

–12

39
For q = 1, y = (x + 3)2 + 1 For q = –1, y = –(x + 3)2 – 1
y y

x
–6 –5 –4 –3 –2 –1 0 1
12
–2
10
–4
8
–6
6
–8
4

2 –10

x –12
–7 –6 –5 –4 –3 –2 –1 0 1

For q = 4, y = (x + 3)2 + 4
For q = 0, y = –(x + 3)2
y y

14 x
–7 –6 –5 –4 –3 –2 –1 0 1
12 –2
10
–4
8
–6
6
–8
4

2 –10

x
–7 –6 –5 –4 –3 –2 –1 0 1 –12

y = –(x + 3)2 + q For q = 1, y = –(x + 3)2 + 1


For q = –4, y = –(x + 3)2 – 4 y
y
2

x x
–7 –6 –5 –4 –3 –2 –1 0 1 –7 –6 –5 –4 –3 –2 –1 0 1
–2 –2

–4 –4

–6 –6

–8 –8

–10
–10

–12

40
For q = 4, y = –(x + 3)2 + 4 y = (x + 3)2 + q
y For q = –4, y = (x + 3)2 – 4
(a) The graph opens upwards.
6
(b) (–5, 0), (–1, 0)
4 (c) (0, 5)
(d) x = –3
2 (e) minimum point (–3, –4)

x For q = –1, y = (x + 3)2 – 1


–7 –6 –5 –4 –3 –2 –1 0 1
(a) The graph opens upwards.
–2 (b) (–4, 0), (–2, 0)
(c) (0, 8)
–4
(d) x = –3
–6 (e) minimum point (–3, –1)

–8
For q = 0, y = (x + 3)2
(a) The graph opens upwards.
(b) (–3, 0)
Step 2:
(c) (0, 9)
y = –(x – 2)2 + q
(d) x = –3
For q = – 4, y = –(x – 2)2 – 4
(e) minimum point (–3, 0)
(a) The graph opens downwards.
(b) No x-intercepts For q = 1 y = (x + 3)2 + 1
(c) (0, –8) (a) The graph opens upwards.
(d) x = 2 (b) No x-intercepts
(e) maximum point (2, – 4) (c) (0, 10)
(d) x = –3
For q = –1, y = –(x – 2)2 – 1
(e) minimum point (–3, 1)
(a) The graph opens downwards.
(b) No x-intercepts For q = 4, y = (x + 3)2 + 4
(c) (0, –5) (a) The graph opens upwards.
(d) x = 2 (b) No x-intercepts
(e) maximum point (2, –1) (c) (0, 13)
(d) x = –3
For q = 0, y = –(x – 2)2 (e) minimum point (–3, 4)
(a) The graph opens downwards.
y = –(x + 3)2 + q
(b) (2, 0)
For q = – 4, y = –(x + 3)2 – 4
(c) (0, – 4)
(a) The graph opens downwards.
(d) x = 2
(b) No x-intercepts
(e) maximum point (2, 0)
(c) (0, –13)
For q = 1, y = –(x – 2)2 + 1 (d) x = –3
(a) The graph opens downwards. (e) maximum point (–3, –4)
(b) (1, 0), (3, 0)
For q = –1, y = –(x + 3)2 – 1
(c) (0, –3)
(a) The graph opens downwards.
(d) x = 2
(b) No x-intercepts
(e) maximum point (2, 1)
(c) (0, –10)
For q = 4, y = –(x – 2)2 + 4 (d) x = –3
(a) The graph opens downwards. (e) maximum point (–3, –1)
(b) (0, 0), (4, 0)
For q = 0, y = –(x + 3)2
(c) (0, 0)
(a) The graph opens downwards.
(d) x = 2
(b) (–3, 0)
(e) maximum point (2, 4)
(c) (0, –9)
(d) x = –3
(e) maximum point (–3, 0)

41
For q = 1, y = –(x + 3)2 + 1 Graph 1 C: y = (x – 1)(x + 6)
(a) The graph opens downwards. Graph 2 y = (x – 1)(x – 6)
(b) (–4, 0), (–2, 0) E: y = x2 – 7x + 6
(c) (0, –8) Graph 3 D: y = –(x – 1)(x –6)
(d) x = –3 Graph 4 y = (x + 1)(x – 6)
(e) maximum point (–3, 1) G: y = x2 – 5x – 6
Graph 5 y = –(x – 1)(x + 6)
For q = 4, y = –(x + 3)2 + 4
F: y = –x2 – 5x + 6
(a) The graph opens downwards.
Graph 6 A: y = –(x + 1)(x + 6)
(b) (–5, 0), (–1, 0)
Graph 7 y = –(x + 1)(x – 6)
(c) (0, –5)
H: y = –x2 + 5x + 6
(d) x = –3
Graph 8 B: y = (x + 1)(x + 6)
(e) maximum point (–3, 4)
4. For the equation y = (x – p)2 + q for q = –4, –1, 0, 1 and 4, the graph
Practise Now 1
opens upwards while for the equation y = –(x – p)2 + q for q = –4,
–1, 0, 1 and 4, the graph opens downwards. (a) x2 + 7x – 8 = 0
5. For the equation y = (x – p)2 + q for q = –4, –1, 0, 1 and 4, the (x + 8)(x – 1) = 0
coordinates of the minimum point of the graph are (p, q). x+8 =0 or x–1 =0
For the equation y = –(x – p)2 + q for q = –4, –1, 0, 1 and 4, the x = –8 x =1
coordinates of the maximum point of the graph are (p, q). x = –8 or x = 1
6. The line of symmetry of each graph is about the line x = p. (b) 6y2 + 7y – 20 = 0
(3y – 4)(2y + 5) = 0
Thinking Time (Page 33) 3y – 4 = 0 or 2y + 5 = 0
2 3y = 4 2y = –5
1. y = –(x – 1) + 4
1 1
Since the graph cuts the x-axis at (3, 0) and (–1, 0), y =1 y = –2
3 2
y = –(x + 1)(x – 3)
1 1
y y = 1 or y = –2
3 2

Practise Now 2
(a) (x + 7)2 = 100
x + 7 = ± 100
y = – (x + 1)(x – 3)
x + 7 = ±10
x + 7 = 10 or x + 7 = –10
x =3 x = –17
x x = 3 or x = –17
–1 0 3 (b) (y – 5)2 = 11
y – 5 = ± 11
2. y = (x – a)2 + b y – 5 = 11 or y – 5 = – 11
For the equation y = (x – a)2 + b, the graph opens upwards. The y = 11 + 5 y = – 11 + 5
coordinates of the minimum point of the graph are (a, b) and the = 8.32 (to 3 s.f.) = 1.68 (to 3 s.f.)
graph is symmetrical about the line x = a. y = 8.32 or y = 1.68
Since the minimum point is at (2, –1), the equation of the curve
can be expressed as y = (x – 2)2 – 1. Practise Now 3
(a) x is 20. Half of this is 10.
Class Discussion (Matching Quadratic Graphs with the
x2 + 20x = [x2 + 20x + 102] – 102
Corresponding Functions)
= (x + 10)2 – 100
For Graphs 1, 2, 4 and 8, the graphs open upwards and so, the function
is of the form y = (x – h)(x – k) , where (h, k) is the minimum point.
For Graphs 3, 5, 6 and 7, the graphs open downwards and so, the function
is of the form y = –(x – h)(x – k), where (h, k) is the maximum point.

42
7 (c) x2 – x – 1 = 0
(b) x is –7. Half of this is – .
2 x2 – x = 1
2 2 2 2
7 7 1 1
x2 – 7x = x 2 – 7 x + – – – x2 – x + – =1+ –
2 2 2 2
2 2
7 49 1 1
= x– – x– =1+
2 4 2 4
1 1 1 5
(c) x is . Half of this is . x– =±
5 10 2 4
2 2
1 1 1 1 1 5 1 5
x2 + x = x2 + x + – x– =
4
or x– =–
4
5 5 10 10 2 2
1
2 5 1 5 1
x+ 1 x=+ x =– +
= – 4 2 4 2
10 100
= 1.62 (to 2 d.p.) = –0.62 (to 2 d.p.)
(d) x2 + 6x – 9 = (x2 + 6x) – 9
x = 1.62 or x = –0.62
x is 6. Half of this is 3.
2. (x + 4)(x – 3) = 15
x + 6x – 9 = [x + 6x + 32] – 32 – 9
2 2
x2 + x – 12 = 15
= (x + 3)2 – 18
x2 + x = 27
2 2
Practise Now 4 1 1
x2 + x + = 27 +
2 2
1. (a) x2 + 6x – 4 =0 1
2
1
x2 + 6x =4 x+ = 27 +
2 2
2 4
6 6
x2 + 6x + =4+ 1 109
2 2 x+ =±
2 4
x2 + 6x + 32 = 4 + 32
1 109 1 109
(x + 3)2 = 13 x+ = or x+ =–
2 4 2 4
x + 3 = ± 13
109 1 109 1
x + 3 = 13 or x + 3 = – 13 x = – x =– –
4 2 4 2
x = 13 – 3 x = – 13 – 3 = 4.72 (to 3 s.f.) = –5.72 (to 3 s.f.)
= 0.61 (to 2 d.p.) = –6.61 (to 2 d.p) x = 4.72 or x = –5.72
x = 0.61 or x = –6.61
(b) x2 + 7x + 5 = 0 Practise Now 5
x2 + 7x = –5
(a) Comparing 2x2 + 3x – 7 = 0 with ax2 + bx + c = 0, we have a = 2,
2 2
7 7 b = 3 and c = –7.
x2 + 7x + = –5 +
2 2
2 –3 ± 32 – 4(2)(–7)
7 49 x=
x+ = –5 + 2(2)
2 4
7 29 –3 ± 65
x+ =± =
2 4 4
= 1.27 (to 3 s.f.), –2.77 (to 3 s.f.)
7 29 7 29
x+ = or x+ =– x = 1.27 or x = –2.77
2 4 2 4
(b) Comparing 5x2 – 8x – 1 = 0 with ax2 + bx + c = 0, we have a = 5,
29 7 29 7
x = – x =– – b = –8 and c = –1.
4 2 4 2
= –0.81 (to 2 d.p.) = –6.19 (to 2 d.p.) –(–8) ± (–8)2 – 4(5)(–1)
x=
x = –0.81 or x = –6.19 2(5)
8 ± 84
=
10
= 1.72 (to 3 s.f.), –0.117 (to 3 s.f.)
x = 1.72 or x = –0.117

43
(c) (x – 1)2 = 4x – 5 2. y = 7 – 4x – 3x2
2
x – 2x + 1 = 4x – 5
x2 – 6x + 6 = 0 x –3 –2 –1 0 1 2
Comparing x2 – 6x + 6 = 0 with ax2 + bx + c = 0, we have a = 1, y –8 3 8 7 0 –13
b = –6 and c = 6.
–(–6) ± (–6)2 – 4(1)(4) y
x=
2(1)
10
6 ± 20
=
2
= 4.73 (to 3 s.f.), 1.27 (to 3 s.f.) 5 y = 7 – 4x – 3x2

x = 4.73 or x = 1.27
(d) (x + 3)(x – 1) = 8x – 7 x
0
x2 + 2x – 3 = 8x – 7 –3 –2 –1 1 2

x2 – 6x + 4 = 0 –5
Comparing x2 – 6x + 4 = 0 with ax2 + bx + c = 0, we have a = 1,
b = –6 and c = 4.
–10
–(–6) ± (–6)2 – 4(1)(6)
x= Scale:
2(1) x-axis: 2 cm represent 1 unit
–15
6 ± 12 y-axis: 2 cm represent 5 units
=
2
From the graph, the x-coordinates of the points of intersection of
= 5.24 (to 3 s.f.), 0.764 (to 3 s.f.)
y = 7 – 4x – 3x2 and the x-axis are x = –2.3 and x = 1.
x = 5.24 or x = 0.764
The solutions of the equation 7 – 4x – 3x2 = 0 are x = –2.3 and
x = 1.
Practise Now 6
1. (i) y = 2x2 – 4x – 1 Practise Now 7

x –2 –1 0 1 2 3 4 1. (i) y = x2 – 6x + 9

y 15 5 –1 –3 –1 5 15 x –1 0 1 2 3 4 5 6

(ii) y 16 9 4 1 0 1 4 9
y
(ii)
15
y
Scale:
10 15 x-axis: 2 cm represent 1 unit
y-axis: 2 cm represent 5 units

5 y = 2x2 – 4x – 1 10

y = x2 – 6x = 9
x 5
–2 –1 0 1 2 3 4
Scale:
–5 x-axis: 2 cm represent 1 unit x
y-axis: 2 cm represent 5 units –1 0 1 2 3 4 5 6

(iii) From the graph, the x-coordinates of the points of intersection


of y = 2x2 – 4x – 1 and the x-axis are x = –0.2 and x = 2.2. (iii) From the graph, the x-coordinate of the point of intersection of
The solutions of the equation 2x2 – 4x – 1 = 0 are x = –0.2 y = x2 – 6x + 9 and the x-axis is x = 3.
and x = 2.2. The solution of the equation x2 – 6x + 9 = 0 is x = 3.

44
2. y = 8x – x2 – 16

x 0 1 2 3 4 5 6 7 8

y –16 –9 –4 –1 0 –1 –4 –9 –16

x
0 1 2 3 4 5 6 7 8

y = 8x – x2 – 16

–5

–10

–15

Scale:
x-axis: 2 cm represent 1 unit
–20 y-axis: 4 cm represent 5 units

From the graph, the x-coordinate of the point of intersection of


y = 8x – x2 – 16 and the x-axis is x = 4.
3
The solution of the equation 8x – x2 – 16 = 0 is x = 4. (b) = 3x – 1
x+2
3
Practise Now 8 (x + 2) = (3x – 1) (x + 2)
x+2
6 3 = (3x – 1)(x + 2)
1. (a) =x+3
x+4 3 = 3x2 + 5x – 2
6 0 = 3x2 + 5x – 5
(x + 4) = (x + 3) (x + 4)
x+4 2
3x + 5x – 5 = 0
6 = (x + 3)(x + 4) Comparing 3x2 + 5x – 5 = 0 with ax2 + bx + c = 0, we have
6 = x2 + 7x + 12 a = 3, b = 5 and c = –5.
0 = x2 + 7x + 6
2
–5 ± 5 2 – 4(3)(–5)
x + 7x + 6 = 0 x=
2(3)
(x + 1)(x + 6) = 0
–5 ± 85
x+1 =0 or x+6 =0 =
6
x = –1 x = –6
= 0.703 (to 3 s.f.), –2.37 (to 3 s.f.)
x = –1 or x = –6
x = 0.703 or x = –2.37

45
4
2. = 2x – 3 –(– 44) ± (– 44)2 – 4(8)(55)
x x=
4 2(8)
x = (2x – 3) x 44 ± 176
x =
4 = x(2x – 3) 16
= 3.58 (to 3 s.f.), 1.92 (to 3 s.f.)
4 = 2x2 – 3x
x = 3.58 or x = 1.92
0 = 2x2 – 3x – 4
3 1
2x2 – 3x – 4 = 0 2. – =2
x–2 (x – 2)2
Comparing 2x2 – 3x – 4 = 0 with ax2 + bx + c = 0, we have a = 2,
3 1
b = –3 and c = – 4. – (x – 2)2 = 2 (x – 2)2
x – 2 (x – 2)2
–(–3) ± (–3)2 – 4(2)(–4)
x= 3 1
2(2) (x – 2)2 – (x – 2)2 =2 (x – 2)2
x–2 (x – 2)2
3 ± 41 3(x – 2) – 1 = 2(x – 2)2
=
4
3x – 6 – 1 = 2(x2 – 4x + 4)
= 2.35 (to 3 s.f.), –0.851 (to 3 s.f.)
3x – 7 = 2x2 – 8x + 8
x = 2.35 or x = –0.851
0 = 2x2 – 11x + 15
2x2 – 11x + 15 =0
Practise Now 9
(x – 3)(2x – 5) =0
1 2 x–3 =0 or 2x – 5 = 0
1. (a) + =5
x–2 x–3 x =3 2x = 5
1 2
+ (x – 2)(x – 3) = 5(x – 2)(x – 3) x = 2.5
x–2 x–3
x = 3 or x = 2.5
1 2
(x – 2)(x – 3) + (x – 2)(x – 3) = 5(x – 2)(x – 3)
x–2 x–3 Practise Now 10
(x – 3) + 2(x – 2) = 5(x – 2)(x – 3)
x – 3 + 2x – 4 = 5(x2 – 5x + 6) (i) AB = 17 – 8 – x
3x – 7 = 5x2 – 25x + 30 = (9 – x) cm
0 = 5x2 – 28x + 37 (ii) By Pythagoras’ Theorem,
5x2 – 28x + 37 = 0 AC2 = AB2 + BC2
Comparing 5x – 28x + 37 = 0 with ax2 + bx + c = 0, we have
2 82 = (9 – x)2 + x2
a = 5, b = –28 and c = 37. 64 = 81 – 18x + x2 + x2
0 = 17 – 18x + 2x2
–(–28) ± (–28)2 – 4(5)(37) 2
x= 2x – 18x + 17 = 0 (shown)
2(5)
(iii) 2x2 – 18x + 17 = 0
28 ± 44
= Comparing 2x2 – 18x + 17 = 0 with ax2 + bx + c = 0, we have a = 2,
10
= 3.46 (to 3 s.f.), 2.14 (to 3 s.f.) b = –18 and c = 17.
x = 3.46 or x = 2.14 –(–18) ± (–18)2 – 4(2)(17)
x=
5 x –1 2(2)
(b) – =7
x–3 x–2 18 ± 188
5 x –1 =
– (x – 2)(x – 3) = 7(x – 2)(x – 3) 4
x–3 x–2 = 7.928 (to 3 d.p.), 1.072 (to 3 d.p.)
5 x –1 x = 7.928 or x = 1.072
(x – 2)(x – 3) – (x – 2)(x – 3) = 7(x – 2)(x – 3)
x–3 x–2
(iv) BC = 1.072 cm
5(x – 2) – (x – 1)(x – 3) = 7(x – 2)(x – 3)
AB = 9 – 1.072 = 7.928 cm
5x – 10 – (x2 – 4x + 3) = 7(x2 – 5x + 6)
1
5x – 10 – x2 + 4x – 3 = 7x2 – 35x + 42 Area of triangle = AB BC
2
9x – 13 – x2 = 7x2 – 35x + 42 1
0 = 8x2 – 44x + 55 = 7.928 1.072
2
2
8x – 44x + 55 = 0 = 4.25 cm2 (to 3 s.f.)
Comparing 8x2 – 44x + 55 = 0 with ax2 + bx + c = 0, we have
a = 8, b = – 44 and c = 55.

46
Practise Now 11 (b) y = –(x – 3)(x + 1)
x2 is –1, the graph opens downwards.
600 600 15
(i) – = When y = 0,
x x+7 60
–(x – 3)(x + 1) = 0
600 600 1
– = x–3 =0 or x + 1) = 0
x x+7 4
600 600 1 x =3 x = –1
x(x + 7) – x(x + 7) = x(x + 7) The graph cuts the x-axis at (–1, 0) and (3, 0).
x x+7 4
1 When x = 0,
600(x + 7) – 600x = x(x + 7)
4 y = –(–3)(1) = 3
1 7 The graph cuts the y-axis at (0, 3).
600x + 4200 – 600x = x2 + x
4 4 y
1 7
4200 = x2 + x
4 4 y = –(x – 3)(x + 1)
1 2 7 3
x + x – 4200 = 0
4 4
x2 + 7x – 16 800 = 0 (shown)
x
(ii) x2 + 7x – 16 800 = 0 –1 0 3
Comparing x2 + 7x – 16 800 = 0 with ax2 + bx + c = 0, we have
a = 1, b = 7 and c = –16 800.

–7 ± 7 2 – 4(1)(–16 800)
x=
2(1)
–7 ± 67 249
= Line of symmetry
2
= 126.16 (to 2 d.p.), –133.16 (to 2 d.p.) (c) y = (3 – x)(x + 5)
x = 126.16 or x = –133.16 x2 is –1, the graph opens downwards.
(iii) x = 126.16 or x = –133.16 (rejected, since x > 0) When y = 0,
600 (3 – x)(x + 5) = 0
Time taken for the return journey = = 4.51 h (to 3 s.f.)
126.16 + 7 3–x =0 or x+5 =0
x =3 x = –5
Practise Now 12 The graph cuts the x-axis at (–5, 0) and (3, 0).
(a) y = (x – 2)(x – 6) When x = 0,
x2 is 1, the graph opens upwards. y = (3)(5) = 15
When y = 0, The graph cuts the y-axis at (0, 15).
(x – 2)(x – 6) = 0 y
x–2 =0 or x–6 =0
x =2 x=6
The graph cuts the y-axis at (0, 12). 15
y y = (3 – x)(x + 5)
Line of symmetry

x
–5 0 3
12

y = (x – 2)(x – 6)

x
0 2 6

47
Practise Now 13 Practise Now 14
2 2
1. (i) y = –(x – 2)2 + 9 6 6
1. (i) x2 – 6x + 6 = x – 6 x + –
2
– – +6
x2 is –1, the graph opens downwards. 2 2
When y = 0, = (x – 3)2 – 3
–(x – 2)2 + 9 = 0 (ii) The coordinates of the minimum point are (3, –3).
–(x – 2)2 = –9 (iii) When x = 0,
(x – 2)2 = 9 y = (0)2 – 6(0) + 6 = 0
x–2 =3 or x – 2 = –3 The graph cuts the y-axis at (0, 6).
x =5 x = –1 y
The graph cuts the x-axis at (5, 0) and (–1, 0).
x=3
When x = 0, y = x2 – 6x + 6
y = –(–2)2 + 9 = 5
The graph cuts the y-axis at (0, 5).
(ii) The coordinates of the maximum point are (2, 9).
(iii) y 6
(2, 9)

y = –(x – 2)2 + 9 x
5
–3
(3, –3)

(iv) The equation of the line of symmetry is x = 3.


x 2 2
1 1
2. (i) x2 + x + 1 = x + x +
–1 5 2
x=2 – +1
2 2
2
1 3
(iv) The equation of the line of symmetry is x = 2. = x+ +
2 4
2. (i) y = (x + 1)2 – 1 1 3
(ii) The coordinates of the minimum point are – , .
x2 is 1, the graph opens upwards. 2 4
When y 0, (iii) When x = 0,
(x + 1)2 – 1 = 0 y = (0)2 + 0 + 1 = 1
(x + 1)2 = 1 The graph cuts the y-axis at (0, 1).
x+1 =1 or x + 1 = –1 y
x =0 x = –2
The graph cuts the x-axis at (0, 0) and (–2, 0).
When x = 0,
y = (1)2 – 1 = 0
The graph cuts the y-axis at (0, 0). y = x2 + x + 1
(ii) The coordinates of the minimum point are (–1, –1).
(iii) y

y = (x + 1)2 – 1

– 1, 3 1
2 4
x
x=–1
2
x
–2 0
1
(iv) The equation of the line of symmetry is x = – .
x = –1 2

(iv) The equation of the line of symmetry is x = –1.

48
Exercise 1A 2. (a) (x + 1)2 = 9

1. (a) 2x2 + 5x – 7 =0 x+1 =± 9


(x – 1)(2x + 7) =0 x + 1 = ±3
x–1 =0 or 2x + 7 = 0 x+1 =3 or x + 1 = –3
x =1 2x = –7 x =2 x = –4
1 x = 2 or x = – 4
x = –3 (b) (2x + 1)2 = 16
2
1 2x + 1 = ± 16
x = 1or x = –3
2 2x + 1 = ±4
(b) 4x2 – 5x – 6 =0 2x + 1 =4 or 2x + 1 = – 4
(x – 2)(4x + 3) =0 2x =3 2x = –5
x–2 =0 or 4x + 3 = 0 1 1
x =1 x = –2
x =2 4x = –3 2 2
3 1 1
x =– x = 1 or x = –2
4 2 2
3 (c) (5x – 4)2 = 81
x = 2 or x = –
4 5x – 4 = ± 81
(c) 7x + x2 – 18 = 0
5x – 4 = ±9
(x – 2)(x + 9) = 0
5x – 4 =9 or 5x – 4 = –9
x–2 =0 or x+9 =0
5x = 13 5x = –5
x =2 x = –9
3
x = 2 or x = –9 x =2 x = –1
5
(d) 4 – 3x – x2 = 0 3
(1 – x)(x + 4) = 0 x = 2 or x = –1
5
1–x =0 or x+4 =0 9
(d) (7 – 3x)2 =
x =1 x = –4 16
x = 1 or x = – 4 9
(e) x(3x – 1) = 2 7 – 3x = ±
16
3x2 – x = 2 3
2 7 – 3x = ±
3x – x – 2 = 0 4
(x – 1)(3x + 2) = 0 3 3
7 – 3x = or 7 – 3x = –
x–1 =0 or 3x + 2 = 0 4 4
x =1 3x = –2 1 3
3x = 6 3x = 7
2 4 4
x =– 1 7
3 x =2 x=2
2 12 12
x = 1 or x = – 7 1
3 x=2 or x = 2
(f) (7 – 3x)(x + 2) = 4 12 12
7x + 14 – 3x2 – 6x = 4 (e) (x + 3)2 = 11
x + 14 – 3x2 = 4 x + 3 = ± 11
x + 10 – 3x2 = 0 x + 3 = 11 or x + 3 = – 11
(2 – x)(3x + 5) = 0
x = 11 – 3 x = – 11 – 3
2–x =0 or 3x + 5 = 0
= 0.32 (to 2 d.p.) = –6.32 (to 2 d.p.)
x =2 3x = –5
x = 0.32 or x = –6.32
2
x = –1 (f) (2x – 3)2 = 23
3
2 2x – 3 = ± 23
x = 2 or x = –1
3 2x – 3 = 23 or 2x – 3 = – 23
2x = 23 + 3 2x = – 23 + 3
x = 3.90 (to 2 d.p.) x = –0.90 (to 2 d.p.)
x = 3.90 or x = –0.90

49
(g) (5 – x)2 = 7 (h) x is –1.4. Half of this is –0.7.
5–x =± 7 x2 – 1.4x = [x2 – 1.4 + (–0.7)2] – (–0.7)2
= (x – 0.7)2 – 0.49
5–x = 7 or 5–x =– 7
4. (a) x2 + 2x – 5 = 0
x =5– 7 x =5+ 7 x2 + 2x = 5
= 2.35 (to 2 d.p.) = 7.65 (to 2 d.p.) x + 2x + 12 = 5 + 12
2

x = 2.35 or x = 7.65
(x + 1)2 = 6
2
1
(h) –x = 10 x+1 =± 6
2
1 x+1 = 6 or x+1 =– 6
– x = ± 10
2 x = 6 –1 x =– 6 –1
1 1 = 1.45 (to 2 d.p.) = –3.45 (to 2 d.p.)
– x = 10 or – x = – 10
2 2 x = 1.45 or x = –3.45
1 1 (b) x2 + 17x – 30 = 0
x = – 10 x= + 10
2 2 x2 + 17x = 30
= –2.66 (to 2 d.p.) = 3.66 (to 2 d.p.) 17
2
17
2

x = –2.66 or x = 3.66 x2 + 17x + = 30 +


2 2
3. (a) x is 12. Half of this is 6. 17
2

x+ 289
x2 + 12x = [x2 + 12x + 62] – 62 = 30 +
2 4
= (x + 6)2 – 36
17 409
(b) x is –6. Half of this is –3. x+ =±
2 4
x2 – 6x + 1 = [x2 – 6x + (–3)2] – (–3)2 + 1
17 409 17 409
= (x – 3)2 – 8 x+ = or x+ =–
2 4 2 4
3
(c) x is 3. Half of this is . 409 17 409 17
2 x = – x =– –
2 2 4 2 4 2
3 3
x2 + 3x – 2 = x + 3x +
2
– –2 = 1.61 (to 2 d.p.) = –18.61 (to 2 d.p.)
2 2
x = 1.61 or x = –18.61
2
3 17 (c) x2 – 12x + 9 = 0
= x+ –
2 4 x2 – 12x = –9
9 x – x + (–6)2 = –9 + (–6)2
2
(d) x is 9. Half of this is .
2 (x – 6)2 = 27
2 2
9 9
x2 + 9x – 1 = x + 9 x + x – 6 = ± 27
2
– –1
2 2
x–6 = 27 or x – 6 = – 27
2
9 85
= x+ – x = 27 + 6 x = – 27 + 6
2 4 = 11.20 (to 2 d.p.) = 0.80 (to 2 d.p.)
1 1 x = 11.20 or x = 0.80
(e) x is . Half of this is .
2 4 (d) x2 – 5x – 5 = 0
2 2
1 1 1 1 x2 – 5x = 5
x = x + x+
2
x2 + – 2 2
2 2 4 4 5 5
2
x2 – 5x + – =5+ –
1 1 2 2
= x+ – 2
4 16 5 45
x– =
2 1 2 4
(f) x is – . Half of this is – .
9 9 5 45
2 2 x– =±
2 2 1 1 2 4
x – x = x2 – x + –
2
– –
9 9 9 9 5 45 5 45
x– = or x– =–
1
2
1 2 4 2 4
= x– –
9 81 45 5 45 5
x= + x =– +
(g) x is 0.2. Half of this is 0.1. 4 2 4 2
x + 0.2x = [x + 0.2x + 0.12] – 0.12
2 2 = 5.85 (to 2 d.p.) = –0.85 ( to 2 d.p.)
= (x + 0.1)2 – 0.01 x = 5.85 or x = –0.85

50
1 5. (a) x(x – 3) = 5x + 1
(e) x2 + x–3 =0
4 x2 – 3x = 5x + 1
1 x2 – 8x =1
x2 + x = 3
4 x2 – 8x + (–4)2 = 1 + (–4)2
2 2
1 1 1 (x – 4)2 = 17
x2 + x+ =3+
4 8 8 x – 4 = ± 17
2
1 193
x+ = x – 4 = 17 or x – 4 = – 17
8 64
x = 17 + 4 x = – 17 + 4
1 193
x+ =± = 8.12 (to 2 d.p.) = –0.12 (to 2 d.p.)
8 64
x = 8.12 or x = –0.12
1 193 1 193 (b) (x + 1)2 = 7x
x+ = or x+ =– 2
8 64 8 64 x + 2x + 1 = 7x
193 1 193 1 x2 – 5x = –1
x= – x =– – 2 2
64 8 64 8 5 5
x2 – 5x + – = –1 + –
= 1.61 (to 2 d.p.) = –1.86 (to 2 d.p.) 2 2
2
x = 1.61 or x = –1.86 5 21
x– =
6 2 2 4
(f) x2 – x + =0
7 49 5 21
6 2 x– =±
x2 – x = – 2 4
7 49
5 21 5 21
6 3
2
2 3
2 x– = or x– =–
2
x – x+ – =– + – 2 4 2 4
7 7 49 7
21 5 21 5
3
2
1 x= + x =– +
x– = 4 2 4 2
7 7 = 4.79 (to 2 d.p.) = 0.21 (to 2 d.p.)
3 1 x = 4.79 or x = 0.21
x– =±
7 7 (c) (x + 2)(x – 5) = 4x
3 1 3 1 x2 – 3x – 10 = 4x
x– = or x– =–
7 7 7 7 x2 – 7x = 10
2 2
1 3 1 3 7 7
x =+ x =– + x2 – 7x + – = 10 + –
7 7 7 7 2 2
2
= 0.81 (to 2 d.p.) = 0.05 (to 2 d.p.) 7
x– 89
x = 0.81 or x = 0.05 =
2 4
(g) x2 + 0.6x – 1 = 0
7 89
x2 + 0.6x = 1 x– =±
2 4
x + 0.6x + 0.32 = 1 + 0.32
2

7 89 7 89
(x + 0.3)2 = 1.09 x– = or x– =–
2 4 2 4
x + 0.3 = ± 1.09 89 7 89 7
x= + x =– +
x + 0.3 = 1.09 or x + 0.3 = – 1.09 4 2 4 2
x = 1.09 – 0.3 x = – 1.09 – 0.3 = 8.22 (to 2 d.p.) = –1.22 (to 2 d.p.)
= 0.74 (to 2 d.p.) = –1.34 (to 2 d.p.) x = 8.22 or x = –1.22
x = 0.74 or x = –1.34 (d) x(x – 4) = 2(x + 7)
(h) x2 – 4.8x + 2 = 0 x2 – 4x = 2x + 14
x2 – 4.8x = –2 x2 – 6x = 14
x2 – 4.8x + (–2.4)2 = –2 + (–2.4)2 x – 6x + (–3)2 = 14 + (–3)2
2

(x – 2.4)2 = 3.76 (x – 3)2 = 23


x – 2.4 = ± 3.76 x – 3 = ± 23
x – 2.4 = 3.76 or x – 2.4 = – 3.76 x–3 = 23 or x – 3 = – 23
x = 3.76 + 2.4 x = – 3.76 + 2.4 x = 23 + 3 x = – 23 + 3
= 4.34 (to 2 d.p.) = 0.46 (to 2 d.p.) = 7.80 (to 2 d.p.) = –1.80 (to 2 d.p.)
x = 4.34 or x = 0.46 x = 7.80 or x = –1.80

51
6. y2 – ay – 6 = 0 (e) Comparing –3x2 – 7x + 9 = 0 with ax2 + bx + c = 0, we have
y – ay = 6 a = –3, b = –7 and c = 9.
2 2
a a –(–7) ± (–7)2 – 4(–3)(9)
y2 – ay + – =6+ – x=
2 2 2(–3)
2
a a2 7 ± 157
y– =6+ =
2 4 –6
= –3.25 (to 3 s.f.), 0.922 (to 3 s.f.)
a a2
y– =± 6+ x = –3.25 or x = 0.922
2 4
(f) Comparing –5x2 + 10x – 2 = 0 with ax2 + bx + c = 0, we have
a a 2 + 24 a = –5, b = 10 and c = –2.
y– =±
2 4
–10 ± 10 2 – 4(–5)(–2)
a a 2 + 24 x=
y– =± 2(–5)
2 4
–10 ± 60
a 2 + 24 =
a –10
y = ±
2 4 = 0.225 (to 3 s.f.), 1.77 (to 3 s.f.)
a ± a 2 + 24 x = 0.225 or x = 1.77
y = 2. (a) x2 + 5x = 21
2
x2 + 5x – 21 = 0
Exercise 1B Comparing x2 + 5x – 21 = 0 with ax2 + bx + c = 0, we have
a = 1, b = 5 and c = –21.
1. (a) Comparing x2 + 4x + 1 = 0 with ax2 + bx + c = 0, we have
a = 1, b = 4 and c = 1. –5 ± 5 2 – 4(1)(–21)
x=
2(1)
– 4 ± 4 2 – 4(1)(1)
x= –5 ± 109
2(1) =
2
– 4 ± 12 = 2.72 (to 3 s.f.), –7.72 (to 3 s.f.)
=
2 x = 2.72 or x = –7.72
= –0.268 (to 3 s.f.), –3.73 (to 3 s.f.)
(b) 10x2 – 12x = 15
x = –0.268 or x = –3.73
10x2 – 12x – 15 = 0
(b) Comparing 3x2 + 6x – 1 = 0 with ax2 + bx + c = 0, we have
Comparing 10x2 – 12x – 15 = 0 with ax2 + bx + c = 0, we have
a = 3, b = 6 and c = –1.
a = 10, b = –12 and c = –15.
–6 ± 6 2 – 4(3)(–1) –(–12) ± (–12)2 – 4(10)(–15)
x=
2(3) x=
2(10)
–6 ± 48
= 12 ± 744
6 =
20
= 0.155 (to 3 s.f.), –2.15 (to 3 s.f.)
= 1.96 (to 3 s.f.), –0.764 (to 3 s.f.)
x = 0.155 or x = –2.15
x = 1.96 or x = –0.764
(c) Comparing 2x2 – 7x + 2 = 0 with ax2 + bx + c = 0, we have
(c) 8x2 = 3x + 6
a = 2, b = –7 and c = 2. 2
8x – 3x – 6 = 0
–(–7) ± (–7)2 – 4(2)(2) Comparing 8x2 – 3x – 6 = 0 with ax2 + bx + c = 0, we have
x=
2(2) a = 8, b = –3 and c = –6.
7 ± 33 –(–3) ± (–3)2 – 4(8)(–6)
= x=
4 2(8)
= 3.19 (to 3 s.f.), 0.314 (to 3 s.f.)
3 ± 201
x = 3.19 or x = 0.314 =
16
(d) Comparing 3x2 – 5x – 17 = 0 with ax2 + bx + c = 0, we have
= 1.07 (to 3 s.f.), –0.699 (to 3 s.f.)
a = 3, b = –5 and c = –17.
x = 1.07 or x = –0.699
–(–5) ± (–5)2 – 4(3)(–17)
x=
2(3)
5 ± 229
=
6
= 3.36 (to 3 s.f.), –1.69 (to 3 s.f.)
x = 3.36 or x = –1.69
52
(d) 4x2 – 7 = 2x
2 –(–7) ± (–7)2 – 4(3)(–3)
4x – 2x – 7 = 0 x=
2(3)
Comparing 4x2 – 2x – 7 = 0 with ax2 + bx + c = 0, we have
7 ± 85
a = 4, b = –2 and c = –7. =
6
–(–2) ± (–2)2 – 4(4)(–7) = 2.70 (to 3 s.f.), –0.370 (to 3 s.f.)
x=
2(4) x = 2.70 or x = –0.370
2 ± 116 (c) (x – 1)2 – 2x = 0
= 2
8 x – 2x + 1 – 2x = 0
= 1.60 (to 3 s.f.), –1.10 (to 3 s.f.) x2 – 4x + 1 = 0
x = 1.60 or x = –1.10 Comparing x2 – 4x + 1 = 0 with ax2 + bx + c = 0, we have
(e) 9 – 5x2 = –3x a = 1, b = –4 and c = 1.
2
5x – 3x – 9 = 0
–(– 4) ± (– 4)2 – 4(1)(1)
Comparing 5x2 – 3x – 9 = 0 with ax2 + bx + c = 0, we have x=
2(1)
a = 5, b = –3 and c = –9.
4 ± 12
–(–3) ± (–3)2 – 4(5)(–9) =
x= 2
2(5) = 3.73 (to 3 s.f.), 0.268 (to 3 s.f.)
3 ± 189 x = 3.73 or x = 0.268
=
10 (d) x(x – 5) = 7 – 2x
= 1.67 (to 3 s.f.), –1.07 (to 3 s.f.) x2 – 5x = 7 – 2x
x = 1.67 or x = –1.07 x2 – 3x – 7 = 0
(f) 16x – 61 = x2 Comparing x2 – 3x – 7 = 0 with ax2 + bx + c = 0, we have
–x2 + 16x – 61 = 0 a = 1, b = –3 and c = –7.
Comparing –x2 + 16x – 61 = 0 with ax2 + bx + c = 0, we have
–(–3) ± (–3)2 – 4(1)(–7)
a = –1, b = 16 and c = –61. x=
2(1)
–16 ± 16 2 – 4(–1)(–61)
x= 3 ± 37
2(–1) =
2
–16 ± 12 = 4.54 (to 3 s.f.), –1.54 (to 3 s.f.)
=
–2 x = 4.54 or x = –1.54
= 6.27 (to 3 s.f.), 9.73 (to 3 s.f.) (e) (2x + 3)(x – 1) –x(x + 2) = 0
x = 6.27 or x = 9.73 2x2 – 2x + 3x – 3 – x2 – 2x = 0
3. (a) x(x + 1) = 1 x2 – x – 3 = 0
x2 + x = 1 Comparing x – x – 3 = 0 with ax2 + bx + c = 0, we have a = 1,
2

2
x +x–1 =0 b = –1 and c = –3.
Comparing x2 + x – 1 = 0 with ax2 + bx + c = 0, we have a = 1,
–(–1) ± (–1)2 – 4(1)(–3)
b = 1 and c = –1. x=
2(1)
–1 ± 12 – 4(1)(–1) 1 ± 13
x= =
2(1) 2
–1 ± 5 = 2.30 (to 3 s.f.), –1.30 (to 3 s.f.)
=
2 x = 2.30 or x = –1.30
= 0.618 (to 3 s.f.), –1.62 (to 3 s.f.) (f) (4x – 3)2 + (4x + 3)2 = 25
x = 0.618 or x = –1.62 16x – 24x + 9 + 16x2 + 24x + 9 = 25
2

(b) 3(x + 1)(x – 1) = 7x 32x2 + 18 = 25


3(x2 – 1) = 7x 32x2 – 7 = 0
3x2 – 3 = 7x Comparing 32x – 7 = 0 with ax2 + bx + c = 0, we have a = 32,
2

2
3x – 7x – 3 = 0 b = 0 and c = –7.
Comparing 3x2 – 7x – 3 = 0 with ax2 + bx + c = 0, we have
0 ± 0 2 – 4(32)(–7)
a = 3, b = –7 and c = –3. x=
2(32)
0 ± 896
=
64
= 0.468 (to 3 s.f.), –0.468 (to 3 s.f.)
x = 0.468 or x = –0.468

53
4. (a) 0.5(x2 + 1) = x Exercise 1C
0.5x2 + 0.5 = x
1. (i) y = 2x2 – 5x + 1
0.5x2 – x + 0.5 = 0
Comparing 0.5x2 – x + 0.5 = 0 with ax2 + bx + c = 0, we have x –1 0 1 2 3 4
a = 0.5, b = –1 and c = 0.5.
b2 – 4ac = (–1)2 – 4(0.5)(0.5) y 8 1 –2 –1 4 13
=0 (ii)
2
Since b – 4ac = 0, the equation has one real solution. y
Scale:
–(–1) ± 0 14
x-axis: 2 cm represent 1 unit
x= y-axis: 1 cm represents 1 unit
2(0.5)
1
= 12
1
=1
x=1 10
y = 2x2 – 5x + 1
3 1
(b) Comparing x2 + 2x – = 0 with ax2 + bx + c = 0, we have
4 2 8
3 1
a = , b = 2 and c = – .
4 2
6
2 2 3 1
b – 4ac = 2 – 4 –
4 2
1 4
=5
2
2
Since b – 4ac > 0, the equation has two real solutions. 2
1
–2 ± 5
2 x
x= 0
3 –1 1 2 3 4
2
4
–2
1
–2 ± 5
2
=
3
2 (iii) From the graph, the x-coordinates of the points of intersection
of y = 2x2 – 5x + 1 and the x-axis are x = 2.3 and x = 0.20.
= 0.230 (to 3 s.f.), –2.90 (to 3 s.f.)
The solutions of the equation 2x2 – 5x + 1 = 0 are x = 2.3 and
x = 0.230 or x = –2.90
x = 0.20.
(c) 5x – 7 = x2
2. (i) y = 7 – 5x – 3x2
x2 – 5x + 7 = 0
Comparing x2 – 5x + 7 = 0 with ax2 + bx + c = 0, we have x –3 –2 –1 0 1 2
a = 1, b = –5 and c = 7.
b2 – 4ac = (–5)2 – 4(1)(7) y –5 5 9 7 –1 –15
= –3
Since b2 – 4ac < 0, the equation has no real solutions.
(d) 3x – 4 = (4x – 3)2
3x – 4 = 16x2 – 24x + 9
16x2 – 27x + 13 = 0
Comparing 16x2 – 27x + 13 = 0 with ax2 + bx + c = 0, we have
a = 16, b = –27 and c = 13.
b2 – 4ac = (–27)2 – 4(16)(13)
= –103
Since b2 – 4ac < 0, the equation has no real solutions.

54
(ii) (ii)
x y

Scale:
15
x-axis: 2 cm represent 1 unit
10 y-axis: 2 cm represent 5 units
10
5 2
y = 7 – 5x – 3x y = 3x2 + 4x – 5
5
x
–3 –2 –1 0 1 2
x
–3 –2 –1 0 1 2
–5
–5
–10 Scale:
x-axis: 2 cm represent 1 unit
–10 y-axis: 1 cm represents 5 units
–15
(iii) From the graph, the x-coordinates of the points of intersection
(iii) From the graph, the x-coordinates of the points of intersection of y = 3x2 + 6x – 5 and the x-axis are x = –2.1 and x = 0.8.
of y = 7 – 5x – 3x2 and the x-axis are x = –2.55 and x = 0.90. The solutions of the equation 3x2 + 4x – 5 = 0 are x = –2.1
The solutions of the equation 7 – 5x – 3x2 = 0 are x = –2.55 and x = 0.8.
and x = 0.90. 5. y = 5 – 2x – x2
3. (i) y = x2 + 6x + 9
x –4 –3 –2 –1 0 1 2
x –5 –4 –3 –2 –1 0
y –3 2 5 6 5 2 –3
y 4 1 0 1 4 9
Scale: y
(ii) x-axis: 2 cm represent 1 unit
y y-axis: 2 cm represent 1 unit 6
Scale: 10
x-axis: 2 cm represent 1 unit
y-axis: 1 cm represents 1 unit 5
8
y = 5 – 2x – x2
4
y = x2 + 6x + 9
6
3
4
2
2
1
x
–5 –4 –3 –2 –1 0
x
–4 –3 –2 –1 0 1 2

(iii) From the graph, the x-coordinate of the point of intersection of –1


y = x2 + 6x + 9 and the x-axis is x = –3.
The solution of the equation x2 + 6x + 9 = 0 is x = –3. –2
4. (i) y = 3x2 + 4x –5

x –3 –2 –1 0 1 2 –3

y 10 –1 –6 –5 2 15 From the graph, the x-coordinates of the points of intersection of


y = 5 – 2x – x2 and the x-axis are x = –3.45 and x = 1.45.
The solutions of the equation 5 – 2x – x2 = 0 are x = –3.45 and
x = 1.45.

55
6. (i) y = 4x2 + 12x + 9 8. (a) P = 2 – 0.1(x – 3)2

x –4 –3 –2 –1 0 1 2 x 0 1 2 3 4 5 6 7 8 9 10

y 25 9 1 1 9 25 49 P 1.1 1.6 1.9 2.0 1.9 1.6 1.1 0.4 –0.5 –1.6 –2.9

(ii) P
y Scale:
3 x-axis: 2 cm represent 1 unit
Scale: P-axis: 2 cm represent $1 million
50
x-axis: 2 cm represent 1 unit
y-axis: 2 cm represent 10 units 2
40
1
P = 2 – 0.1(x – 3)2
30
x
0 1 2 3 4 5 6 7 8 9 10
20
–1
10
–2
x
–4 –3 –2 –1 0 1 2
–3
–10
–4
–20
(b) P = 0.
From the graph, the x-coordinate of the point of intersection of From the graph, the x-coordinate of the point of intersection of
y = 4x2 + 12x + 9 and the x-axis is x = –1.5. P = 2 – 0.1(x – 3)2 and the x-axis is x = 7.5.
The solution of the equation 4x2 + 12x + 9 = 0 is x = –1.5. x = 7.5.
7. y = 10x – 25 – x2

x 0 1 2 3 4 5 6 7 8 9 10

y –25 –16 –9 –4 –1 0 –1 –4 –9 –16 –25

y Scale:
x-axis: 1 cm represents 1 unit
y-axis: 2 cm represent 5 units
x
0 1 2 3 4 5 6 7 8 9 10

–5

–10
y = 10x – 25 – x2

–15

–20

–25

From the graph, the x-coordinate of the point of intersection of


y = 10x – 25 – x2 and the x-axis is x = 5.
The solution of the equation 10x – 25 – x2 = 0 is x = 5.

56
9. (a) y = 200 + 7x – 6x2
–1 ± 12 – 4(2)(–8)
x=
y 2(2)
–1 ± 65
250 =
4
Scale: = 1.77 (to 3 s.f.), –2.27 (to 3 s.f.)
x-axis: 1 cm represents 1 m x = 1.77 or x = –2.27
y-axis: 2 cm represent 50 cm
7
(b) 3x – 1 =
200 x+4
7
(3x – 1) (x + 4) = (x + 4)
x+4
(3x – 1)(x + 4) = 7
3x2 + 12x – x – 4 = 7
150 3x2 + 11x – 11 = 0
y = 200 + 7x – 6x2
Comparing 3x2 + 11x – 11 = 0 with ax2 + bx + c = 0, we have
a = 3, b = 11 and c = –11.
–11 ± 112 – 4(3)(–11)
x=
100 2(3)
–11 ± 253
=
6
= 0.818 (to 3 s.f.), –4.48 (to 3 s.f.)
x = 0.818 or x = –4.48
50
x +1
(c) =x
5–x
x +1
(5 – x) = x (5 – x)
5–x
x x + 1 = 5x – x2
0 1 2 3 4 5 6
0 = –x2 + 4x – 1
2
–x + 4x – 1 = 0
(b) (i) From the graph, the x-coordinate of the point of intersection Comparing –x2 + 4x – 1 = 0 with ax2 + bx + c = 0, we have
of y = 200 + 7x – 6x2 and the x-axis is x = 6.4. a = –1, b = 4 and c = –1.
The solution of the equation 200 + 7x – 6x2 = 0 is – 4 ± 4 2 – 4(–1)(–1)
x = 6.4. x=
2(–1)
(ii) From the graph, when the balloon is 50 cm above the ground,
– 4 ± 12
y = 50, x = 5.6. =
–2
The horizontal distance from the foot of the platform
= 0.268 (to 3 s.f.), 3.73 (to 3 s.f.)
when the balloon is 50 cm above the ground is 5.6 m.
x = 0.268 or x = 3.73
(c) t = 6.4. After t
7
level which is not valid in this case. (d) x+ =9
x
7
Exercise 1D x+ x =9 x
x
2
8 x + 7 = 9x
1. (a) = 2x + 1 2
x x – 9x + 7 = 0
8 = x(2x + 1) Comparing x2 – 9x + 7 = 0 with ax2 + bx + c = 0, we have
8 = 2x2 + x a = 1, b = –9 and c = 7.
0 = 2x2 + x – 8 –(–9) ± (–9)2 – 4(1)(7)
2x2 + x – 8 = 0 x=
2(1)
Comparing 2x2 + x – 8 = 0 with ax2 + bx + c = 0, we have
9 ± 53
a = 2, b = 1 and c = –8. =
2
= 8.14 (to 3 s.f.), 0.860 (to 3 s.f.)
x = 8.14 or x = 0.860

57
x +1 (iii) Since the numbers are positive, x = 3.
(e) 2x + 1 =
x–5 12
=3
x +1 3+1
(2x + 1) (x – 5) = (x – 5)
x–5 12
(2x + 1)(x – 5) = x + 1 =4
3
2x2 – 10x + x – 5 = x + 1 The two numbers are 3 and 4.
2x2 – 9x – 5 = x + 1 2 5x
3. (a) =
2x2 – 10x – 6 = 0 x +1 3– x
x2 – 5x – 3 = 0 2 5x
(x + 1)(3 – x) = (x + 1)(3 – x)
Comparing x2 – 5x – 3 = 0 with ax2 + bx + c = 0, we have x +1 3– x
a = 1, b = –5 and c = –3. 2(3 – x) = 5x(x + 1)
6 – 2x = 5x2 + 5x
–(–5) ± (–5)2 – 4(1)(–3)
x= 0 = 5x2 + 7x – 6
2(1)
(5x – 3)(x + 2) = 0
5 ± 37
= 5x – 3 = 0 or x+2 =0
2
5x = 3 x = –2
= 5.54 (to 3 s.f.), –0.541 (to 3 s.f.)
3
x = 5.54 or x = –0.541 x=
5
5x
(f) = 3x + 1 3
x+4 x= or x = –2
5
5x (x – 2)(x – 3)
(x + 4) = (3x + 1) (x + 4) 2
x+4 (b)
(x – 1)(x + 2)
=
3
5x = (3x + 1)(x + 4) (x – 2)(x – 3) 2
5x = 3x2 + 12x + x + 4 (x – 1)(x + 2) = (x – 1)(x + 2)
(x – 1)(x + 2) 3
5x = 3x2 + 13x + 4 2
0 = 3x2 + 8x + 4 (x – 2)(x – 3) = (x – 1)(x + 2)
3
2
3x + 8x + 4 = 0 2
x2 – 3x – 2x + 6 = (x2 + 2x – x – 2)
(3x + 2)(x + 2) = 0 3
3x + 2 = 0 or x+2 =0 2 2 2
x – 5x + 6 = (x + x – 2)
3x = –2 x = –2 3
2 2 2 2 2 4
x =– x – 5x + 6 = x + x –
3 3 3 3
2 1 2 17 22
x = – or x = –2 x – x+ =0
3 3 3 3
12 12 x2 – 17x + 22 = 0
2. (i) – =1
x +1 Comparing x – 17x + 22 = 0 with ax2 + bx + c = 0, we have
2
x
12 12 a = 1, b = –17 and c = 22.
x(x + 1) – x(x + 1) = 1 x(x + 1)
x x +1
–(–17) ± (–17)2 – 4(1)(22)
12(x + 1) – 12x = x(x + 1) x=
2(1)
12x + 12 – 12x = x2 + x
12 = x2 + x 17 ± 201
=
2 2
x + x – 12 = 0 (shown)
= 15.6 (to 3 s.f.), 1.41 (to 3 s.f.)
(ii) x2 + x – 12 = 0
x = 15.6 or x = 1.41
(x + 4)(x – 3) = 0
x+4 =0 or x–3 =0
x = –4 x =3
x = – 4 or x = 3

58
x(x – 3) 3 –1 ± 12 – 4(1)(–10)
4. =
(x + 1) 2
5 x=
2(1)
x(x – 3) 2 3
(x + 1) = (x + 1)2 –1 ± 41
(x + 1)2 5 =
3 2
x(x – 3) = (x + 1)2 = 2.70 (to 3 s.f.), –3.70 (to 3 s.f.)
5
3 2 x = 2.70 or x = –3.70
x2 – 3x = (x + 2x + 1)
5 x–2 1
(c) + =1
2 3 2 6 3 5 2x – 3
x – 3x = x + x+
5 5 5 x–2 1
+ 5(2x – 3) = 1 5(2x – 3)
2 2 21 3 5 2x – 3
x – x– =0
5 5 5 x–2 1
5(2x – 3) + 5(2x – 3)= 1 5(2x – 3)
2x2 – 21x – 3 = 0 5 2x – 3
Comparing 2x2 – 21x – 3 = 0 with ax2 + bx + c = 0, we have (x – 2)(2x – 3) + 5 = 5(2x – 3)
a = 2, b = –21 and c = –3. 2x2 – 3x – 4x + 6 + 5 = 10x – 15
–(–21) ± (–21)2 – 4(2)(–3) 2x2 – 7x + 11 = 10x – 15
x= 2x2 – 17x + 26 = 0
2(2)
(2x – 13)(x – 2) = 0
21 ± 465
= 2x – 13 = 0 or x – 2 = 0
4
2x = 13 x =2
= 10.6 (to 3 s.f.), –0.141 (to 3 s.f.)
x = 6.5
x = 10.6 or x = –0.141
x = 6.5 or x = 2
x 4
5. (a) = –1 3 2
2 x (d) + =5
x x +1
x 4 3 2
x =–1 x +
2 x x(x + 1) = 5 x(x + 1)
x x +1
x2 4
=x –1 3 2
2 x x(x + 1) + x(x + 1)= 5x(x + 1)
x2 x x +1
=4–x 3(x + 1) + 2x = 5x(x + 1)
2
x2 3x + 3 + 2x = 5x2 + 5x
+x–4 =0 5x + 3 = 5x2 + 5x
2
2
x + 2x – 8 = 0 5x2 – 3 = 0
(x + 4)(x – 2) = 0 5x2 = 3
x+4 =0 or x–2=0 3
x2 =
x = –4 x=2 5
x = – 4 or x = 2 3
x =±
2 x +1 5
(b) =1–
x+5 5 = 0.775 (to 3 s.f.),
2 x +1 –0.775 (to 3 s.f.)
+ =1
x+5 5 x = 0.775 or x = –0.775
2 x +1
+ 5(x + 5) = 1 5(x + 5)
x+5 5
2 x +1
5(x + 5) + 5(x + 5)= 1 5(x + 5)
x+5 5
10 + (x + 1)(x + 5) = 5(x + 5)
10 + x2 + 5x + x + 5 = 5x + 25
x2 + 6x + 15 = 5x + 25
x2 + x – 10 = 0
Comparing x + x – 10 = 0 with ax2 + bx + c = 0, we have
2

a = 1, b = 1 and c = –10.

59
1 1 3 5 4
(e) + = (g) =2–
x+2 x–2 8 x 2 (x 2)2
1 1 3 5 4
+ (x + 2)(x – 2) = (x + 2)(x – 2) + =2
x+2 x–2 8 x 2 (x 2)2
1 1 3 5 4
(x + 2)(x – 2) + (x + 2)(x – 2) = (x + 2)(x – 2) + (x – 2)2 = 2 (x – 2)2
x+2 x–2 8 x – 2 (x 2)2
3 2 5 4
(x – 2) + (x + 2) = (x – 4) (x – 2)2 + (x – 2)2= 2(x – 2)2
8 x 2 (x 2)2
3 2 3 5(x – 2) + 4 = 2(x – 2)2
2x = x –
8 2 5x – 10 + 4 = 2(x2 – 4x + 4)
0=
3 2
x – 2x –
3 5x – 6 = 2x2 – 8x + 8
8 2 0 = 2x2 – 13x + 14
3 2 3 2
2x – 13x + 14 = 0
x – 2x – =0
8 2 Comparing 2x2 – 13x + 14 = 0 with ax2 + bx + c = 0, we have
2
3x – 16x – 12 = 0
a = 2, b = –13 and c = 14.
Comparing 3x – 16x – 12 = 0 with ax2 + bx + c = 0, we have
2

a = 3, b = –16 and c = –12. –(–13) ± (–13)2 – 4(2)(14)


x=
2(2)
–(–16) ± (–16)2 – 4(3)(–12)
x= 13 ± 57
2(3) =
4
16 ± 400 = 5.14 (to 3 s.f.), 1.36 (to 3 s.f.)
=
6 x = 5.14 or x = 1.36
2 x
= 6, – 5
3 (h) + =1
x –1 (x – 1)2
2
x = 6 or x = – 5 x
3 + (x – 1)2 = 1 (x – 1)2
x – 1 (x – 1)2
7 x +1 1
(f) – = x
x –1 x+3 2 5
(x – 1)2 + (x – 1)2 = (x – 1)2
7 x +1 x –1 (x – 1)2
– 1
(x – 1)(x + 3) = (x – 1)(x + 3) 5(x – 1) + x = (x – 1)2
x –1 x + 3 2
7 x +1 1 5x – 5 + x = x2 – 2x + 1
(x – 1)(x + 3) – (x – 1)(x + 3) = (x – 1)(x + 3) 6x – 5 = x2 – 2x + 1
x –1 x+3 2
1 0 = x2 – 8x + 6
7(x + 3) – (x + 1)(x – 1) = (x – 1)(x + 3) 2
2 x – 8x + 6 = 0
1 2 Comparing x2 – 8x + 6 = 0 with ax2 + bx + c = 0, we have
7x + 21 – (x2 – 1) = (x + 3x – x – 3)
2 a = 1, b = –8 and c = 6.
1 2
7x + 21 – x2 + 1 = (x + 2x – 3) –(–8) ± (–8)2 – 4(1)(6)
2 x=
2(1)
1 2 3
7x + 22 – x2 = x +x– 8 ± 40
2 2 =
3 2 47 2
0 = x – 6x – = 7.16 (to 3 s.f.), 0.838 (to 3 s.f.)
2 2
3 2 47 x = 7.16 or x = 0.838
x – 6x – =0
2 2 112 – 2 x
6. (i) Length of rectangle =
3x2 – 12x – 47 = 0 2
Comparing 3x – 12x – 47 = 0 with ax2 + bx + c = 0, we have
2 = (56 – x) cm
a = 3, b = –12 and c = – 47. (ii) Area of rectangle = 597 cm2
x(56 – x) = 597
–(–12) ± (–12)2 – 4(3)(– 47)
x= 56x – x2 = 597
2(3)
56x – x2 – 597 = 0
12 ± 708 x2 – 56x + 597 = 0 (shown)
=
6
= 6.43 (to 3 s.f.), –2.43 (to 3 s.f.)
x = 6.43 or x = –2.43

60
(iii) x2 – 56x + 597 = 0 8. Let the length of the smaller square be x.
Comparing x2 – 56x + 597 = 0 with ax2 + bx + c = 0, we have Area of smaller square = x2
a = 1, b = –56 and c = 597. Area of larger square = 9x2
–(–56) ± (–56)2 – 4(1)(597) Length of square = ± 9 x 2
x= = 3x or –3x (reject, x > 0)
2(1)
56 ± 748 Total perimeter of both squares = 4x + 4(3x)
= = 200 cm
2
= 41.67 (to 2 d.p.), 14.33 (to 2 d.p.) 16x = 200
x = 41.67 or x = 14.33 x = 12.5
(iv) Since the length of a rectangle usually refers to the longer side, Perimeter of larger square = 12(12.5)
When x = 14.33, = 150 cm
Breadth = 14.33 cm 9. Let the length of one square be x and the length of the other square
Length = 56 – 14.33 = 41.67 cm be y.
By Pythagoras’ Theorem, Since total length of the wire is 100 cm,
(Length of diagonal)2 = Length2 + Breadth2 4x + 4y = 100
= 41.672 + 14.332 4y = 100 – 4x
y = 25 – x — (1)
Length of the diagonal = 41.67 2 + 14.332
Since total area of the squares is 425 cm2,
= 44.1 cm (to 3 s.f.)
x2 + y2 = 425 — (2)
AP PQ
7. (i) Given = , Subst. (1) into (2):
AB BC
6x x x2 + (25 – x)2 = 425
= x + (625 – 50x + x2) = 425
2
3x + 5 1
6x 2x2 – 50x + 200 = 0
=x
3x + 5 x2 – 25x + 100 = 0
6x (x – 20)(x – 5) = 0
(3x + 5) = x (3x + 5)
3x + 5 x – 20 = 0 or x–5=0
6x = x(3x + 5) x = 20 x=5
6x = 3x2 + 5x From (1), when x = 20, y = 5;
0 = 3x2 – x when x = 5, y = 20.
2
3x – x = 0 (shown) The lengths of the sides of the two squares are 5 cm and 20 cm.
(ii) 3x2 – x = 0 10. Let the original number of students be x.
x(3x – 1) = 0 120
x =0 or 3x – 1 = 0 x
3x = 1 120 120
Amount each student pays if 2 join = $ –2 =$
1 x x+2
x=
3 120 120
–2 =
1 x x+2
x = 0 or x =
3 120 – 2 x 120
=
1 x x+2
(iii) Since x > 0, x =
3 (120 – 2x)(x + 2) = 120x
PB = AB – AP 120x + 240 – 2x2 – 4x = 120x
1 1 2x2 + 4x – 240 = 0
= 3 +5 –6
3 3 x2 + 2x – 120 = 0
=6–2 (x – 10)(x + 12) = 0
= 4 cm x – 10 = 0 or x + 12 = 0
x = 10 x = –12
(rejected, since x > 0)
There were originally 10 students in the group.

61
11. Let the number of members in 2016 be x. 350
13. (i) Av. amount of rice ordered in Jan 2009 = kg
4200 x
Subscription fee of each member in 2016 = cents
x 350
(ii) Av. amount of rice ordered in Jan 2012 = kg
4200 x + 0.15
Subscription fee of each member in 2017 = – 10 cents
x 350 350
(iii) – = 30
4500 x x + 0.15
= cents
x + 20 350(x + 0.15) 350 x
– = 30
4200
– 10 =
4500 x(x + 0.15) x(x + 0.15)
x x + 20 350(x + 0.15) – 350 x
4200 – 10 x 4500 = 30
= x(x + 0.15)
x x + 20 350(x + 0.15) – 350x = 30x(x + 0.15)
(4200 – 10x)(x + 20) = 4500x
350x + 52.5 – 350x = 30x2 + 4.5x
4200x +84 000 – 10x2 – 200x = 4500x
30x2 + 4.5x – 52.5 =0
10x2 + 500x – 84 000 = 0
60x2 + 9x – 105 =0
x2 + 50x – 8400 = 0
20x2 + 3x – 35 = 0 (Shown)
(x – 70)(x + 120) = 0 2
(iv) 20x + 3x – 35 = 0
x – 70 = 0 or x + 120
(4x – 5)(5x + 7) = 0
x = 70 x = –120
4x – 5 = 0 or 5x + 7 = 0
(rejected, since x > 0)
4x = 5 5x = –7
There were 70 members in 2016.
2
60 x = 1.25 x = –1
(rejected since
12. (i) No. of pages printed by Printer A in 1 minute = 5
x x > 0)
60 Price per kilogram of rice in Jan 2012 = 1.25 + 0.15
(ii) No. of pages printed by Printer B in 1 minute =
x+2 = $1.40
60 60 2 8
(iii) + = 144 14. (i) Time taken by Rui Feng = + h
x x+2 x x +1
60(x + 2) 60 x
+ = 144 10
x(x + 2) x(x + 2) (ii) Time taken by Jun Wei = h
x
60(x + 2) + 60 x 10 2 8 40
= 144 – + =
x(x + 2) x x x +1 60
60(x + 2) + 60x = 144x(x + 2) 10 2 8 2
– – =
60x + 120 + 60x = 144x2 + 288x x x x +1 3
144x2 + 168x – 120 =0 8 8 2
– =
6x2 + 7x – 5 = 0 (Shown) x x +1 3
(iv) 2
6x + 7x – 5 = 0 8 8 2
– x(x + 1) = x(x + 1)
(2x – 1)(3x + 5) = 0 x x +1 3
2x – 1 = 0 or 3x + 5 = 0 8 8 2
x(x + 1) – x(x + 1) = x(x + 1)
x x +1 3
2x = 1 3x = –5
2 2 2
1 2 8(x + 1) – 8x = x + x
x= x = –1 3 3
2 3
2 2 2
1 2 8x + 8 – 8x = x + x
x= or x = –1 3 3
2 3
2 2 2
1 8 = x + x
(v) Since x > 0, x = 3 3
2
60 2 2 2
No. of pages printed by Printer B in 1 minute = 0 = x + x–8
x+2 3 3
2 2 2
60 x + x–8 =0
= 3 3
1
+2 2x2 + 2x – 24 = 0
2
x2 + x – 12 = 0 (Shown)
60
=
1
2
2
= 24
144
Time taken by Printer B to print 144 pages =
24
= 6 minutes
62
(iii) x2 + x – 12 = 0 1500 1500 30
(x + 4)(x – 3) = 0 (iii) – =
x x + 50 60
x+4 =0 or x–3 =0 1500 1500 1
x = –4 x =3 – x(x + 50) = x(x + 50)
x x + 50 2
x = – 4 or x = 3
1500 1500 1
Since the running speed cannot be a negative value, x = – 4 x(x + 50) – x(x + 50) = x(x + 50)
x x 50 2
is rejected. 1
2 8 1500(x + 50) – 1500x = x(x + 50)
(iv) Time taken by Rui Feng = + 2
3 3+1 1 2
1500x + 75 000 – 1500x = x + 25x
2 8 2
= +
3 4 1 2
75 000 = x + 25x
2 2
= +2 h 1 2
3 0= x + 25x – 75 000
= 2 h 40 minutes 2
700 700 1 2
15. (i) + = 20 x + 25x – 75 000 = 0
x x – 30 2
700 700 x2 + 50x – 150 000 = 0 (Shown)
+ x(x – 30) = 20 x(x – 30) 2
(iv) x + 50x – 150 000 = 0
x x – 30
Comparing x2 + 50x – 150 000 = 0 with ax2 + bx + c = 0, we
700 700
x(x – 30) + x(x – 30) = 20x(x – 30) have a = 1, b = 50 and c = –150 000.
x x – 30
700(x – 30) + 700x = 20x2 – 600x –50 ± 50 2 – 4(1)(–150 000)
x=
700x – 21 000 + 700x = 20x2 – 600x 2(1)
1400x – 21 000 = 20x2 – 600x –50 — 602 500
0 = 20x2 – 2000x + 21 000 =
2
2
20x – 2000x + 21 000 = 0 = 363.10 (to 2 d.p.), – 413.10 (to 2 d.p.)
x2 – 100x + 1050 = 0 (Shown) x = 363.10 or x = – 413.10
(ii) x2 – 100x + 1050 = 0 (v) Since x > 0, x = 363.10
Comparing x2 – 100x + 1050 = 0 with ax2 + bx + c = 0, we have Time taken by Pump B
a = 1, b = –100 and c = 1050. 1500
=
–(–100) ± (–100)2 – 4(1)(1050) 363.10 + 50
x= = 3.63 mins (to 3 s.f.)
2(1)
= 3 minutes 38 seconds (to the nearest second)
100 ± 5800
= 17. (i) At Samy’s Money Exchange,
2
S$x = US$1
= 88.08 (to 2 d.p.), 11.92 (to 2 d.p.)
1
x = 88.08 or x = 11.92 S$2000 = US$ 2000
x
(iii) Since average speed of the car > 30 km/h, x = 88.08
2000
700 = US$
Time taken for return journey = x
88.08
(ii) At Chan’s Money Exchange,
= 7.95 hours (to 3 s.f.)
S$(x + 0.05) = US$1
1500
16. (i) No. of minutes taken by Pump A = 1
x S$1000 = US$ 1000
1500 x + 0.05
(ii) No. of minutes taken by Pump B =
x + 50 1000
= US$
x + 0.05

63
2000 1000 1 2 3
(iii) + = 2370 (b) + + =0
x x + 0.05 x x –1 x +1
2000 1000 1 2 3
+ x(x + 0.05) = 2370 x(x + 0.05) + + x(x – 1)(x + 1) = 0 x(x – 1)(x + 1)
x x + 0.05 x x –1 x +1
2000 1000 1 2
x(x + 0.05) + x(x + 0.05) = 2370x(x + 0.05) x(x – 1)(x + 1) + x(x – 1)(x + 1)
x x + 0.05 x x –1
2000(x + 0.05) + 1000x = 2370x(x + 0.05) 3
+ x(x – 1)(x + 1) =0
2000x + 100 + 1000x = 2370x2 + 118.5x x +1
3000x + 100 = 2370x2 + 118.5x (x – 1)(x + 1) + 2x(x + 1) + 3x(x – 1) =0
0 = 2370x2 – 2881.5x x2 – 1 + 2x2 + 2x + 3x2 – 3x =0
– 100 6x2 – x – 1 =0
2
2370x – 2881.5x – 100 = 0 (2x – 1)(3x + 1) =0
237x2 – 288.15x – 10 = 0 (Shown) 2x – 1 = 0 or 3x + 1 = 0
2
(iv) 237x – 288.15x – 10 = 0 2x =1 3x = –1
Comparing 237x2 – 288.15x – 10 = 0 with ax2 + bx + c = 0, we 1 1
x= x =–
have a = 237, b = –288.15 and c = –10. 2 3
1 1
–(–288.15) ± (–288.15)2 – 4(237)(–10) x= or x = –
x= 2 3
2(237) 1 2
(c) – =1
288.15 ± 9 2.510.4225 x2 – 9 3– x
= 1 2
474 – =1
= 1.25 (to 2 d.p.), –0.03 (to 2 d.p.) (x + 3)(x – 3) –(x – 3)
x = 1.25 or x = –0.03 1 2
+ =1
(v) Since x > 0, x =1.25 (x + 3)(x – 3) x–3
At Chan’s Money Exchange, 1 2
+ (x + 3)(x – 3) = 1 (x + 3)(x – 3)
US$1 = S$(1.25 + 0.05) (x + 3)(x – 3) x – 3
= S$1.30 1 2
(x + 3)(x – 3) + (x + 3)(x – 3)
4 x (x + 3)(x – 3) x–3
18. (a) =
x –1 2 x + 3x – 5
2
= (x + 3)(x – 3)
4 x 1 + 2(x + 3) = (x + 3)(x – 3)
=
x –1 (x – 1)(2 x + 5) 1 + 2x + 6 = x2 – 9
4 x
– =0 7 + 2x = x2 – 9
x –1 (x – 1)(2 x + 5)
x2 – 2x – 16 = 0
4 x Comparing x – 2x – 16 = 0 with ax2 + bx + c = 0, we have
2
– (x – 1)(2x + 5) = 0 (x – 1)(2x + 5)
x – 1 (x – 1)(2 x + 5) a = 1, b = –2 and c = –16.
4 x
(x – 1)(2x + 5) – (x – 1)(2x + 5) = 0 –(–2) ± (–2)2 – 4(1)(–16)
x –1 (x – 1)(2 x + 5) x=
2(1)
4(2x + 5) – x = 0
2 ± 68
8x + 20 – x = 0 =
2
7x + 20 = 0
= 5.12 (to 3 s.f.), –3.12 (to 3 s.f.)
7x = –20
x = 5.12 or x = –3.12
6
x = –2
7

64
3 x +1 Exercise 1E
(d) + 2 =1
x–3 x – 5x + 6
1. (a) y = (x + 1)(x + 3)
3 x +1
+ =1 x2 is 1, the graph opens upwards.
x–3 (x – 2)(x – 3)
When y = 0,
3 x +1
+ (x – 2)(x – 3) = 1 (x – 2)(x – 3) (x + 1)(x + 3) = 0
x – 3 (x – 2)(x – 3)
x+1 =0 or x + 3 = 0
3 x +1
(x – 2)(x – 3) + (x – 2)(x – 3) x = –1 x = –3
x–3 (x – 2)(x – 3)
The graph cuts the x-axis at (–1, 0) and (–3, 0).
= (x – 2)(x – 3)
When x = 0,
3(x – 2) + (x + 1) = (x – 2)(x – 3)
y = (1)(3) = 3
3x – 6 + x + 1 = x2 – 3x – 2x + 6
The graph cuts the y-axis at (0, 3).
4x – 5 = x2 – 5x + 6
2 y
x – 9x + 11 = 0
Comparing x2 – 9x + 11 = 0 with ax2 + bx + c = 0, we have line of symmetry
a = 1, b = –9 and c = 11.
–(–9) ± (–9)2 – 4(1)(11)
x=
2(1)
y = (x + 1)(x + 3)
9 ± 37
=
2
= 7.54 (to 3 s.f.), 1.46 (to 3 s.f.)
3
x = 7.54 or x = 1.46
19.
from Sandy Land to White City and against the wind from White x
–3 –1 0
City to Sandy Land.
Let the speed of the wind be x km/h.
450 450 1
+ =5
165 + x 165 – x 2
(b) y = (x – 2)(x + 4)
450 450
+ (165 + x)(165 – x) x2 is 1, the graph opens upwards.
165 + x 165 – x
When y = 0.
11 (x – 2)(x + 4) = 0
= (165 + x)(165 – x)
2 x–2 =0 or x+4=0
450 450 x =2 x = –4
(165 + x)(165 – x) + (165 + x)(165 – x)
165 + x 165 – x The graph cuts the x-axis at (2, 0) and (– 4, 0).
11 When x = 0,
= (165 + x)(165 – x)
2 y = (–2)(4) = –8
11 The graph cuts the y-axis at (0, –8).
450(165 – x) + 450(165 + x) = (165 + x)(165 – x)
2
y
11
74 250 – 450x + 74 250 + 450x = (27 225 – x2)
2
148 500 = 149 737.5 – 5.5x2
line of symmetry
5.5x2 = 1237.5 y = (x – 2)(x + 4)
x2 = 225
x = ± 225
= ±15
Since the speed cannot be a negative value, x = 15.
The speed of the wind is 15 km/h.
x
–4 0 2

–8

65
(c) y = –(x + 1)(x – 5) (e) y = (3 – x)(x + 2)
x2 is –1, the graph opens downwards. x2 is –1, the graph opens downwards.
When y = 0, When y = 0,
–(x + 1)(x – 5) = 0 (3 – x)(x + 2) = 0
x+1 =0 or x–5 =0 3–x =0 or x+2 =0
x = –1 x =5 x =3 x = –2
The graph cuts the x-axis at (–1, 0) and (5, 0). The graph cuts the x-axis at (3, 0) and (–2, 0).
When x = 0, When x = 0,
y = –(1)(–5) = 5 y = (3)(2) = 6
The graph cuts the y-axis at (0, 5). The graph cuts the y-axis at (0, 6).
y y

line of symmetry line of symmetry


6
y = –(x + 1)(x – 5)

5 y = (3 – x)(x + 2)

x x
–1 5 –2 3

(d) y = –(x – 1)(x + 6)


x2 is –1, the graph opens downwards.
When y = 0,
(f) y = (2 – x)(4 – x)
–(x – 1)(x + 6) = 0
x2 is 1, the graph opens upwards.
x–1 =0 or x+6 =0
When y = 0,
x =1 x = –6
(2 – x)(4 – x) = 0
The graph cuts the x-axis at (1, 0) annd (–6, 0).
2–x =0 or 4–x =0
When x = 0,
x =2 x =4
y = –(–1)(6) = 6
The graph cuts the x-axis at (2, 0) and (4, 0).
The graph cuts the y-axis at (0, 6).
When x = 0,
y
y = (2)(4) = 8
line of symmetry The graph cuts the y-axis at (0, 8).
y

6
y = –(x – 1)(x + 6) y = (2 – x)(4 – x)

x 8
–6 1

x
2 4

66
2. (a) y = x2 + 2 When x = 0,
x2 is 1, the graph opens upwards. y = (0 – 3)2 + 1 = 10
When y = 0, The graph cuts the y-axis at (0, 10).
x2 + 2 = 0 The coordinates of the minimum point are (3, 1).
x2 = –2 The equation of the line of symmetry is x = 3.
The graph does not cut the x-axis. y x=3
When x = 0,
y = 02 + 2 = 2
The graph cuts the y-axis at (0, 2).
The equation of the line of symmetry is x = 0.
y
y = (x – 3)2 + 1
x=0 10

y = x2 + 2
(3, 1)
x
0

(d) y = (x + 1)2 – 3
2 x2 is 1, the graph opens upwards.
(0, 2)
The coordinates of the minimum point are (–1, –3).
0 The equation of the line of symmetry is x = –1.
When y = 0,
(b) y = –x2 – 6 (x + 1)2 – 3 = 0
x2 is –1, the graph opens downwards. (x + 1)2 = 3
When y = 0, x+1 =± 3
–x2 – 6 = 0
x+1 = 3 or x+1 =– 3
x2 = –6
The graph does not cut the x-axis. x = 3 –1 x =– 3 –1
When x = 0, = 0.732 (to 3 s.f.) = –2.73 (to 3 s.f.)
y = –(0)2 – 6 = –6 The graph cut the x-axis at (0.732, 0) and (–2.73, 0).
The graph cuts the y-axis at (0, –6). When x = 0,
The coordinates of the maximum point are (0, –6). y = (0 + 1)2 – 3 = –2
The equation of the line of symmetry is x = 0. The graph cuts the y-axis at (0, –2).

y y

x=0 x = –1
y = (x + 1)2 – 3
x
0
–6 (0, –6)

y = –x2 – 6

x
–2.73 0 0.732
–2
(–1, –3)

(c) y = (x – 3)2 + 1 (e) y = –(x + 2)2 + 3


2
x is 1, the graph opens upwards. x2 is –1, the graph opens downwards.
When y = 0, The coordinates of the minimum point are (–2, 3).
(x – 3)2 + 1 = 0 The equation of the line of symmetry is x = –2.
(x – 3)2 = –1
The graph does not cut the x-axis.
67
When y = 0, 3 3
3. (i) x2 + x=x x+
–(x + 2)2 + 3 = 0 4 4
–(x + 2)2 = –3 3
(ii) y = x2 + x
(x + 2)2 = 3 4
3
x+2 =± 3 =x x+
4
x+2 = 3 or x+2=– 3
x2 is 1, the graph opens upwards.
x = 3 –2 x=– 3 –2
When y = 0,
= –0.268 (to 3 s.f.) = –3.73 (to 3 s.f.)
3
The graph cut the x-axis at (–0.268, 0) and (–3.73, 0). x x+ =0
4
When x = 0, 3
y = –(0 + 2)2 + 3 = –1 x = 0 or x = –
4
The graph cuts the y-axis at (0, –1). 3
The graph cuts the x-axis at (0, 0) and – , 0 .
y 4
When x = 0,
x = –2
3
y=0 0+ =0
(–2, 3) 4
The graph cuts the y-axis at (0, 0).
x
–3.73 –0.268 0 y

line of symmetry 3
–1 y = x2 + x
4
y = –(x + 2)2 + 3

(f) y = –(x – 4)2 – 1


x
x2 is –1, the graph opens downwards. 3 0

The coordinates of the minimum point are (4, –1). 4

The equation of the line of symmetry is x = 4. 4. y = –(x2 – x)


When y = 0, x2 is –1, the graph opens downwards.
–(x – 4)2 – 1 = 0 When y = 0,
(x – 4)2 = –1 –(x2 – x) = 0
The graph does not cut the x-axis. –x(x – 1) = 0
When x = 0, x = 0 or x = 1
y = –(0 – 4)2 – 1 = –17 The graph cuts the x-axis at (0, 0) and (1, 0).
The graph cuts the y-axis at (0, –17). When x = 0,
y y = –(02 – 0) = 0
x=4
The graph cuts the y-axis at (0, 0).
x
0 (4, –1) y

line of symmetry
x
0 1
y = –(x – 4)2 – 1
y = –(x2 – x)
–17

68
5. (i) x2 + x – 6 = (x + 3)(x – 2) (ii) y = x2 – 8x + 5
(ii) y = x2 + x – 6 = (x – 4)2 – 11
= (x + 3)(x – 2) x2 is 1, the graph opens upwards.
x2 is 1, the graph opens upwards. When y = 0,
When y = 0, (x – 4)2 – 11 = 0
(x + 3)(x – 2) = 0 (x – 4)2 = 11
x = –3 or x = 2 x – 4 = ± 11
The graph cuts the x-axis at (–3, 0) and (2, 0).
x – 4 = 11 or x – 4 = – 11
When x = 0,
y = (0 + 3)(0 – 2) = –6 x = 11 + 4 x = – 11 + 4
The graph cuts the y-axis at (0, –6). = 7.32 (to 3 s.f.) = 0.683 (to 3 s.f.)
The graph cuts the x-axis at (7.32, 0) and (0.683, 0).
y
When x = 0,
line of symmetry y = (0 – 4)2 – 11 = 5
y = x2 + x – 6
The graph cuts the y-axis at (0, 5).
y

y = x2 – 8x + 5

x
–3 2

–6 5

x
0 0.683 7.32
6. y = x2 – 4x + 3
= (x – 1)(x – 3)
(4, –11)
x2 is 1, the graph opens upwards.
When y = 0,
(x – 1)(x – 3) = 0 (iii) The coordinates of the minimum point are (4, –11).
x–1=0 or x–3 =0 (iv) The equation of the line of symmetry is x = 4.
x=1 x =3 8. y = x2 + 3x + 1
2 2
The graph cuts the x-axis at (1, 0) and (3, 0). 3 3
= x + 3x +
2
– +1
When x = 0, 2 2
2
y = (–1)(–3) = 3 3 5
= x+ –
The graph cuts the y-axis at (0, 3). 2 4
y x2 is 1, the graph opens upwards.
line of symmetry
When y = 0,
2 2
y = x – 4x + 3 3 5
x– – =0
2 4
2
3 5
x– =
2 4
3 5
x– =±
2 4
3 5 3 5
x– = or x– =–
3 2 4 2 4
5 3 5 3
x + x = x =– +
0 1 3 4 2 4 2
= 2.62 (to 3 s.f.) = 0.382 (to 3 s.f.)
The graph cuts the x-axis at (2.62, 0) and (0.382, 0).
2 2
8 8
7. (i) x2 – 8x + 5 = x – 8 x + –
2
– – +5
2 2
= (x – 4)2 – 11

69
When x = 0, y
2
3 5
y= 0– – =1 x=–1
2 4 2
The graph cuts the y-axis at (0, 1).
1 1
The coordinates of the minimum point are –1 , –1 .
2 4
1 2
The equation of the line of symmetry is x = –1 . 1 3
2 y= x+ +
2 4
y
x = –1 1
2
1
– 1, 3
2 4
x
0

y = x2 + 3x + 1

10. –x2 + 10x – 4 = –(x2 – 10x + 4)


2 2
10 10
0 =– x 2 – 10 x + – – – +4
x 2 2
2.62 0.382
= –(x – 5)2 + 21
–1 1 , –1 1 = –(x – p)2 + q where p = 5 and q = 21
2 4 2
y = –(x – 5) + 21
1 3 x2 is –1, the graph opens downwards.
9. (i) Since the minimum point is – , ,
2
2 4 The coordinates of the maximum point are (5, 21).
1 3 When y = 0,
y= x– – +
2 4 –(x – 5)2 + 21 = 0
= (x – h)2 + k (x – 5)2 = 21
1 3
h=– ,k= x – 5 = ± 21
2 4
2 x–5 = 21 or x – 5 = – 21
1 3
(ii) y = x+ +
2 4 x = 21 + 5 x = – 21 + 5
x2 is 1, the graph opens upwards. = 9.58 (to 3 s.f.) = 0.417 (to 3 s.f.)
1 3 The graph cuts the x-axis at (9.58, 0) and (0.417, 0).
The coordinates of the minimum point are – , . When x = 0,
2 4
When y = 0, y = –(0 – 5)2 + 21 = – 4
2 The graph cuts the y-axis at (0, – 4).
1 3
x+ + =0 The equation of the line of symmetry is x = 5.
2 4
2 y
1 3
x+ =–
2 4 x=5
(5, 21)
The graph does not cuts the x-axis.
When x = 0,
2 y = –x2 + 10x – 4
1 3
y= 0+ + =1
2 4
The graph cuts the y-axis at (0, 1). x
0 0.417 9.58

–4

70
Review Exercise 1 2. (a) 2x2 + 6x + 1 = 0
Comparing 2x2 + 6x + 1 = 0 with ax2 + bx + c = 0, we have
1. (a) x2 + 8x + 5 =0
a = 2, b = 6 and c = 1.
x2 + 8x = –5
8
2
8
2 –6 ± 6 2 – 4(2)(1)
x2 + 8x + = –5 + x =
2 2 2(2)
x2 + 8x + 42 = –5 + 42 –6 ± 28
=
(x + 4)2 = 11 4
x + 4 = ± 11 = –0.177 (to 3 s.f.), –2.82 (to 3 s.f.)
x = –0.177 or x = –2.82
x + 4 = 11 or x + 4 = – 11
(b) 3x2 – 7x – 2 = 0
x = 11 – 4 x = – 11 – 4
Comparing 3x2 – 7x – 2 = 0 with ax2 + bx + c = 0, we have
= –0.683 (to 3 s.f.) = –7.32 (to 3 s.f.)
a = 3, b = –7 and c = –2.
x = –0.683 or x = –7.32
(b) x2 + 7x – 3 = 0 –(–7) ± (–7)2 – 4(3)(–2)
x =
x2 + 7x = 3 2(3)
2 2 7 ± 73
7 7 =
x2 + 7x + =3+ 6
2 2
2
= 2.59 (to 3 s.f.), –0.257 (to 3 s.f.)
7 61 x = 2.59 or x = –0.257
x+ =
2 4 (c) – 4x2 + x + 5 = 0
7 61 Comparing – 4x2 + x + 5 = 0 with ax2 + bx + c = 0, we have
x+ =±
2 4 a = – 4, b = 1 and c = 5.
7 61 7 61 –1 ± 12 – 4(– 4)(5)
x+ = or x+ =–
2 4 2 4 x = 2(– 4)
61 7 61 7
x = – x =– – –1 ± 81
4 2 4 2 =
–8
= 0.405 (to 3 s.f.) = –7.41 (to 3 s.f.) 1
x = 0.405 or x = –7.41 = –1 or 1
4
(c) x2 – 11x – 7 = 0 1
x = –1 or x = 1
x2 – 11x = 7 4
2 2
11 11 (d) 3x2 = 5x + 1
x2 – 11x + – =7+ –
2 2 3x2 – 5x – 1 = 0
11
2
149 Comparing 3x2 – 5x – 1 = 0 with ax2 + bx + c = 0, we have
x– =
2 4 a = 3, b = –5 and c = –1.

11 149 –(–5) ± (–5)2 – 4(3)(–1)


x– =± x =
2 4 2(3)

11 149 11 149 5 ± 37
x– = or x– =– =
2 4 2 4 6
= 1.85 (to 3 s.f.), –0.180 (to 3 s.f.)
149 11 149 11
x = + x =– + x = 1.85 or x = –0.180
4 2 4 2
4
= 11.6 (to 3 s.f.) = –0.603 3. (a) (x – 3)2 =
25
(to 3 s.f.)
4
x = 11.6 or x = –0.603 x–3 =±
25
(d) x2 + 1.2x = 1
2 2
2
1.2 1.2 x–3 =±
2
x + 1.2x + =1+ 5
2 2 2 2
x–3 = or x–3=–
(x + 0.6)2 = 1.36 5 5
x + 0.6 = ± 1.36 2 2
x = +3 x=– +3
5 5
x + 0.6 = 1.36 or x + 0.6 = – 1.36
2 3
=3 =2
x = 1.36 – 0.6 x = – 1.36 – 0.6 5 5
= 0.566 (to 3 s.f.) = –1.77 (to 3 s.f.) 2 3
x = 3 or x = 2
x = 0.566 or x = –1.77 5 5

71
(b) (4 – x)2 = 12
1 2
(b) x+ x+ =0
4 – x = ± 12 2 3
4 – x = 12 or 4 – x = – 12 2 1 1
x2 + x+ + =0
x = 4 – 12 x = 4 + 12 3 2 3
= 0.536 (to 3 s.f.) = 7.46 (to 3 s.f.) 7 1
x+ x+ =0
x = 0.536 or x = 7.46 6 3
(c) (x – 1)(x + 3) = 9 6x2 + 7x + 2 = 0, where a = 6, b = 7 and c = 2.
x2 + 3x – x – 3 = 9 5
6. (a) x–1 =
x2 + 2x – 12 = 0 x+7
Comparing x2 + 2x – 12 = 0 with ax2 + bx + c = 0, we have 5
(x – 1) (x + 7) = (x + 7)
a = 1, b = 2 and c = –12. x+7
(x – 1)(x + 7) = 5
–2 ± 2 2 – 4(1)(–12)
x = x2 + 7x – x – 7 = 5
2(1) x2 + 6x – 12 = 0
–2 ± 52 Comparing x2 + 6x – 12 = 0 with ax2 + bx + c = 0, we have
=
2 a = 1, b = 6 and c = –12.
= 2.61 (to 3 s.f.), – 4.61 (to 3 s.f.)
–6 ± 6 2 – 4(1)(–12)
x = 2.61 or x = –4.61 x =
2(1)
(d) x(x + 4) = 17
x2 + 4x = 17 –6 ± 84
=
2
x + 4x – 17 = 0 2
Comparing x2 + 4x – 17 = 0 with ax2 + bx + c = 0, we have = 1.58 (to 3 s.f.), –7.58 (to 3 s.f.)
a = 1, b = 4 and c = –17. x = 1.58 or x = –7.58
x –1 2x
– 4 ± 4 2 – 4(1)(–17) (b) =
x = x+4 x–3
2(1) x –1 2x
– =0
– 4 ± 84 x+4 x–3
=
2 x –1 2x
– (x + 4)(x – 3) = 0 (x + 4)(x – 3)
= 2.58 (to 3 s.f.), –6.58 (to 3 s.f.) x+4 x–3
x = 2.58 or x = –6.58 x –1 2x
(x + 4)(x – 3) – (x + 4)(x – 3) = 0 (x + 4)(x – 3)
4. (i) 2x2 – 7x + 4 = 0 x+4 x–3
Comparing 2x2 – 7x + 4 = 0 with ax2 + bx + c = 0, we have (x – 1)(x – 3) – 2x(x + 4) = 0
a = 2, b = –7 and c = 4. x2 – 3x – x + 3 – 2x2 – 8x = 0
–x2 – 12x + 3 = 0
–(–7) ± (–7)2 – 4(2)(4)
x = Comparing –x – 12x + 3 = 0 with ax2 + bx + c = 0, we have
2

2(2) a = –1, b = –12 and c = 3.


7 ± 17
= –(–12) ± (–12)2 – 4(–1)(3)
4 x =
2(–1)
= 2.78 (to 2 d.p.), 0.72 (to 2 d.p)
x = 2.78 or x = 0.72 12 ± 156
=
(ii) 2(y – 1)2 – 7(y – 1) + 4 = 0 –2
Let (y – 1) be x. = –12.2 (to 3 s.f.), 0.245 (to 3 s.f.)
2x2 – 7x + 4 = 0 x = –12.2 or x = 0.245
Since x = 2.78 (to 2 d.p.), 0,72 (to 2 d.p.),
y – 1 = 2.78 or y – 1 = 0.72
y = 3.78 y = 1.72
6
5. (a) (x – 2) x – =0
7
6 12
x2 – x – 2x + =0
7 7
20 12
x2 – x+ =0
7 7
7x2 – 20x + 12 = 0, where a = 7, b = –20 and c = 12.

72
1 x 1 3
(c) – 5x = 5 (f) + =
x x +1 5 x–2
1 3 x 1
– 5x x=5 x – =
x x–2 x +1 5
1 – 5x2 = 5x 3 x 1
2 – (x – 2)(x + 1) = (x – 2)(x + 1)
5x + 5x – 1 = 0 x – 2 x +1 5
Comparing 5x2 + 5x – 1 = 0 with ax2 + bx + c = 0, we have 3 x 1
a = 5, b = 5 and c = –1. (x – 2)(x + 1) – (x – 2)(x + 1) = (x – 2)(x + 1)
x–2 x +1 5
–5 ± 5 2 – 4(5)(–1) 1
x = 3(x + 1) – x(x – 2) = (x – 2)(x + 1)
2(5) 5
1 2
–5 ± 45 3x + 3 – x2 + 2x = (x + x – 2x – 2)
= 5
10 1 2
= 0.171 (to 3 s.f.), –1.17 (to 3 s.f.) 5x + 3 – x2 = (x – x – 2)
5
x = 0.171 or x = –1.17 1 2 1 2
5 x 5x + 3 – x2 = x – x–
(d) =3– 5 5 5
x x–3 6 2 26 17
5 x x – x– =0
+ =3 5 5 5
x x–3 6x2 – 26x – 17 = 0
5 x Comparing 6x – 26x – 17 = 0 with ax2 + bx + c = 0, we have
2
+ x(x – 3) = 3 x(x – 3)
x x–3 a = 6, b = –26 and c = –17.
5 x
x(x – 3) + x(x – 3) = 3x(x – 3) –(–26) ± (–26)2 – 4(6)(–17)
x x–3 x =
5(x – 3) + x(x) = 3x(x – 3) 2(6)
5x – 15 + x2 = 3x2 – 9x 26 ± 1084
2
=
2x – 14x + 15 = 0 12
Comparing 2x2 – 14x + 15 = 0 with ax2 + bx + c = 0, we have = 4.91 (to 3 s.f.), –0.577 (to 3 s.f.)
a = 2, b = –14 and c = 15. x = 4.91 or x = –0.577
5 3 2
–(–14) ± (–14)2 – 4(2)(15) (g) – 2 =
x = x–2 x –4 7
2(2)
5 3 2
– =
14 ± 76 x–2 (x – 2)(x + 2) 7
=
4 5 3 2
= 5.68 (to 3 s.f.), 1.32 (to 3 s.f.) – (x – 2)(x + 2) = (x – 2)(x + 2)
x – 2 (x – 2)(x + 2) 7
x = 5.68 or x = 1.32
5 3
2 1 (x – 2)(x + 2) – (x – 2)(x + 2)
(e) + =5 x–2 (x – 2)(x + 2)
x +1 x–3
2
2 1 = (x – 2)(x + 2)
+ (x + 1)(x – 3) = 5 (x + 1)(x – 3) 7
x +1 x – 3 2
5(x + 2) – 3 = (x2 – 4)
2 1 7
(x + 1)(x – 3) + (x + 1)(x – 3) = 5(x + 1)(x – 3)
x +1 x–3 2 2 8
5x + 10 – 3 = x –
2(x – 3) + (x + 1) = 5(x + 1)(x – 3) 7 7
2x – 6 + x + 1 = 5(x2 – 3x + x – 3) 2 2 8
5x + 7 = x –
3x – 5 = 5(x2 – 2x – 3) 7 7
3x – 5 = 5x2 – 10x – 15 2 2 57
x – 5x – =0
2
5x – 13x – 10 = 0 7 7
2
2x – 35x – 57 = 0
Comparing 5x – 13x – 10 = 0 with ax2 + bx + c = 0, we have
2

(2x + 3)(x – 19) = 0


a = 5, b = –13 and c = –10.
2x + 3 = 0 or x – 19 = 0
–(–13) ± (–13)2 – 4(5)(–10) 2x = –3 x = 19
x =
2(5) 1
x = –1
13 ± 369 2
=
10 1
x = –1 or x = 19
= 3.22 (to 3 s.f.), –0.621 (to 3 s.f.) 2
x = 3.22 or x = –0.621

73
1 x+3 8. (i) y = –x2 + 5x – 4
(h) + =2
2x + 1 2 x2 – 5 x – 3 y = –(x2 – 5x + 4)
1 x+3 5
+ =2 x is –5. Half of this is – .
2x + 1 (2 x + 1)(x – 3) 2
1 x+3 y = –(x2 – 5x + 4)
+ (2x + 1)(x – 3)
2 x + 1 (2 x + 1)(x – 3) 2 2
5 5
=2
(2x + 1)(x – 3)
=– x2 – 5 x + – – – +4
2 2
1 x+3
(2x + 1)(x – 3) + (2x + 1)(x – 3)
2x + 1 (2 x + 1)(x – 3) 5
2
25
=– x– – +4
= 2(2x + 1)(x – 3) 2 4
(x – 3) + (x + 3) = 2(2x + 1)(x – 3) 2
5 9
x – 3 + x + 3 = 2(2x2 – 6x + x – 3) =– x– –
2 4
2x = 2(2x2 – 5x – 3)
2
2x = 4x2 – 10x – 6 5 9
=– x– +
4x2 – 12x – 6 = 0 2 4
2x2 – 6x – 3 = 0 (ii) x2 is –1, the graph opens downwards.
Comparing 2x – 6x – 3 = 0 with ax2 + bx + c = 0, we have
2
When y = 0,
2
a = –2, b = –6 and c = –3. 5 9
– x– + =0
–(–6) ± (–6) – 4(2)(–3)
2 2 4
x = 2
2(2) 9 5
x– =
4 2
6 ± 60
= 5 3 5 3
4 x– = or x– =–
2 2 2 2
= 3.44 (to 3 s.f.), –0.436 (to 3 s.f.)
3 5 3 5
x = 3.44 or x = –0.436 x= + x =– +
2 2 2 2
7. (i) y = x2 – 7x + 12
=4 =1
y = (x – 3)(x – 4)
The graph cuts the x-axis at (4, 0) and (1, 0).
(ii) y = (x – 3)(x – 4)
When y = 0,
x2 is 1, the graph opens upwards. 2
5 9
When y = 0, y=– 0– + = –4
2 4
(x – 3)(x – 4) = 0
x–3 =0 or x–4=0 The graph cuts the y-axis at (0, – 4).
1 1
x =3 x=4 The coordinates of the maximum point are 2 , 2 .
2 4
The graph cuts the x-axis at (3, 0) and (4, 0).
1
When x = 0, The equation of the line of symmetry is x = 2
2
y = (–3)(– 4) = 12
y
The graph cuts the y-axis at (0, 12).
y 21, 21
2 4
21
4
12 y = x2 – 7x + 12

x
0 1 4

–4
x
0 3 4 x = 21
2

74
9. Let the smaller positive integer be x, then the bigger positive integer –(–5) ± (–5)2 – 4(1)(–16 250)
is x + 1. x=
2(1)
1 1 1
– = 5 ± 65 025
x x +1 12 =
2
1 1 1
– x(x + 1) = x(x + 1) = 130 or –125 (rejected since x > 0)
x x +1 12 In November 2013,
1 1 1
x(x + 1) – x(x + 1) = x(x + 1) A$100 = S$130
x x +1 12
130
1 A$1250 = S$ 1250
(x + 1) – x = x(x + 1) 100
12
= S$1625
1 2 1
x+1–x = x + x 40
12 12 11. (i) Time taken by Farhan = h
x
1 2 1
1= x + x 40
12 12 (ii) Time taken by Khairul = h
x – 30
1 2 1
x + x–1 =0 40 40 10
12 12 (iii) – =
x2 + x – 12 =0 x – 30 x 60
(x – 3)(x + 4) =0 40 40 1
– =
x–3 = 0 or x + 4 = 0 x – 30 x 6
40 40 1
x =3 x = –4 (rejected since – x(x – 30) = x(x – 30)
x – 30 x 6
x is a positive
40 40 1
integer) x(x – 30) – x(x – 30) = x(x – 30)
x – 30 x 6
x+1=3+1=4
1
The two numbers are 3 and 4. 40x – 40(x – 30) = x(x – 30)
6
10. (i) In November 2013, 1
S$x = A$100 40x – 40x + 1200 = x2 – 5x
6
100 1
S$650 = A$ 650 1200 = x2 – 5x
x 6
65 000 1 2
= A$ x – 5x – 1200 = 0
x 6
In December 2013, x2 – 30x – 7200 = 0 (Shown)
2
S$(x – 5) = A$100 (iv) x – 30 – 7200 = 0
100 Comparing x2 – 30x – 7200 = 0 with ax2 + bx + c = 0, we have
S$650 = A$ 650
x–5 a = 1, b = –30 and c = –7200.
65 000
= A$ –(–30) ± (–30)2 – 4(1)(–7200)
x–5 x =
2(1)
65 000 65 000
– = 20 30 ± 29 700
x–5 x =
65 000 2
65 000
(ii) – = 20 = 101.17 (to 2 d.p.) or –71.17 (to 2 d.p.)
x–5 x
6500 6500 (v) Since x > 0, x = 101.17
– =2 40
x–5 x Time taken by Khairul =
6500 6500 101.17 – 30
x(x – 5) – x(x – 5) = 2 x(x – 5) = 0.562 h
x–5 x
6500x – 6500(x – 5) = 2x(x – 5) = 33.7 minutes (to 3 s.f.)
6500x – 6500x + 32 500 = 2x2 – 10x 12. (i) In November 2013,
2x2 – 10x – 32 500 = 0 6000
No. of litres of petrol that could be bought =
x2 – 5x – 16 250 = 0 x
Comparing x – 5x – 16 250 = 0 with ax2 + bx + c = 0, we have
2 (ii) In December 2013,
a = 1, b = –5 and c = –16 250. 6000
No. of litres of petrol that could be bought =
x + 10

75
6000 6000 3 14. (a) (i) 60 + 25x – x2 = 0
(iii) – =1
x x + 10 7 –x2 + 25x + 60 = 0
6000 6000 10 Comparing –x2 + 25x + 60 = 0 with ax2 + bx + c = 0, we have
– =
x x + 10 7 a = –1, b = 25 and c = 60.
6000 6000 10
– x(x + 10) = x(x + 10) –25 ± 25 2 – 4(–1)(60)
x x + 10 7 x=
2(–1)
6000 6000 10
x(x + 10) – x(x + 10) = x(x + 10) –25 ± 865
x x + 10 7 =
10 –2
6000(x + 10) – 6000x = x(x + 10)
7 = –2.2 (to 1 d.p.) or 27.2 (to 1 d.p.)
10 2 100 (ii) The positive solution in (a)(i) represents the horizontal
6000x + 60 000 – 6000x = x + x
7 7 distance of the stone from the foot of the tower.
10 2 100 (b)
60 000 = x + x y Scale:
7 7 x-axis: 2 cm represent 2 m
10 2 100 220 y-axis: 2 cm represent 20 m
x + x – 60 000 = 0
7 7
10x2 + 100x – 420 000 = 0
200
x2 + 10x – 42 000 = 0 (Shown)
2
(iv) x + 10x – 42 000 = 0
Comparing x2 + 10x – 42 000 = 0 with ax2 + bx + c = 0, we have 180

a = 1, b = 10 and c = –42 000.


160
–10 ± 10 2 – 4(1)(– 42000)
x =
2(1) y = 60 + 25x – x2
140
–10 ± 168 100
=
2
= 200 or –210 120
(v) Since x > 0, x = 200
In December 2013, 100
No. of litres of petrol that could be bought with $60
6000 80
=
200 + 10
= 28.57 (to 4 s.f.)
No. of litres of petrol that could be bought with $34 60

28.57
= 34
60 40
= 16.2 (to 3 s.f.)
13. (i) x) m 20
x) m
2
(ii)
x
(35 –2x)(22 – 2x) = 400 0 5 10 15 20 25
770 – 70x – 44x + 4x2 = 400
4x2 – 114x + 770 = 400
(c) (i) From the graph,
4x2 – 114x + 370 = 0
The greatest height reached by the stone = 216 m
2x2 – 57x + 185 = 0 (Shown)
2 (ii) From the graph,
(iii) 2x – 57x + 185 = 0
When the stone is 180 m above sea level, y = 180,
Comparing 2x2 – 57x + 185 = 0 with ax2 + bx + c = 0, we have
The horizontal distance from the foot of the tower = 6.5 m
a = 2, b = –57 and c = 185.
or 18.5 m
–(–57) ± (–57)2 – 4(2)(185)
x =
2(2)
57 ± 1769
=
4
= 24.76 (to 2 d.p.) or 3.74 (to 2 d.p.)
(iv)

76
15. x minutes. Comparing x2 – 16x – 25 = 0 with ax2 + bx + c = 0, we have a = 1,
b = –16 and c = –25.
(x + 5) minutes. –(–16) ± (–16)2 – 4(1)(–25)
1 x=
of the tank while the 2(1)
x+5
1 16 ± 356
of the tank. =
x 2
1 1 1 = 17.4 (to 3 s.f.) or –1.43 (to 3 s.f.) (rejected since speed is positive)
+ = Speed of boat in still water = 17.4 km/h
x x+5 11
1
9
1 1 9 Challenge Yourself
+ =
x x+5 100
1. Let x be the digit in the tens place and (6 – x) be the digit in the ones
1 1 9
+ x(x + 5) = x(x + 5) place.
x x+5 100 1
1 1 9 x(6 – x) = (x 10 + 6 – x)
x(x + 5) + x(x + 5) = x(x + 5) 3
x x 5 100 1
2
9 6x – x = (9x + 6)
(x + 5) + x = x(x + 5) 3
100 6x – x2 = 3x + 2
9 2 9
2x + 5 = x + x x2 – 3x + 2 = 0
100 20
(x – 1)(x – 2) = 0
9 2 31
x – x–5 =0 x = 1 or x = 2
100 20
9x2 – 155x – 500 = 0 The original number is 15 or 24.
Comparing 9x – 155x – 500 = 0 with ax2 + bx + c = 0, we have
2 2. Since x = h and x = k are the roots of the equation ax2 + bx + c = 0,
a = 9, b = –155 and c = –500. ax2 + bx + c = (x – h)(x – k)
ax2 + bx + c = x2 – (h + k)x + hk
–(–155) ± (–155)2 – 4(9)(–500) b c
x= x2 + x + = x2 – (h + k)x + hk
2(9) a a
155 ± 42 025
=
18 b
= –(h + k)
7 a
= 20 or –2 (rejected since time cannot be negative)
9 b
– =h+k
a
while by the larger pipe is 25 minutes. c
= hk
16. When the boat travels upstream, it moves against the current, hence a
the speed is (x – 5) km/h. b c
h + k = – and hk =
a a
When the boat travels back (downstream), it moves with the current,
hence the speed is (x + 5) km/h.
12 12 1
+ =1
x–5 x+5 2
12 12 3
+ =
x–5 x+5 2
12 12 3
+ (x – 5)(x + 5) = (x – 5)(x + 5)
x–5 x+5 2
12 12 3
(x – 5)(x + 5) + (x – 5)(x + 5)= (x – 5)(x + 5)
x–5 x+5 2
3
12(x + 5) + 12(x – 5) = (x – 5)(x + 5)
2
3
12x + 60 + 12x – 60 = (x2 – 25)
2
3 75
24x = x2 –
2 2
3 2 75
x – 24x – =0
2 2
3x2 – 48x – 75 = 0
x2 – 16x – 25 = 0

77
Chapter 2 Further Functions
TEACHING NOTES
Suggested Approach
This is an extension of the topic of functions that students have learnt previously in Book 2. Teachers should ensure that students
are able to grasp the important concept of how to substitute a given domain into the equations. For example, for the function
f(x) = x2 + 2x – 3, a domain k would give: f(k) = k2 + 2k – 3.

Section 2.1: Functions Involving Higher Order Expressions


Teachers should guide students with learning the technique to correctly substitute variables and form equations
in order to find unknowns. As the pre-requisite for students to manipulate functions is the ability to solve
simultaneous equations, teachers could revise the methods of solving a pair of simultaneous linear equations.

Section 2.2: Inverse Functions


Students may find it confusing to identify the relation that if f(x) = y, then f–1(y) = x. Teachers should break it
down and explain that in order to find the inverse function, x needs to be expressed in terms of y, i.e. make x
the subject. Teachers could first revise the process of expressing one variable in terms of another and give some
practice questions to the students. For example, for the equation y = 3x + 2, expressing x in terms of y would
y–2
give x = .
3

78
WORKED SOLUTIONS Practise Now 3
Practise Now (Page 43) f(x) = 8x + 3
2 Let y = 8x + 3
f(3) = 6(3) +3(3) + 1
f(x) = y and f–1(y) = x
= 64
1
x = (y – 3)
8
Practise Now 1
–1 1
f (y) = (y – 3)
(i) f(3x) = 4(3x)2 – 5(3x) + 2 8
= 36x2 – 15x + 2 –1 1
f (x) = (x – 3)
(ii) f(2x + 3) = 4(2x + 3)2 – 5(2x + 3) + 2 8
= 4(4x2 + 12x + 9) – 10x – 15 + 2
= 16x2 + 48x + 36 – 10x – 13 Practise Now 4
= 16x2 + 38x + 23 f(x) = 7x – 4
(iii) f(x – 3) = 4(x2 – 3)2 – 5(x2 – 3) + 2
2
Let y = 7x – 4
= 4(x4 – 6x2 + 9) – 5x2 + 15 + 2 f(x) = y and f–1(y) = x
= 4x4 – 24x2 + 36 – 5x2 + 17 1
x = (y + 4)
= 4x4 – 29x2 + 53 7
–1 1
f (y) = (y + 4)
Practise Now 2 7
–1 1
f(3) = a(3)2 + 3b Hence f (x) = (x + 4)
7
= 9a+ 3b
–1 1
= 15 — (1) f (10) = (10 + 4)
7
f(–2) = a(–2)2 + b(–2) =2
= 4a – 2b 1
=8 f–1(– 4) = (– 4 + 4)
7
2b = 4a – 8 =0
b = 2a – 4 — (2) 1 1 1
f–1 = +4
Subst. (2) into (1) 7 7 7
9a + 3(2a – 4) = 15 29
=
9a + 6a – 12 = 15 49
15a = 27
4 Practise Now 5
a =1
5
2
4 f(x) =
From (2), b = 2 1 –4 x–5
5
2
2 Let y =
=– x–5
5
f(x) = y and f–1(y) = x
4 2
a=1 ,b=– y(x – 5) = 2
5 5
4 2 2 xy – 5y = 2
f(1) = 1 (1) – (1) 2 + 5y
5 5 x=
2 y
=1 2
5 = +5
4 2 y
f(–5) = 1 (–5)2 – (–5)
5 5 2
f–1(y) = +5
= 47 y
2
Hence f–1(x) = +5
x
f–1 x = 0.

79
2 1
f–1(6) = +5 (v) F(5) = (5)(5 + 1)
6 2
1 = 15
=5
3 1
2 F(4) = (4)(4 + 1)
f–1(–3) = +5 2
–3 = 10
1 F(5) – F(4) = 15 – 10
=4
3 =5
1 2
f–1 = +5 1
4 1 (vi) F(7) = (7)(7 + 1)
2
4
= 28
= 13 1
F(6) = (6)(6 + 1)
2
Practise Now 6 = 21
f(x) = px + q F(7) – F(6) = 28 – 21
f–1(3) = 6 f(6) = 3 =7
Thus, f(3) = 15 and f(6) = 3. 1
(vii) F(x + 1) = (x + 1)[(x + 1) + 1]
3p + q = 15 — (1) 2
1
6p + q = 3 — (2) = (x + 1)(x + 2)
2
(2) – (1): 3p = –12
1
p = –4 (viii) F(x – 1) = x(x – 1)
2
Subst. p = – 4 into (1): 3(– 4) + q = 15
1 1
q = 15 + 12 F(x) – F(x – 1) = x(x + 1) – x(x – 1)
2 2
= 27 1
= x[(x + 1) – (x – 1)]
2
Exercise 2A 1
= x(2)
2
1. g(x) = x2 + 5
=x
(i) g(a) = a2 + 5
1
(ii) g(a + 1) = (a + 1)2 + 5 (ix) F(x2) = x2(x2 + 1)
2
= a2 + 2a + 1 + 5
3. h(x) = x2 – 5x + 4
= a2 + 2a + 6
(i) h(2a) = (2a)2 – 5(2a) + 4
(iii) g(a + 1) – g(a – 1)
= 4a2 – 10a + 4
= (a2 + 2a + 6) – [(a – 1)2 + 5]
h(a) = a2 – 5a + 4
= a2 + 2a + 6 – (a2 – 2a + 1 + 5)
h(2a) – h(a)
= 4a
= (4a2 – 10a + 4) – (a2 – 5a + 4)
1
2. F(x) = x(x + 1) = 3a2 – 5a
2
(ii) a2 – 5a + 4 = 0
1
(i) F(1) = (1)(1 + 1) (a – 4)(a – 1) = 0
2
=1 a–4=0 or a–1 =0
1 a=4 a =1
(ii) F(2) = (2)(2 + 1) (iii) h(a2) = (a2)2 – 5(a2) + 4
2
=3 = a4 – 5a2 + 4
1 h(a) = a2 – 5a + 4
(iii) F(3) = (3)(3 + 1)
2 h(a2) + h(a) = (a4 – 5a2 + 4) + (a2 – 5a + 4)
=6 = a4 – 4a2 – 5a + 8
1
(iv) F(x – 1) = (x – 1)[(x – 1) + 1]
2
1
= x(x – 1)
2

80
4. g(x) = mx + c 2. f(x) = x – 7
g(1) = m + c = 5 — (1) Let y = f(x)
g(5) = 5m + c = – 4 — (2) f(x) = y and f–1(y) = x
(2) – (1): 4m = –9 y=x–7
1 x=y+7
m = –2
4 f–1(y) = y + 7
1 1 Hence f–1 : x ! x + 7
Subst. m = –2 into (1): –2 + c = 5
4 4 f–1(3) = 3 + 7
1 = 10
c =5+2
4 f–1(7) = 7 + 7
1
=7 = 14
4 –1
f (–5) = –5 + 7
1 1
m = –2 ,c=7 =2
4 4
1 1
1 1 f–1 = +7
g(3) = –2 (3) + 7 3 3
4 4
1
1 =7
= 3
2
3. g(x) = 3x + 4
1 1
g(– 4) = –2 (– 4) + 7 Let y = g(x)
4 4
1 g(x) = y and g–1(y) = x
= 16 y = 3x + 4
4
5. h(x) = px2 + qx + 2 y–4
x=
h(2) = p(2)2 + q(2) + 2 3
= 4p + 2q + 2 y–4
g–1(y) =
= 34 3
–1 x–4
2p + q = 16 Hence g : x !
3
q = 16 – 2p — (1)
–1 1
h(–3) = p(–3)2 + q(–3) + 2 g (3) = (3 – 4)
3
= 9p – 3q + 2
1
= 29 =–
3
9p – 3q = 27 — (2) 1
–1
Subst. (1) into (2): 9p – 3(16 – 2p) = 27 g (–4) = (– 4 – 4)
3
9p + 6p – 48 = 27 2
= –2
15p = 75 3
p =5 1 1 1
From (1), q = 16 – 2(5) g–1 = –4
2 3 2
=6 1
p = 5, q = 6 = –1
6
h(4) = 5(4)2 + 6(4) + 2 3 1 3
g–1 – = – –4
= 106 4 3 4
h(–2) = 5(–2)2 + 6(–2) + 2 7
= 10 = –1
12
4. h(x) = 5x + 6
Exercise 2B Let y = h(x)
1 h(x) = y and h–1(y) = x
1. f(x) = x–3
4 y = 5x + 6
Let y = f(x) y–6
x=
f(x) = y and f–1(y) = x 5
1 –1 y–6
y= x–3 h (y) =
4 5
x = 4y + 12
= 4(y + 3)
f–1(y) = 4(y + 3)
Hence f–1(x) = 4(x + 3)

81
x–6 6. g(x) = 6x – 8
Hence h–1 : x !
5 For g(x) = 10, 6x – 8 = 10
6–6 6x = 18
h–1(6) =
5 x =3
=0 For g(x) = 40, 6x – 8 = 40
10 – 6 6x = 48
h–1(10) =
5 x =8
4 For g(x) = – 4, 6x – 8 = – 4
=
5 6x = 4
2
– –6 2
2 x=
–1
h – = 5 3
5 5
32 For g(x) = –6, 6x – 8 = –6
=– 6x = 2
25
7 1
= –1 x=
25 3
1 3
12 – 6 7. f(x) = 7 – x
–1 1 2 5
h 12 =
2 5 Let y1 = f(x)
13 f(x) = y1 and f–1(y1) = x
=
10 3
y1 = 7 – x
3 5
=1
10 7 – y1
x=
5. f(x) = 8 – 3x 3
Let y = f(x) 5
f(x) = y and f–1(y) = x 5(7 – y1)
=
y = 8 – 3x 3
8 y –1 5(7 – y1)
x= f (y1) =
3 3
8 y –1 5(7 – x)
–1
f (y) = Hence f (x) =
3 3
8 x 1
–1
Hence f (x) = g(x) = x – 6
3 4
8 9 Let y2 = g(x)
–1
f (9) = g(x) = y2 and g–1(y2) = x
3
1 1
=– y2 = x – 6
3 4
8 (–12) y +6
–1
f (–12) = x = 2
3 1
2 4
=6 = 4(y2 + 6)
3
1 g–1(y2) = 4(y2 + 6)
8– 3
–1 1 2 Hence g–1(x) = 4(x + 6)
f 3 =
3 3 5(7 – 3)
(i) f–1(3) =
5 3
=1
9 2
=6
3 3
8– –
3 16 5[7 – (–17)]
f–1 – = (ii) f–1(–17) =
16 3 3
35 = 40
=2
48 (iii) g–1(5) = 4(5 + 6)
= 44
(iv) g–1(–6) = 4(–6 + 6)
=0

82
5(7 – 2) 5x
(v) f–1(2) = 10. f(x) =
3 2 – 4x
1 Let y = f(x)
=8
3 f(x) = y and f–1(y) = x
g–1(1) = 4(1 + 6) 5x
y=
= 28 2 – 4x
1 y(2 – 4x) = 5x
f–1(2) + g–1(1) = 8 + 28
3 2y – 4xy = 5x
1 5x + 4xy = 2y
= 36
3 x(5 + 4y) = 2y
–1 5(7 – 4) 2y
(vi) f (4) = x=
3 5 + 4y
=5 –1 2y
f (y) =
g–1(4) = 4(4 + 6) 5 + 4y
= 40 –1 2x
Hence f (x) =
f (4) – g–1(4) = 5 – 40 5 + 4x
–1

= –35 5
f–1(x x = 0 i.e. x = –
8. f(x) = ax2 + bx 4
f(–2) = a(–2)2 + b(–2) 2(4)
f–1(4) =
= 4a – 2b 5 + 4(4)
= 20 — (1) 8
=
21
f(4) = a(4)2 + b(4)
2(–6)
= 16a + 4b f–1(–6) =
5 + 4(–6)
= 32 — (2)
–12
(1) 2: 8a – 4b = 40 — (3) =
–19
(2) + (3): 24a = 72 12
a =3 =
19
Subst. a = 3 into (1): 4(3) – 2b = 20 3x – 1
11. f(x) =
2b = –8 x–2
b = –4 Let y = f(x)
9. h(x) = px2 + qx f(x) = y and f–1(y) = x
h(1) = p + q = 2 — (1) 3x – 1
y =
h–1(36) = 3 x–2
h(3) = p(3)2 +3q y(x – 2) = 3x – 1
= 9p + 3q xy – 2y – 3x = –1
= 36 x(y – 3) = 2y – 1
3p + q = 12 — (2) 2y – 1
x =
(2) – (1): 2p = 10 y–3
p =5 –1 2y – 1
f (y) =
Subst. p = 5 into (1) y–3
5+q=2 –1 2x – 1
Hence f (x) =
x–3
q = –3
f–1(x x – 3 = 0 i.e. x = 3.
p = 5, q = –3
2(5) – 1
h(–1) = 5(–1)2 – 3(–1) f–1(5) =
5–3
=5+3
9
=8 =
2
h(2) = 5(2)2 – 3(2) 1
= 20 – 6 =4
2
= 14 2(7) –1
f–1(7) =
7–3
13
=
4
1
=3
4

83
12. f(x) = ax + b 2
Subst. p = 1 into (1):
f(1) = a + b = 1 — (1) 3
f–1(5) = 2 2
1 + q = –5
f(2) = 2a+ b = 5 — (2) 3
(2) – (1): a = 4 2
q = –5 – 1
Subst. a = 4 into (1): 3
2
4+b=1 = –6
3
b = –3
2 2
a = 4, b = –3 p=1 , q = –6
3 3
f(x) = 4x – 3
2 2
Let f(x) be y f(x) = 1 x – 6
3 3
f(x) = y and f–1(y) = x Let f(x) be y
y = 4x – 3 f(x) = y and f–1(y) = x
y+3 2 2
x= y =1 x–6
4 3 3
y+3 2
f–1(y) = y+6
4 3
x=
x+3 1
2
Hence f–1(x) =
4 3
7 + 3 =
3
y+
2
f–1(7) =
4 5 3
1 3
=2 = y+4
2 5
1 3
–5 + 3 f–1(y) = y + 4
–1 1 2 5
f –5 =
2 4 3
5 Hence f–1(x) = x + 4
=– 5
8 15. g(x) = mx + c
13. f(x) = ax + b –1
g (–3) = 0
f(1) = a + b = 3 — (1) g(0) = c = –3
f–1(7) = 5 g–1(1) = 2
f(5) = 5a+ b = 7 — (2) g(2) = 2m + c = 1 — (1)
(2) – (1): 4a = 4 Subst. c = –3 into (1):
a =1 2m – 3 = 1
Subst. a = 1 into (1): 2m = 4
1+b=3 m =2
b =2 m = 2, c = –3
a = 1, b = 2 g(x) = 2x – 3
f(x) = x + 2 Let g(x) be y
Let f(x) be y g(x) = y and g–1(y) = x
f(x) = y and f–1(y) = x y = 2x – 3
y=x+2 y+3
x=y–2 x=
2
f–1(y) = y – 2 –1 y+3
g (y) =
Hence f–1(x) = x – 2 2
14. f(x) = px + q –1 x+3
Hence g (x) =
f(1) = p + q = –5 — (1) 2
f(–2) = –2p + q = –10 — (2) g(5) = 2(5) – 3
(1) – (2): 3p = 5 =7
2 4+3
p =1 g–1(4) =
3 2
1
=3
2

84
Review Exercise 2 3. f(x) = x2 – 3x + 5
(i) f(a) = a2 – 3a + 5
1. f(x) = 6x – 9
(ii) f(2a) = (2a)2 – 3(2a) + 5
(i) f(3) = 6(3) – 9
= 4a2 – 6a + 5
=9
(iii) f(a + 3) = (a + 3)2 – 3(a + 3) + 5
f(– 4) = 6(– 4) – 9
= a2 + 6a + 9 – 3a – 9 + 5
= –33
= a2 + 3a + 5
(ii) Let f(x) be y
(iv) f(a2) = (a2)2 – 3(a2) + 5
f(x) = y and f–1(y) = x
= a4 – 3a2 + 5
y = 6x – 9
(v) f(a + 2) = (a2 + 2)2 – 3(a2 + 2) + 5
2

y+9
x= = a4 + 4a2 + 4 – 3a2 – 6 + 5
6
y+9 = a4 + a2 + 3
–1
f (y) = 4. f(x) = 10x – 7
6
–1 x+9 g(x) = 3 – x
1
Hence f : x !
6 4
–1 3+ 9 (i) (a) f(2) = 10(2) – 7
(iii) f (3) =
6 = 13
=2 1
g(4) = 3 – (4)
–8 + 9 4
f–1(–8) =
6 =2
1 f(2) + g(4) = 13 + 2
=
6 = 15
2. (i) f(x) = 2px + 3q (b) f(–1) = 10(–1) – 7
f(2) = 4p + 3q = 17 — (1) = –17
f–1(5) = –1 1
f(–1) = –2p + 3q = 5 — (2) g(8) = 3 – (8)
4
(1) – (2): 6p = 12 =1
p =2 f(–1) – g(8) = –17 – 1
Subst. p = 2 into (1): = –18
4(2) + 3q = 17 (c) 3f(3) = 3[10(3) – 7]
3q = 17 – 8 = 3(23)
=9 = 69
q =3 1
2g(3) = 2 3 – (3)
p = 2, q = 3 4
f(x) = 4x + 9 1
=2 2
f(4) = 4(4) + 9 4
= 25 1
=4
f(–5) = 4(–5) + 9 2
= –11 1
3f(3) – 2g(3) = 69 – 4
(ii) Let f(x) be y 2
f(x) = y and f–1(y) = x 1
= 64
y = 4x + 9 2
y–9 (ii) For f(x) = 2g(x),
x= 1
4 10x – 7 = 2 3 – x
–1 y–9 4
f (y) = 1
4 10x – 7 = 6 – x
–1 x–9 2
Hence f (x) = 1
4 10 x = 13
–1 10 – 9 2
f (10) = 5
4 x=1
1 21
=
4
–16 – 9
f–1(–16) =
4
1
= –6
4
85
(iii) For f(2p) = g(4p), Let g(x) be y
1 g(x) = y and g–1(y) = x
10(2p) – 7 = 3 – (4p)
4 1 5
y=– x+4
20p – 7 = 3 – p 12 12
21p = 10 5
4 –y
10 x= 12
p=
21 1
(iv) Let y1 = f(x) 12
f(x) = y1 and f–1(y1) = x = 53 – 12y
y1 = 10x – 7 g–1(y) = 53 – 12y
y +7 Hence g–1(x) = 53 – 12x
x= 1 6. (i) h(x) = x2 – 7x + 6
10
–1 y +7 h(a2) = (a2)2 – 7(a2) + 6
f (y1) = 1
10 = a4 – 7a2 + 6
–1 x+7 h(a) = a2 – 7a + 6
Hence f (x) =
10 h(a2) – h(a)
Let y2 = g(x) = (a4 – 7a2 + 6) – (a2 – 7a + 6)
g(x) = y2 and g–1(y2) = x = a4 – 8a2 + 7a
1 (ii) For h(k) = 0, k2 – 7k + 6 = 0
y2 = 3 – x
4 (k – 6)(k – 1) = 0
3 – y2 k – 6 = 0 or k–1 =0
x=
1
k =6 k =1
4
k = 6 or k = 1
= 4(3 – y2)
(iii) h(2a) = (2a)2 – 7(2a) + 6
= 12 – 4y2
= 4a2 – 14a + 6
g–1(y2) =12 – 4y2
h(a + 1) = (a + 1)2 – 7(a + 1) + 6
Hence g–1(x) = 12 – 4x
= a2 + 2a + 1 – 7a – 7 + 6
x+7
(v) f–1(x) = = a2 – 5a
10
h(2a) – h(a + 1)
3 +7
f–1(3) = = (4a2 – 14a + 6) – (a2 – 5a)
10
=1 = 3a2 – 9a + 6
g–1(x) = 12 – 4x 7. h(x) = 5x2 + 2x + 1
g–1(4) = 12 – 4(4) (i) h(2x) = 5(2x)2 + 2(2x) + 1
= –4 = 5(4x2) + 4x + 1
f (3) – g–1(4) = 1 – (– 4)
–1 = 20x2 + 4x + 1
=5 (ii) h(x + 1) = 5(x + 1)2 + 2(x + 1) + 1
5. g(x) = px + q = 5(x2 + 2x + 1) + 2x + 2 + 1
g(5) = 5p + q = 4 — (1) = 5x2 + 10x + 5 + 2x + 2 + 1
–1
g (3) = 17 = 5x2 + 12x + 8
g(17) = 17p + q = 3 — (2) (iii) h(x + 1) = 5x2 + 12x + 8
(2) – (1): 12p = –1 h(x – 1) = 5(x – 1)2 + 2(x – 1) + 1
1 = 5(x2 – 2x + 1) + 2x – 2 + 1
p=– = 5x2 – 10x + 5 + 2x – 2 + 1
12
1 = 5x2 – 8x + 4
Subst. p = – into (1):
12 h(x + 1) – h(x – 1)
1 = (5x2 + 12x + 8) – (5x2 – 8x + 4)
5 – +q=4
12 = 20x + 4
5
q =4
12
1 5
p=– ,q=4
12 12
1 5
g(x) = – x+4
12 12

86
Challenge Yourself
Domain of inverse function = range of function
= {2, 6, 7}
Range of inverse function = Domain of function
= {–3, 1, 3, 5}
Since the element 2 in the domain has two images, –3 and 5, the inverse
is not a function.

87
Chapter 3 Linear Inequalities
TEACHING NOTES
Suggested Approach
Teachers can begin this chapter by linking to students’ prior knowledge of solving simple linear equations in one variable. By
replacing the equality sign with inequality signs, teachers can emphasise that the variable can take more than one value.

Since students have learnt simple inequalities in the form ax > b, ax ! b, ax < b and ax " b , where a and b are integers, teachers
can remind students that “solving an inequality” involves finding all the solutions that satisfy the inequality, which is akin to
solving a simple linear equation. To help students better understand linear inequalities and see how it can be applied in our
daily lives, teachers may get students to give some real-life examples where inequalities are involved.

Section 3.1: Inequalities


Teachers should recap with students how to solve simple linear inequalities and to represent the solution on a
number line. The use of number lines will help students to visualise and understand the meanings of <, >, " and
! (see Investigation: Inequalities). Teachers should guide students when solving linear inequalities that involve
reversing the inequality signs when multiplying or dividing the inequalities by a negative number as this may be
confusing to them. Teachers can use actual numbers to explain how the signs will change when multiplying and
dividing by a negative number. Teachers can get students to explore the relationship between the solution of an
inequality and that of the corresponding linear equation (see Thinking Time on page 57 of the textbook).

Section 3.2: Problem Solving involving Inequalities


In problem solving involving inequalities, students must work on their mathematical process of interpretation
and thinking skills. Teachers can guide students to understand terms such as ‘at most’, ‘at least’, ‘not more than’
and ‘not lesser than’ and how to form an inequality and solve it to find the answer to the problem.

Section 3.3: Solving Simultaneous Linear Inequalities


In solving simultaneous linear inequalities, teachers should guide students on how to solve two linear inequalities
separately and to consider only the common solutions of the inequalities after representing both inequalities
on a number line. The use of number lines in this section is essential to help students visualise the solving of
simultaneous linear inequalities. Teachers should highlight to students that there may not always be a solution
to the simultaneous linear inequalities (see Worked Example 8 on page 63 of the textbook).

Section 3.4: Limits of Accuracy


Teachers should highlight to students that the concept of limits of accuracy is somewhat an application of rounding
off. Students should be familiar with rounding up a number with the digit ‘5’, for instance 6.75 is often taken to
be 6.8 (correct to the nearest 1 decimal place). In theory, rounding up 6.75 to 6.8 has an error of 0.5. Rounding
it down to 6.7 would also give an error of 0.5. Teachers should reinforce that for the case of limits of accuracy,
the upper bound should be taken as the first number that would be rounded up. For instance, the upper and lower
bound for 20 cm (correct to the nearest cm) would be 20.5 cm and 19.5 cm respectively. Teachers can impress
upon students that the upper bound is not 20.4 cm, as numbers such as 20.45 and 20.488 will not be accounted
for.

Challenge Yourself
x x
Since z = , teachers can ask students to find the greatest and least possible values of to find the limits in
y y
which z must lie.

Teachers can get the students to first factorise the denominator, x2 – 14x + 49 and observe that value of the
denominator is more than zero.

88
WORKED SOLUTIONS Thinking Time (Page 57)
Investigation (Properties of Inequalities) 1. ax + b = c, where a, b and c are constants and a > 0
Step 1: Arrange the terms such that the constants are all on one side
1. Multiplication
LHS = 10 (–5) If x > y of the equation, i.e. ax = c – b.
by a negative
= –50 and d < 0, Step 2: Divide both sides by a to solve for x. The equality sign
number on both –50 < –30 Yes
RHS = 6 (–5) then remains.
sides of the
= –30 dx < dy. The steps will not change if a < 0.
inequality 10 > 6
2. ax + b > c, where a, b and c are constants and a > 0
Division by a Step 1: Arrange the terms such that the constants are all on one side
LHS = 10 ÷ (–5) If x > y
negative number of the inequality, i.e. ax > c – b.
= –2 and d < 0,
on both sides of –2 < –1.2 Yes Step 2: Divide both sides by a to solve for x. The inequality sign
RHS = 6 ÷ (–5) then
the inequality x y (>) remains.
= –1.2 < .
10 > 6 d d The steps will change if a < 0 such that the inequality sign will
Table 3.1 change to <.
3. ax + b ! c, where a, b and c are constants and a > 0
2. Yes, the conclusions drawn from Table 3.1 apply to 10 ! 6.
Step 1: Arrange the terms such that the constants are all on one side
The following conclusions hold for x ! y:
of the inequality, i.e. ax ! c – b.
x y
x ! y and d < 0, then dx " dy and " . Step 2: Divide both sides by a to solve for x. The inequality sign
d d
(!) remains.
The following conclusions hold for x < y:
The step will change if a < 0 such that the inequality sign will change
x y
x < y and d < 0, then dx > dy and > . to ".
d d
The following conclusions hold for x " y: 4. The solutions of ax + b > c and ax + b < c do not include the solution
x y of its corresponding linear equation ax + b = c.
x " y and d < 0, then dx ! dy and ! . The solutions of ax + b ! c and ax + b " c include the solution of
d d
its corresponding linear equation ax + b = c.
Investigation (Inequalities)
Performance Task (Page 63)
1. (a) (ii) 6 + 2 = 8 < 12 + 2 = 14
Postage rates for parcels to Thailand
(iii) 6 – 4 = 2 < 12 – 4 = 8
(b) If 6 < 12 and a is a real number, then 6 + a < 12 + a and Mass (m g) Postage (cents)
6 – a < 12 – a. 0 < m " 20 65
(c) If 12 > 6 and a is a real number, then 12 + a > 6 + a and
20 < m " 30 90
12 – a > 6 – a.
30 < m " 40 115
2. (a) (i) –6 < 12
(ii) –6 + 2 = – 4 < 12 + 2 = 14 40 < m " 50 140
(iii) –6 – 4 = –10 < 12 – 4 = 8 50 < m " 60 165
(b) If –6 < 12 and a is a real number, then –6 + a < 12 + a and
–6 – a < 12 – a. Postage rates for parcels to New Zealand and United Kingdom
(c) If 12 > –6 and a is a real number, then 12 + a > –6 + a and
Mass (m g) Postage (cents)
12 – a > –6 – a.
3. (a) (i) 6 > –12 0 < m " 20 110
(ii) 6 + 2 = 8 > –12 + 2 = –10 20 < m " 30 145
(iii) 6 – 4 = 2 > –12 – 4 = –16 30 < m " 40 180
(b) The addition or subtraction of a positive number does not change 40 < m " 50 215
the inequality sign.
50 < m " 60 250
4. Yes, the conclusion applies.
The information is taken from
Journal Writing (Page 56) http://www.singpost.com/send-documentparcel/postage-rates.
Other real life applications of inequalities: html#Airmail
BMI, grades, credit limits, text messaging, travel times, weight limits,

for vehicles, etc.


Teachers should note that the list is not exhaustive.

89
Practise Now 1 Practise Now 4
1. (a) –6x > –30 1. (a) 15x + 1 < 5(3 + x)
6x < 30 15x + 1 < 15 + 5x
30 15x + 1 – 5x < 15 + 5x – 5x
x<
6 10x + 1 < 15
x<5 10x + 1 – 1 < 15 – 1
10x < 14
2 3 4 5 6 14
x <
10
(b) –8x " 32
2
8x ! –32 x <1
5
32
x !– 16 y y +1
8 (b) !
3 2
x ! –4
16 y y +1
3 2 !3 2
3 2
–5 –4 –3 –2 –1 32y ! 3(y + 1)
2. –3x < –13 32y ! 3y + 3
3x > 13 32y – 3y ! 3y + 3 – 3y
13 29y ! 3
x> 3
3 y!
1 29
x >4 1 1
3 (c) (z – 4) " (z + 1) + 2
The smallest integer value of x is 5. 2 3
1 1
2 3 (z – 4) " 2 3 (z + 1) + 2
2 3
Practise Now 2
3(z – 4) " 2(z + 1) + 6(2)
(a) x–3 !7 3z – 12 " 2z + 2 + 12
x–3+3 !7+3 3z – 12 " 2z + 14
x ! 10 3z – 12 – 2z " 2z + 14 – 2z
z – 12 " 14
8 9 10 11 12
z – 12 + 12 " 14 + 12
z " 26
(b) –2y + 4 >3 3 1 1
2. (p – 2) + > (p – 1)
–2y + 4 – 4>3–4 4 2 2
–2y > –1 3 1 1
2y<1 4 ( p – 2) + >4 (p – 1)
4 2 2
2y 1 3(p – 2) + 2 > 2(p – 1)
<
2 2 3p – 6 + 2 > 2p – 2
1 3p – 4 > 2p – 2
y<
2 3p – 4 – 2p > 2p – 2 – 2p
p – 4 > –2
–2 –1 0 1 2 p–4+4 >–2+4
p >2
Practise Now 3 Smallest perfect square value of p is 4 = 22
5–x < –9
5–5–x < –9 – 5
–x < –14
x > 14

12 13 14 15 16

(i) Smallest prime value of x is 17


(ii) Smallest perfect cube value of x is 27 = 33

90
Practise Now 5 Practise Now 8
Let x 1. Solving the two linear inequalities separately,
x + 76 + 89 8x + 13 " 4x – 3 and 4x – 3 < 5x – 11
! 75
3 8x + 13 – 4x " 4x – 3 – 4x 4x – 3 – 5x < 5x – 11 – 5x
x + 76 + 89 4x + 13 " – 3 –x – 3 < – 11
3 !3 75
3 4x + 13 – 13 " –3 – 13 –x – 3 + 3 < –11 + 3
x + 76 + 89 ! 225 4x " –16 –x < –8
x + 165 ! 225 –16
x + 165 – 165 ! 225 – 165 x" x >8
4
x ! 60 x " –4
Representing x " – 4 and x > 8 on a number line,

Practise Now 6
–5 – 4 –3 –2 –1 0 1 2 3 4 5 6 7 8 9
Let x and y be the number of $10 notes and $5 notes respectively. The simultaneous linear inequalities have no solution.
x + y = 12 — (1) 2. Solving the two linear inequalities separately,
x 10 + y 5 < 95 y–2 2y + 1 2y + 1
< and "3
i.e. 10x + 5y < 95 — (2) 3 5 5
y–2 2y + 1 2y + 1
From (1), 3 5 <3 5 5 "5 3
3 5 5
y = 12 – x — (3)
5(y – 2) < 3(2y + 1) 2y + 1 " 15
Substitute (3) into (2):
5y – 10 < 6y + 3 2y + 1 – 1 " 15 – 1
10x + 5(12 – x) < 95
5y – 10 – 6y < 6y + 3 – 6y 2y " 14
10x + 60 – 5x < 95
14
5x + 60 < 95 –y – 10 < 3 y"
2
5x + 60 – 60 < 95 – 60 –y – 10 + 10 < 3 + 10 y "7
5x < 35 –y < 13
35 y > –13
x<
5
Representing y > –13 and y " 7 on a number line,
x <7
The maximum number of $10 notes that Vishal has is 6.

–14 –13 –12 –11 –10 –9 –8 –7 –6 –5 –4 –3 –2 –1 0 1 2 3 4 5 6 7 8


Practise Now 7
The solutions satisfying both inequalities lie in the overlapping
Solving the two linear inequalities separately,
shaded region, i.e. –13 < x " 7.
2x – 3 " 7 and 2x + 1 > –3x – 4
2x – 3 + 3 " 7 + 3 2x + 1 + 3x > –3x – 4 + 3x
Practise Now (Page 65)
2x " 10 5x + 1 > –4
10 (a) 375, 325
x" 5x + 1 – 1 > – 4 – 1
2 (b) 1800.5, 1799.5
x "5 5x > –5 (c) 85, 75
–5
x>
5 Practise Now 9
x > –1
Upper bound limits of 5 m = 5.5
Representing x " 5 and x > –1 on a number line,
Upper bound of perimeter = 4(5.5)
= 22 m
–2 –1 0 1 2 3 4 5 6
The solutions satisfying both inequalities lie in the overlapping shaded
region, i.e. –1 < x " 5.

91
Practise Now 10 2. (a) a+2 <3
a+2–2<3–2
Maximum length of reel = 25.5 m
a <1
Minimum length of reel = 24.5 m
Maximum length used for scarf = 6.5 m
–1 0 1 2 3
Minimum length used for scarf = 5.5 m
Maximum amount of wool remaining (b) b–3 "4
= maximum length of reel – minimum length used b–3+3"4+3
= 25.5 – 5.5 b "7
= 20 m
Minimum amount of wool remaining 5 6 7 8 9
= minimum length of reel – maximum length used
(c) –c + 3 >5
= 24.5 – 6.5
–c + 3 – 3 >5–3
= 18 m
–c >2
c < –2
Practise Now 11
Maximum value of width = 40.5 m –4 –3 –2 –1 0
Minimum value of width = 39.5 m
(d) 4–d !4
Maximum value of length = 60.5 m
4–d–4 !4–4
Minimum value of length = 59.5 m
–d !0
Maximum area = 40.5 60.5
d "0
= 2450.25 m2
Minimum area = 39.5 59.5
= 2350.25 m2 –2 –1 0 1 2
2350.25 m2 ! area < 2450.25 m2 (e) –2e – 1 ! 2
–2e – 1 + 1 ! 2 + 1
Practise Now 12 –2e ! 3
For 1 litre of water, 3
e"
Upper bound area = 20.5 m2 –2
Lower bound area = 19.5 m2 1
e " –1
For the farm, 2
Upper bound area = 60.5 m2
Lower bound area = 59.5 m2 – 4 –3 –2 –1 0
60.5 (f) 2 + 5f < 0
Maximum amount of water needed =
19.5
2 + 5f – 2 < 0 – 2
= 3.10 litres (to 3 s.f.)
5f < – 2
59.5
Minimum amount of water needed = 2
20.5 f <–
5
= 2.90 litres (to 3 s.f.)

–2 –1 20 1 2
Exercise 3A –
5
1. (a) If x > y, then –6x < –6y (g) g–7 "1–g
x y g–7+g "1–g+g
(b) If x < y, then >
–30 –30 2g – 7 " 1
(c) If x " y, then – 4x ! – 4y 2g – 7 + 7 " 1 + 7
x y 2g " 8
(d) If x ! y, then "
–10 –10 8
(e) 5 + h < 7 + h g"
2
(f) 5 – k < 7 – k g "4

2 3 4 5 6

92
(h) 5h > 4(h + 1) 3. 7 + 2x " 16
5h > 4h + 4 7 + 2x – 7 " 16 – 7
5h – 4h > 4h + 4 – 4h 2x " 9
h >4 9
x"
2
2 3 4 5 6 1
x "4
2
(i) 8j + 3 < 2(7 – j)
8j + 3 < 14 – 2j
1
8j + 3 + 2j < 14 – 2j + 2j 3 4 4 5 6
2
10j + 3 < 14 (i) Largest integer value of x is 4
10j + 3 – 3 < 14 – 3 (ii) Largest perfect square value of x is 4 = 22
10j < 11 4. 3 – 4x > 3x – 18
11 3 – 4x – 3 > 3x – 18 – 3
j<
10 – 4x > 3x – 21
1
j <1 – 4x – 3x > 3x – 21 – 3x
10
–7x > –21
–21
0 11
1
2
x <
10 –7
x <3
(j) 4k + 5 ! 2(–2k)
4k + 5 ! –4k
1 2 3 4 5
4k + 5 + 4k ! –4k + 4k
(i) Prime value of x is 2
8k + 5 ! 0
(ii) Yes, x = 0 is less than 3.
8k + 5 – 5 ! 0 – 5
5. (a) –5x < 25
8k ! –5
25
5 x>
k!– –5
8
x > –5

–1 5 0 1 2
– –6 –5 –4 –3 –2
8
(k) 2(m – 5) " 2 – m (b) –12x ! 138
2m – 10 " 2 – m 138
x"
2m – 10 + m " 2 – m + m –12
3m – 10 " 2 1
x " –11
3m – 10 + 10 " 2 + 10 2
3m " 12
12 –10 –11 1 –12
m" –11
3 2
m "4 (c) –y " –7
y !7
2 3 4 5 6
(l) 3(1 – 4n) > 8 – 7n 6 7 8 9 10

3 – 12n > 8 – 7n (d) –9y > –35


3 – 12n + 7n > 8 – 7n + 7n 9y < 35
3 – 5n >8 35
3 – 5n – 3 >8–3 y<
9
–5n >5 8
y<3
5 9
n<
–5
n <–1 2 3 84
3
9

–3 –2 –1 0

93
(e) 4(p + 1) < –3(p – 4) c+4 c +1
(c) >
4p + 4 < –3p + 12 4 3
4p + 4 + 3p < –3p + 12 + 3p c+4 c +1
4 3 >4 3
7p + 4 < 12 4 3
7p + 4 – 4 < 12 – 4 3(c + 4) > 4(c + 1)
7p <8 3c + 12 > 4c + 4
8 3c + 12 – 4c > 4c + 4 – 4c
p< –c + 12 > 4
7
1 –c + 12 – 12 > 4 – 12
p <1
7 –c > –8
c <8
1 2–d 3– d 1
0 11 2 (d) < +
7 2 4 2
(f) 6 – (1 – 2q) ! 3(5q – 2) 2–d 3– d 1
4 <4 +
6 – 1 + 2q ! 15q – 6 2 4 2
5 + 2q ! 15q – 6 2(2 – d) < (3 – d) + 2
5 + 2q – 15q ! 15q – 6 – 15q 4 – 2d < 5 – d
5 – 13q ! –6 4 – 2d + d < 5 – d + d
5 – 13q – 5 ! –6 – 5 4–d <5
–13q ! – 11 4–d–4 <5–4
–11 –d < 1
q"
–13 d > –1
11 1 2 1
q" (e) (e – 2) + < (e – 4)
13 4 3 6
1 2 1
12 (e – 2) + < 12 (e – 4)
11 1
4 3 6
0 2
13 3(e – 2) + 8 < 2(e – 4)
4a 3e – 6 + 8 < 2e – 8
6. (a) !2 3e + 2 < 2e – 8
3
4a 3e + 2 – 2e < 2e – 8 – 2e
3 !3 2
3 e + 2 < –8
4a ! 6 e + 2 – 2 < –8 – 2
6 e < –10
a!
4 f +1 3f +1 3f –1
(f) + " +2
1 2 4 4
a !1
2 f +1 3f +1 3f –1
4 + "4 +2
2b – 1 3b 2 4 4
(b) >
3 5 2( f + 1) + (3f + 1) " (3f – 1) + 8
2b – 1 3b
3 5 >3 5 2f + 2 + 3f + 1 " 3f – 1 + 8
3 5
5f + 3 " 3f + 7
5(2b – 1) > 3(3b)
5f + 3 – 3f " 3f + 7 – 3f
10b – 5 > 9b
2f + 3 – 3 "7–3
10b – 5 – 9b > 9b – 9b
2f "4
b–5 >0
4
b–5+5 >0+5 f"
2
b >5
f "2

94
1 1 1 1 3x + 2
(g) (3g + 4) – (g + 1) !1– (g + 5) 8. (i) (2x – 7) "
5 3 3 3 2
1 1 1 1 3x + 2
5 3 (3g + 4) – (g + 1) !5 3 1 – (g + 5) 3 2 (2x – 7) "3 2
5 3 3 3 2
3(3g + 4) – 5(g + 1)! 15 – 5(g + 5) " 3(3x + 2)
2(2x – 7)
9g + 12 – 5g – 5! 15 – 5g – 25 4x – 14" 9x + 6
4g + 7! –10 – 5g 4x – 14 – 9x" 9x + 6 – 9x
4g + 7 + 5g! –10 – 5g + 5g –5x – 14"6
9g + 7! –10 –5x – 14 + 14" 6 + 14
9g + 7 – 7! –10 – 7 –5x" 20
9g! –17 20
x!
–17 –5
g ! x ! –4
9
8 (ii) Smallest value of x2 is (0)2 = 0
g ! –1
9
h 3 h Exercise 3B
(h) 4 + <3 –5
3 4 2 1. Let the number of tickets Kate can buy be x.
4 12 3 10.50x " 205
h+ < h – 15
3 4 2 205
x "
4 3 3 3 10.50
h + 3 – h < h – 15 – h
3 2 2 2 11
x " 19
1 21
– h + 3 < –15
6 The number of tickets Kate can buy is 19.
1 2. Since the area of rectangle = length breadth,
– h + 3 – 3 < –15 – 3
6 x y = 24
1 Factors of 24: 1 24, 2 12, 3 8, 4 6
– h < –18
6
Hence, where x > y, the possible pairs of integer values of x and y
1
6 – h < 6 –18 are x = 24, y = 1; x = 12, y = 2; x = 8, y = 3; x = 6, y = 4.
6
3. (a) Solving the two linear inequalities separately,
–h < –108
x–4 "3 and 3x ! –6
h > 108
–6
1 p x–4+4 "3+4 x!
7. (2 – p) – 3 ! 3
6 10
x "7 x ! –2
1 p
6 10 (2 – p) – 3 ! 6 10 Representing x " 7 and x ! –2 on a number line,
6 10
10(2 – p) – 180 ! 6p
20 – 10p – 180 ! 6p
–3 –2 –1 0 1 2 3 4 5 6 7 8
–10p – 160 ! 6p
–10p – 160 – 6p ! 6p – 6p The solutions satisfying both inequalities lie in the overlapping
–16p – 160 ! 0 shaded region, i.e. –2 " x " 7.
–16p – 160 + 160 ! 0 + 160 (b) Solving the two linear inequalities separately,
–16p ! 160 2x + 5 < 15 and 3x – 2 > –6
160 2x + 5 – 5 < 15 – 5 3x – 2 + 2 > –6 + 2
p" 2x < 10 3x > – 4
–16
p " –10 –4
x <5 x>
The largest possible value of p is –10. 3
1
x > –1
3
1
Representing x < 5 and x > –1 on a number line,
3

–2 –1 1 –1 0 1 2 3 4 5
3
The solutions satisfying both inequalities lie in the
1
overlapping shaded region, i.e. –1 < x < 5.
3
95
4. (a) Solving the two linear inequalities separately, (c) Solving the two linear inequalities separately,
5x – 1 < 4 and 3x + 5 ! x + 1 3x – 3 < x – 9 and x – 9 < 2x
5x – 1 + 1 < 4 + 1 3x + 5 – x ! x + 1 – x 3x – 3 – x < x – 9 – x x – 9 – 2x < 2x – 2x
5x < 5 2x + 5 ! 1 2x – 3 < –9 –x – 9 < 0
x <1 2x + 5 – 5 ! 1 – 5 2x – 3 + 3 < –9 + 3 –x – 9 + 9 < 0 + 9
2x ! –4 2x < –6 –x < 9
x ! –2 x < –3 x > –9
Representing x < 1 and x ! –2 on a number line, Representing x < –3 and x > –9 on a number line,

–3 –2 –1 0 1 2 –10 –9 –8 –7 –6 –5 –4 –3 –2
The solutions satisfying both inequalities lie in the overlapping The solutions satisfying both inequalities lie in the overlapping
shaded region, i.e. –2 " x < 1. shaded region, i.e. –9 < x < –3.
The integer values of x which satisfy both inequalities are (d) Solving the two linear inequalities separately,
–2, –1 and 0. 2x " x + 6 and x + 6 < 3x + 5
(b) Solving the two linear inequalities separately, 2x – x " x + 6 – x x + 6 – 3x < 3x + 5 – 3x
2x – 5 ! 1 and 3x – 1 < 26 x "6 –2x + 6 < 5
2x – 5 + 5 ! 1 + 5 3x – 1 + 1 < 26 + 1 –2x + 6 – 6 < 5 – 6
2x ! 6 3x < 27 –2x < –1
x !3 x<9 –1
x>
Representing x ! 3 and x < 9 on a number line, –2
1
x>
2
1
2 3 4 5 6 7 8 9 10 Representing x " 6 and x > on a number line,
2
The solutions satisfying both inequalities lie in the overlapping
shaded region, i.e. 3 " x < 9.
The integer values of x which satisfy both inequalities are 3, 011 2 3 4 5 6 7
4, 5, 6, 7 and 8. 2
5. (a) Solving the two linear inequalities separately, The solutions satisfying both inequalities lie in the overlapping
–4 " 2x and 2x " 3x – 2 1
shaded region, i.e. < x " 6.
2
2x ! –4 2x – 3x " 3x – 2 – 3x
6. Let the number of days be x.
x ! –2 –x " –2
8x + 20 " 100
–2
x! 8x + 20 – 20 " 100 – 20
–1
x!2 8x " 80
Representing x ! –2 and x ! 2 on a number line, x " 10
The maximum number of days is 10.
7. Let the three consecutive integers be x, (x + 1) and (x + 2).
–2 –1 0 1 2 3 x + (x + 1) + (x + 2) < 75
The solutions satisfying both inequalities lie in the overlapping 3x + 3 < 75
shaded region, i.e. x ! 2. 3x + 3 – 3 < 75 – 3
(b) Solving the two linear inequalities separately, 3x < 72
1 – x < –2 and –2 " 3 – x x < 24
1 – x – 1 < –2 – 1 –2 + x " 3 – x + x Largest possible integer value of x = 23
–x < –3 –2 + x " 3 Largest possible integer = 23 + 2 = 25
–3 Cube of the largest possible integer = 253 = 15 625
x> –2 + x + 2 " 3 + 2 8. Let the number of correct answers obtained be x.
–1
x >3 x "5 5x – 2(30 – x) " 66
Representing x > 3 and x " 5 on a number line, 5x – 60 + 2x " 66
7x – 60 " 66
7x – 60 + 60 " 66 + 60
2 3 4 5 6 7x " 126
The solutions satisfying both inequalities lie in the overlapping x " 18
shaded region, i.e. 3 < x " 5. The maximum number of correct answers obtained is 18.

96
9. Let x and y be the number of $5 notes and $2 notes respectively. Representing x < 5 17 – 2 and x > 5 17 – 3 on a number line,
x + y = 50 — (1)
x 5 + y 2 > 132
i.e. 5x + 2y > 132 — (2) 17 18 19
From (1),
y = 50 – x — (3) The solutions satisfying both inequalities lie in the overlapping
Substitute (3) into (2): shaded region, i.e. 5 17 – 3 < x < 5 17 – 2.
5x + 2(50 – x) > 132 The integer value of x is 18.
5x + 100 – 2x > 132 12. Solving the two linear inequalities separately,
3x + 100 > 132 3x – 2 ! 10 and 10 ! x + 4
3x + 100 – 100 > 132 – 100 3x – 2 + 2 ! 10 + 2 10 – x ! x + 4 – x
3x > 32 3x ! 12 10 – x ! 4
32 x !4 10 – x – 10 ! 4 – 10
x>
3 –x ! –6
2 x "6
x > 10
3 Representing x ! 4 and x " 6 on a number line,
The minimum number of $5 notes that he has is 11.
10. Solving the two linear inequalities separately,
1 1 1 1 3 4 5 6 7
x–4 > x and x+1 < x+3
2 3 6 8 The solutions satisfying both inequalities lie in the overlapping
1 1 1 1 1 1 1 1
x–4– x > x– x x+1– x < x+3– x shaded region, i.e. 4 " x " 6.
2 3 3 3 6 8 8 8
The prime value of x is 5.
1 1
x–4 >0 x+1 <3 13. (a) Solving the two linear inequalities separately,
6 24
3–a "a–4 and a – 4 " 9 – 2a
1 1
x–4+4 >0+4 x+1–1 <3–1 3–a–a "a–4–a a – 4 + 2a " 9 – 2a + 2a
6 24
1 1 3 – 2a " – 4 3a – 4 " 9
x >4 x <2 3 – 2a – 3 " – 4 – 3 3a – 4 + 4 " 9 + 4
6 24
1 1 –2a " –7 3a " 13
6 x >6 4 24 x < 24 2
6 24 –7 13
a ! a"
x > 24 x < 48 –2 3
Representing x > 24 and x < 48 on a number line, 1 1
a !3 a"4
2 3
1 1
Representing a ! 3 and a " 4 on a number line,
10 20 24 30 40 48 50 2 3
The solutions satisfying both inequalities lie in the overlapping
shaded region, i.e. 24 < x < 48.
2 3 1 4 1 5
The prime values of x which satisfy both inequalities are 29, 31, 3 4
2 3
37, 41, 43 and 47.
The solutions satisfying both inequalities lie in the overlapping
11. Solving the two linear inequalities separately, 1 1
shaded region, i.e. 3 " a " 4 .
x + 2 < 5 17 and 5 17 < x + 3 2 3
x + 2 – 2 < 5 17 – 2 5 17 – x < x + 3 – x (b) Solving the two linear inequalities separately,
1–b <b–1 and b – 1 < 11 – 2b
x < 5 17 – 2 5 17 – x < 3
1–b–b <b–1–b b – 1 + 2b < 11 – 2b + 2b
5 17 – x – 5 17 < 3 – 5 17 1 – 2b < –1 3b – 1 < 11
–x < 3 – 5 17 1 – 2b – 1 < – 1 – 1 3b – 1 + 1 < 11 + 1
x > –(3 – 5 17 ) –2b < – 2 3b < 12
–2 12
x > 5 17 – 3 b> b<
–2 3
b >1 b<4

97
Representing b > 1 and b < 4 on a number line, Representing a ! 4 and a " – 4 on a number line,

0 1 2 3 4 5 – 4 –3 –2 –1 0 1 2 3 4
The solutions satisfying both inequalities lie in the overlapping The solutions satisfying both inequalities lie in the overlapping
shaded region, i.e. 1 < b < 4. shaded region, i.e. – 4 ! a ! 4.
(c) Solving the two linear inequalities separately, (b) Solving the two linear inequalities separately,
3 – c < 2c – 1 and 2c – 1 < 5 + c b b b
" +1 and +1 "b–1
3 – c – 2c < 2c – 1 – 2c 2c – 1 – c < 5 + c – c 3 2 2
3 – 3c < –1 c–1<5 b b b b b
– " +1– +1–b "b–1–b
3 – 3c – 3 < –1 – 3 c–1+1<5+1 3 2 2 2 2
–3c < – 4 c<6 b b
– "1 – + 1 " –1
–4 6 2
c> b b
–3 6 – "6 1 – + 1 – 1 " –1 – 1
1 6 2
c >1 b
3 –b " 6 – " –2
1 2
Representing c > 1 and c < 6 on a number line, b
3 b ! –6 2 – " 2 –2
2
–b " – 4
0 111 2 3 4 5 6 7 –4
b!
3 –1
The solutions satisfying both inequalities lie in the overlapping b !4
1 Representing b ! –6 and b ! 4 on a number line,
shaded region, i.e. 1 < c < 6.
3
(d) Solving the two linear inequalities separately,
3d – 5 < d + 1 and d + 1 ! 2d + 1 –6 –5 –4 –3 –2 –1 0 1 2 3 4
3d – 5 – d < d + 1 – d d + 1 – 2d ! 2d + 1 – 2d The solutions satisfying both inequalities lie in the overlapping
2d – 5 < 1 –d + 1 ! 1 shaded region, i.e. b ! –6.
2d – 5 + 5 < 1 + 5 –d + 1 – 1 ! 1 – 1 (c) Solving the two linear inequalities separately,
2d < 6 –d ! 0 c–2
d <3 d"0 2(1 – c) > c – 1 and c–1 "
7
Representing d < 3 and d " 0 on a number line, c–2
2 – 2c > c – 1 7 (c – 1) " 7
7
2 – 2c – c > c – 1 – c 7(c – 1) " c – 2
–1 0 1 2 3 4 2 – 3c > –1 7c – 7 " c – 2
The solutions satisfying both inequalities lie in the overlapping 2 – 3c – 2 > –1 – 2 7c – 7 – c " c – 2 – c
shaded region, i.e. 0 ! d < 3. –3c > –3 6c – 7 " –2
14. (a) Solving the two linear inequalities separately, –3
c< 6c – 7 + 7 " –2 + 7
a a –3
+3 !4 and 4! +6
4 2 c <1 6c " 5
a a a a 5
+3–3!4–3 4– ! +6– c"
4 2 2 2 6
a a 5
!1 4– !6 Representing c < 1 and c " on a number line,
4 2 6
a a
4 !4 1 4– –4"6–4
4 2
a 0 51 2
a !4 – !2 6
2 The solutions satisfying both inequalities lie in the overlapping
a 5
2 – !2 2 shaded region, i.e. ! c < 1.
2 6
–a ! 4
a " –4

98
(d) Solving the two linear inequalities separately, (b) Solving the two linear inequalities separately,
2d 2d d 1 3x + 2 < 19 and 19 < 5x – 4
d–5 < and ! +
5 5 2 5 3x + 2 – 2 < 19 – 2 19 – 5x < 5x – 4 – 5x
2d 2d d 1 3x < 17 19 – 5x < – 4
5 (d – 5) < 5 5 !5 +
5 5 2 5 17
x< 19 – 5x – 19 < – 4 – 19
d 1 3
5(d – 5) < 2d 2d ! 5 + 2
2 5 x <5 –5x < –23
3
5d
5d – 25 < 2d 2d ! +1 –23
2 x>
–5
5d 5d 5d
5d – 25 – 2d < 2d – 2d 2d – ! +1– 3
2 2 2 x>4
5
d
3d – 25 < 0 – !1 2 3
2 Representing x < 5 and x > 4 on a number line,
3 5
d
3d – 25 + 25 < 0 + 25 2 – !2 1
2
3d < 25 –d ! 2 4 3 5 2 6
4 5
25 5 3
d < d " –2
3 The solutions satisfying both inequalities lie in the overlapping
1 3 2
d <8 shaded region, i.e. 4 < x < 5 .
3 5 3
1 The integer value of x
Representing d < 8 and d " –2 on a number line,
3 (c) Solving the two linear inequalities separately,
– 4 ! 7 – 3x and 7 – 3x ! 2
– 4 + 3x ! 7 – 3x + 3x 7 – 3x – 7 ! 2 – 7
–2 –1 0 1 2 3 4 5 6 7 8 1
8 – 4 + 3x ! 7 –3x ! –5
3
The solutions satisfying both inequalities lie in the overlapping –5
– 4 + 3x + 4 ! 7 + 4 x"
1 –3
shaded region, i.e. –2 ! d < 8 .
3 2
3x ! 11 x "1
15. (a) Solving the two linear inequalities separately, 3
3x – 5 < 26 and 26 ! 4x – 6 11
x!
3x – 5 + 5 < 26 + 5 26 – 4x " 4x – 6 – 4x 3
3x < 31 26 – 4x ! –6 2
x !3
31 3
x< 26 – 4x – 26 ! –6 – 26 2 2
3 Representing x ! 3 and x " 1 on a number line,
1 3 3
x < 10 – 4x ! –32
3
–32
x" 1 2 2 3 24
–4 1 3
3 3
x"8
1 The solutions satisfying both inequalities lie in the overlapping
Representing x < 10 and x " 8 on a number line, 2 2
3 shaded region, i.e. 1 ! x ! 3 .
3 3
The integer values of x which satisfy both inequalities are 2
7 8 9 10 10 1 11
and 3.
3 (d) Solving the two linear inequalities separately,
The solutions satisfying both inequalities lie in the overlapping –10 < 7 – 2x and 7 – 2x ! –1
1 –10 + 2x < 7 – 2x + 2x 7 – 2x – 7 ! –1 – 7
shaded region, i.e. 8 ! x < 10 .
3 –10 + 2x < 7 –2x ! –8
The integer values of x which satisfy both inequalities are 8, –8
–10 + 2x + 10 < 7 + 10 x"
9 and 10. –2
2x < 17 x"4
17
x<
2
1
x <8
2

99
1 (iv) (a) 995, 1050 (b) 995 " x < 1050
Representing x < 8 and x ! 4 on a number line,
2 (v) (a) 0.0345, 0.0355 (b) 0.0345 " x < 0.0355
(vi) (a) 71 500, 72 500 (b) 71 5000 " x < 72 500
2. 0.6m3 = 0.6 106 = 600 000 cm3
3 4 5 6 7 8 1 9
8 Upper bound = 600 000.5
2
The solutions satisfying both inequalities lie in the overlapping Lower bound = 599 999.5
1 3. Upper bound = 0.0045 s
shaded region, i.e. 4 " x < 8 .
2 Lower bound = 0.0035 s
The integer values of x which satisfy both inequalities are 4, 0.0035 " x < 0.0045
5, 6, 7 and 8. 4. (a) 775, 725
16. Given that 0 " x " 7 and 1 " y " 5, (b) 35, 25
(a) Greatest value of x + y = 7 + 5 (c) 758.5, 757.5
= 12 (d) 1500.5, 1499.5
(b) Least value of x – y = 0 – 5 (e) 497.5, 492.5
= –5 5. Upper bounds of the lengths (m) = 6.5, 8.5, 10.5
(c) Largest value of xy = (7)(5) Lower bounds of the lengths (m) = 5.5, 7.5, 9.5
= 35 Upper bound perimeter = 6.5 + 8.5 + 10.5
x 0 = 25.5 m
(d) Smallest value of =
y 5 Lower bound perimeter = 5.5 + 7.5 + 9.5
=0 = 22.5 m
(e) Least value of x2 = 02 6. Maximum number of students = 649
=0 Maximum number of girls = 649 – 297
Greatest value of x2 = 72 = 352
= 49 7. Maximum length of original string = 50.05 cm
17. Given that – 4 " a " –1 and –6 " b " –2, Minimum length of original string = 49.95 cm
(a) Least value of a + b = – 4 + (–6) Maximum length of cut string = 25.05 cm
= –4 – 6 Minimum length of cut string = 24.95 cm
= –10 Maximum length of string left = 50.05 – 24.95
(b) Greatest value of a – b = –1 – (–6) = 25.1 cm
= –1 + 6 Minimum length of string left = 49.95 – 25.05
=5 = 24.9 cm
(c) Smallest value of ab = (–1)(–2) 24.9 " l < 25.1
=2 8. Radius of trunk = 20 cm = 200 mm
a –4 Minimum radius = 198.5 mm = 19.85 cm
(d) Largest value of =
b –2 Minimum circumference = 2 (19.85)
=2
= 125 cm (to 3 s.f.)
(e) Least value of a2 = (–1)2
9. Maximum speed = 65 km/h
=1
Minimum speed = 55 km/h
Greatest value of a2 = (– 4)2
Maximum time taken = 3.5 h
= 16
Minimum time taken = 2.5 h
(f) Largest value of b – a = (–6)2 – (– 4)
2
Maximum distance = 65 3.5
= 36 + 4
= 227.5 km
= 40
Minimum distance = 55 2.5
–1 1
18. (a) False. If a = –1 and b = –2, then = which is less than 1. = 137.5 km
–2 2
137.5 " d < 227.5
(b) True
10. Maximum mass = 35 g
(c) True
Minimum mass = 25 g
35
Exercise 3C Maximum volume =
2.4
1. (i) (a) 4.55, 4.65 (b) 4.55 " x < 4.65 = 14.6 cm3 (to 3 s.f.)
(ii) (a) 0.725, 0.735 (b) 0.725 " x < 0.735 25
Minimum volume =
(iii) (a) 675, 685 (b) 675 " x < 685 2.4
= 10.4 cm3 (to 3 s.f.)

100
11. For 1 litre of paint, 1 1
(d) d >1+ d
Upper bound area = 3.5 m2 2 3
Lower bound area = 2.5 m2 1 1 1 1
d– d >1+ d– d
For the wall, 2 3 3 3
Upper bound area = 700.5 m2 1
d >1
Lower bound area = 699.5 m2 6
700.5 1
Maximum amount of paint needed = 6 d >6 1
2.5 6
= 280.2 litres d >6
12. For 1 bottle,
Upper bound capacity = 1.5 l 4 5 6 7 8
Lower bound capacity = 0.5 l (e) 2(e – 3) "1
For the tank, 2e – 6 "1
Upper bound capacity = 55 l 2e – 6 + 6 "1+6
Lower bound capacity = 45 l 2e "7
55 7
Maximum number of bottles of water drawn = e "
0.5 2
= 110 1
e "3
2
Review Exercise 3
1 2 3 1 4 5
1. (a) a–2 !3 3
2
a–2+2!3+2
(f) 5( f – 4) ! 2f
a !5
5f – 20 ! 2f
5f – 20 – 2f ! 2f – 2f
3 4 5 6 7
3f – 20 ! 0
(b) 2b + 1 < 5 – 4b
3f – 20 + 20 ! 0 + 20
2b + 1 + 4b < 5 – 4b + 4b
3f ! 20
6b + 1 <5
20
6b + 1 – 1 <5–1 f!
3
6b <4 2
4 f !6
b< 3
6
2 4 5 6 27 8
b< 6
3 3
(g) –3 – g > 2g – 7
–2 –1 0 2 1 2 –3 – g – 2g > 2g – 7 – 2g
3 –3 – 3g > –7
1 –3 – 3g + 3 > –7 + 3
(c) c" c–1
2 –3g > –4
1 1 1 –4
c– c" c–1– c g<
2 2 2 –3
1 1
c " –1 g <1
2 3
1
2 c " 2 –1
2
c " –2 –1 0 1 1 2 3
1
3

– 4 –3 –2 –1 0

101
(h) 18 – 3h < 5h – 4 d–2 2d + 3 5
18 – 3h – 5h < 5h – 4 – 5h (d) < +
3 5 8
18 – 8h < –4 d–2 2d + 3 5
– <
18 – 8h – 18 < – 4 – 18 3 5 8
–8h < –22 d – 2 2d + 3 5
3 5 – <3 5
–22 3 5 8
h>
–8 75
5(d – 2) – 3(2d + 3) <
3 8
h >2
4 75
5d – 10 – 6d – 9 <
8
75
1 2 33 4 –d – 19 <
2 8
4
75
a a –d – 19 + 19 < + 19
2. (a) 3+ >5+ 8
4 3
3
a a –d < 28
4 3 3+ >4 3 5+ 8
4 3
3
36 + 3a > 60 + 4a d > –28
8
36 + 3a – 4a > 60 + 4a – 4a 1 2 1
36 – a > 60 (e) (e + 2) ! + (e – 1)
3 3 4
36 – a – 36 > 60 – 36 1 1 2
(e + 2) – (e – 1) !
–a > 24 3 4 3
a < –24 1 1 2
4b 2b 3 4 (e + 2) – (e – 1) ! 3 4
(b) –5 <3– 3 4 3
9 3
4(e + 2) – 3(e – 1) ! 8
4b 2b
9 –5 <9 3– 4e + 8 – 3e + 3 ! 8
9 3
e + 11 ! 8
4b – 45 < 27 – 6b
e + 11 – 11 ! 8 – 11
4b – 45 + 6b < 27 – 6b + 6b
e ! –3
10b – 45 < 27
2f –5 f +3 2( f + 1)
10b – 45 + 45 < 27 + 45 (f) 5– " +
6 2 3
10b < 72
2f –5 f + 3 2( f + 1)
1 6 5– "6 +
b <7 6 2 3
5
30 – (2f – 5) " 3( f + 3) + 4( f + 1)
4c 3 1
(c) – !c– 30 – 2f + 5 " 3f + 9 + 4f + 4
9 4 2
35 – 2f " 7f + 13
4c 3 1
9 4 – !9 4 c– 35 – 2f – 7f " 7f + 13 – 7f
9 4 2
16c – 27 ! 36c – 18 35 – 9f " 13
16c – 27 – 36c ! 36c – 18 – 36c 35 – 9f – 35 " 13 – 35
–20c – 27 ! –18 –9f " –22
–20c – 27 + 27 ! –18 + 27 –22
f !
–20c ! 9 –9
4
9 f !2
c "– 9
20
3. (a) Solving the two linear inequalities separately,
5–a "a–6 and a – 6 " 10 – 3a
5–a–a "a–6–a a – 6 + 3a " 10 – 3a + 3a
5 – 2a " –6 4a – 6 " 10
5 – 2a – 5 " –6 – 5 4a – 6 + 6 " 10 + 6
–2a " –11 4a " 16
–11
a! a "4
–2
1
a !5
2

102
1 (d) Solving the two linear inequalities separately,
Representing a ! 5 and a " 4 on a number line,
2 2d + 1 ! d and d > 3d – 20
2d + 1 – d ! d – d d – 3d > 3d – 20 – 3d
2 3 4 5 1 6 7 d+1 !0 –2d > –20
5
2 –20
d+1–1 !0–1 d<
The simultaneous linear inequalities have no solution. –2
(b) Solving the two linear inequalities separately, d ! –1 d < 10
4 – b < 2b – 1 and 2b – 1 < 7 + b Representing d ! –1 and d < 10 on a number line,
4 – b – 2b < 2b – 1 – 2b 2b – 1 – b < 7 + b – b
4 – 3b < –1 b–1 <7
4 – 3b – 4 < –1 – 4 b–1+1 <7+1 –1 0 1 2 3 4 5 6 7 8 9 10
–3b < –5 b <8 The solutions satisfying both inequalities lie in the overlapping
–5 shaded region, i.e. –1 " d < 10.
b>
–3 4. (a) Largest integer value of x = 14
2 (b) Largest prime value of x = 13
b >1
3 1
2 (c) Largest rational value of x = 14
Representing b > 1 and b < 8 on a number line, 2
3 5. 27 – 2x " 8
27 – 2x – 27 " 8 – 27
–2x " –19
1 2 2 3 4 5 6 7 8
1 –19
3 x!
–2
The solutions satisfying both inequalities lie in the overlapping 1
2 x !9
shaded region, i.e. 1 < b < 8. 2
3
1
(c) Solving the two linear inequalities separately, (a) Least value of x = 9
2
1 1
4c – 1 < and " 3c + 2 (b) Least integer value of x = 10
2 2
6. (a) Solving the two linear inequalities separately,
1 1
4c – 1 + 1 < +1 – 3c " 3c + 2 – 3c 5x > 69 – 2x and 27 – 2x ! 4
2 2
5x + 2x > 69 – 2x + 2x 27 – 2x – 27 ! 4 – 27
1 1
4c < 1 – 3c " 2 7x > 69 –2x ! –23
2 2
1 69 –23
1 x > x"
1 1 1 7 –2
c< 2 – 3c – "2– 6 1
4 2 2 2 x >9 x " 11
3 1 7 2
c< –3c " 1 6 1
8 2 Representing x > 9 and x " 11 on a number line,
1 7 2
1
c! 2
–3
1 9 6 10 11 1 12
c !– 9 11
7 2
2
3 1 The solutions satisfying both inequalities lie in the overlapping
Representing c < and c ! – on a number line,
8 2 6 1
shaded region, i.e. 9 < x " 11 .
7 2
The integer values of x which satisfying both inequalities are
–1 1 0 3 1 10 and 11.

2 8
The solutions satisfying both inequalities lie in the overlapping
1 3
shaded region, i.e. – " c < .
2 8

103
(b) Solving the two linear inequalities separately, 9. Let the side of a square be x cm.
–10 ! x < – 4 and 2 – 5x < 35 4x ! 81
2 – 5x – 2 < 35 – 2 81
x !
–5x < 33 4
33 1
x> x ! 20
–5 4
3 1
x > –6 Greatest possible value of x = 20
5 4
3 1 1
Representing –10 ! x < – 4 and x > – 6 on a number line, Greatest possible area of the square = 20 20
5 4 4
= 410.1 cm2 (to 4 s.f.)
10. x 3 + 5 ! 20
–10 –9 –8 –7
–6
3 –6 –5 –4 3x + 5 ! 20
5 3x + 5 – 5 ! 20 – 5
The solutions satisfying both inequalities lie in the overlapping 3x ! 15
3 x !5
shaded region, i.e. –6 < x < – 4.
5 The maximum number of sheets of writing paper that he can use
The integer values of x which satisfy both inequalities are – 6 is 5.
and –5. 11. Let the age of Rui Feng now be x years. Then the age of Lixin is
7. Given that –1 ! x ! 5 and 2 ! y ! 6, (x – 3) years.
(a) Greatest value of y – x = 6 – (–1) x + (x – 3) ! 50
=6+1 2x – 3 ! 50
=7 2x – 3 + 3 ! 50 + 3
Least value of y – x = 2 – 5 2x ! 53
= –3 53
x!
x 5 2
(b) Greatest value of =
y 2 1
1 x ! 26
=2 2
2 The maximum possible age of Rui Feng now is 26 years.
x –1 The maximum possible age of Lixin now is 23 years.
Least value of =
y 2 The maximum possible age of Lixin 5 years ago is 18 years.
1
=– 12. Let x be the number of 20-cent coins.
2
16 1 + x 0.20 ! 22
x2 52
(c) Greatest value of = 16 + 0.20x ! 22
y 2
16 + 0.20x – 16 ! 22 – 16
25
= 0.20x ! 6
2
1 6
= 12 x!
2 0.20
x2 0 x ! 30
Least value of = The maximum number of 20-cent coins that he has is 30.
y 6
=0 13. Let x be the number of questions Amirah answered wrongly.
8. Given that –3 ! x ! 7 and 4 ! y ! 10, 2 (15 – x) – 1 x > 24
(a) Smallest value of x – y = –3 – 10 2(15 – x) – x > 24
= –13 30 – 2x – x > 24
x 7 30 – 3x > 24
(b) Largest value of =
y 4 30 – 3x – 30 > 24 – 30
3 –3x > – 6
=1
4 –6
x <
(c) Largest value of x2 – y2 = 72 – 42 –3
= 49 – 16 x <2
= 33 The maximum number of questions Amirah answered wrongly
(d) Smallest value of x3 + y3 = (–3)3 + 43 is 1.
= –27 + 64
= 37

104
15. Upper bound mass of 20 cabbages = 92.5 kg Challenge Yourself
Lower bound mass of 20 cabbages = 91.5 kg
x
Upper bound average mass of one cabbage 1. z =
y
92.5
= x 8
20 Greatest = = 40
y 0.2
= 4.625 kg
x 6
Lower bound average mass of one cabbage Least = = 12
y 0.5
91.5 12 ! z ! 40
=
20 3x – 5 3x – 5
= 4.575 kg 2. =
x 2 – 14 x + 49 (x – 7)2
16. (i) Least possible individual masses
3x – 5
= 35 kg, 35 kg, 45 kg, 55 kg, 55 kg, 65 kg >0
x 2 – 14 x + 49
Least possible total mass
3x – 5
= 35 + 35 + 45 + 55 + 55 + 65 >0
(x – 7)2
= 290 kg
3x – 5
(ii) Greatest possible individual masses Since (x – 7)2 is positive, 3x – 5 must be more than 0 for
(x – 7)2
= 45 kg, 45 kg, 55 kg, 65 kg, 65 kg, 75 kg to be more than 0.
Greatest possible total mass 3x – 5 > 0
= 45 + 45 + 55 + 65 + 65 + 75 3x > 5
= 350 kg 5
17. Maximum dimensions = 20.5 cm, 30.5 cm, 15.5 cm x>
3
Maximum volume = 20.5 30.5 15.5 2
x >1
= 9691.375 cm3 3
18. Least possible lengths of sides = 5.5 cm, 6.5 cm, 7.5 cm
Least possible perimeter = 5.5 + 6.5 + 7.5
= 19.5 cm
19. Upper bound of radius = 9.55 cm
Lower bound of radius = 9.45 cm
(a) Upper bound circumference = 2 (9.55)
= 60.00 cm (to 2 d.p.)
Lower bound circumference = 2 (9.45)
= 59.38 cm (to 2 d.p.)
(b) Upper bound area = (9.55)2
= 286.52 cm2 (to 2 d.p.)
Lower bound area = (9.45)2
= 280.55 cm2 (to 2 d.p.)

105
Chapter 4 Indices and Standard Form
TEACHING NOTES
Suggested Approach
In Book 1, the students have been introduced to writing numbers in index notation. In this chapter, they will learn the laws of
indices, zero and negative indices and rational indices.

Teacher should consider using the Investigation activities provided in the textbook to allow students to explore the laws of indices
for numbers before moving on to variables. It is not advisable to state all the laws of indices to the students when teaching this
chapter. After the students are familiar with laws of indices introduced in Section 4.2, where all the indices are positive integers,
teachers can extend it to Section 4.3: Zero and Negative Indices and Section 4.4: Rational Indices.

Teacher should also conduct more discussions on how compound interest and standard form are used in real life.

Section 4.1: Indices


This section gives students a better understanding on the meanings of the base and the index represented in an
index notation. Teachers may start on this chapter by giving scenarios where indices are involved and ask the
students to represent their answers in index notation, like what they have learnt in Book 1 (see Investigation:
Indices). Teachers should guide students along as they learn how to describe and compare numbers written in
index form (see Class Discussion: Comparing Numbers written in Index Form).

Section 4.2: Laws of Indices


Teachers should provide simple numerical examples to illustrate each law of indices. Ample examples should
be given to the students to master each law first before moving on to the next law (see Investigation: Law 1 of
Indices, Investigation: Law 2 of Indices, Investigation: Law 3 of Indices, Investigation: Law 4 of Indices and
Investigation: Law 5 of Indices).

Teachers should clarify any common misconceptions students may have or difficulties they may encounter when
working on questions involving the use of a few laws of indices (see Journal writing on page 83 of the textbook).

Section 4.3: Zero and Negative Indices


Teachers may ask the students to explore the meaning of zero and negative indices through activities instead
of only asking them to state the definition of such indices (see Investigation: Zero Index and see Investigation:
Negative Indices).

It is important to emphasise to the students the meaning of ‘evaluate’ and ‘leaving your answer in positive index
form. Teachers should also highlight the importance of recognising where the brackets are placed in a question
(see Thinking Time on page 87 of the textbook).

Section 4.4: Rational Indices


In Book 1, students have learnt about the square root and cube root of a number. Teacher may wish to extend on
this by introducing the meaning of positive nth root and radical expression.

Teachers should highlight to students to consider the need for the base to be positive in rational indices (see
Thinking Time on page 92 of the textbook).

106
Section 4.5: Standard Form
Teacher may begin this section by getting students to explore how standard forms are being expressed by giving
them some examples of very large and small numbers for them to express these numbers in standard form (see
Class Discussion: Standard Form). Teachers should highlight the difference between numbers expressed in
standard form and numbers not expressed in standard form so that students can better identify which expressions
are in standard form.

For the introduction of common prefixes used in our daily lives, teachers may use the range of prefixes used
in our daily lives (see page 100 of the textbook) to get the students to give more examples of prefixes that they
encounter in their daily lives and to practise reading prefixes.

Some students may find it difficult to manipulate numbers in standard form using a calculator. Teachers should
give them time and guide them through some examples on using the calculator to evaluate numbers represented
in standard form.

107
WORKED SOLUTIONS 4. am an = (a a … … a) (a a … a)
Investigation (Indices) m times n times
=a a … a
Amount of allowance on the 31st day of the month = 231 m + n times
= 2 147 483 548 cents = am + n
$21.5 million
Investigation (Law 2 of Indices)
Class Discussion (Comparing Numbers written in Index
3 3 3 3 3
Form) 1. 35 ÷ 32 =
3 3
1. 210 means 2 multiplied by itself 10 times, while 102 means 10 = 33
multiplied by itself. = 35 – 2
6
2. 210 = (2 2 2 2 2) (2 2 2 2 2) 10 10 10 10 10 10 10
2. =
= 322 > 102 10 4 10 10 10 10
3. 37 = 3(3)6 = 3(9)3 > 73 = 102
4. = 106 – 4
Value of ab Value of ba a a a a a a a
3. a7 ÷ a3 =
23 = 8 32 = 9 a a a
= a4
24 = 16 42 = 16
= a7 – 3
34 = 81 43 = 64 m times

35 = 243 53 = 125 4. am ÷ an = a a … … a
a a … a
45 = 1024 54 = 625 n times
6
4 = 4096 4
6 = 1296 = a a … a
m – n times
(i) If a and b are positive integers such that b > a, ab = ba when = am – n
a = 2 and b = 4.
It is not easy to prove that this is the only solution; students are Investigation (Law 3 of Indices)

(ii) If a and b are positive integers such that b > a, ab < ba when 1. (25)2 = 25 25
a = 1, i.e. 12 < 21, 13 < 31, 14 < 41, etc and when a = 2 and b = 3, = 25 + 5 (using Law 1 of indices)
ab < ba, i.e. 23 < 32. = 25 2
5. In general, if a and b are positive integers such that b > a, then 2. (10 ) = 104 104 104
4 3

ab > ba, with some exceptions when a = 2 and b ! 4 and when = 104 + 4 + 4 (using Law 1 of indices)
a = 1. = 104 3
3. (am)n = (am am … am)
Investigation (Law 1 of Indices) n times
n times
2 3
1. 7 7 = (7 7) (7 7 7) = am + m + … + m
=7 7 … 7 = am n
= 75
= 72 + 3 Investigation (Law 4 of Indices)
2. 64 5
6 = (6 6 6 6) (6 6 6 6 6)
=6 6 … 6 1. 23 73 = (2 2 2) (7 7 7)
= 69 = (2 7) (2 7) (2 7)
= 64 + 5 = (2 7)3
3. a3 4
a = (a a a) (a a a a) 2. (–3)2 (– 4)2 = (–3) (–3) (– 4) (– 4)
=a a … a = [(–3) (– 4)] [(–3) (– 4)]
= a7 = [(–3) (– 4)]2
= a3 + 4 3. an bn = (a a … a) (b b … b)
n times n times
= (a b) (a b) … (a b)
n times
= (a b)n

108
Class Discussion (Is (a + b)n = an + bn? Is (a – b)n = an – bn?)
(xy2)4 (3x2 y)4 = (x4y2 4) (34x2 4y4) (Law 4 and Law 3) (a + b)n an + bn
= (x4y8) (81x8y4) Example: a = 3, b = 2, n = 4
= 81x4 + 8y8 + 4 (Law 1) (3 + 2)4 = 54 = 625
= 81x12y12 34 + 24 = 81 + 16 = 97
(a – b) a – bn
n n

(xy2)4 (3x2y)4 = [(xy2) (3x2y)]4 (Law 4) Example: a = 3, b = 2, n = 4


= (3x1 + 2y2 + 1)4 (Law 1) (3 – 2)4 = 14 = 1
= (3x3y3)4 34 – 24 = 81 – 16 = 65
= 34x3 4y3 4 (Law 3)
= 81x12y12 Investigation (Zero Index)
1. Index Form Value
Investigation (Law 5 of Indices)
4
3 81
3 83 3 3
1. 8 ÷ 5 = 3 3 27
5
8 8 8 32 9
= 1
5 5 5 3 3
8 8 8 3 0
1
=
5 5 5 Table 4.1
3
8 2. 81 (i.e. 34) must be divided by 3 to get 27 (i.e. 33).
=
5 3. 27 (i.e. 33) must be divided by 3 to get the value of 32.
(–12)4 4. 32 must be divided by 3 to get the value of 31.
2. (–12)4 ÷ (–7)4 =
(–7)4 5. (a) 31 must be divided by 3 to get the value of 30.
(–12) (–12) (–12) (–12) 6.
= Index Form Value
(–7) (–7) (–7) (–7)
4
(–2) 16
(–12) (–12) (–12) (–12)
= (–2)3 –8
(–7) (–7) (–7) (–7)
2
(–12)
4 (–2) 4
= (–2) 1
–2
(–7)
n times (–2)0 1
a a … a Table 4.2
3. an ÷ bn =
b b … b 7.
n times
a a a Thinking Time (Page 87)
= …
b b b
LHS = –50 = –(50) = –1
n times
n
RHS = (–5)0 = 1
a –50 0
=
b
Investigation (Negative Indices)
Journal Writing (Page 83)
3 1.
2 x2 Index Form Value
1. (i) To simplify , Law 3 of indices must be applied to the
y 32 9
entire expression. 1
3 3
Nora applied Law 3 of Indices to the algebraic terms x and y 0
3 1
but she did not apply the same law to the number.
1
Farhan applied Law 3 of indices to the number correctly but he 3–1
3
applied Law 1 of Indices to the algebraic terms x and y which
1
is the wrong law. 3–2
3 9
2 x2 23 x2 3
8 x6 Table 4.3
(ii) = =
y y 3
y3

109
2. Investigation (Rational Indices)
Index Form Value
2 2 1
(–2)2 4 (a) 5 3 = 5 3

1 1
(–2) –2 = (5 ) 2 3

0
(–2) 1 3
= 52
–1 1
(–2) – 2 1 2
2 (b) 5 3 = 5 3
1
1 3 2
(–2)–2 = (5 )
4 = ( 3 5 )2
Table 4.4
3. Thinking Time (Page 94)

Thinking Time (Page 89) 1. If a and b are real numbers, and m and n are rational numbers, then
m+n
Law 1 of Indices: am an = a if a > 0
1. If a and b are real numbers, and m and n are integers, then Law 2 of Indices: m
a ÷a =an m–n
if a > 0
m+n
Law 1 of Indices: am an = a if a Law 3 of Indices:
mn
(am)n = a if a > 0
m n m–n
Law 2 of Indices: a ÷a =a if a Law 4 of Indices:
n
an bn = (a b) if a, b > 0
mn
Law 3 of Indices: (am)n = a if a a
n

Law 4 of Indices:
n
an bn = (a b) if a, b Law 5 of Indices: an ÷ bn = if b > 0
n
b
a
Law 5 of Indices: an ÷ bn = if b 2. (i) In Law 1, it is necessary for a > 0 otherwise am or an is not
b
–2
2. (i) In Law 1, it is necessary for a 0–1, it is (ii) In Law 4, it is necessary for a, b > 0 otherwise an or bn is not

–2
(ii) In Law 4, it is necessary for a, b 0–2, it is
3. (–1) (–1) a, b > 0 but in this case,
a = b = –1 < 0.
3. (i) If m = n in Law 2, then LHS = an ÷ an = 1 and
RHS = an – n = a0 = 1.
Class Discussion (Standard Form)
So a0 is a special case of Law 2.
1 1. The powers of 10 are all positive integers.
(ii) If m = 0, then RHS = a0 – n = n and
a 2. The powers of 10 are all negative integers.
1 3. (v) 2.9 104
LHS = a0 ÷ an = 1 ÷ an = n
a (vi) 3 108 m/s
1 (vii) 3.8 10–5 cm
So a–n = n is a special case of Law 2.
a (viii) 2.99 10–23 g

Class Discussion (Rational Indices) Performance Task (Page 102)


1 1
Let p = 5 . Then p3= ( 5 )3
3 3 1 GB = 1 073 741 824 bytes
1 3 All computer data is stored in a binary format as either a one or a zero.
= 5 3 (Using Law 3 of Indices)
Hence each level is an increment of 2 to the 10th power or 1024. As
= 51
such, 1 GB = 230.
=5
27 = 128 MB
p= 35
28 = 256 MB
In this case, there is only one possible value of p.
29 = 512 MB
1
3
Hence, 5 3 = 5.
Thinking Time (Page 104)
1. 57 910 000 km = 5.791 107 km
Thinking Time (Page 92)
5 945 900 000 km = 5.9459 109 km
1
1. If a < 0, then a n = n a 3683 km
2. 3 683 000 m/h = = 3683 km/h = 3.683 103 km/h
1
n
1h
2. If a = 0, then 0 = 0 n
n is a positive integer.

110
3. 0.000 000 0004 m = 4 10–10 m (c) (–2c2d 5)5 = (–2)5 c2 5 d 5 5
26
500 000 000 000 000 000 000 000 000 = 5 10 = –32 c10 d25
2
26 4 10 –10 = –32c10d25
Total volume of air molecules = 5 10 (d) (m2n)4 (m4n3)5 = (m2 4n4) (m4 5n3 5)
2
= 6.28 107 m3 (to 3 s.f.) = (m8n4) (m20n15)
4. 100 trillion = 100 1012 = 1 1014 = m8 + 20n4 + 15
1 106 = 1 million 2 109 = 2 106 103 = 2000 million = m28n19
42 000 000 = 4.2 107 (– p 7 q 5 )2
(e) (–p7q5)2 ÷ (3p3q2)3 =
(3 p 3 q 2 )3
Practise Now 1 (–1)2 p 7 2
q5 2

=
(a) 4 7 5
4 =4 7+5
33 p 3 3 q 2 3

= 412 p q 14 10
=
(b) (–3)6 (–3) = (–3)6 + 1 27 p 9 q 6
= (–3)7
p14 – 9 q10 – 6
(c) a 12 8
a =a 12 + 8 =
27
= a20
p5 q4
(d) 2xy4 3x5y3 = 6x1 + 5y4 + 3 =
27
= 6x6y7
Practise Now 5
Practise Now 2
213
(a) 97 ÷ 93 = 97 – 3 (a) 213 ÷ 73 =
73
= 94 3
21
(b) (– 4)8 ÷ (– 4) = (– 4)8 – 1 =
7
= (– 4)7
10 6 10 – 6 = 33
(c) a ÷ a = a
2615
= a4 (b) (265)3 ÷ 1315 =
1315
27 x 9 y 4
(d) 27x9y4 ÷ 9x6y3 = 26
15

9 x6 y3 =
13
= 3x9 – 6y4 – 3
= 215
= 3x3y 3
p2 q7 p6 q7
(c) ÷ = 3 ÷
Practise Now 3 q p5 q p5

1. (a) (63)4 = 63 4 p6 q7
=
= 612 q3 p5
(b) (k ) = k5 9
5 9 p6 + 5
=
= k45 q 3+ 7
(c) (3q)6 (34)q = 36q 34q p11
=
= 36q + 4q q10
3
= 310q 3x 2 27 x 7 27 x 6 27 x 7
(d) ÷ = ÷ 21
2. x (x ) ÷ (x ) = x10
8 3 n n 2
x3 x 21
x 9
x
x8 x3n ÷ x2n = x10 27 x 6
x 21
x8 + 3n – 2n = x10 =
x9 27 x 7
8 + n = 10
27 x 6 + 21
n =2 =
27 x 9 + 7

Practise Now 4 x 27
=
x16
(a) 37 87 = 247
= x27 – 16
(b) (5b4)3 = 53 b4 3
= x11
= 125 b12
= 125b12

111
Practise Now 6 16d –2 e 16d –2 e
(c) = 3
–1 3
(2d e) 2 d –1 3 e 3
1. (a) 20150 = 1
(b) (–7)0 = 1 16d –2 e
=
(c) 3y0 = 3(1) 8 d –3 e 3
=3 2d –2 + 3
=
(d) (3y)0 = 30 y0 e 3–1
=1 1 2d
= 2
=1 e
2. (a) 3 33 ÷ 32 = 30 + 3 – 2
0 5f 0
(d) 5f 0 ÷ 3( f –2)2 =
= 31 3( f –2 )2
=3 5
=
(b) 30 + 32 = 1 + 9 3 f –4
= 10 5f 4
=
3
Practise Now 7 18g –6
(e) 18g–6 ÷ 3(g–2)2 =
1 3(g –2 )2
(a) 6–2 =
62 18g –6
=
1 3g –4
=
36 6
=
1 g –4 + 6
(b) (–8)–1 =
(–8)1 6
=
1 g2
=–
8
4 6h 2 4
4
–3
1 (f) 6h2 ÷ 2h–2 – h h3 – –4
= –h h3 –
(c) = h 2h –2 h –4
3
5 4 = 3h2 + 2 – h1 + 3 – 4h4
5
= 3h4 – h4 – 4h4
1 = –2h4
=
64
125
Practise Now 9
64
=1÷ (a) By prime factorisation, 256 = 4 4 4 4 = 44.
125
4
125 256 = 4 4 4 4 4
=1
64 =4
61 (b) By prime factorisation, 1024 = 4 4 4 4 4 = 45.
=1
64 5
1024 = 5 4 4 4 4 4
–1
1 1 =4
(d) = –1
9 1 (c) By prime factorisation, 8 = 2 2 2 = 23 and 27 = 3 3 3 = 33.
9
3
8 3
2 2 2
1 =
=1÷ 27 3 3 3
9
2
=1 9 =
3
=9

Practise Now 10
Practise Now 8
1

(a) a–1 a3 ÷ a–2 = a–1 + 3 – (–2) (a) 36 2 = 36


= a4 =6
(b) (b–5c2)–3 = b–5 –3c2 –3
= b15c–6
b15
= 6
c

112
1
5
–1 1 3
3 m
(b) 8 3
= (b) 5
m ÷ m2 =
8 3
m2
1
3
1 m5
= =
8 1
(m 2 ) 3
1 1
= m5
2 = 2
1
1 1 3 m3
(c) (–125) = 3
1–2
–125 = m5 3

1 – 7
= = m 15
3
–125 1
= 7
1 m 15
=–
5 –1 –3 – 1 5 –1
(c) (m –3 n 5 ) 3
= m 3
n 3

–5
Practise Now 11 = mn 3

2 m
1. (a) 64 3 = ( 3 64 )2 = 5
n3
= 42 –1 –1
–1 –1
= 16 m 3n 4 m 3n 4
(d) =
–1 – 1 –2
–3 1
3
5 (m 2 n 3 )–2 m2 –2
n 3

(b) 32 5
= –1 –1
32 m n 3 4

3
= 2
–4
1 m n 3
= 5
32 – 1 – (–4 ) – 1 – 2
3
= m 3
n 4 3

1 11 – 11
= = m n 3 12
2
11
1 m3
= =
8 11
n 12
3
(c) 1001.5 = 100 2 1 –2 1 2 1 –4 1 –2 2
3
(e) (25m 2 n –4 ) 2 (m 3 n 5 )2 = (25 2 m 2
n 2
)(m 3 2
n 5
)
= ( 100 )
–2 6 –4
= (10)3 = (5mn )(m n 5 )
= 1000 –2 – 4
= 5m1+ 6 n 5
1
3 n n 3
2. (a) a = (a ) – 14
n = 5m 7 n 5

= a 3
5m 7
= 14
1 1 n5
(b) =
5 1
x2 (x ) 2 5
–1 –1 1
(f) (m 2 n 7 ) 5
(m 5 n –5 ) = (m 2 n 7 ) (m 5 n –5 ) 5
1
= 2 –1 5 1 –5 1
x5 = (m 2 n 7 ) (m 5
n 5
)
–2
–1
= x 5
= (m n ) 2 7
(mn ) –1

1
2 +1 – 7 –1
= m n
Practise Now 12 8
3 –7
= m n
5 1 2 5 1
(a) (m 2 ) 6 m3 = m 6
m3 m3
5 1 = 8
= m 3
m 3
n7
5+1
= m3 3

= m2

113
Practise Now 13 (c) (2.4 108) ÷ (6 104)
= ( 2 . 4 10x 8 ) ÷
(a) 5x = 125
5x = 53 ( 6 10x 4 ) =
x =3
= 4000
1
(b) 7y = = 4 103
49
3.5 10 –5
1 (d)
7y = 2 1.4 10 8
7
7y = 7–2 = ( 3 . 5 10x – 5 )

y = –2 ÷ ( 1 . 4 10x 8 ) =
(c) 8z = 16
–13
(2 ) = 24
3 z = 2.5 10
23z = 24 (e) 1.14 105 + 4.56 104
3z = 4 = 1 . 1 4 10x 5 ÷
1
z =1 4 . 5 6 10x 4 =
3
= 159 600
Practise Now 14 = 1.60 105 (to 3 s.f.)
(f) 4 104 – 2.6 106
1. (a) 5 !300 000 = 5.3 106
8 = 4 10x 4 – 2 . 6 10x 6 =
(b) 600
! 000 000 = 6 10
(c) 0.000 048
!
= 4.8 10–5 = –2 560 000
(d) 0.000 000 000 !21 = 2.1 10–10 = –2.56 106
2. (a) 6
1.325 10 = 1 325 000 2.37 10 –3 + 3.25 10 –4
(g)
(b) 4.4 10–3 = 0.0044 4.1 10 5
= ( 2 . 3 7 10x – 3 + 3 . 2 5
Practise Now 15
10x – 4 ) ÷ 4 . 1 10x 5 =
–6
(a) 1 micrometre = 10 metres –9
= 6.57 10 (to 3 s.f.)
25.4 micrometres = 25.4 10–6 metres
6.3 10 6
= 2.54 10–5 metres (h)
1.5 10 2 – 3 10 –1
(b) 10 mm = 1 cm
1 = 6 . 3 10x 6 ÷ ( 1 . 5
2340 mm = 2340 cm
10
10x 2 – 3 10x – 1 ) =
= 234 cm
4
= 2.34 102 cm = 4.21 10 (to 3 s.f.)
(c) 1 terabyte = 1012 bytes
4.0 terabytes = 4.0 1012 bytes Practise Now 17
1 MB = 106 bytes
Practise Now 16 512 MB = 512 106 bytes
(a) (1.14 105) (4.56 104) = 5.12 108 bytes
= ( 1 . 1 4 10x 5 )
5.12 10 8
No. of photographs that can be stored =
640 10 3
( 4 . 5 6 10x 4 ) = = 800
= 5.20 109 (to 3 s.f.)
(b) (4.2 10– 4) (2.6 102)
Exercise 4A
= ( 4 . 2 10x – 4 ) 1. (a) 23 27 = 23 + 7
= 210
( 2 . 6 10x 2 ) =
(b) (– 4) (– 4)5 = (– 4)6 + 5
6

= 0.1092 = (– 4)11
= 1.09 10–1 (to 3 s.f.) 8
(c) x x = x 3 8+3

= x11
(d) (3y ) (8y7) = 24y2 + 7
2

= 24y9

114
4 4
2. (a) 58 ÷ 55 = 58 – 5 x x
(f) – = –1
= 53 y2 y2
(b) (–7) ÷ (–7)4 = (–7)11 – 4
11
x4
= (–1)4
= (–7)7 y2 4
7 3 7–3
(c) 6x ÷ x = 6x x4
= 6x4 =1
y8
–15 y 9 x4
(d) (–15y9) ÷ 5y4 = =
5 y4 y8
= –3y9 – 4 5. (a) h2k h11k9 = h2 + 11 k1 + 9
= –3y5 = h13k10
3. (a) (92)4 = 92 4 (b) (–m n ) 4m11n9 = –1 4 m7 + 11 n3 + 9
7 3

= 98 = – 4 m18 n12
(b) (h ) = h2 5
2 5
= – 4m18n12
= h10 (c) 11p q 2p q = 11 p6 + 3 2 q7 + 10
6 7 3 10

(c) 314 (52)7 = 314 52 7 = 22 p9 q17


= 314 514 = 22p9q17
= (3 5)14 h9 k 6
= 1514 (d) h9k6 ÷ h5k4 =
h5 k 4
(d) 23 93 = (2 9)3 = h9 – 5k6 – 4
= 183 = h4k2
(e) (2k ) = 23 k6 3
6 3
15m 8 n 7
= 8 k18 (e) 15m8n7 ÷ 3m2n =
3m 2 n
= 8k18 = 5m8 – 2n7 – 1
(f) (–3x y ) = (–3)4 x6 4 y2
6 2 4 4
= 5m6n6
= 81 x24 y8
–10 x 5 y 6
= 81x24y8 (f) (–10x5y6) ÷ (–2xy3) =
–2 xy 3
1413 = 5x5 – 1y6 – 3
4. (a) 1413 ÷ 713 =
713 = 5x4y3
13
=2
6. (a) (a2)3 a5 = a2 3 a5
95 4
5 4 20
(b) (9 ) ÷ 3 = 20 = a6 a5
3
= a6 + 5
(32 )20
= = a11
320
(b) (b ) (b ) = b3 7 b4 5
3 7 4 5

32 20 = b21 b20
=
320 = b21 + 20
340 = b41
= 20
3 (c) (c ) ÷ (– c ) = c6 5 ÷ (– c2)
6 5 2

= 340 – 20 c 30
= 320 =
–c 2
5
m m5 c 30 – 2
(c) = =
2 25 –1
m5
= = – c28
32
3 (–3d 3 )2
3 33 (d) (–3d3)2 ÷ (2d)3 =
(d) = 2 3 (2d )3
n2 n
27 (–3)2 d 3 2

= =
n6 23 d 3
6 9 d6
p4 p4 6 =
(e) = 8 d3
q q6
9
p 24 = d6 – 3
= 8
q6 9d 3
=
8

115
2
(e 3 )5 2a 2 a 2a 2 a2
(e) (e3)5 ÷ (– e2)4 = 8. (a) =
(–e2 )4 b b2 b b2 2
e3 5 2a 2 a2
= =
(–1)4 e2 4 b b4

e15 2a 2 + 2
=
= b1+ 4
1 e8
2a 4
= e15 – 8 =
b5
= e7
3 2
(4 f 6 )3 c c3 c3 c3 2

(f) (4f 6)3 ÷ (–2f 3)3 = (b) =


(–2 f 3 )3 d2 2d d2 3 2 2
d2
43 f 6 3 c3 c6
= = 6
(–2)3 f 3 3 d 4 d2
3+ 6
c
64 f 18 =
= 4 d6+2
–8 f 9
c9
= –8 f 18 – 9 =
4d 8
= –8f 9 4
3e 3 27e9 34 e 3 4
27e9
7. (a) (ab2)3 (2a2b)3 = a3 b2 3 23 a2 3 b3 (c) ÷ 11
= ÷
f2 f f2 4 f 11
= a3 b6 8 a6 b3
81 e12 27e9
= 8 a3 + 6 b6 + 3 = 8
÷ 11
f f
= 8 a9 b9
81 e 12 f 11
= 8a9b9 =
f8 27 e9
(b) c d (–5c d ) = c2d 2 (–5)2 c3 2 d 3 2
2 2 3 3 2

= c2 d2 25 c6 d 6 81 12 – 9 11 – 8
= e f
= 25 c2 + 6 d 2 + 6 27
= 25 c8 d 8 = 3e3f 3
= 25c8d 8 g2
6
–3g 5
3
g2 6
(–3)3 g 5 3
(d) ÷ = ÷
(8e f )5 3 2
h3 2h 2 h3 6
23 h2 3
(c) (8e5f 3)2 ÷ (e3f )3 =
(e 3 f )3 g12 –27 g15
= ÷
82 e5 2
f3 2
h18 8 h6
= 3 3 3
e f g12 8 h6
= 18
64 e10 f6 h –27 g15
= 9 3 8 12 – 15 6 – 18
e f =– g h
10 – 9 27
= 64 e f 6–3
3 8 –3 –12
= 64 e f =– g h
27
= 64ef 3
8
16g 8 h 7 =–
(d) 16g8h7 ÷ (–2g3h2)3 = 27g 3 h12
(–2g 3 h 2 )3
(2 x 2 y)3 (5 xy 4 )3 23 x2 3 y3 53 x 3 y4 3
16g 8 h 7 9. (a) = 2
= (10 xy 3 )2 4 xy 10 x 2 y 3 2 4 xy
(–2)3 g 3 3 h 2 3

8 x6 y3 125 x 3 y12
16g 8 h 7 =
= 100 x 2 y 6 4 xy
–8 g 9 h 6
1000 x 6 + 3 y 3+12
= –2g8 – 9h7 – 6 =
400 x 2 +1 y 6 +1
= –2g–1h
5 x 9 y15
2h =
=– 2 x 3 y7
g
5
= x9 – 3 y15 – 7
2
5 x 6 y8
=
2

116
8 x8 y4 (4 x 2 y 2 )2 8 x8 y4 42 x2 2 y2 2 Substitute (1) into (2):
(b) = 2 (8 – b) – b = – 4
(2 xy 2 )2 (3xy) 2
2 x 2 y2 2
32 x 2 y2
8 – 2b = – 4
8 x8 y4 16 x 4 y 4
= 2b = 12
4 x2 y4 9 x 2 y2
b =6
128 x 8 + 4 y 4 + 4 a=8–6
=
36 x 2 + 2 y 4 + 2 =2
32 x12 y 8 a = 2, b = 6
=
9 x 4 y6
32 Exercise 4B
= x12 – 4 y8 – 6
9
1. (a) 170 = 1
32 x 8 y 2
= 2
0
9 (b) – =1
7
(2 xy 2 )5 25 x 5 y2 5
(c) = (c) 4a0 = 4(1)
(4 x 2 y)2 (xy 3 ) 4 2 x 2 2 y 2 xy 3
=4
32 x5 y10
= 4 (d) –8b0 = –8(1)
16 x y xy 3
2
= –8
32 x 5 y10 (e) (72cd 2)0 = 1
=
16 x 4 +1 y 2 + 3 (f) 7(e8)0 = 7(1)
32 x5 y10 =7
= 2. (a) 20 24 = 20 + 4
16 x5 y5
= 2 x5 – 5 y10 – 5 = 24
= 2 x0 y5 = 16
= 2y5 2 0 72 70
(b) 7 7 ÷ 7 =
7
4 x 2 y4 8 x 4 y2 32 x2 + 4 y4 + 2
(d) = 2 = 72 + 0 – 1
(4 x 2 y 2 )2 4 x 2 2 y2 2
=7
32 x 6 y6 0 2
(c) 8 – 8 = 1 – 64
=
16 x 4 y4 = –63
= 2 x6 – 4 y6 – 4 (d) 63 + 60 – 6 = 216 + 1 – 6
= 2 x2 y2 = 211
= 2x2y2 1
3. (a) 7–3 = 3
(2 p 3 q 4 )4 (4 p 2 q)2 pa + b 7
10. ÷ = a–b
(–3q )5 2
9 q 1
=
a+b
343
24 p3 4
q4 4 2
p2 2 q2 = p
÷ 4 1
(–3)2 q 5 2
9 q a–b
(b) (–5)–1 =
a+b
(–5)1
16 p12 q16 ÷ 16 p4 q2 = p 1
9 q10 9 qa – b =–
5
16 p12 q16 9 pa + b –2
= 3 1
9 q10 16 p4 q 2
qa – b (c) = 2
4 3
12 16 a+b
p q 1 p 4
=
q10 p4 q2 qa – b 1
=
12 16 a+b 9
p q p
= a–b
p4 q10 + 2 q 16
9
p12 – 4 pa b =1÷
= a–b 16
q12 –16
q 16
=1
p8 pa + b 9
= a–b
q –4
q 7
=1
a+b =8 9
a = 8 – b — (1)
a – b = –4 — (2)

117
1
–1 1
5 1 1 4

(d) = (c) (16) 4


=
3 5 16
3 1
=
5 4
16
=1÷
3 1
=
3 2
=1
5 3
(d) 41.5 = 4 2
3
= = ( 4 )3
5
2 –2 –4 (7 2 )–2 = 23
4. (a) (7 ) ÷ 7 =
7 –4 =8
–4 5
7 1 3
= –4 –5
7 (e) 8 3 =
8
=1
1
1 =
(b) 50 – 5–2 = 50 – 2 ( 3 8 )5
5
1 1
=1– =
25 25
24 1
= =
25 32
–1 2
3 5 3
(f) (–1000) 3 = ( –1000 )
2
(c) (215)0 + =1+
5 3
= (–10)2
2
=2 = 100
3 1
–2 2 4
3 24 0
7. (a) a = a4
(d) 3 2015 = 9 1 1
4 3 (b)
3 2 3
b 2 = (b )
16 2 1
= 9 = b 3
9 2
= 16 = b 3

2 2 1
5. (a) By prime factorisation, 196 = 2 2 7 7=2 7.
(c) ( 5 c )4 = (c 5 )4
1 4
196 = 2 2 7 7
= c5
= 14 4

3
= c5
(b) By prime factorisation, 125 = 5 5 5=5. 1
1
3 3 (d) = 1
125 = 5 5 5 6
d d6
=5 –1
(c) By prime factorisation, 32 = 2 2 2 2 2 = 25. = d 6

1 1
5
1 5
1 (e) = 4
= 8
e4 e8
32 2 2 2 2 2
1 1
= = 1
2 e2
(d) By prime factorisation, 16 = 2 2 2 2 = 24 = e
–1
2

and 81 = 3 3 3 3 = 34.
1 1
16 2 2 2 2 (f) 5
= 5
3
4
= 4 ( f) f3
81 3 3 3 3
–5
2 = f 3
=
3 8. (a) 11a = 1331
1
6. (a) 812 = 81 11a = 113
=9 a=3
1
b 1
(b) (–27) 3 = 3 –27 (b) 2 =
128
= –3 1
2b = 7
2
2b = 2–7
b = –7

118
(c) 9c = 35 –3
j2 j2 –3

32c = 35 (f) (j2k–1)–3 = j2 –3


k–1 –3
k3 k3 –3

2c = 5 j–6

1 = j –6 k3
c =2 k –9
2 = j –6 – 6 k3 + 9
(d) 10d = 0.01
= j –12 k12
1
10d = k 12
100 = 12
j
10d = 10–2
d = –2 (m 5 n 3 ) (m 2 )–2 m 5 n 3 m 2 –2
(g) =
5a 4 3a 2 (m –1n)2 m –1 2 n 2
9. (a) 5a4 3a2 ÷ a–3 =
a –3 m 5 n 3 m –4
=
5 3 a4 + 2 m –2 n 2
=
a –3 m5 – 4 n3
=
15 a 6 m –2 n 2
=
a –3 m n3
=
= 15 a6 a3 m –2 n 2
= 15 a6 + 3 = m1 + 2 n3 – 2
= 15a9 = m3n
–24b –6 6 6
(b) –24b–6 ÷ (3b–3)2 = (h) (5p)3 – 10p 7p2 + = 125p3 – 10p 7p2 + –3
(3b –3 )2 p –3 p
–24b –6 = 125p3 – 70p3 + 6p3
=
32 b –3 2 = 61p3
1 1
–24b –6 10. (a) a 3
a = a2 a3
=
9 b –6 1+1
= a2 3
2
= –2 3+2
3 = a6 6

5
(3c)0 = a6
(c) (3c)0 ÷ (c–3d5)–2 =
(c –3 d 5 )–2 3
1 1
(b) b2 ÷ 6
b = (b 2 ) 3 ÷ b 6
1
= –3 –2 5 –2 2 1
c d b 3
= 1
1 b 6
= 6 –10
c d 2

10
b3
d = 1
= b6
c6
2–1
(4e –6 f 3 )2 42 e –6 2 f 3 2 = b3 6
(d) = 4–1
8e12 f 6 8e12 f 6 = b6 6

16 e –12 f 6 = b6
3
=
8e12 f 6 1

6–6
= b2
2 f
= 4+1
e12 +12 (c) c 5
4 1 –2 c5 2
c2 ÷ c 5 =
–2
2 f0 c 5
= 24 4+1+2
e = c 5 2 5

2 8 + 5 + 4
= 24 = c 10 10 10
e 17
(e) (3g–3h–1)2 (– 4g3h–2)2 = c 10
= (32 g–3 2 h–1 2) ((– 4)2 g3 2
h–2 2)
= (9 g– 6 h–2) (16 g6 h– 4)
= 9 16 g– 6 + 6 h–2 – 4
= 144 g0 h–6
144
= 6
h

119
1 –3 –1 1
1 –1 –3 d 10 d 2
g –2 h 2 2
25 2
(d) d 10 ÷ d 5
d 2 = –1
(d) =
d 5 25 g –2 h 2
= d
1 – 3– –1
10 2 5 ( ) 25 2
1

1 – 15 + 2
= –2 1 2 1
= d 10 10 10 g 2
h 2

–6
= d 5 5
= –1
1 g h
= 6
d5 5g
=
–1 1 h
(e) (e –3 f 4 ) 2
= 1
1

(e f ) –3 4 2
1 1
(4 j 4 k ) 2
4 3 –2
(e) (4 j k ) ÷ 2h k 2 = –1
1 2h 3k 2

=
–3 1 4 1 1
e 2
f 2 4 1 1
42 j 2
k2
1 = –1
= 2h 3k 2
–3 2
e 2
f 1
3 2 j2 k2
e2 =
= 2 –1
f 2h 3k 2

2 1+1
–4 3 2 3 –4 3
(f) (g h ) = g 3 2 h
3 5 2 5 2 j2 k2 2
=
–6 h3
= gh 5
j2k
g =
= 6 h3
h5 –1
1 1 –2 1 3 1 4 1 –5 1
–1 (m 3n 4 )4
–2
11. (a) (a b ) 3 3 4
(a b ) = (a –5 2 3
b 3
) (a 2
b 2
) (f) (m 3 n 4 )4 ÷ 5
32m 4 n –8 = 5
32m 4 n –8
–2 2 –5
= (a 3
b) (a b ) 2
–1 4
–2 +2 1– 5 m3 4
n 4
= a 3
b 2 = 1
4 –8 5
4 –3 (32m n )
= a 3
b 2

a
4
3 m12 n –1
=
= 3
1 4 1 –8 1
b2 32 5
m 5
n 5

3 3 3 –2 4 5 –2 5 m12 n –1
(b) (c –3 d 5 )–2 (c –3 d 5 )–2 = (c –3 –2
d5 ) (c 5 d 5
) = 4 –8
6 –6 4 –2
2 m 5
n 5
= (c d 5) (c d )
12 – 4 –1+ 8
6+4 –6 –2 m 5
n 5
=c d 5
=
2
10 – 16 56 3
=c d 5
m 5
n5
c10 =
= 16 2
56 3
d5 m n5 5
=
–1 –1 –1 –1 2
e 3f 4
e 3f 4
(c) = 3 –4
– 1 –2 – 1 –2
2
(e f ) 3
e 2 –2
f 3 x –4 y 7 z –6 x 5 y 2 z –6
12. (a)
–1 –1 x 3 y –1 z 3 x –3 y –5 z 4
e 3f 4
=
e– 4 f3
2
x –4 3
y7 3
z –6 3 x5 –4
y 2 – 4 z –6 –4

=
–1 +4 –1 – 2 x3 3
y –1 3
z3 3 x –3 –4
y –5 – 4 z 4 –4

= e 3
f 4 3

11 – 11
x –12 y 21 z –18 x –20 y –8 z 24
= e f 3 12 =
x9 y –3 z9 x12 y 20 z –16
11
e3 x –12 – 20 y 21– 8 z –18 + 24
= =
x 9 +12 y –3+ 20 z 9 –16
11
f 12

x –32 y13 z 6
=
x 21 y17 z –7
= x–32 – 21 y13 – 17 z6 + 7
= x–53 y– 4 z13
z13
= 53 4
x y
120
3 –2 (ii) 300 GHz = 300 109 Hz
x 3 y –4 z 7 x –4 yz –5
(b) ÷ = 3 1011 Hz
x –5 y 2 x 7 y –3
= 3 105 106 Hz
x3 3
y–4 3
z7 3
x –4 –2
y –2 z –5 –2
= 3 105 MHz
= ÷
x –5 3
y 2 3
x 7 –2
y –3 –2 4. (i) a pm = 70 10–12 m
9 –12 21 8 –2 = 7 10–11 m
x y z x y z10
= ÷ (ii) b nm = 0.074 10–9 m
x –15 y6 x –14 y6
= 7.4 10–1 m
x9 y –12 z 21 x –14 y 6 (iii) a : b = 7.0 10–11 : 7.4 10–11
= –15 6
x y x y –2 z10
8
= 35 : 37
9 –14 –12 + 6
x y z 21 5. c Mm = 1500 106 m
= –15 + 8
x y6 – 2 z10 = 1.5 109 m
x –5 y –6 z 21 d Tm = 5.91 1012 m
=
x –7 y 4 z10 d 5.91 1012
100% = 100%
c 1.5 10 9
= x–5 + 7 y–6 – 4 z21 – 10
= 394 000%
= x2 y–10 z11
= 3.94 105 %
x 2 z11
= 6. (a) (2. 34 105) (7.12 10– 4)
y10
= ( 2 . 3 4 10x 5 )
ab n cn d bn + 2 ab n cn d cn + 3
(c) ÷ n+3 =
bc cd c bc cd bn + 2 ( 7 . 1 2 10x – 4 ) =
n n+n+3
a
b c d = 166.608
=
b1+ n + 2 c 2 d = 1.67 102 (to 3 s.f.)
a bn c2 n + 3 d (b) (5.1 10–7) (2.76 10–3)
=
b n + 3 c2 d
= ( 5 . 1 10x – 7 )
= a bn – (n + 3) c2n + 3 – 2
= a b–3 c2n + 1 ( 2 . 7 6 10x – 3 ) =
ac 2 n +1 = 1.4076 10 –9
=
b3 = 1.41 10–9 (to 3 s.f.)
(a + b) n
(a + b) n+3
(a + b)n abc (c) (13.4 104) ÷ (4 105)
(d) ÷ =
(a + b)n + 3
2 2
bc abc bc
= ( 1 3 . 4 10x 4 )
(a + b)n abc
=
(a + b)n + 3 bc 2 ÷ ( 4 10x 5 ) =
(a + b) a n
= 0.335
=
(a + b)n (a + b)3 c = 3.35 10–1
a 3 10 –4
= (d)
c(a + b)3 9 10 –8
= ( 3 10x – 4 ) ÷ ( 9 10x
Exercise 4C
– 8 ) =
1. (a) 85
! 300
= 8.53 104
= 3330 (to 3 s.f.)
(b) 52
! 700 000
= 5.27 107
= 3.33 103
(c) 0.000 !23 = 2.3 10–4
–8 (e) 2.54 103 + 3.11 104
(d) 0.000 000 094
! = 9.4 10
2. (a) 3
9.6 10 = 9600 = 2 . 5 4 10x 3
(b) 4 105 = 400 000 + 3 . 1 1 10x 4 =
(c) 2.8 10–4 = 0.000 28
= 33 640
(d) 1 10–6 = 0.000 001
= 3.36 104 (to 3 s.f.)
3. (i) 300 000 000 Hz = 3 108 Hz
(f) 6 105 – 3.1 107
= 3 102 106 Hz
= 3 102 MHz = 6 10x 5 – 3 . 1 10x 7 =

= –30 400 000


= –3.04 107

121
4.37 10 –4 + 2.16 10 –5 8. (a) 2P 4Q = 2(7.5 103) 4(5.25 104)
(g) = 3.15 109
3 10 –3
(b) Q – P = (5.25 104) – (7.5 103)
= ( 4 . 3 7 10x – 4
= 4.5 104
+ 2 . 1 6 10x – 5 ) 9. x + 8y = (2 10–3) + 8(7 10–4)
= 7.6 10–3
÷ 3 10x – 3 =
10. (a) MN = (3.2 106) (5.0 107)
= 0.153 (to 3 s.f.) = 1.6 1014
= 1.53 10–1 M 3.2 10 6
(b) =
2.4 10 –10 N 5.0 10 7
(h)
7.2 10 –6 – 3.5 10 –8 = 6.4 10–2
= 2 . 4 10x – 1 0 ÷ ( 7 . 2 M
11. R =
EI
10x – 6 – 3 . 5 10x – 8 ) = 6 10 4
=
–5 (4.5 10 8 ) (4 10 2 )
= 3.35 10 (to 3 s.f.)
7. (a) (1.35 10–4)3 = 3.33 10–7 (to 3 s.f.)
12. (i) 300 000 000 m/s = 3 108 m/s
= ( 1 . 3 5 10x – 4 ) xy 3 =
(ii) 778.5 million km = 778.5 106 km
–12
= 2.46 10 (to 3 s.f.) = 778.5 106 103 m
(b) 6(3.4 103)2 = 7.785 1011 m
= 6 ( 3 . 4 10x 3 ) xy 2 = Distance
Time taken =
Speed
= 69 360 000
7.785 1011
= 6.94 107 (to 3 s.f.) =
3 10 8
(c) 1.21 10 8 = 2595 seconds
= ( 1 . 2 1 10x 8 ) = = 43 minutes 15 seconds
13. (i) Distance travelled by rockets in 4 days = 4.8 105 km
= 1.1 104
4.8 10 5
3
Distance travelled by rockets in 12 days = 12
(d) 9.261 10 6 4
= 1.44 106 km
= shift 3
( 9 . 2 6 1 10x 6 ) =
Distance
(ii) Speed =
= 210 Time
= 2.1 102 4.8 10 5
=
2.3 10 –2 4.7 10 3 4
(e) = 1.2 105 km/day
2 10 3
4.8 10 7
= ( 2 . 3 10x – 2 4 . 7 Time taken =
1.2 10 5
10x 3 ) ÷ 2 10x 3 = = 400 days
14. (i) Increase in population = 5.45 108 – 4.20 108
= 0.05405 = 1.25 108
= 5.41 10–2 (to 3 s.f.) 1.17 10 9
(ii) = 2.15 (to 3 s.f.)
8 10 2 + 2.5 10 3 5.45 10 8
(f)
2 10 –2 – 3.4 10 –3 1.23 10 9
(iii) = 1.69 (to 3 s.f.)
7.28 10 8
= ( 8 10x 2 + 2 . 5 10x 3 )

÷ ( 2 10x – 2 – 3 . 4 10x

– 3 ) =

= 199 000 (to 3 s.f.)


= 1.99 105

122
Review Exercise 4 3 –2 2
2 9 23 2
(d) ÷ = ÷
1. (a) (a3b) (a4b3) = a3 + 4b1 + 3 5 2 53 9
= a7b4 8 22
6a 5 b 4 = ÷
125 92
(b) (6a5b4) ÷ (2a3b2) =
2a 3b 2
8 4
= 3a5 – 3b4 – 2 = ÷
125 81
= 3a2b2
8 81
(c) (–3a b ) = (–3) a3 3 b5
3 5 3 3 3 =
125 4
= –27a9b15 37
3 =1
2a 2 b 16a 5 8a 2 3b 3 16a 5 125
(d) ÷ = ÷
b3 ab 7 b3 3 ab 7 4. (a) By prime factorisation, 81 = 3 3 3 3 = 34.
4
8a 6 b 3 ab 7 81 = 4 3 3 3 3
=
b9 16a 5 =3
(b) By prime factorisation, 27 = 3 3 3 = 33
8a 6 +1b 3+ 7
= and 125 = 5 5 5 = 53.
16a 5 b 9
3
27 3
3 3 3
8a 7 b10 =
125 5 5 5
=
16a 5 b 9
3
=
a 7 – 5 b10 – 9 5
=
2 3
(c) 161.5 = 16 2
a2b
= = ( 16 )3
2
= 43
2. (a) 524 ÷ 58 = 524 – 8
= 64
= 516 3
1 1 –3 1 5
(b) = 3 = 5–3 (d) 1024 5
=
125 5 1024
1 3
(c) 5
5 = 55 1
=
5
1024
3. (a) 52 ÷ 5–1 50 = 52 – (–1) + 0 3
1
= 53 =
4
= 125
1
–2 –2 1 1 =
(b) 2 – 3 = 2 – 2 64
2 3 –4 4
1 1 3 x x4
= – 5. (a) = =
4 9 x 3 81
9 4 1
= – (b) 3 ÷ x–3 = 3 ÷ =3 x3 = 3x3
36 36 x3
5 6. (a) (x3y–2) (x–3y5) = x3 – 3 y–2 + 5
=
36 = x0 y3
–1
–2
1 1 = y3
(c) 3 + – (–3)0 = 2 + 3 – 1
3 3
(5 x 2 y 3 )0
1 (b) (5x2y3)0 ÷ (–2x–3y5)–2 =
= +3–1 (–2 x –3 y 5 )–2
9
1
1 =
=2 (–2 x –3 y 5 )–2
9
= (–2x–3y5)2
= (–2)2 x–3 2
y5 2

= 4x–6y10
4 y10
=
x6

123
4 3 8. (a) 4– 6 4x = 1
x2 x5 x8 x15
(c) ÷ = –12 ÷ 21 4– 6 + x = 40
y –3 y7 y y
–6 + x = 0
x8 y 21
= –12 x =6
y x15
(b) x–3 = 7
= x8 – 15y21 – (–12) 1
= x–7y33 =7
x3
y 33 3
7x = 1
=
x7 1
x3 =
(3x –2 y 5 )2 (–2 x 3 y –2 )2 9 x –4 y10 4 x 6 y –4 7
(d) =
4 6
9x y 9 x 4 y6 (c) 512 5–2 ÷ 5x = 25
512 – 2 – x = 52
36 x –4 + 6 y10 – 4
=
9 x 4 y6 510 – x = 52
10 – x = 2
36 x 2 y 6 x =8
=
9 x 4 y6 9. (a) 16a = 8
4 (24)a = 23
=
x2 24a = 23
3 1
3
7. (a) 5
p3 8p = p 5 (8p) 3 4a = 3
3 1 3
= p5 2p 3 a=
3+1
4
= 2p 5 3
(b) 2015b = 20150
14 b =0
= 2p 15

(p q ) (p q ) = (p q ) (p )
3 –2
3 10 c
–3 5 3
4 –2 –3 – 2 3 –2 4 3 –2 3 (c) = 0.01
(b) 5 5 3 5 3 5
q 3
10

= (p q ) (p q )
1
2 –2
5
12
5 –2 10c – 1 =
100
2 + 12 –2 –2 1
= p 5
q 5 10c – 1 = 2
10
22 – 12
= p q 5 5 10c – 1 = 10–2
22 c – 1 = –2
p5 c = –1
= 12
q5 2d – 6
(d) = 29
2 –2 2 –2 2
p3q 5
p3q 5
2d – 6 – 1 = 29
(p q )
(c) = 3
–1 –3
2 5
p –6 q 5
2d – 7 = 2 9
2 +6
d–7 =9
p3
= 3+2 d = 16
q5 5
10. (a) (6.4 106) (5.1 10–3)
20
p3 = ( 6 . 4 10x 6 ) ( 5 . 1
=
q
10x
( )
5
– 3 ) =
–1 2 –3 2 –5 10 –3 2
1
3
(d) p q 3
27( p q ) = p q 3
(27( p q )) 3
= 32 640

= p 3 q10
–5
(
3 p
–3 1
3
q
2 1
3
) = 3.26 104 (to 3 s.f.)
(b) (2.17 10–5) ÷ (7 104)
–5 2
= p q10 3
3 p –1q 3
= ( 2 . 1 7 10x – 5 )
– 5 –1 10 + 2
= 3p 3 3
q ÷ ( 7 10x 4 ) =
–10
–8 32
= 3.1 10
= 3p 3q 3

32
3q 3
= 8
p3

124
(c) (3.17 104) + (2.26 105) 13. (i) 149 597 870 700 nm = 1.496 1011 m (to 4 s.f.)
Distance
= ( 3 . 1 7 10x 4 ) (ii) Time =
Speed
+ ( 2 . 2 6 10x 5 ) = 1.496 1011
=
= 257 700 3 10 8 m/s
= 2.58 105 (to 3 s.f.) = 499 s (to 3 s.f.)
(d) (4.15 10–3) – (5.12 10–4) 14. (i) 1 Mm = 106 m
= ( 240 Mm = 240 106 m
4 . 1 5 10x – 3 )
= 2.4 108 m
– ( 5 . 1 2 10x – 4 ) = Distance
(ii) Speed of rocket =
= 3.64 10–3 (to 3 s.f.) Time
5.1 10 –6 – 2.34 10 5 =
1m
(e)
4.87 10 –3 + 9 10 –2 8000 ns
1m
= ( 5 . 1 10x – 6 – 2 . 3 4 =
8000 10 –9 s
10x 5 ) ÷ ( 4 . 8 7 10x = 1.25 105 m/s
Distance
– 3 + 9 10x – 2 ) = Time taken =
Speed
6
= –2.47 10 (to 3 s.f.) 2.4 10 8 m
=
8.43 10 + 6.8 10 7 8
1.25 10 5 m/s
(f)
(1.01 10 4 )3 = 1920 s
= ( 8 . 4 3 10x
7 + 6 . 8 15. (i) Mass of water molecule = 2(1.66 10–24) + (2.66 10–23)
= 2.99 10–23 g (to 3 s.f.)
10x 8 ) ÷ ( 1 . 0 1 10x 4 )
280
(ii) Approx. no. of water molecules =
x3 = 2.99 10 –23
= 9.36 1024 g (to 3 s.f.)
= 7.42 10–4 (to 3 s.f.)
11. (a) a – b = 110 000 000 – 12 100 000
Challenge Yourself
= 1.1 108 – 1.21 107
4 4
= 9.79 107 1. 23 = (2(3 ) = 281
3 3
24 = 2(4 ) = 264
(b) 3
ab = 3
(1.1 10 8 ) (1.21 10 7 )
So, 234 < 243 < 243 = 264 < 234 = 281.
3 4 4
= 1.331 1015 32 = 3(2 ) = 316
42 (42)
= 1.1 105 3 = 3 = 316
(c) 6c = 6(0.000 007)2
2
So, 316 < 324 < 342.
2 2
= 6(7 10–6)2 43 = 4(3 ) = 49
3 3
= 2.94 10–10 42 = 4(2 ) = 48
ac (1.1 10 8 ) (7 10 –6 ) So, 4 < 49 < 423 < 430.
8

(d) = 4
b 1.21 10 7 316 < 430 = 260 < 23 = 281
4
= 6.36 10–5 (to 3 s.f.) Hence 23 is the largest.
12. (i) 1 nm = 10–9 m 2. 31, 32, 33, 34, 35, 36, 37, …
7 nm = 7 10–9 m 3, 9, 27, 81, 243, 729, 2187, …
Circumference = d We can observe that the last digit of 3n are in the sequence:
= 3.142(7 10–9) 3, 9, 7, 1, 3, 9, 7, 1, …
= 2.20 10–8 m (to 3 s.f.) Since 2015 ÷ 4 gives a remainder of 3, hence the largest digit of
(ii) Area = r 2
32015 is 7.
2
7 10 –9
= 3.142
2
= 3.85 10–17 m2 (to 3 s.f.)

125
3. Let x = 2 + 2 + 2 + … .

( )
2

Then x2 = 2 + 2 + 2 +…

x2 = 2 + 2 + 2 +…
x2 =2+x
2
x –x–2 =0
(x – 2)(x + 1) =0
x = 2 or x = –1

Since 2 + 2 + 2 + … > 0, hence x = 2.

126
Revision Exercise A1 3. (a) 7x = 24 ÷ 42
–3 3 7x = 24 ÷ (22)2
1 2
1. (a)
2
=
1
=8 7x = 24 ÷ 24
7x =1
–3
(b) 0.04–1.5 = 0.04 2
7x = 70
3
1 2 x =0
= 1
0.04 (b) (2 y + 3) 2 = 5
1 2
= ( 25 )3 (2 y + 3) 2 = 52
3
=5 2y + 3 = 52
= 125 2y = 25 – 3
–1 –1
9 2 25 2 2y = 22
(c) 1 =
16 16 y = 11
1
16 2 4. (i) 3 – 5p > 17
=
25 3 – 5p – 3 > 17 – 3
–5p > 14
16
= 14
25 p<
–5
4 p < –2.8
=
5 (ii) p < –2.8
11 5 4
(d) 92.5 ÷ 27 3 = 9 2 ÷ 27 3

= ( 9 )5 ÷ ( 3 27 )4 –4 –3 –2.8 –2
= 35 ÷ 34 The greatest integer value of p is –3.
= 35 – 4 5. Let x be the number.
=3 Solving the two linear inequalities separately,
1 –1 1 2x – 5 < 12 and 3x – 1 > 12
(e) 9 2 – 0.36 2 = 9 –
0.36
2x – 5 + 5 < 12 + 5 3x – 1 + 1 > 12 + 1
1
=3– 2x < 17 3x > 13
0.6
17 13
1 x < x>
=1 2 3
3
1 1
2. (a) a5 ÷ a–2 = a5 – (–2) x <8 x >4
2 3
= a5 + 2
1 1
= a7 Representing x < 8 and x > 4 on a number line,
1
2 3
(b) b4 ÷ b b–7 = b4 ÷ b 2 b–7
4 – 1 + (–7 )
= b 2

–3.5 4 41 5 6 7 8 81 9
=b 3 2
1
= The solutions satisfying both inequalities lie in the overlapping
b 3.5
1 1
5 5 shaded region, i.e. 4 < x < 8 .
c –3 d c 2 d –2 3 2
(c) =
c 2 d –2 c –3 d The possible integer values of the number are 5, 6, 7 and 8.
= (c2 – (–3)d –2 – 1)5
= (c5d –3)5
= c25d –15
c 25
= 15
d

127
6. f(x) = 10x – 3 330
9. Time taken for Train A to travel from P to Q = h
Let y = 10x – 3 x
f(x) = y and f–1(y) = x 330
Time taken for Train B to travel from Q to P = h
y+3 x–5
x= 330 330 1
10 – =
y+3 x–5 x 2
–1
f (y) = 330 330
10 1
x(x – 5) – = x(x – 5)
–1 x+3 x–5 x 2
Hence f (x) =
10 1
x(330) – 330(x – 5) = x(x – 5)
–1 9+3 2
f (9) =
10 1 5
330x – 330x + 1650 = x2 – x
1 2 2
=1
5 1 2 5
1650 = x – x
–1 (–3) + 3 2 2
f (–3) =
10 1 2 5
x – x – 1650 = 0
=0 2 2
1 x2 – 5x – 3300 = 0
– +3
1 2 (x – 60)(x + 55) = 0
f–1 – =
2 10 x – 60 = 0 or x + 55 = 0
1 x = 60 x = –55
=
4 (rejected since x > 0)
7. (i) 1 nanometre = 10–9 metres 330
Time taken for Train A to travel from P to Q = = 5.5 h
8.8 nanometres = 8.8 10–9 metres 60
Circumference = d 330
Time taken for Train B to travel from Q to P = =6h
= 3.142(8.8 10–9) 60 – 5
= 2.76 10–8 m (to 3 s.f.) 10. (i) By Pythagoras’ Theorem,
(ii) Area = r2 AS2 = PA2 + PS2
2
8.8 10 –9 = y2 + 42
= 3.142
2 = y2 + 16
= 6.08 10–17 m2 (to 3 s.f.) By Pythagoras’ Theorem,
8. Let the side of the larger square be x cm and that of the smaller AB2 = AQ2 + BQ2
square be y cm. = (8 – y)2 + 32
4x – 4y = 100 = y2 – 16y + 64 + 9
x – y = 25 = y2 – 16y + 73
x = y + 25 — (1) (ii) By Pythagoras’ Theorem,
x – 3y2 = 325
2
— (2) BS2 = BR2 + RS2
Substitute (1) into (2): BS2 = (4 –3)2 + 82
(y2 + 25)2 – 3y2 = 325 BS2 = 12 + 82
y + 50y + 625 – 3y2 = 325
2
BS2 = 65 — (1)
50y + 300 – 2y2 = 0 By Pythagoras’ Theorem,
25y + 150 – y2 = 0 BS2 = AS2 + AB2
y2 – 25y – 150 = 0 BS2 = (y2 + 16) + (y2 – 16y + 73)
(y – 30)(y + 5) = 0 BS2 = 2y2 – 16y + 89 — (2)
y – 30 = 0 or y+5 =0 Substitute (1) into (2):
y = 30 y = –5 (rejected since y > 0) 2y2 – 16y + 89 = 65
When y = 30, 2y2 – 16y + 24 = 0
x = 30 + 25 = 55 y2 – 8y + 12 = 0 (Shown)
The lengths of a side of each of the squares are 30 cm and 55 cm.

128
(iii) y2 – 8y + 12 = 0
(y – 2)(y – 6) = 0
y–2 =0 or y–6 =0
y =2 y =6
y = 2 or y = 6
When y = 2,
AB2 = 22 – 16(2) + 73 = 45
AB = 45 cm
AS2 = 22 + 16 = 20
AS = 20 cm
5
Area of !ABS = AB AS
2
5
= 45 20
2
2
= 15 cm
When y = 6,
AB2 = 62 – 16(6) + 73 = 13
AB = 13 cm
AS2 = 62 + 16 = 52
AS = 52 cm
5
Area of !ABS = AB AS
2
5
= 13 52
2
2
= 13 cm
The possible values of the area of !ABS are 15 cm2 or
13 cm2.
11. (i) Area of picture = 160 cm2
(16 – 2x) (20 – 2x) = 160
320 – 32x – 40x + 4x2 = 160
320 – 72x + 4x2 = 160
160 – 72x + 4x2 = 0
4x2 – 72x + 160 = 0
x2 – 18x + 40 = 0 (shown)
2
(ii) x – 18x + 40 = 0
Comparing x2 – 18x + 40 = 0 with ax2 + bx + c = 0, we have
a = 1, b = –18 and c = 40.
–(–18) ± (–18)2 – 4(1)(40)
x=
2(1)
18 ± 164
=
2
= 15 (to 2 s.f.), 2.6 (to 2 s.f.)
x = 15 or x = 2.6
(iii) Width of border = 2.6 cm

129
Revision Exercise A2 5. Solving the two linear inequalities separately,
–3 ! 2q + 7 and 2q + 7 < 23
1. (a) (2ab2)3 = 23 a3 b2 3 = 8a3b6
2q + 7 " –3 2q + 7 – 7 < 23 – 7
(b) c3 c–2 ÷ c0 = c3 + (–2) – 0
2q + 7 – 7 " –3 – 7 2q < 16
= c3 – 2
2q " –10 q <8
=c
–2 2 q " –5
2 d
(c) = Representing q " –5 and q < 8 on a number line,
d 2
d2
=
4
–5 – 4 –3 –2 –1 0 1 2 3 4 5 6 7 8
2
(d) 2 ÷ 4e–3 = The solutions satisfying both inequalities lie in the overlapping
4e –3
2 shaded region, i.e. –5 ! q < 8.
= 2 4x 3 1
2 e –3 6. – !x–2
5 10 4
21–2
= 4x 3 3 1 3
e –3 – + !x–2 +
5 10 10 4 10
2 –1
= –3 4x 19
e !x–1
5 20
e3
= 4x 19
2 –x !x–1 –x
5 20
5 3
= e 1 19
2 – x ! –1
5 20
2. 5–6 ÷ 5p = 1250
19
5–6 – p = 1 –1
20
5–6 – p = 50 x "
1
–6 – p = 0 –
5
p = –6 3
1 1 x "9
= –6 = 26 = 64 4
2p 2 (a) Smallest prime value of x = 11
3. (a) 102x + 3 = 0.001 (b) Smallest integer value of x = 10
1 3
102x + 3 = (c) Smallest rational value of x = 9
1000 4
1 2
102x + 3 = 7. f(x) =
10 3 3x – 2
102x + 3 = 10–3 Let y = f(x)
2x + 3 = –3 f(x) = y and f–1(y) = x
2x = –6 2
y=
3x – 2
x = –3
y(3x – 2) = 2
1
(b) y–2 = 3xy – 2y = 2
81
1 1 3xy = 2 + 2y
= 2 + 2y
y2 81 x=
y2 = 81 3y
–1 2 + 2y
y = ± 81 f (y) =
3y
= ±9 2 1+ y
1
=
(c) (2z – 1) 3 = 2 3 y
1 3
(2z – 1) 3 = 23 2 1
= +1
3 y
2z – 1 = 8
2 1
2z = 9 Hence f–1(x) = +1
3 x
z = 4.5
4. (a) Largest possible value of x2 – y = 32 – 3 = 6 f–1(x x=0
2 1
(b) Smallest possible value of
x
=
1 f–1(6) = +1
y 6 3 6
7
=
9
130
1 10. (i) By Pythagoras’ Theorem,
8. (i) Area of the trapezium PQRS = RQ (SR + PQ)
2 XY 2 = OX 2 + OY 2
1 92 = (r + 5)2 + (4 + r)2
= (a + 1) (2a + 1 + 3a + 2)
2 81 = r2 + 10r + 25 + r2 + 8r + 16
1 81 = 2r2 + 18r + 41
= (a + 1)(5a + 3) cm2
2 2
2r + 18r – 40 = 0
1
(ii) (a + 1)(5a + 3) = 9 r2 + 9r – 20 = 0 (shown)
2
(ii) r2 + 9r – 20 = 0
(a + 1)(5a + 3) = 18
Comparing r2 + 9r – 20 = 0 with ax2 + bx + c = 0, we have
5a2 + 3a + 5a + 3 = 18
a = 1, b = 9 and c = –20.
5a2 + 8a – 15 = 0 (shown)
2
(iii) 5a + 8a – 15 = 0 –9 ± 9 2 – 4(1)(–20)
r=
Comparing 5a2 + 8a – 15 = 0 with ax2 + bx + c = 0, we have 2(1)
a = 5, b = 8 and c = –15. –9 ± 161
=
–8 ± 8 2 – 4(5)(–15) 2
x = = 1.84 (to 3 s.f.), –10.8 (to 3 s.f.)
2(5)
OX = 1.84 + 5 = 6.84 cm
–8 ± 364 OY = 4 + 1.84 = 5.84 cm
=
10 40
= 1.11 (to 2 d.p.), –2.71 (to 2 d.p.) 11. h
x
x = 1.11 or x = –2.71 10
(iv) PQ = 3(1.108) + 2 Time taken for the last 10 km = h
x–5
= 5.32 cm (to 3 s.f.) 40 10 40
+ =2
9. x h. x x–5 60
x + 2) h. 40 10 8
+ =
1 x x–5 3
of the tank while the
x+2 x(x – 5)
40
+
10
= x(x – 5)
8
1 x x–5 3
of the tank.
x 8
40(x – 5) + 10x = x(x – 5)
1 1 1 3
+ =
x x+2 1
20 8 2 40
60 40x – 200 + 10x = x – x
3 3
1 1 3 8 40
+ = 50x – 200 = x2 – x
x x+2 4 3 3
1 1 3 8 2 190
+ x(x + 2) = x(x + 2) x – x + 200 = 0
x x+2 4 3 3
2
1 1 3 8x – 190x + 600 = 0
x(x + 2) + x(x + 2) = x(x + 2)
x x+2 4 4x2 – 95x + 300 = 0
3 (x – 20)(4x – 15) = 0
(x + 2) + x = x(x + 2)
4 x – 20 = 0 or 4x – 15 = 0
3 3 x = 20 4x = 15
2x + 2 = x2 + x
4 2 3
3 2 1 x = 3 (rejected)
x – x–2 =0 4
4 2
3x2 – 2x – 8 = 0
20 km/h.
(x – 2)(3x + 4) = 0
x – 2 = 0 or 3x + 4 = 0
x =2 3x = – 4
1
x = –1
3
(rejected since x > 0)
The time taken by the larger pipe is 2 hours while by the smaller

131
Chapter 5 Application of Mathematics in Practical Situations
TEACHING NOTES
Suggested Approach
Teachers can get students to discuss examples of percentages, which are used in everyday life. Although the concepts covered
in this chapter are applicable to the real world, students might not have encountered the need to be familiar with them and
hence might not identify with the situations easily. Teachers should prepare more relatable material, such as advertisements on
discounted products, to allow students to appreciate the application of mathematics in practical situations.

Section 5.1: Profit and Loss


The definitions of profit and loss should be made clear to students, whereby:

Profit = Selling price – Cost price

Loss = Cost price – Selling price.

Teachers should also emphasise the difference between the expression of profit and loss as a percentage of the
cost price and the calculation of percentage gain or loss in terms of the selling price, that may occur in some
business transactions.
Profit Loss
100% 100%
Cost price Cost price

Profit Loss
Percentage gain = 100% Percentage loss = 100%
Selling price Selling price

Thus, teachers should remind students to read the questions carefully in order to ascertain the correct percentage
to report.

Section 5.2: Discount, Taxation and Commission


These real-world concepts would be useful for students when they start to work and plan their finances. However,
teachers should note that students may not encounter terms such as discount, GST and commission often, and
thus should explain the terms clearly before going through the topic.

The examples and figures used in the textbook are those applicable in Singapore, but the theory involved would be
relevant for use in other countries. Teachers may supplement the questions with current figures and tax brackets
in Pakistan, for instance, and further discuss with students the implications and importance of taxes in a country.

Section 5.3: Simple Interest and Compound Interest


Teachers may apply the prevailing interest rate in the region to an investment example using both simple interest
and compound interest, in order to illustrate the effect of the significant difference in the final amount. Teachers
should highlight to students that the computation of interest would be different depending on whether a simple
or compounded interest is charged, and hence students need to be careful when faced with such questions.

Section 5.4: Hire Purchase


To assert the real-world context of this section, teachers may show students some advertisements on posters or
other promotional material that feature the availability of a hire purchase alternative. Teachers can also suggest
to students to think about whether Shop A having a cheaper interest rate for the hire purchase of the exact same
item as compared to Shop B implies that a buyer should get the item from Shop A. There may be other hidden
terms and conditions that make Shop B’s item more attractive, such as a longer period of warranty for instance.

132
Section 5.5: Money Exchange
Teachers may wish to conduct a class exercise by asking students to find out the current exchange rates of the
local currency against prominent currencies such as the US Dollar, Euro, Sterling Pound, Japanese Yen etc.
compared to five to ten years ago. Based on the the trend, students can try to predict which currency would be a
good investment to make. Teachers can highlight to students that when exchanging money, the money changer
would offer both a buying and selling rate, and ensure that students are clear about the difference. Teachers can
then explain why the exchange of one currency to another, and back to the previous currency, will usually result
in a loss.

133
WORKED SOLUTIONS Investigation (Percentage Point)
Investigation (Discount, Service Charge and GST) 1. Increase from 5% to 7% = 7% – 5%
7 = 2%
1. GST as calculated by Kate = $14.40
100 2
Percentage increase from 5% to 7% = 100%
= $1.01 (to the nearest cent) 5
GST as calculated by the restaurant = 40%
7 10 2. Yes, Jun Wei is right in saying that the increase from 5% to 7% is
= $14.40 + $14.40 an increase of 40%. Please see solution to Question 1.
100 100
7 3. While the percentage increase from 5% to 7% is 40%, the difference
= ($14.40 + $1.44)
100 between 5% and 7% is 2%. The term ‘percentage point’ is used to
7 describe the difference between two percentages.
= $15.84
100
= $1.11 (to the nearest cent)
Class Discussion (Body Mass index)
2. GST is an acronym for Goods and Services Tax, thus the tax is also
imposed on the service charge, which is 10% of the subtotal. 1. –
3. If the discount is given before the service charge and GST are taken 2. Medical practitioners in Singapore make use of the BMI to determine
into account, the bill received will be as follows: which risk category you belong to as shown in the Table 5.2 of the
textbook.
Yummy Restaurant With this information, they will outline patients’ health risks with
Fish and Chips: $ 8.50 increasing obesity and provide the necessary advices such as to start
Chicken Chop: $ 9.50 to eat more healthily and increase the activity level to lose weight.
Subtotal: $ 18.00 3.
Discount: –$ 3.60 mobile phone charges and housing loan rate.
Subtotal: $ 14.40
Service Charge 10%: $ 1.44 Performance Task (Page 127)
GST 7%: $ 1.1088
Total: $ 16.9488
out the different interest rates as well as charges offered by the different
If the discount is given after the service charge and GST have been credit card companies such as DBS, UOB, OCBC, Citibank, Standard
taken into account, the bill received will be as follows:

Yummy Restaurant
Investigation (Simple Interest and Compound Interest)
Fish and Chips: $ 8.50
Chicken Chop: $ 9.50 PRT
1. Interest =
Subtotal: $ 18.00 100
Service Charge 10%: $ 1.80 1000 2 3
=
GST 7%: $ 1.386 100
Subtotal: $ 21.186 = $60
Discount: –$ 4.2372 Total amount after 3 years = $1000 + $60
= $1060
Total: $ 16.9488
2. 1st year: Principal P1 = $1000
4. If the discount is given before the service charge and GST are taken Interest I1 = $1000 2%
into account, both the service charge and GST will be calculated = $20
based on a smaller amount, i.e. $14.40, and thus the service charge Total amount at the end of the 1st year,
and GST will have already been discounted. A1 = P1 + I1
If the discount is given after the service charge and GST have = $1000 + $20
been taken into account, both the service charge and GST will be = $1020
calculated based on a greater amount, i.e. $18, and thus the discount 2nd year: Principal P2 = A1 = $1020
will be given on the service charge and GST as well. Interest I2 = $1020 2%
Hence, it makes no difference whether the discount is given before = $20.40
or after the service charge and GST are taken into account as the Total amount at the end of the 2nd year,
total bill will still be the same. A2 = P2 + I2
= $1020 + $20.40
= $1040.40

134
3rd year: Principal P3 = A2 = $1040.40 2. 88% of cost price = $16.50
Interest I3 = $1040.40 2% $16.50
1% of cost price =
= $20.808 88
Total amount at the end of the 3rd year, 100% of cost price =
$16.50
100
A3 = P3 + I3 88
= $1040.40 + $20.808 = $18.75
= $1061.21 (to the nearest cent) The cost price of the book is $18.75.
3. Interest offered by Bank B = $1061.21 – $1000
= $61.21 Practise Now 3
Difference in amount of interest offered by Bank A and Bank B 1800
Cost price of 1800 eggs = $1.20
= $61.21 – $60 12
= $1.21 = $180
Bank B offers a higher interest of $1.21.
133
= $180
Journal Writing (Page 136) 100
= $239.40
2. ‘Buying rate’ means the rate at which the money changer will buy
95
the foreign currency and ‘selling rate’ means the rate at which it will Number of eggs that the shopkeeper can sell = 1800
100
sell the currency.
= 1710
3. No, both ‘buying rate’ and ‘selling rate’ are independent. It depends
$239.40
on how much the money changer wants to earn from the deal. Selling price of each egg =
1710
= $0.14
Practise Now 1
$240 – $180 Practise Now 4
1. (a) Required percentage = 100%
$180
$60 $100 – $88
= 100% 1. Percentage discount = 100%
$180 $100
1 $12
= 33 % = 100%
3 $100
$6000 – $5000 = 12%
(b) Required percentage = 100%
$5000 94
2. Sale price of washing machine = $600
$1000 100
= 100%
$5000 = $564
= 20%
127 Practise Now 5
2. (a) Selling price of gold chain = $500
100
(i) 91% of marked price = $1274
= $635
94 $1274
(b) Selling price of car = $78 400 1% of marked price =
100 91
= $73 696 $1274
100% of marked price = 100
91
= $1400
Practise Now 2
The marked price of the laptop is $1400.
1. 135% of cost price = $1282.50 95
(ii) Sale price of laptop after a 5% discount = $1400
$1282.50 100
1% of cost price =
135 = $1330
$1282.50 96
100% of cost price = 100 Sale price of laptop after a further discount of 4% = $1330
135 100
= $950 = $1276.80
The cost price of the smartphone is $950. No, the sale price would not be $1274.

135
Practise Now 6 Practise Now 8
7
1. GST payable = $85
100
On the next $2000, tax payable = 7% of $2000
= $5.95 = $140
Total amount of money the man has to pay for article Total tax payable = $690
= $85 + $5.95
= $90.95
Practise Now 9
2. 107% of marked price = $642
$642 Total reliefs = $3000 + 2($4000) + $28 500 + $3500
1% of marked price =
107 = $43 000
$642 Taxable income = $284 000 – $43 000
100% of marked price = 100
107 = $241 000
= $600 Tax
The marked price of the printer is $600. First $200 000 : $20 750
$241 000
Next $41 000 at 18% : 18 $41 000 = $7380
Practise Now 7 100
Income tax payable = $20 750 + $7380
15
1. Discount = $6.90 = $28 130
100
= $1.035
Service charge = 10% (marked price – discount) Practise Now 10
10 2
= ($6.90 – $1.035) 1. Amount of commission the agent receives = $528 000
100 100
10 = $10 560
= $5.865
100 2. 3.5% of selling price = $25 375
= $0.5865 $25 375
GST payable = 7% (marked price – discount + service charge) 1% of selling price =
3.5
7 $25 375
= ($6.90 – $1.035 + $0.5865) 100% of selling price = 100
100 3.5
7 = $725 000
= $6.4515
100 The selling price of the piece of property is $725 000.
= $0.451 605
Total amount payable Practice Now 11
= marked price – discount + service charge + GST payable
= $6.90 – $1.035 + $0.5865 + $0.451 605 (a) Amount of interest the man has to pay at the end of 1 year
= $6.90 (to the nearest cent) 5.5
= $150 000
100
2. 117.7% of price after discount = $23.54
= $8250
$23.54
1% of price after discount = Amount of interest the man has to pay at the end of 3 years
117.7
$23.54 = $8250 3
100% of price after discount = 100 = $24 750
117.7
= $20 Total amount he owes the bank
The price of the set meal after discount is $20. = $150 000 + $24 750
80% of marked price = $20 = $174 750
$20 (b) Total amount of interest Shirley earns
1% of marked price =
80 = $6720 – $6000
$20 = $720
100% of marked price = 100
80 Amount of interest Shirley earns per last year
= $25 3
= $6000
The marked price of the set meal is $25. 100
= $180
Time taken for her investment to grow to $6720
$720
=
$180
= 4 years

136
Practise Now 12 Practise Now 13
1. P = $3000, R = 5, n = 4 20
(i) Downpayment = $900
At the end of 4 years, total amount accumulated is 100
R
n = $180
A = P 1+ Remaining amount = $900 – $180
100
4
= $720
5
= 3000 1 + Amount of interest Amirah owes at the end of 1 year
100
10
= $3646.52 (to the nearest cent) = $720
100
Total interest, I = A – P = $72
= $3646.52 – $3000 Amount of interest Amirah owes at the end of 4 years
= $646.51 = $72 4
2. (a) P = $1500, R = 2, n = 2 = $288
At the end of 2 years, total amount accumulated is Total amount to be paid in monthly instalments
n
R = $720 + $288
A = P 1+
100 = $1008
2
2 $1008
= 1500 1 + Monthly instalment = (4 years = 48 months)
100 48
= $21
= $1560.60 (to the nearest cent)
(ii) Total amount Amirah pays for the air conditioner
Total interest, I = A – P
= $1008 + $180
= $1560.60 – $1500
= $1188
= $60.60
(iii) She has to pay $(1188 – 900) = $288 more for buying the air
(b) Since interest is calculated monthly,
conditioner on hire purchase.
2 1
P = $1500, R = = , n = 2 12 = 24
12 6
At the end of 2 years, total amount accumulated is Practise Now 14
n
R 1. (a) (i) NZ$1 = S$0.9451
A = P 1+
100 NZ$2360 = S$0.9451 2360
24
1 = S$2230.44 (to the nearest cent)
6 (ii) 100 = S$2.8542
= 1500 1 +
100 2.8542
1 = S$
= $1561.16 (to the nearest cent) 100
Total interest, I = A – P 2.8542
25 600 = S$ 25 600
= $1561.16 – $1500 100
= $61.16 = S$730.68 (to the nearest cent)
3. Since interest is calculated yearly, (b) (i) S$1.7624 = 1
P = $4000, A = 4243.60, n = 2 1
S$1 =
At the end of 2 years, total amount accumulated is 1.7624
n 1
R S$5690 = 5690
A = P 1+ 1.7624
100
2
= 3229 (to the nearest )
R
$4243.60 = $4000 1 + (ii) S$4.2136 = 100 baht
100
2 100
4243.60 R S$1 = baht
= 1+ 4.2136
4000 100
100
R S$7460 = 7460
1+ = 1.0609 4.2136
100 = 177 046 (to the nearest baht)
R
= 1.0609 – 1 2. HK$100 = S$16.235
100
16.235
R = 100( 1.0609 – 1) HK$1 = S$
100
=3 16.235
HK$35 000 = S$ 35 000
The interest rate is 3%. 100
= S$5682.25

137
Amount of remaining Singapore dollars 2. (i) 35% of cost price = $280
= S$(5682.25 – 3500) $280
1% of cost price =
= S$2182.25 35
S$16.242 = HK$100 $280
100% of cost price = 100
100 35
S$1 = HK$ = $800
16.242
Amount of Hong Kong dollars they receive The cost price of the refrigerator is $800.
100 (ii) Selling price of refrigerator = $800 + $280
= HK$ 2182.25
16.242 = $1080
= HK$13 436 (to the nearest dollar) 3. Cost price of 5 kg of mixture = 2 $8 + 3 $6
= $16 + $18
Exercise 5A = $34
Selling price of 5 kg of mixture = 20 $2.55
1. (a)
= $51
= $5
$51 – $34
$5 Required percentage = 100%
Required percentage = 100% $51
$40
$17
= 12.5% = 100%
$51
(b) Loss = $600 – $480
1
= $120 = 33 %
3
$120 4. Selling price of one dozen of roses = 12 $1.20
Required percentage = 100%
$600
= $14.40
= 20%
$18 – $14.40
104 Required percentage = 100%
(c) Selling price = $88 000 $14.40
100
$3.60
= $91 520 = 100%
$14.40
= 25%
= $3520 5. 75% of price Devi buys from Shirley = $360
77.5
(d) Selling price = $5680 $360
100 1% of price Devi buys from Shirley =
75
= $4402 $360
Loss = $5680 – $4402 100% of price Devi buys from Shirley = 100
75
= $1278 = $480
1 Devi buys the fax machine from Shirley at $480.
(e) 117 % of cost price = $28.14
4 125% of cost price = $480
$28.14 $480
1% of cost price = 1% of cost price =
1 125
117
4
$480
$28.14 100% of cost price = 100
100% of cost price = 100 125
1
117 = $384
4
Shirley paid $384 for the fax machine.
= $24 6. Total number of apples Raj buys = 200 60
Cost price = $24 = 12 000
Cost price of 12 000 apples = 200 $28
= $4.14 = $5600
(f) 93% of cost price = $506.85
$506.85 price
1% of cost price =
93 180
= $5600
$506.85 100
100% of cost price = 100
93 = $10 080
= $545 85
Number of apples that Raj can sell = 12 000
Cost price = $545 100
Loss = $545 – $506.85 = 10 200
= $38.15 $10 080
Selling price per apple =
10 200
= $0.99 (to the nearest cent)
138
$1500 6. (a)
7. Cost price of each article =
300 On the next $5400, tax payable = 2% of $5400
= $5 = $108
120 Total tax payable = $108
Selling price of each of the 260 articles = $5
100 (b)
= $6 On the next $8400, tax payable = 3.50% of $8400
50 = $294
Selling price of each of the remaining 40 articles = $6
100
Total tax payable = $494
= $3
(c)
Selling price of articles = 260 $6 + 40 $3
On the next $32 000, tax payable = 7% of $32 000
= $1560 + $120
= $2240
= $1680
Total tax payable = $2790
$1680 – $1500
Required percentage = 100% (d)
$1500
On the next $25 000, tax payable = 11.5% of $25 000
$180
= 100% = $2875
$1500
= 12% Total tax payable = $6225
2.5
7. (a) Amount of commission the agent receives = $650 000
100
Exercise 5B
= $16 250
$580 – $464 (b) 2.5% of selling price = $12 000
1. Percentage discount = 100%
$580 $12 000
$116 1% of selling price =
= 100% 2.5
$580 $12 000
= 20% 100% of selling price = 100
2.5
88 = $480 000
2. Sale price of folding table = $45
100 The selling price of the house is $480 000.
= $39.60
3. (i) 7% of marked price = $49 8. (i) 87.5% of marked price = $700
$49 $700
1% of marked price = 1% of marked price =
7 87.5
$49 $700
100% of marked price = 100 100% of marked price = 100
7 87.5
= $700 = $800
The marked price of the television set is $700. The marked price of the air conditioner is $800.
(ii) Sale price of television set = $700 – $49 (ii) Sale price of air conditioner after a 10% discount
= $651 90
= $800
7 100
4. GST payable = $270
100 = $720
= $18.90 Sale price of air conditioner after a further discount of 2.5%
Total amount of money Ethan has to pay for microwave oven 97.5
= $270 + $18.90 = $720
100
= $288.90 = $702
5. 107% of marked price = $1391 No, the sale price would not be $700.
$1391 75
1% of marked price = 9. Price of seafood fried rice after discount = $9.50
107 100
$1391 = $7.125
100% of marked price = 100
107 117.7
= $1300 Total amount payable = $7.125
100
The marked price of the electronic gadget is $1300. = $8.39 (to the nearest cent)

139
10. Total reliefs = $3000 + $4000 + 2($5000) + $18 600 15. Let the amount of money be taxable at 7% be $x.
= $35 600 Amount of tax payable for $x at 7%
Taxable income = $185 000 – $35 600 = $1474 – $550
= $149 400 = $924
Tax Hence,
First $120 000 : $7950 7
$x = $924
$149 400 100
Next $29 400 at 15% : 15 $29 400 = $4410
100 0.07x = 924
Income tax payable = $7950 + $4410 x = 13 200
= $12 360 Total reliefs
11. Property tax payable yearly = $3000 + $2000 + $4(4000) + 2($5000) + $15 600 + $200
10 = $46 800
= $28 800
100 Gross annual income = $46 800 + $40 000 + $13 200
= $2880 = $100 000
Property tax payable for 6 months
$2880 Exercise 5C
=
2
1. Principal Interest Time Simple Amount
= $1440
12. Amount of commission Lixin receives = $1220 – $500 rate Interest
= $720 (a) $12 000 8% 7 years $6720 $18 720
4% of Lixin’s sales = $720 (b) $500 11% 4 years $220 $720
$720 (c) $300 9% 4 years $108 $408
1% of Lixin’s sales =
4 (d) $3000 4% 10 years $1200 $4200
$720 (e) $3600 5% 2 years $360 $3960
100% of Lixin’s sales = 100
4 (f) $1800 7% 18 months $189 $1989
= $18 000
(g) $4500 6% 2 years $540 $5040
Lixin’s sales for that month are $18 000.
(h) $1200 5% 18 months $90 $1290
13. Let the marked price of the sofa set be $x.
Sale price of sofa set = 90% 80% 75% $x 2. Amount of interest paid = $55
= 90% 75% 80% $x Let the sum of money borrowed be $x.
= 80% 90% 75% $x 5
$55 = 12% $x
= 80% 75% 90% $x 12
= 75% 90% 80% $x 12 5
x = 55 ÷ ÷
= 75% 80% 90% $x 100 12
= $0.54x = 1100
Thus the sale price of the sofa set is the same regardless of the order The sum of money was $1100.
Khairul chooses to arrange the 3 discounts. 2.25
3. Total interest = 25 $6400
100
14. 117.7% of price after discount = $10.13
= $3600
$10.13
1% of price after discount = 6
117.7 4. Annual interest on $800 investment = $800
100
$10.13
100% of price after discount = 100 = $48
117.7
= $8.61 (to the nearest cent) 7
Annual interest on $1200 investment = $1200
100
The price of the ramen after discount is $8.61.
= $84
82% of marked price = $8.61
Total annual interest = $48 + $84
$8.61
1% of marked price = = $132
82
6
$8.61 5. Amount of interest earned per year = $1250
100% of marked price = 100 100
82
= $10.50 (to the nearest cent) = $75
$750
The marked price of the ramen is $10.50. Time taken for interest to grow to $750 =
$75
= 10 years

140
$119 7 (b) P = $700, R = 11, n = 3
6. Interest rate = 100% ÷
$4800 12 At the end of 3 years, total amount accumulated is
n
1 R
=4 %
4 A = P 1+
100
7. Amount of interest Rui Feng has to pay at the end of 1 year 3
11
6 = 700 1 +
= $48 000 100
100
= $957.34 (to the nearest cent)
= $2880
Total interest, I = A – P
Amount of interest Rui Feng has to pay at the end of 2 years
= $957.34 – $700
= $2880 2
= $257.34
= $5760
3
Total amount of money he has to pay at the end of 2 years (c) P = $5000, R = 11 , n = 2
4
= $48 000 + $5760 At the end of 2 years, total amount accumulated is
= $53 760 n
R
8. Total amount of interest the man earns A = P 1+
100
= $18 900 – $16 800 2
3
= $2100 11
4
Amount of interest the man earns per last year = 5000 1 +
100
5
= $16 800 = $6244.03 (to the nearest cent)
100
Total interest, I = A – P
= $840
= $6244.03– $5000
Time taken for his investment to grow to $16 800
= $1244.03
$2100
= (d) P = $1200, R = 4, n = 3
$840
At the end of 3 years, total amount accumulated is
1
= 2 years n
2 R
A = P 1+
9. P = $5000, R = 8, n = 3 100
3
At the end of 3 years, total amount accumulated is 4
n = 1200 1 +
R 100
A = P 1+ = $1349.84 (to the nearest cent)
100
3 Total interest, I = A – P
8
= 5000 1 + = $1349.84 – $1200
100
= $6298.56 = $149.84
Total interest, I = A – P 1
(e) P = $10 000, R = 7 , n = 2
= $6298.56 – $5000 2
At the end of 2 years, total amount accumulated is
= $1298.56
n
10. (a) P = $450, R = 10, n = 2 R
A = P 1+
At the end of 2 years, total amount accumulated is 100
2
R
n 1
A = P 1+ 7
2
100 = 10 000 1 +
2 100
10
= 450 1 + = $11 556.25
100
Total interest, I = A – P
= $544.50
= $11 556.25 – $10 000
Total interest, I = A – P
= $1556.25
= $544.50 – $450
1
= $94.50 11. P = $5000, R = 5 , n = 3
4
At the end of 3 years, total amount accumulated is
n
R
A = P 1+
100
3
1
5
4
= 5000 1 +
100
= $5829.57 (to the nearest cent)

141
12. Amount of interest from end March to June 17. (a) Since interest is calculated monthly,
3 3 5.68
= $600 P = $15 000, R = , n = 6 12 = 72
100 12 12
= $4.50 At the end of 6 years, total amount accumulated is
n
Amount of interest from end June to December R
3 A = P 1+
6 100
= $(600 + 400)
100 12 72
5.68
= $15
12
Total amount in the bank at the end of the year = 15 000 1 +
100
= $600 + $4.50 + $400 + $15 = $21 074.13 (to the nearest cent)
= $1019.50 (b) Since interest is calculated half-yearly,
13. Initially, at 3.5% interest rate, interest received 5.68
3.5 1 P = $15 000, R = = 2.84, n = 6 2 = 12
= $6400 2
100 2 At the end of 6 years, total amount accumulated is
= $112 n
R
At new 4% interest rate, interest received A = P 1+
100
4 1 12
= $6400 2.84
100 2 = 15 000 1 +
= $128 100
= $20 991.14 (to the nearest cent)
Difference in amount of interest = $128 – $112
18. Since interest is calculated yearly,
= $16
P = $5000, A = $5800, n = 5
14.
At the end of 5 years, total amount accumulated is
1
7 n
4 2 R
= $4000 A = P 1+
100 100
= $580 R
5

Interest received during next 5 years $5800 = $5000 1 +


100
7.6 R
5
= $4000 5 5800
100 = 1+
5000 100
= $1520
R
Total amount at the end of 7 years 1+ = 5 1.16
100
= $4000 + $580 + $1520 R
= $6100 = 5 1.16 – 1
100
15. Let the sum of money deposited by Daniel be $x. R = 100( 5 1.16 – 1)
3 1
3 3 = 3.01 (to 3 s.f.)
4 x – 2 x = 50
The interest rate is 3.01%.
100 100
1 19. Since interest is calculated quarterly,
4 x = 50 4.2
P = $96.60, R = = 1.05, n = 1 4 = 4
100 4
x = 20 000
n
The sum of money Daniel deposits is $20 000. R
A = P 1+
16. Interest received at 2.75% interest rate 100
2.75 1.05
4
= $20 000 P + $96.60 = P 1 +
100 100
= $550
P + $96.60 = 1.01054P
Interest received at x% interest rate = $550 – $50
(1.01054 – 1)P = $96.60
= $500
96.60
$500 P =$
New simple interest, x% = 100% 1.0105 4 – 1
$20 000
= $2264.09 (to the nearest cent)
= 2.5%
x = 2.5

142
20. Since interest is calculated monthly, Exercise 5D
1
12 1. (a) (i) Amount paid by hire purchase
P = $800, R = 2 = 25 , n = 12
12 24 = $(40 10) + $50
At the end of 1 year, total amount accumulated is = $450
n
R Additional amount paid = $450 – $360
A = P 1+ = $90
100
2 $90
25 (ii) Percentage of cash price = 100%
$360
24
= 800 1 + = 25%
100
= $905.93 (to the nearest cent) (b) (i) Amount paid by hire purchase
Total interest, I = A – P = $(75 12) + $150
= $905.93 – $800 = $1050
= $105.93 Additional amount paid = $1050 – $900
21. Since interest is calculated daily, = $150
2 $150
P = $9000, R = ,n=3 (ii) Percentage of cash price = 100%
365 $900
At the end of 3 days, total amount accumulated is 2
= 16 %
n 3
R
A = P 1+ (c) (i) Amount paid by hire purchase
100
3 = $(500 36) + $10 000
2 = $28 000
365
= 9000 1 + Additional amount paid = $28 000 – $25 000
100
= $3000
= $9001.48 (to the nearest cent)
$3000
22. (i) Kate should invest in Company B since the interest earned is Percentage of cash price = 100%
$25 000
higher.
= 12%
(ii) For Company A,
2. (a) (i) Amount paid by hire purchase
PRT
I = 10
100 = $(9 24) + $ 200
100
8000 4.9 4
= = $216 + $20
100
= $236
= $1568
(ii) Additional amount = $236 – $200
For Company B,
= $36
Since interest is calculated half-yearly,
Percentage saved by paying cash
4.8
P = $8000, R = = 2.4, n = 4 2 = 8 36
2 = 100%
At the end of 6 years, total amount accumulated is 200
n = 18%
R
A = P 1+ (b) (i) Amount paid by hire purchase
100
8 15
2.4 = $(18 20) + $ 350
= 8000 1 + 100
100 = $360 + $52.50
= $9671.41 (to the nearest cent) = $412.50
Total interest, I = A – P (ii) Additional amount = $412.50 – $350
= $9671.41 – $8000 = $62.50
= $1671.41 Percentage saved by paying cash
Difference in interest earned = $1671.41 – $1568 62.50
= $103.41 = 100%
350
= 17.9% (to 3 s.f.)

143
(c) (i) Amount paid by hire purchase (ii) Difference as percentage of cash price
25 1200
= $(52 30) + $ 1600 = 100%
100 8000
= $1560 + $400 = 15%
= $1960 (c) (i) Downpayment = $200
(ii) Additional amount = $1960 – $1600 Remaining amount = $1200 – $200
= $360 = $1000
Percentage saved by paying cash Amount of interest owed at the end of 1 year
360 15
= 100% = $1000
1600 100
= 22.5% = $150
3. (a) (i) Downpayment = $100 1
Amount of interest owed at the end of 1 years
Remaining amount = $800 – $100 3
= $700 1
= $150 1
Amount of interest owed at the end of 1 year 3
8 = $200
= $700 Total amount to be paid in monthly instalments
100
= $56 = $1000 + $200
Total amount to be paid in monthly instalments = $1200
= $700 + $56 $1200
Monthly instalment = $
= $756 16
$756 = $75 (to the nearest cent)
Monthly instalment = (ii) Difference as percentage of cash price
12
= $63 200
= 100%
(ii) Difference as percentage of cash price 1200
56 2
= 100% = 16 %
800 3
= 7% $(2198 – 1798)
4. (a) Percentage discount = 100%
$2198
(b) (i) Downpayment = $3200
= 18.2% (to 1 d.p.)
Remaining amount = $8000 – $3200
(b) Hire purchase price = $55 38
= $4800
= $2090
Amount of interest owed at the end of 1 year
Difference = $2198 – $2090
10
= $4800 = $108
100
= $480 (c) Total amount of interest = $2090 – $1798
1 = $292
Amount of interest owed at the end of 2 years Amount of interest at the end of 1 year
2
1 $292
= $480 2 = 12
2 38
= $1200 = $92.21 (to the nearest cent)
Total amount to be paid in monthly instalments Let the rate of simple interest be x%.
= $4800 + $1200 x
$92.21 = $1798
= $6000 100
$6000 92.21(100)
Monthly instalment = $ x=
30 1798
= $200 = 5.13 (to 3 s.f.)
The rate of simple interest charged for hire purchase is 5.13%.

144
5. (i) Downpayment 2. (a) £1 = ¤1.19
15 £320 = 320 ¤1.19
= $3200
100 = ¤380.80
= $480 (b) ¤1.19 = £1
Remaining amount 1
¤1 = £
= $3200 – $480 1.19
= $2720 1
¤956 = £ 956
Amount of interest the man owes at the end of 1 year 1.19
9.5 = £803 (to the nearest £)
= $2720
100 3. (a) S$1.70 = US$1
= $258.40 1
S$1 = US$
Amount of interest the man owes at the end of 2 years 1.70
= $258.40 2 1
S$8500 = US$ 8500
= $516.80 1.70
Total amount to be paid in monthly instalments = US$5000
= $2720 + $516.80 (b) After one year, amount in the bank
= $3236.80 104
= US$5000
Monthly instalment 100
= US$5200
$3236.80
= (2 years = 24 months) US$1 = S$1.65
24
= $134.87 (to the nearest cent) US$5200 = 5200 S$1.65
(ii) Total amount the man pays for the computer system = S$8580
= $3236.80 + $480 Amount made from investment
= $3716.80 = S$(8580 – 8500)
(iii) He has to pay $(3716.80 – 3200) = $516.80 more for buying = S$80
the computer system on hire purchase. 4. (a) US$1 = S$1.4103
25 US$765 = 765 S$1.4103
6. Downpayment = $x = S$1078.88 (to the nearest cent)
100
= $0.25x (b) Rp 100 = S$0.0111
Remaining amount = $x – $0.25x 0.0111
Rp 1 = S$
= $0.75x 100
Amount of interest the man owes at the end of 30 months 0.0111
Rp 2 560 000 = S$ 2 560 000
100
12 30
= $0.75x = S$284.16
100 12
= $0.975x 5. (a) S$1.0373 = A$1
Total amount to be paid in monthly instalments 1
S$1 = A$
= $52 30 1.0373
1
= $1560 S$350 = A$ 350
1.0373
Hence 0.975x = 1560
= A$337 (to the nearest A$)
x = 1600
(b) S$2.8250 = 100
100
Exercise 5E S$1 =
2.8250
1. (a) US$1 = 34.77B 100
S$5670 = 5670
US$150 = 150 34.77B 2.8250
= 5215.50B = 200 708 (to the nearest )
(b) 34.77B = US$1
1
1B = US$
34.77
1
1617B = US$ 1617
34.77
= US$46.51 (to the nearest cent)

145
6. NZ$100 = S$94.85 $60 000 – $57 000
3. Percentage discount = 100%
94.85 $60 000
NZ$1 = S$
100 $3000
= 100%
94.85 $60 000
NZ$3200 = S$ 3200
100 = 5%
= S$3035.20 4. Marked price of 200 textbooks = 200 $20
Amount of remaining Singapore dollars = $4000
= S$(3035.20 – 2560.20) 93.5
Amount of money the school has to pay = $4000
= S$475 100
S$97.65 = NZ$100 = $3740
100 15
S$1 = NZ$ 5. (a) Value-added tax payable = $20
97.65 100
Amount of New Zealand dollars they receive = $3
100
= NZ$ 475
97.65 = $26 – $23
= NZ$486.43 (to the nearest cent) = $3
(b) (i) Price the retailer buys each bread toaster at from the
Review Exercise 5 manufacturer
1. $36 120
= $20
100
= $432
= $24
Selling price of cameras = $1800 + $432
25
= $2232 Value-added tax payable = $24
100
$432
Required percentage = 100% = $6
$2232
Price a customer has to pay for a bread toaster
11
= 19 % = $24 + $6 + $3
31
2. 125% of cost price of Item A = $48 = $33

$48 130
1% of cost price of Item A = (ii) Selling price of a bread toaster = $33
125 100
= $42.90
$48
100% of cost price of Item A = 100
125
= $38.40 = $42.90 – $24 – $6
The cost price of Item A is $38.40. = $12.90
80% of cost price of Item B = $48
$48 = 25 $12.90
1% of cost price of Item B =
80 = $322.50
$48 6. 117.7% of price after discount = $7.94
100% of cost price of Item B = 100
80 $7.94
1% of price after discount =
= $60 117.7
The cost price of Item B is $60. $7.94
100% of price after discount = 100
Total cost price of the two items = $38.40 + $60 117.7
= $98.40 = $6.75 (to the nearest cent)
Total selling price of the two items = 2 $48 The price of the noodles after discount is $6.75.
= $96 90% of marked price = $6.75
Net loss = $98.40 – $96 $6.75
1% of marked price =
= $2.40 90
$2.40 $6.75
Required percentage = 100% 100% of marked price = 100
$98.40 90
= $7.50 (to the nearest cent)
18
=2 % The marked price of the noodles is $7.50.
41

146
7. Total reliefs = $3000 + 2($5000) + $16 000 + $750 11. Since interest is calculated half-yearly,
= $29 750 4
l = $5800, R = = 2, n = 3 2 = 6
Taxable income = $80 000 – $29 750 2
= $50 250
n
Tax R
A = P 1+
First $40 000 : $550 100
$80 000 2
6
7
Next $10 250 at 7% :
100
$10 250 = $717.50 P + $5800 = P 1 +
100
Income tax payable = $550 + $717.50 P + $5800 = 1.026P
= $1267.50 (1.026 – 1)P = $5800
8. Amount of commission the agent receives 5800
P =$
5 2.25 1.02 6 – 1
= $50 000 + ($240 000 – $50 000)
100 100 = $45 972 (to the nearest dollar)
5 2.25 12. (i) Downpayment
= $50 000 + $190 000
100 100 15
= $450
= $2500 + $4275 100
= $6775 = $67.50
9. Amount of interest earned for $600 at the end of 2011 Remaining amount
3 = $450 – $67.50
= $600
100 = $382.50
= $18 Amount of interest owed at the end of 1 year
Amount of interest earned for $1000 at the end of 2012 12
= $382.50
3 100
= $1000
100 = $45.90
= $30 Amount of interest owed at the end of 18 months
Total amount Khairul has in the bank at the end of 2013 18
= $45.90
= $1000 + $18 + $30 + $30 12
= $1078 = $68.85
10. (a) Since interest is calculated monthly, Total amount to be paid in monthly instalments
4.12 = $382.50 + $68.85
P = $15 000, R = , n = 3 12 = 36
12 = $451.35
At the end of 3 years, total amount accumulated is Monthly instalment
n
R $451.35
A= P 1+ =
100 18
4.12
36 = $25.08 (to the nearest cent)
2 (ii) Total amount the man pays for the printer
= 15 000 1 + = $450 + $68.85
100
= $16 969.85 (to the nearest cent) = $518.85
(b) Since interest is calculated half-yearly, 13. Number of litres of petrol that the car used
4.12 1850
P = $15 000, R = = 2.06, n = 3 2 = 6 =
12
2
At the end of 3 years, total amount accumulated is 1
= 154 litres
n 6
R
A= P 1+ Amount of euros spent on petrol
100
1
2.06
6 = 154 ¤0.80
= 15 000 1 + 6
100 = ¤123.33
= $16 952.14 (to the nearest cent) £100 = ¤116.50
116.50
£1 = ¤
100

147
Amount of remaining euros
116.50
=¤ 75
100
= ¤87.375
Total amount they spent for the short trip
= ¤3500 + ¤123.33 + ¤87.375
= ¤3711
¤115 = £100
100
¤1 = £
115
Amount of Sterling pounds that they exchanged
100
=£ 3711
115
= £3227 (to the nearest pound)

Challenge Yourself
7
(i) GST paid by Lixin = $500
100
= $35
7
(ii) GST paid by Jun Wei = $500
107
= $32.71 (to the nearest cent)
(iii) GST on $500 is $35 which is the same answer as in (i).
The shopkeeper is not complaining about it because he rather pays
a GST of $32.71 than a GST of $35 to the government.
(iv) The amount paid by each customer at Shops B and C is $500. As far
as the government is concerned, this amount must be inclusive of
GST. Another way of looking at this is to ask how the government
can keep track of the shops which absorb GST and charge them a
different GST amount. All the shops will tell the government that the

lower
amount is inclusive of GST regardless of whether the shops charge
or absorb GST.
(v) Yes, it makes a difference. The difference is the original selling
price of the TV before the government announces that they will
charge GST. Shop C has been selling the TV for $500 and decides
to absorb GST after the announcement, so it still sells the TV for
$500 (inclusive of GST). If Shop C decides not to absorb GST, they
will sell the TV for $500 (before GST) or $535 (inclusive of GST),
just like what Shop A does. Since Shop B has been selling the TV for
about $467.29 and decides to charge GST after the announcement,
it sells the TV for $500 (inclusive of GST) now.

148
Chapter 6 Coordinate Geometry
TEACHING NOTES
Suggested Approach
Teachers should revise with the students on what they have learnt about functions and linear graphs in Book 1 before teaching
this chapter. As students may have difficulty distinguishing equations parallel to the x- and y- axes, or may incorrectly refer to
the equation of the x-axis as x = 0 and the equation of the y-axis as y = 0, teachers should highlight these common mistakes to
the students and give them ample practice so that they can better grasp the concepts in coordinate geometry.

Section 6.1: Gradient of a Straight Line


Students have learnt in Book 2 that the gradient of a straight line is the ratio of the vertical change to the
horizontal change. Teachers can get the students to recall this and build upon what they have learnt to find the
formula for gradient given two points on a line (see Class Discussion: Finding the Gradient of a Straight Line).

In this section, the students will learn that the gradient of a straight line can be positive, negative, zero or undefined
(see Investigation: Gradient of a Straight Line). Teachers should not only teach students how to find the gradient
given two points, but also how to use the gradient to determine the coordinates of a point on the line.

Section 6.2: Length of Line Segment


Teachers should recall Pythagoras’ Theorem with the students before showing them how the formula for the
length of any line segment PQ is given by (x2 – x1 )2 + (y2 – y1 )2 is proven. The formula is used to evaluate
the distance between two given points P(x1, y1) and Q(x2, y2). While students might ask whether the length of any
line segment PQ can also be given by (x1 – x2 )2 + (y1 – y2 )2 , teachers can get the students to check if that is
true (see Thinking Time on page 150 of the textbook).

Section 6.3: Equation of a Straight Line


Students have learnt in Book 1 how the graph of a straight line in the form y = mx + c changes when either m or
c varies. Teachers should recap the equations of a vertical line and a horizontal line and introduce the gradient-
intercept form of the equation of a straight line, y = mx + c.

Teachers can guide the students along and show them the different cases and ways of finding the equation of a
straight line (see Journal Writing on page 158 of the textbook).

Section 6.4: Parallel and Perpendicular Lines


Teachers should demonstrate to students how the gradients of parallel and perpendicular lines are related, and
prove this with the use of trigonometric ratios (see Class Discussion: Parallel and Perpendicular Lines). Once
students have understood the properties of parallel and perpendicular lines, they should be able to see that
obtaining the equation of a line is possible when given the gradient of either a parallel or perpendicular line as
well as the coordinates of a point on the line.

Challenge Yourself
Students should observe that the question states that AB is parallel to OC. Hence, they should equate the gradients
of AB and OC together to find the proof.

149
WORKED SOLUTIONS 5. (a) When y2 – y1 > 0 and x2 – x1 < 0, the sign of the gradient is
Class Discussion (Finding the Gradient of a Straight Line) negative.
(b) When y2 – y1 < 0 and x2 – x1 > 0, the sign of the gradient is
1. (i) In Fig. 6.3(a), negative.
vertical change
Gradient of AB = (c) When the signs of y2 – y1 and x2 – x1 are the same, the sign of
horizontal change
the gradient is positve.
6
= (d) When y2 – y1 = 0, i.e. y1 = y2, the gradient of the line is zero.
3
(e) When x2 – x1 = 0, i.e. x1 = x2
=2
In Fig. 6.3(b),
vertical change
Thinking Time (Page 150)
Gradient of AB =
horizontal change (x2 – x1 )2 + (y2 – y1 )2 = [–(– x2 + x1 )]2 + [–(– y2 + y1 )]2
–3
= = (x1 – x2 )2 + (y1 – y2 )2
1
= –3 Yes, they are equal.
(ii) The gradient of each line segment is the same. This is because
the points lie on the same line segment. Journal Writing (Page 158)
y – y1 y – y2
2. Gradient of AB = 2 or 1 Case 1: Given the gradient m and the y-intercept c, substitute m and c
x2 – x1 x1 – x2
into y = mx + c
y2 – y1 7–4 3
3. (a) Gradient = = Case 2: Given the gradient m and the coordinates of a point (a, b),
x2 – x1 3 – (–1) 4
substitute the coordinates of the point and the gradient into
y2 – y1 –11 – (–3) –8 4
(b) Gradient = = = =– y = mx + c y-intercept. Then, substitute m and c into
x2 – x1 2 – (–4) 6 3
y = mx + c
y2 – y1 3– 3 0 Case 3: Given the coordinates of two points (a, b) and (c, d
(c) Gradient = = = =0
x2 – x1 –4 – 6 –10 gradient of the line using the coordinates of the two points.
y2 – y1 8 – (–1) 9 Substitute the coordinates of the point and the gradient into
(d) Gradient = = =
x2 – x1 2–2 0 y = mx + c y-intercept. Then, substitute m and c into
y = mx + c
Thinking Time (Page 145)
Thinking Time (Page 161)
y2 – y1 –(– y2 + y1 ) y – y2
= = 1
x2 – x1 –(– x2 + x1 ) x1 – x2 5–3
1. Gradient of AB =
Yes, they are equal. 3– 2
2
=
Investigation (Gradient of a Straight Line) 1
=2
Coordinates Gradient of Sign of 9–5
y2 – y1 x2 – x1
of End-points Line segment Gradient Gradient of BC =
5–3
4 5–1 0 – (–2) 4
(a) A(–2, 1) and B(0, 5) =2 positive =
2 =4 =2 2
=2
3 = –1 8–5 4–7
9–3
(b) C(7, 5) and D(4, 8) negative
–3 =3 = –3 Gradient of AC =
5–2
6
–3 3 3–6 –4 – (–2) =
(c) E(–2, 6) and F(–4, 3) = positive 3
–2 2 = –3 = –2
=2
0 =0 1–1 The three points are collinear since the gradients are equal.
(d) G(1, 1) and H(3, 1) zero 3–1=2
2 =0 2. The points are collinear.

0 6–3 –4 – (–4)
(e) I(–4, 3) and J(–4, 6)
3 =3 =0

Table 6.1

150
Class Discussion (Parallel and Perpendicular Lines) Practise Now 2
1. Case 1: (i) 1 and 2 are equal (corresponding !s); the gradients y2 – y1
Gradient of line, = –3
of these lines are equal, i.e. m1 = m2. x2 – x1
(ii) Gradients are equal, thus l1 and l2 are parallel. h – (–9)
= –3
2. Case 2: (i) m1 = tan –3 – 4
SR h+9
= = –3
PS –7
(ii) m2 = tan h+9 = –3(–7)
sin h+9 = 21
= h = 12
cos
sin(180° – )
=
– cos(180° – ) Practise Now 3
= –tan (180° – )
(a) Length of line segment CD = (x2 – x1 ) + (y2 – y1 )
2 2
PS
=–
SR
= (3 – 6) + (–2 – 2)
2 2

SR PS
m1m2 = –
= (–3) + (–4)
2 2
PS SR
= –1
1 = 9 + 16
m1 = – (shown)
m1 = 25
(iii) If l1 and l2 are perpendicular, m1m2 = –1.
= 5 units
(iv) If m1m2 = –1, l1 and l2 are perpendicular.
(b) Length of line segment MN = (x2 – x1 ) + (y2 – y1 )
2 2

Class Discussion (Perpendicular Lines)


= [6 – (–1)] + (–4 – 5)
2 2

2 2 2 2 2 2 2 2
Yes. Note that AB = 2 + 6 , BC = 12 + (k – 1) and AC = 10 +
= 7 + (–9)
2 2
(k + 5). Thus, 102 + (k + 5)2 = 4 + 46 + 144 + (k – 1)2. Solving this
equation would give k = 5. = 49 + 81

= 130
Practise Now 1
= 11.4 units (to 3 s.f.)
y2 – y1
(a) Gradient of CD =
(c) Length of line segment PQ = (x2 – x1 ) + (y2 – y1 )
2 2
x2 – x1
3–1
= (8 – 2) + (7 – 7)
2 2
=
6–3
2 = 62 + 02
=
3
y – y1 = 36 + 0
(b) Gradient of HK = 2
x2 – x1 = 36
–2 – (–7) = 6 units
=
0–5
5
=
–5
= –1
y – y1
(c) Gradient of MN = 2
x2 – x1
1–1
=
16 – (–4)
0
=
20
=0

151
Practise Now 4 Since CF = DF,

(a) Let the coordinates of the point E be (0, k). ( p – 4)2 + 1 = ( p + 2)2 + 49

CE = (x2 – x1 ) + (y2 – y1 ) ( ( p – 4)2 + 1 )2= ( ( p + 2)2 + 49 )2


2 2

(p – 4)2 + 1 = (p + 2)2 + 49
= (0 – 4)2 + [k – (–1)]2
p – 8k + 16 + 1 = p2 + 4k + 4 + 49
2

= (–4)2 + (k + 1)2 p2 – 8k + 17 = p2 + 4k + 53
–8k – 4k = 53 – 17
= 16 + (k + 1)2
–12k = 36
DE = (x2 – x1 )2 + (y2 – y1 )2 k = –3
Coordinates of F are (–3, 0)
= [0 – (–2)] + (k – 7)
2 2
1
Area of !OEF = base height
= 2 + (k – 7)
2 2
2
1
= OF OE
= 4 + (k – 7)
2
2
Since CE = DE, 1 1
= 3 2
2 4
16 + (k + 1)2 = 4 + (k – 7)2
3
= 3 units2
( 16 + (k + 1)2 )2= ( 4 + (k – 7)2 )2 8

16 + (k + 1)2 = 4 + (k – 7)2 y
16 + k2 + 2k + 1 = 4 + k2 – 14k + 49
k2 + 2k + 17 = k2 – 14k + 53 7

2k + 14k = 53 – 17 6
16k = 36 5
36 4
k =
16 3
1
2 E 0, 2 1
k =2 4
4
1
1 F(–3, 0)
Coordinates of E are 0, 2 x
4 –3 –2 –1 O 1 2 3 4
(b) Let the coordinates of the point F be (p, 0). –1
C(4, –1)
CF = (x2 – x1 ) + (y2 – y1 )
2 2

= ( p – 4)2 + [0 – (–1)]2 Practise Now 5

= ( p – 4)2 + 12 1. DE2 = (2 – 6)2 + (3 – 1)2


= (– 4)2 + 22
= ( p – 4)2 + 1 = 16 + 4
DF = (x2 – x1 ) + (y2 – y1 )
2 2 = 20
EF2 = (–1 – 2)2 + (–3 – 3)2
= [ p – (–2)] + (0
2
7)2 = (–3)2 + (–6)2
= 9 + 36
= ( p + 2) + (–7)
2 2

= 45
= ( p + 2)2 + 49 DF2 = (–1 – 6)2 + (–3 – 1)2
= (–7)2 + (– 4)2
= 49 + 16
= 65
Since DE2 + EF2 = 20 + 45
= 65
= DF2,
the triangle is a right-angled triangle with "DEF = 90°. (Converse
of Pythagoras’ Theorem)

152
2. PQ2 = [6 – (–3)]2 + (3 – 1)2 Practise Now 8
= 92 + 22
(a) If A, B and C are collinear, they lie on the same straight line,
= 81 + 4
i.e. gradient of AB = gradient of AC
= 85
9 – (k + 4) 9 – (k + 3)
QR2 = (1 – 6)2 + (8 – 3)2 =
0 – (k + 1) 0 – 2k
= (–5)2 + 52
5–k 6–k
= 25 + 25 =
–k – 1 –2k
= 50 –10k + 2k = k2 – 5k – 6
2

PR2 = [1 – (–3)2 + (8 – 1)2 k2 – 5k + 6 = 0


= 42 + 72 (k – 3)(k – 2) = 0
= 49 + 16 k–3=0 or k–2=0
= 65 k=3 k=2
Since QR2 + PR2 = 50 + 65 k = 2 or 3
= 115 (b) If AB is parallel to CD,
PQ2, gradient of AB = gradient of CD
the triangle is not a right-angled triangle. 9 – (k + 4) (k + 6) – (k + 3)
=
0 – (k + 1) (2k + 2) – 2k
Practise Now 6 5 k 3
=
–k – 1 2
1. Since (–1, 2) lies on the line y = 5x + a, the coordinates (–1, 2) must
10 – 2k = –3k – 3
satisfy the equation,
k = –13
i.e. 2 = 5(–1) + c
= –5 + c
Practise Now 9
c =7
2. Since (6, 8) lies on the line y = –4x + b, the coordinates (6, 8) must Since ABC = 90, gradient of AB gradient of BC = –1
satisfy the equation, –2 – 1 k – (–2)
= –1
i.e. 8 = –4(6) + b –1 – 0 2 – (–1)
= –24 + b i.e.
b = 32 k+2
3 = –1
3
Practise Now 7 k + 2 = –1
k = –3
3–1
(a) Gradient of AB =
5 – (–2)
Practise Now 10
2
= Gradient of DC = gradient of AB
7
2 8–2
Equation of AB is in the form y = x+c =
7 16 – 4
Since (–2, 1) lies on the line, 6
=
2 12
1 = (–2) + c 1
7 =
4 2
1=– +c Since BC is perpendicular to AB,
7
11 gradient of BC = –2
c= 1
7 Equation of DC is y – 10 = (x – 5)
2 11 2
Equation of AB is y = x + 2y – 20 = x – 5
7 7
(b) C(6, 4) and D(–4, 4) have the same y-coordinate of value 4. 2y = x + 15
CD is a horizontal line with equation y = 4. Equation of BC is y – 8 = –2(x – 16)
(c) E(–3, 5) and F(–3, 8) have the same x-coordinate of value –3. y = –2x + 32 + 8
EF is a vertical line with equation x = –3. y = –2x + 40

153
Exercise 6A 3
3. Gradient of line =
5
1– 0 p – (–7) 3
1. (a) Gradient = =
–2 – 0 4 – (–3) 5
1 p+7 3
=– =
2 7 5
7 – (–3) 5(p + 7) = 21
(b) Gradient =
1– 2
5p + 35 = 21
10
= 5p = –14
–1
4
= –10 p = –2
5
8–4
(c) Gradient = 4. Gradient of AB = 3
–5 – (–2)
–3 – 8
4 =3
= k – 3k
–5 + 2
–11
4 =3
=– –2k
3
–11 = –6k
–8 – 7
(d) Gradient = 5
1 – (–4) k =1
–15 6
= 2
5 5. Gradient =
= –3 a
1– a 2
6 – (–5) =
(e) Gradient = 2a – 9 a
2 – (–2)
a(1 – a) = 2(2a – 9)
11
= a – a2 = 4a – 18
4 2
9–9 a + 3a – 18 = 0
(f) Gradient = (a + 6)(a – 3) = 0
6 – (–7)
0 a+6 =0 or a – 3 = 0
=
13 a = –6 or a = 3
=0 6. Gradient of PQ = Gradient of PR
1–1 –9 – (–11) –3 – (–11)
2. Gradient of AB = =
7–0 k 6 2k – 6
0 2 8
= =
7 k–6 2k – 6
=0 2(2k – 6) = 8(k – 6)
4 –1 4k – 12 = 8k – 48
Gradient of AE =
6–0 4k = 36
3 k =9
=
6 7. Since the points are collinear, i.e. they lie on a straight line,
1 Gradient of PQ = Gradient of PR
=
2 –2 – (–3) z – (–3)
0–5 =
Gradient of DC = 3– 2 8–2
6–0 1 z+3
5 =
=– 1 6
6 6 =z+3
4–5
Gradient of DE = z =3
6–0
1
=–
6

154
8. Gradient of AB =2 (c) EF = (8 – (–1)) + (–3 – 4)
2 2

2t 2 + 7 – t
=2 = 9 2 + (–7)2
7–2
2t 2 + 7 – t = 130
=2
5 = 11.4 units (to 3 s.f.)
2t2 + 7 – t = 10
2t2 – 3 – t =0 (d) GH = (–4 – (–10))2 + (–7 – 2)2
(2t – 3)(t + 1) =0 = 6 2 + (–9)2
2t – 3 =0 or t+1=0
= 117
2t =3 t = –1
1 = 10.8 units (to 3 s.f.)
t =1 2. AB = 10 units
2
1 ( p – 0)2 + (0 – p)2 = 10
t=1 or t = –1
2
p 2 + p 2 = 10
1– 6
9. (i) Gradient of AB =
2–0 2 p 2 = 10
–5
= 2p2 = 100
2
p2 = 50
5
=– p = ± 50
2
3–1 = ±7.07 units (3 s.f.)
Gradient of BC =
7–2 3. (a) Since R lies on the y-axis, x-coordinate of R = 0.
2 Let the coordinates of point R be (0, y).
=
5 PR = QR
8–3
Gradient of CD = (0 – (–2))2 + (y – 6)2 = (0 – 9)2 + (y – 3)2
5–7
5
= (2)2 + (y – 6)2 = (–9)2 + (y – 3)2
–2
5 4 + (y – 6)2 = 81 + (y – 3)2
=– 2
2 4 + y – 12y + 36 = 81 + y2 – 6y + 9
8–6 40 – 12y = 90 –6y
Gradient of AD =
5–0 6y = –50
2 1
= y = –8
5 3
5 1
(ii) Gradient of AB = Gradient of CD = – Coordinates of R are 0, –8
2 3
2 (b) Since S lies on the x-axis, y-coordinate of S = 0.
Gradient of BC = Gradient of AD =
5 Let the coordinates of point S be (x, 0).
They are equal. PS = QS

Exercise 6B (x – (–2))2 + (0 – 6)2 = (x – 9)2 + (0 – 3)2

(x + 2)2 + (–6)2 = (x – 9)2 + (–3)2


1. (a) AB = (9 – 2) + (7 – 3)
2 2

(x + 2)2 + 36= (x – 9)2 + 9


= 72 + 4 2 2
x + 4x + 4 + 36 = x2 – 18x + 81 + 9
= 65 4x + 40 = 90 – 18x
= 8.06 units (to 3 s.f.) 22x= 50
3
(b) CD = (–5 – 3) + (9 – 6)
2 2
x =2
11
= (–8)2 + 32 3
Coordinates of S are 2 , 0
11
= 73
= 8.54 units (to 3 s.f.)

155
4. Since W lies on the y-axis, x-coordinate of W = 0. (ii) AC = [3 – (–2)]2 + (4 – 1)2
Let the coordinates of point W be (0, y).
Since W is equidistant from M and from N. = 5 2 + 32
MW = NW = 34
(0 – 3) + (y – 7)
2 2
= (0 – 11) + [ y – (–6)]
2 2
= 5.83 units (to 2 d.p.)
(iii)
(–3)2 + (y – 7)2 = (–11)2 + (y + 6)2 y
2 2
9 + (y – 7)
= 121 + (y + 6) C(3, 4)
D(0, 4)
9 + y2 – 14y + 49
= 121 + y2 + 12y + 36 4
y2 – 14y + 58
= y2 + 12y + 157 3
–14y – 12y= 157 – 58
2
–26y
= 99
21 1 B(1, 1)
y = –3 A(–2, 1)
26 x
21 –2 –1 0 1 2 3 4
Coordinates of W are 0, –3
26
5. (i) AB = 12 units From the graph, the coordinates of D are (0, 4).
(iv) Length of perpendicular from B to CK = 3 units
BC = (2 – 8)2 + (6 – (–2))2
Area of !BCK = 12 units2
= (–6)2 + 8 2 1
base height = 12
2
= 100 1
CK 3 = 48
= 10 units 2
1 1
AC = (2 – (–4))2 + (6 – (–2))2 (t – 3) 3 = 12 or (3 – t) 3 = 12
2 2
= 62 + 82 3(t – 3) = 24 3(3 – t) = 24
3t – 9 = 24 9 – 3t = 24
= 100 3t = 33 3t = –15
= 10 units t = 11 t = –5
Perimeter of !ABC = 12 + 10 + 10 t = 11 or –5
= 32 units 7. (i) AB = 3 units
1
Area of !ABC = base height BC = [3 – (–1)]2 + [–1 – (–2)]2
2
1 = 4 2 + 12
= 12 8
2
= 48 units2 = 17
(ii) Let the length of the perpendicular from A to BC be x units. = 4.12 units (to 3 s.f.)
Area of !ABC = 48 units2
AC = [3 – (–1)]2 + (–1 – 1)2
1
base height = 48
= 4 + (–2)
2 2
2
1
BC x = 48 = 20
2
1 = 4.47 units (to 3 s.f.)
10 x = 48
2 (ii) Length of the perpendicular from C to AB = 4 units
5x = 48 1
Area of !ABC = base height
x = 9.6 2
The length of the perpendicular from A to BC is 9.6 units. 1
= AB 4
1 2
6. (i) Area of !ABC = base height 1
2 = 3 4
1 2
= 3 3 = 6 units2
2
= 4.5 units2

156
(iii) Length of the perpendicular from B to CE = 4 units 10. PQ2 = (3 – 3)2 + (1 – 4)2
Area of !BCE = 14 units2 = 02 + (–3)2
1 =9
base height = 14
2 QR = (8 – 3)2 + (4 – 1)2
2

1 = 52 + 32
CE 4 = 14
2 = 25 + 9
2CE = 14 = 34
CE = 7 units PR = (8 – 3)2 + (4 – 4)2
2

Possible values of k = 6 or –8 = 52 + 02
8. [1 – (1 – t )]2 + (2t – 1)2 = 11 – 9t = 25
Since PQ2 + PR2 = 9 + 25
( [1 – (1 – t )]2 + (2t – 1)2 )2 = ( 11 – 9t )2
= 34
[1 – (1 – t)]2 + (2t – 1)2 = 11 – 9t = QR2,
t2 + 4t2 – 4t + 1 = 11 – 9t the triangle is a right-angled triangle with "QPR = 90°. (Converse
5t2 – 4t + 1 = 11 – 9t of Pythagoras’ Theorem)
5t2 + 5t – 10 =0 1
Area of !PQR = PQ PR
t2 + t – 2 =0 2
(t + 2)(t – 1) =0 1
= 3 5
t+2 =0 or t–1 =0 2
t = –2 t =1 = 7.5 units2
t = –2 or t = 1 Let the length of the perpendicular from P to QR be x units.
9. (i) AB = 6 units Area of !PQR = 7.5 units2
1
BC = (2 – 5)2 + (5 – 2)2 base height = 7.5
2
= (–3)2 + 32 1
QR x = 7.5
2
= 18 1
34 x = 7.5
2
AC = [2 – (–1)]2 + (5 – 2)2 x = 2.57 (to 3 s.f.)
= 32 + 32 The length of the perpendicular from P to QR is 2.57 units.
11. y
= 18
Since BC = AC, the points A, B and C are the vertices of an 16 R(5, 15)
isosceles triangle. 14
(ii) Length of the perpendicular from C to AB = 3 units 12
1 10
Area of !ABC = base height
2
8
1
= 6 3 6
2
= 9 units2 4 Q(5, 4)
y 2 P(1, 3)
C(2, 5) x
0 1 2 3 4 5 6 7
5
4
PR = (5 – 1)2 + (15 – 3)2
3
A(–1, 2) 2 B(5, 2) = 4 2 + 12 2
1 = 160 units
x
–1 0 1 2 3 4 5 Length of the perpendicular from P to QR = 4 units

157
1 4 – (–5) 9
Area of !PQR = base height (d) Gradient of GH = =
2 4 – (–6) 10
1 9
= QR 4 Equation of GH is in the form y = x+c
2 10
1 Since (–6, –5) lies on the line,
= 11 4
2 9
–5 = (–6) + c
= 22 units2 10
Let the length of the perpendicular from Q to PR be h units. 2
–5 = –5 + c
Area of !PQR = 22 units2 5
1 2
base height = 22 c =
2 5
1 9 2
PR h = 22 y= x+
2 10 5
1 –7 – (–4) –3
160 h = 22 (e) Gradient of IJ = = = –1
2 1 – (–2) 3
h = 3.48 (to 3 s.f.) Equation of IJ is in the form y = –x + c
The length of the perpendicular from Q to PR is 3.48 units. Since (–2, – 4) lies on the line,
– 4 = (–1)(–2) + c
Exercise 6C –4 = 2 + c
c = –6
1. Since the point (1, 2) passes through the line y = –x + c,
y = –x – 6
2 = –1 + c
–1 – (–5) 4 2
c =3 (f) Gradient of KL = = =
–1 – (–7) 6 3
2. Since the point (–3, 3) passes through the line y = 4x + c, 2
3 = 4(–3) + c Equation of KL is in the form y = x + c
3
3 = –12 + c Since (–1, –1) lies on the line,
c = 15 2
–1 = (–1) + c
–1 – 0 –1 3
3. (a) Gradient of AB = = = = –1
1– 0 1 2
–1 = – + c
y-intercept = 0 3
Equation of AB is y = –x 1
c =–
5–3 2 3
(b) Gradient of CD = = =2
2 –1 1 2 1
y= x–
Equation of CD is in the form y = 2x + c 3 3
Since (1, 3) lies on the line, (g) M(8, 0) and N(–9, 0) have the same y-coordinate of value 0.
3 = 2(1) + c MN is a horizontal line with equation y = 0.
3 =2+c (h) O(0, 0) and P(0, 7) have the same x-coordinate of value 0.
c =1 OP is a horizontal line with equation x = 0.
y = 2x + 1 4. (a) y = mx + c
3– 4 –1 1 1
(c) Gradient of EF = = = 0 = (0) + c
–2 – 2 –4 4 3
1 c =0
Equation of EF is in the form y = x + c
4 1
y = x
Since (2, 4) lies on the line, 3
1 (b) y = mx + c
4 = (2) + c
4 1 = 3(1) + c
1 1 =3+c
4 = +c
2 c = –2
7 y = 3x – 2
c =
2 (c) y = mx + c
1 7 –5 = –3(2) + c
y = x+
4 2 –5 = –6 + c
c =1
y = –3x + 1

158
(d) y = mx + c 8. For the line 2x – 5 = ky,
1 ky = 2x – 5
7= – (5) + c
2 2 5
y = x–
5 k k
7 =– +c
2 2
Gradient of the line 2x – 5 = ky is .
19 k
c=
2 For the line (k + 1)x = 6y – 3,
1 19 6y = (k + 1)x + 3
y =– x+
2 2 k +1 3
(e) y = mx + c y = x–
6 6
4 = (0)(5) + c k +1 1
y = x–
c =4 6 2
y =4 k +1
Gradient of the line (k + 1)x = 6y – 3 is .
(f) y = mx + c 6
a = a(0) + c Since both lines have the same gradient,
c =a 2 k +1
=
y = ax + a k 6
5. y = mx + c 12 = k(k + 1)
Gradient of line = 2 12 = k2 + k
2
c=0 k + k – 12 = 0
Equation of line is in the form y = 2x + 0 (k + 4)(k – 3) = 0
Equation of line is y = 2x. k + 4 = 0 or k – 3 = 0
6. (a) Since the line is a horizontal line, the gradient is 0 and the k = – 4 or k = 3
y-intercept = 1. x y
9. (i) + =1
Equation of line is y = 1. 3 2
(b) x y
6 + =6 1
there is no y-intercept. 3 2
Equation of line is x = 1.5. 2x + 3y = 6
(c) Since (0, –1) and (1, 0) lie on the line, 3y = 6 – 2x
0 – (–1) 1 6 – 2x
Gradient of line = = =1 y =
1– 0 1 3
c = –1 6 2x
y = –
Equation of the line is y = x – 1. 3 3
2
(d) Since (0, 1) and (2, 0) lie on the line, y =– x+2
3
0 –1 1
Gradient of line = =– 2
2–0 2 (ii) y = – x + 2
3
c=1
2
1 The gradient of the line is – .
Equation of the line is y = – x + 1. 3
2
2
7. (i) Length of perpendicular from C to AB = 4 units (iii) y = – x+2
3
1 Where the line cuts the x-axis, y = 0.
Area of !ABC = base height
2
2
1 0 =– x+2
= 2 4 3
2
2
= 4 units2 x =2
3
5–3 2 1 2x = 6
(ii) Gradient of line passing through B and C = = =
5 –1 4 2
x =3
5 –1 4
(iii) Gradient of line passing through A and C = = =1 The coordinates of the point is (3, 0).
5 –1 4
Equation of AC is in the form y = x + c
Since (1, 1) lies on the line,
1 = (1)(1) + c
1 =1+c
c =0
y =x

159
2 (ii) Where l intersects the line x = 2
10. (i) Given the gradient of line = – ,
3 5x + 6y + 30 = 0
2 5(2) + 6y + 30 = 0
Equation of the line is in the form y = – x+c
3 10 + 6y + 30 = 0
Since (–3, 5) lies on the line, 6y + 40 = 0
2 6y = – 40
5 = –3 – +c
3 2
5=2+c y = –6
3
c=3 2
The coordinates of the point of intersection are 2, –6 .
2 3
Equation of the line is y = – x + 3.
3 (iii) Since the line has the same gradient as l,
2 5x + 6y + 30 = 0
(ii) Given that (p, 3) passes through the line y = – x + 3,
3 6y = –5x – 30
2 5 30
3 =– p+3 y =– x–
3 6 6
2 5
p =0 y =– x–5
3 6
p =0 5
11. Given that the line has the same gradient as the line 2y = 5x + 7, Gradient of the line = –
6
2y = 5x + 7 Since (3, –1) lies on the line,
5x + 7 5
y = –1 = – (3) + c
2 6
5 7 5
y = x+ –1 = – + c
2 2 2
5 3
Gradient of the line = c =
2 2
Since (3, –2) lies on the line, 5 3
Equation of the line is y = – x + .
5 6 2
–2 = (3) + c
2 (iv) Since (3, –1) lies on the line and the gradient is 0,
15 –1 = 0(3) + c
–2 = +c
2 c = –1
19 Equation of the line is y = –1.
c =–
2 12 – 3 9
5 19 14. (a) (i) Gradient of line l = = =3
Equation of the line is y = x – . 3– 0 3
2 2 (ii) Equation of the line is in the form y = 3x + c
12. (i) y = mx + c
Since (0, 3) lies on the line,
1 = 3(3) + c
3 = 3(0) + c
1 =9+c
c =3
c = –8
y = 3x + 3
y = 3x – 8
(b)
(ii) y = 3x – 8 — (1) x=3
y
y=x — (2)
12 B(3, 12)
Substitute (1) into (2):
3x – 8 = x 10

2x = 8 8
x =4 6
y =4 4
A(0, 3) C(6, 3)
The coordinates of the point of intersection are (4, 4). 2
13. (i) Where l crosses the x-axis, y = 0 x
0 1 2 3 4 5 6
5x + 6y + 30 = 0
5x + 6(0) + 30 = 0
The coordinates of the point of intersection are (6, 3).
5x + 30 = 0
5x = –30
x = –6
The coordinates of the point are (–6, 0).

160
15. Since the line has the same gradient as the x-axis, the gradient is 0. Where the line intersects the x-axis, y = 0
mx = ny + 2 2 17
y = x+
ny = mx – 2 7 7
m 2 2 17
y = x– 0 = x+
n n 7 7
m 2 17
=0 x =–
n 7 7
m =0 2x = –17
For the line to be parallel to the y-axis, n = 0. 1
x = –8
16. (i) 3x + 4y = 24 2
Where l crosses the x-axis at the point A, y = 0 1
The coordinates of the point are –8 , 0 .
3x + 4y = 24 2
(ii) y
3x + 4(0) = 24
3x = 24 R(2, 7)
7
x =8
6
The coordinates of the point A are (8, 0).
5 y=5
Where l crosses the y-axis at the point B, x = 0 Q(9, 5)
4
3x + 4y = 24
3(0) + 4y = 24 3
P(2, 3)
4y = 24 2
y =6 1
The coordinates of the point B are (0, 6). x
0 1 2 3 4 5 6 7 8 9
(ii) AB = (0 – 8) + (6 – 0)
2 2

= 82 + 62 The coordinates of R are (2, 7).

= 100 (iii) PQ = (9 – 2)2 + (5 – 3)2


= 10 units = 72 + 22
(iii) Let the coordinates of C be (k, k).
= 53
3k + 4k = 24
7k = 24 = 7.28 units (to 3 s.f.)
24 (iv) Length of perpendicular from Q to RP = 7 units
k = 1
7 Area of !PQR = base height
3 2
k =3 1
7 = 4 7
3 3 2
The coordinates of the point C are 3 , 3 . = 14 units2
7 7
3 Let the length of perpendicular from R to PQ be h units.
3 –0 Area of !PQR = 14 units2
(iv) Gradient of line OC = 7 =1
3 1
3 –0 PQ h = 14
7 2
Since (0, 0) lies on the line, 1
53 h = 14
0 = 0(1) + c 2
c=0 h = 3.85 units (to 3 s.f.)
Equation of the line is y = x. The length of perpendicular from R to PQ is 3.85 units.
5–3 2
17. (i) Gradient of the line = = Exercise 6D
9–2 7
Since (2, 3) lies on the line, 1. (a) Gradient = 8
2 1
3 = (2) + c (b) Gradient = –
7 2
4 –1
3= +c 2. (a) Gradient =
7 3
17 1
c= =–
7 3
2 17
Equation of the line is y = x + .
7 7

161
–1 (b) 42x – 7y = 5
(b) Gradient =
3 7y = 42x – 5
4 5
4 y = 6x –
= 7
3 1
3. (a) Since the points O, A and B are collinear, Gradient of line = –
6
gradient of OA = gradient of OB 1
Equation of line is y –(–6) = –[x – (–1)]
3k – 0 6–0 6
=
2–0 4k – 0 1 1
y+6 =– x–
3k 6 6 6
=
2 4k 1 37
y =– x–
12k2 = 12 6 6
k2 = 1 6y = –x – 37
k = ±1 (c) 3x + y = 17
(b) Since OA is parallel to BC, y = –3x + 17
gradient of OA = gradient of BC Gradient of line = –3
3k – 0 7–6 Equation of line is y – 8 = –3(x – 4)
=
2–0 10k – 4 k y = –3x + 12 + 8
3k 1 y = –3x + 20
=
2 6k (d) y + 2x = 13
18k2 = 2 y = –2x + 13
2 1
k2 = Gradient =
18 2
1 1
= Equation of line is y –(–3) = (x – 2)
9 2
1 1
k =± y+3 = x–1
3 2
4. Gradient of AB gradient of BC = –1 2y = x – 8
4 –1 3 – (–3)
Gradient of AB = 6. Gradient of AB =
–1 – 1 2 – (–1)
3 6
=– =
2 3
2 =2
Gradient of BC =
3 1
k–4 2 Gradient of BC = –
= 2
6 – (–1) 3 k–3 1
3k – 12 = 12 + 2 =–
(k + 5) – 2 2
3k = 26 2k – 6 = –k – 3
26 3k = 3
k =
3 k =1
5. (a) 3y + 7x = 29
7. PR = 12 units
3y = –7x + 29
PQ = (–1 – 6) + (3 – 8)
2 2
7 29
y=– x+
3 3 = 74 units
7
QR = (6 – 11) + (8 – 1)
Gradient of line = – 2 2
3
7 = 74 units
Equation of line is y – 5 = – [x – (–2)]
3 Since PQ = QR, the points P, Q and R are vertices of an isosceles
7 14
y =– x– +5 triangle. (shown)
3 3
3y = – 7x + 1

162
8–3 2–4
Gradient of PQ = (ii) Gradient of RP =
6 – (–1) 4 – (–2)
5 2
= =–
7 6
1– 8 1
Gradient of QR = =–
11 – 6 3
7 –4 – 4
=– Gradient of RS =
8 6 – (–2)
Since gradient of PQ gradient of QR = –1, PQ is perpendicular to 8
=–
QR and PQR is a right angle. 8
8. (i) Let point C be (k, 0). = –1
Since the gradients of RP and RS are not equal, points P, R and
AB = (6 – 0)2 + (6 – 4)2
S are not collinear.
= 40 units –12 – (–6)
10. (a) Gradient of AB =
BC = (6 – k )2 + (6 – 0)2 3 – (–1)
6
= (6 – k )2 + 36 units =–
4
(6 – k )2 + 36 = 40 =–
3
(6 – k)2 + 36 = 40 2
36 – 12k + k2 + 36 – 40 = 0 6 – (–6)
Gradient of AC =
k2 – 12k + 32 = 0 k – (–1)
12
(k – 4)(k – 8) = 0 =
k +1
k–4 =0 or k–8 =0
Since A, B and C are collinear,
k =4 or k =8
12 3
Coordinates of C are (4, 0) or (8, 0). =–
k +1 2
(ii) Let point D be the point (0, p).
24 = –3k – 3
6–4
Gradient of AB = 3k = –27
6–0
k = –9
2
= (b) If AB is perpendicular to AC,
6
2
1 Gradient of AC =
= 3
3
12 2
Gradient of DB = –3 =
k +1 3
6– p
= –3 36 = 2k + 2
6–0
2k = 34
6 – p = –18
k = 17
p = 24
3 – (–3)
Coordinates of D are (0, 24). 11. (i) Gradient of AB =
2 – (–1)
9. (i) Let the point P be (2m, m).
6
RP = SP =
3
(–2 – 2m)2 + (4 – m)2 = (6 – 2m)2 + (–4 – m)2 =2
k–3
(4 + 8m + 4 m 2 ) + (16 – 8m + m 2 ) = Gradient of BC =
6–2
(36 – 24 m + 4 m 2 ) + (16 + 8m + m 2 ) Since AB is perpendicular to BC,
k–3 1
20 + 5m 2 = 52 – 16m + 5m 2 =–
6–2 2
20 + 5m2 = 52 – 16m + 5m2 2k – 6 = –6 + 2
16m = 32 2k = 2
m =2 k =1
Coordinates of P are (4, 2).

163
1 – (–3) 12 – (–6)
(ii) Gradient of AC = 14. (a) Gradient of AB =
6 – (–1) 3 – (–1)
4 18
= =
7 4
12. Let the points be A(2, 1), B(1, 5), C(–2, –1) and D(–1, –5). 9
=
5 1 2
Gradient of AB =
1 2 6 – (–6)
Gradient of AC =
= –4 k – (–1)
5 ( 1) 12
Gradient of CD = =
1 ( 2) k +1
= –4 Since A, B and C are collinear,
Since gradient of AB = gradient of CD, AB // CD. 12 9
=
5 ( 1) k +1 2
Gradient of BC =
1 ( 2) 24 = 9k + 9
6 9k = 15
=
3 15
k=
=2 9
5 1 2
Gradient of AD = =1
1 2 3
6 (b) Since AB is perpendicular to AC,
=
3 12 2
=–
=2 k +1 9
Since gradient of BC = gradient of AD, BC // AD, 108 = –2k – 2
the 4 points are the vertices of a parallelogram. 2k = –110
13. Let the points be A(5, 8), B(3, 5), C(5, 2) and D(7, 5). k = –55
8 5 12 – 6
Gradient of AB = (c) Gradient of BC =
5 3 3– k
3 Since BC is perpendicular to AC,
=
2 12 – 6 12
= –1
Gradient of CD =
5 2 3– k k +1
7 5 144 – 72
= –1
=
3 (3 – k )(k + 1)
2 72
= –1
Since gradient of AB = gradient of CD, AB // CD. 3k + 3 – k 2 – k
5 2 72 = k2 – 2k – 3
Gradient of BC =
3 5 k2 – 2k – 75 = 0
3 k2 – 2k + 1 = 76
=–
2 (k – 1)2 = 76
8 5
Gradient of AD = k – 1 = ± 76
5 7
3 k = 1 ± 2 19
=–
2 7–2
15. (a) Gradient of AB =
Since gradient of BC = gradient of AD, BC // AD. 6 –1
5
Length of AB = (5 – 3) + (8 – 5)
2 2
=
5
= (2) + (3)
2 2
=1
= 13 units Gradient of perpendicular bisector = –1
1+ 6 2 + 7
Length of BC = (5 – 3) + (2 – 5)
2 2
Midpoint of AB = ,
2 2
= (2)2 + (–3)2 7 9
= ,
= 13 units 2 2
Since AB = BC, AB // CD and BC // AD,
the 4 points are vertices of a rhombus.

164
Equation of perpendicular bisector is Review Exercise 6
9 7
y – = –1 x – 1. (i) y = mx + c
2 2
–3 = 2(0) + c
7 9
y = –x + + c = –3
2 2
y+x =8 y = 2x – 3
7–2 (ii) Given that the line passes through the point (4, k),
(b) Gradient of BC = k = 2(4) – 3
6–7
= –5 k=8–3
1 k=5
Gradient of perpendicular bisector =
5 2. (i) 6x + 2y = 7
6+7 7+2 2y = –6x + 7
Midpoint of BC = ,
2 2 –6 x + 7
y=
13 9 2
= ,
2 2 6 7
y =– x+
Equation of perpendicular bisector is 2 2
9 1 13 7
y– = x– y = –3x +
2 5 2 2
1 13 9 Gradient of the line = –3
y = x– +
5 10 2 (ii) Given that the line passes through (3, 5) and the gradient = –3,
1 16 y = mx + c
y = x+
5 5 5 = –3(3) + c
5y = x + 16 5 = –9 + c
8–0 c = 14
16. (i) Gradient of AC =
–2 – 4 –3 – 5 –8
8 3. Gradient of AB = = = –8
=– 2 –1 1
6 Given that the line passes through the origin, (0, 0),
4
=– y = mx + c
3
0 = –8(0) + c
(ii) AB = [4 – (–3)]2 + (–1)2 c=0
y = –8x
= (7)2 + (–1)2
4. (i) 3x – 4y = 24
= 50 units 4y = 3x – 24
BC = [–3 – (–2)2 + (1 – 8)2 y =
3x – 24
4
= (–1)2 + (–7)2 3 24
y = x–
= 50 units 4 4
AB = BC 3
y = x–6
1– 0 4
Gradient of AB = 3
–3 – 4 Gradient of l =
1 4
=– (ii) 3x – 4y = 24
7
8 –1 Where l crosses the x-axis at the point A, y = 0
Gradient of BC =
–2 – (–3) 3x – 4y = 24
=7 3x – 4(0) = 24
Since gradient of AB gradient of BC = –1, 3x = 24
AB is perpendicular to BC and !ABC = 90°. x =8
(iii) Length of one side = 50 units The coordinates of the point A are (8, 0).
Where l crosses the y-axis at the point B, x = 0
Area of square = ( 50 )2 3x – 4y = 24
= 50 units2 3(0) – 4y = 24
– 4y = 24
y = –6
The coordinates of the point B are (0, –6).

165
AB = (0 – 8)2 + (–6 – 0)2 6 6
(iv) Given that the line passes through M 2 , 3 and is parallel
7 7
= 8 +6
2 2 to the y-axis,

= 100
= 10 units 6
x=2
7
–3 – 0 3
(iii) Gradient of OM = =– 5 –1 4 2
4–0 4 6. (i) Gradient of l = = =
5 – (–1) 6 3
Given that the line passes through B(0, –6),
(ii) Given that the line passes through B(5, 5),
y = mx + c
y = mx + c
3
–6 = – (0) + c 2
4 5 = (5) + c
3
c = –6
1
3 5 =3 +c
y =– x–6 3
4
5
0–6 3 c=
5. (i) Gradient of AB = =– 3
8–0 4
2 5
Given that the line passes through A(0, 6), y= x+
3 3
y = mx + c
(iii) Length of perpendicular from B to AC = 4 units
3
6 = – (0) + c 1
4 Area of !ABC = 5 4
2
c =6
= 10 units2
3
y =– x+6
4 (iv) BC = (4 – 5)2 + (1 – 5)2
3
(ii) y = – x + 6 — (1) = (–1) + (– 4)
2 2
4
y=x+1 — (2) = 17
Substitute (1) into (2):
= 4.12 units (to 2 d.p.)
3
– x+6 =x+1 7. (i) 3y = 2x + 8
4
When l crosses the x-axis at the point A, y = 0
3
– x–x =1–6 3(0) = 2x + 8
4
3 2x = –8
–1 x = –5 x = –4
4
6 The coordinates of the point A are (– 4, 0).
x =2
7 2t + 8
(ii) Let the x-coordinate be t, then the y-coordinate is .
6 3
y =2 +1
7 Since the y-coordinate is twice the x-coordinate,
6 2t + 8
=3 2t =
7 3
6 6 6t = 2t + 8
The coordinates of M are 2 , 3 .
7 7 4t = 8
6 6 t =2
(iii) Given that the line passes through M 2 , 3 and is parallel
7 7 2t = 4
to the x-axis,
The coordinates of the point K are (2, 4).
Gradient of the line = 0
y = mx + c (iii) Length of perpendicular from K to AH = 4 units
6 6 1
3 =0 2 +c Area of !ABC = 8 4
7 7 2
6 = 16 units2
c =3
7
6
y=3
7

166
3– 0 3 (iii) Given that l2 has the same gradient as l1,
8. (i) Gradient of the line = = =3
0 – (–1) 1 5y + 12x – 60 = 0
Given that the line passes through B(0, 3), 5y = –12x + 60
y = mx + c 12 60
y =– x+
3 = 3(0) + c 5 5
c =3 12
y =– x + 12
y = 3x + 3 5
(ii) AB = h 12
Gradient of l2 = –
5
[0 – (–1)]2 + (3 – 0)2 = h Given that the line passes through (0, –2),
12 + 32 = h y = mx + c
12
10 = h –2 = – (0) + c
5
( 10 )2 = ( h )2 c = –2
12
10 = h y=– x–2
5
h = 10
(iv)
(iii) Since the point (–5, k) lies on BA produced, y
y = 3x + 3
12 Q(0, 12)
k = 3(–5) + 3
k = –12 10
(iv) 8
y
6
4
4 y=x+1
B 2
3 R(–5, 0) P(5, 0)
x
2 –7 –6 –5 –4 –3 –2 –1 0 1 2 3 4 5 6 7
1 C(2, 1)
A The coordinates of the point R are (–5, 0).
x
–2 –1 0 1 2 3 10. AB = h + 11
DC = 2 + t
The coordinates of the point C are (2, 1).
Perpendicular height of trapezium ABCD = 6 units
9. (i) 5y + 12x – 60 = 0
Area of trapezium ABCD = 84 units2
Where l1 crosses the x-axis at the point P, y = 0
1
5y + 12x – 60 = 0 (AB + DC) 6 = 84
2
5(0) + 12x – 60 = 0 1
12x = 60 (h + 11 + 2 + t) 6 = 84
2
x =5 3(h + 13 + t) = 84
The coordinates of the point P are (5, 0). h + 13 + t = 28
Where l1 crosses the y-axis at the point Q, x = 0 t = 15 – h
5y + 12x – 60 = 0 11. (i) Since the line through C at B is a horizontal line,
5y + 12(0) – 60 = 0 the equation of the line y = – 4.
5y = 60 (ii) Since A is a point on the y-axis, x = 0
y = 12 2y – 5x = 4
The coordinates of the point Q are (0, 12). 2y – 5(0) = 4
(ii) PQ = (0 – 5)2 + (12 – 0)2 2y = 4
y =2
= (–5)2 + 12 2 The coordinates of the point A are (0, 2).
= 169
= 13 units

167
Since B lies on line AB and its y-coordinate = – 4, 13. 3x + 5y = 13
2(– 4) – 5x = 4 5y = –3x + 13
– 8 – 5x = 4 3 13
y =– x+
5x = –12 5 5
2 –1
x = –2 Gradient of line =
5 3

2 5
The coordinates of the point B are –2 , – 4 . 5
5 =
3
1 2
Area of !ABC = 2 6 3 5 10
2 5 Equation of the line is y – = x–
2 11 3 11
= 7.2 units
5 50 3
(iii) AB = l y = x – +
3 33 11
2 5 41
2 = x–
–2 –0 + (– 4 – 2)2 = l 3 33
5
33y = 55x – 41
2
2 14. Gradient of line = 2
–2 + (–6)2 = l
5 Equation of the line is y – 3 = 2(x – 2)
41.76 = l y = 2x – 4 + 3
= 2x – 1
( 41.76 )2 = ( l )2
41.76 = l
l = 41.76 Challenge Yourself
(iv) Let the length of the perpendicular from C to AB be h units. a2 – b2 (a + b)(a – b)
Area of !ABC = 7.2 units2 Gradient of AB = = =a+b
a–b a–b
1 c2
AB h = 7.2 Gradient of OC = =c
2 c
1 Since AB is parallel to OC,
41.76 h = 7.2
2 Gradient of AB = Gradient of OC
h = 2.23 units (to 3 s.f.) a + b = c (proven)
2–2
(v) Gradient of AD = =0
0 – 2.4
Gradient of BC = 0
Since AD = 2.4 units = BC,
hence ABCD is a parallelogram (proven).
12 – 15 –3
12. (i) Gradient = = = 0.1
20 – 50 –30
Given that (20, 12) lies on the line,
12 = 0.1(20) + c
12 = 2 + c
c = 10
y = 0.1x + 10
(ii) The value of the y-intercept represents the length of the spring
when it is suspended freely without a mass attached to it.

168
Chapter 7 Graphs of Functions and Graphical Solution
TEACHING NOTES
Suggested Approach
This chapter serves as an introduction to the important concepts of relations and functions. Before plotting graphs of functions,
revise the choice of scales and labelling of scales on both axes. Students are often weak in some of these areas. Teachers should
encourage the students to draw the curves free hand, as well as to use curved rules to assist them.

It will be worthwhile to ask students to remember the general shapes of quadratic graphs, the “U” shape and the upside down
“U” shape. Also, students should remember the general shapes of cubic, reciprocal and exponential graphs. This will help
them to identify and rectify errors when they sketch or plot a few points wrongly and if the shapes of their graphs look odd.

Section 7.1: Graphs of Cubic Functions


Teachers should help students to see how the coefficient of x³ affects the shape of the graph. Teachers can ask
students who are confident and have grasped the concept behind graphs of cubic functions to explain to the
class (see Investigation: Graphs of Cubic Functions). Teachers will facilitate the class through the activity as
the students lead and learn together.

Section 7.2: Graphs of Reciprocal Functions


When introducing graphs of reciprocal functions, teachers should bring the students’ attention to the marginal
notes on the four quadrants on the Cartesian plane and the order of rotational symmetry about a particular
a a
point (see Information and Recall on page 176). Students need to know why the graphs of y = and y = 2 do
x x
not intersect with both axes and the values obtained when any real number is divided by zero see Investigation:
a a
Graphs of y = and Investigation: Graphs of 2 .
x x
When the students are able to understand the shapes of the graphs of reciprocal functions, they can go on to further
describe the graphs in the cases where a < 0 and where a > 0 (see Thinking Time on page 177 of the textbook).

Section 7.3: Graphs of Exponential Functions


Teachers should help students to see how the values of a and k affect the shape of the graphs of y = ax and
y = kax. Teachers can ask students who are confident and have grasped the concept behind graphs of exponential
functions to explain to the class (see Investigation: Graphs of y = ax and y = kax) while the teachers facilitate.

Now that the students have learnt the graphs of power and exponential functions, teachers can get them to match
such graphs with their corresponding functions and see whether they are able to differentiate between the different
types of graphs (see Class Discussion: Matching Graphs of Power Functions with the Corresponding Functions).

Section 7.4: Gradient of a Curve


Teachers can first revise what was covered in Chapter 6, where the gradient of a straight line is the ratio of
vertical change to horizontal change. Students may have difficulty in drawing the tangent to the curve. They may
draw the line too close to the curve or out of the point. Teachers can guide them along and suggest to them to
pick two points, say A and B, each with equal distance and near the point of contact, and then draw the tangent
parallel to line AB at the point of contact.

Section 7.5: Applications of Graphs in Real-World Contexts


Students tend to make mistakes with the quantities “distance”, “time” and “speed”. As such, teachers should
revise the formula for speed as the distance per unit time. Teachers should highlight to the students the difference
between finding the instantaneous speed and the average speed. The total distance travelled by an object has to
include the distance travelled in the returning journey when there is such a case, while the total time taken has
to include the time taken when the object is at rest or stationary.

Teachers can introduce the many different applications of graphs in real-world contexts through the worked
examples and activities (see Class Discussion: Linear Distance-Time Graphs and Thinking Time on pages 193
and 194 of the textbook).

169
For graphs involving the rate of flow of liquid in various shapes and sizes of containers, teachers should guide
the students along and teach them how to deduce the rate of increase of depth when the shape of the container
changes (see Thinking Time on page 197 of the textbook).

Some common mistakes that students make are that they may write 1 hour 15 minutes as 1.15 h instead of 1.25 h.
Students are also prone to mistakes when different units are given in a problem. For example, when a problem
gives the speed as v km/h and the time as t minutes, teachers can highlight to the students to convert the units
where needed. Students are encouraged to search on the internet for more real life examples related to graphs
so that they can familiarise themselves with the units used in the real world.

Challenge Yourself
From the acceleration-time graph, students should observe the change in acceleration in three different time
periods, i.e. between t = 1 and t = 2, between t = 2 and t = 4 and between t = 4 and t = 6. To deduce how the
motion of the object is and sketch a possible speed-time graph for that, students need to know that the gradient
of a speed-time graph gives the acceleration.

170
WORKED SOLUTIONS (e) y = –2x3
Investigation (Graphs of Cubic Functions) y

1. (a) y = x3
y

x
O

x
O

(f) y = –5x3
y
(b) y = 2x3
y

x
O
x
O

2. Where a > 0, the graph takes the shape


(c) y = 5x3
y y

x
O

x
O
Where a > 0, the graph takes the shape
y

(d) y = –x3 x
O
y

x
O

171
3. (a) y = x3 – x2 + 1 (e) y = –x3 + 2x2 + 1
y y

x
O x
O

(b) y = x3 + 4x2 – 3
y (f) y = –x3 – 0.5x – 1
y

x
O
x
O

(c) y = x3 + x
y
4. x3 is positive, the graph takes the shape

x
O
x
O

x3 is negative, the graph takes the shape


(d) y = –x3 + x2 – 2
y
y

x
O
x
O

172
a 1. (i) For a > 0, the graph consists of two parts that lie in the 1st and
Investigation Graphs of y = 3rd quadrants.
x
1 (ii) For a < 0, the graph consists of two parts that lie in the 2nd and
y=
x 4th quadrants.
y 2. There is rotational symmetry of order 2 about the origin, i.e. it maps
onto itself twice by rotation in 360°.
3. No. The curves get very close to the x-axis and y-axis but will never
touch them.

x
Thinking Time (Page 177)
O
(a) When a > 0, the equation of the line of symmetry is y = x.
(b) When a < 0, the equation of the line of symmetry is y = –x.

a
Investigation Graphs of y =
x2
5 2
y= y=
x x2
y y

x
O

x
O

1 4
y=– y=
x x2
y y

x
O

x
O

3
y=–
x
y

x
O

173
1 (b) y = 3x
y=–
x2 y
y

x
O
1

x
O

(c) y = 4x
y

3
y=–
x2
y

x x
O O

(d) y = 5x
y

1. (i) For a > 0, the graph consists of two parts that lie in the 1st and
2nd quadrants.
(ii) For a < 0, the graph consists of two parts that lie in the 3rd and 1
4th quadrants.
2. The y-axis is the line of symmetry, i.e. the graph is symmetrical x
O
about the y-axis.
3. No. The curves get very close to the x-axis and y-axis but will never
touch them. 2. (a) (0, 1)
(b) As x increases, y
Investigation (Graphs of y = ax and y = kax) (c) No, it does not.
3. As a increases, the steepness of the graph increases.
1. (a) y = 2x 4. (a) y = 2x
y
y

1 1
x
O x
O

174
(b) y = 3(2x) 5. (a) The coordinates of the point where the graph intersects the y-axis
y in 4(a) are (0, 1).
The coordinates of the point where the graph intersects the y-axis
in 4(b) are (0, 3).
The coordinates of the point where the graph intersects the y-axis
3
in 4(c) are (0, 5).
The coordinates of the point where the graph intersects the y-axis
in 4(d) are (0, –1).
x The coordinates of the point where the graph intersects the y-axis
O
in 4(e) are (0, –4).
(b) In graphs 4(a), 4(b) and 4(c), as x increases and tends to the right
(c) y = 5(2x) of the graph, the value of y increases very rapidly and approaches
y 4(d) and 4(e), as x increases and tends
to the right of the graph, the value of y decreases very rapidly

(c) No, the graphs do not intersect the x-axis.


6. If k > 0, the values of y are always positive, i.e. the graph lies entirely
5 above the x-axis.
If k < 0, the values of y are always negative, i.e. the graph lies entirely
below the x-axis.
x
O
Journal Writing (Page 184)
(i) As the number of months x increases, the number of members y
(d) y = –2x
increases dramatically. The curve tends towards the vertical as x
y
increases.
(ii) More real-life applications of exponential graphs include bacterial
decay, population growth, money investment, half-lives of unstable,
x
O radioactive atoms.
–1 Teachers should take note that the list is not exhaustive.

Class Discussion (Matching Graphs of Power Functions


with the Corresponding Functions)
y

(e) y = –4(2x)
y
x
O

x
O
Graph 1

a
Graph 1 is a graph of y = , where a < 0.
x
–4 6
The function of Graph 1 is B: of y = – .
x

175
y y

x x
O O

Graph 2 Graph 6
x
Graph 2 is a graph of y = ka , where k < 0. Graph 6 is a graph of y = kax, where k > 0.
The function of Graph 2 is F: of y = –2(6x). The function of Graph 6 is D: of y = 5x.
y y

x x
O O

Graph 3 Graph 7

a a
Graph 3 is a graph of y = , where a < 0. Graph 7 is a graph of y = , where a > 0.
x2 x2
3 5
The function of Graph 3 is E: of y = – 2 . The function of Graph 7 is C: of y = .
x 2 x2
y y

x x
O O

Graph 4 Graph 8

a Graph 8 is a graph of y = ax3, where a < 0.


Graph 4 is a graph of y = , where a > 0.
x The function of Graph 8 is H: of y = –3x3.
1
The function of Graph 4 is G: of y = .
2x Class Discussion (Linear Distance-Time Graphs)
y
1. From 0900 to 0930, since the graph is a horizontal line,
gradient = 0. This gradient represents the speed of the cyclist which
is 0 km/h. The cyclist is resting from 0900 to 0930.
x 50 – 20
O 2. From 0930 to 1030, gradient = = 30 km/h, the cyclist travels
1
at a constant speed of 30 km/h.
–50 1
3. From 1030 to 1200, gradient = = –33 km/h, the cyclist
Graph 5 1.5 3
1
Graph 5 is a graph of y = ax3, where a > 0. travels at a constant speed of 33 km/h and returns to his starting
3
The function of Graph 5 is A: of y = 2x3. point.

Total distance travelled


4. .
Total time taken
20 + 0 + 30 + 50
is wrong.
4
50 + 50 100
Hence the average speed of the cyclist = = = 25 km/h.
12 – 8 4

176
Thinking Time (Page 194) Since the cross-sectional area of the container is non-uniform, the
graph of Fig. 7.9(c) consists of a curve that becomes less and less
A: For an exchange rate between Singapore dollars and Hong Kong
steep as the container gets wider at the top.
dollars where S$1 = HK$6, S$10 can be exchanged for HK$60. The
graph will be a linear graph with the point (10, 60).
Practise Now 1
Graph 5 matches scenario A.
B: Since the height of water in the water catchment increased at a x –3 –2 –1 0 1 2 3
constant rate from 10 cm to 60 cm, the graph is a linear line that 3
y=x +2 –25 –6 1 2 3 10 29
begins from 10 cm and ends at 60 cm.
Graph 2 matches scenario B. y = –x3 – 2 25 6 –1 –2 –3 –10 –29
C: When Mr Neo drove at a constant speed of 60 km/h, the speed-time
graph will be a horizontal line initially. When he suddenly applied y Scale:
x-axis: 2 cm represent 1 unit
the brakes and came to a stop, his speed decreases until he comes y-axis: 2 cm represent 10 units
30
to a stop when his speed reaches 0 km/h.
Graph 6 matches scenario C.
20
D: The battery level in a smartphone decreased non-uniformly from y = x3 + 2
60% to 15%. The graph is non-uniform and it will begin at 60 and
end at 15. 10
Graph 4 matches scenario D.
E: The temperature of a substance in a freezer decreased from 60 °C x
–3 –2 –1 0 1 2 3
to 15 °C in 20 minutes. The graph will begin at 60 °C and end at
15 °C at the 20-minute mark. –10 y = –x3 – 2
Graph 1 matches scenario E.
F: The plant grew slowly to a height of 15 cm when it was kept indoors
–20
for 4 weeks, then grew more quickly to a height of 60 cm when it
was placed outdoors for the next 4 weeks. The graph will consist of
two lines, of which the second line is steeper since the plant grew –30

more quickly.
Graph 3 matches scenario F. (i) Consider y = x3 + 2.
From the graph, when x = 2.5, y = 17.5
Thinking Time (Page 197) Consider y = –x3 – 2.
From the graph, when x = 2.5, y = –17.5
1. The graph of Fig. 7.9(a) is a straight line because the container has (ii) Consider y = x3 + 2.
a uniform cross-sectional area. From the graph, when y = 15, x = 2.35
2. Consider y = –x3 – 2.
Height (h cm)
From the graph, when y = 15, x = –2.55

a
h
Time (t s)
t2
Fig. 7.9(b)

The graph of Fig. 7.9(b) consists of two straight lines, of which the
second line is steeper because the container is narrower at the top.

Height (h cm)

a
h
Time (t s)
t3
Fig. 7.9(c)

177
Practise Now 2 Practise Now 3

x –5 –4 –3 –2 –1 –0.5 –0.3 0.3 0.5 1 2 3 4 5 x –4 –3 –2 –1 –0.5 0.5 1 2 3 4


y –0.6 –0.75 1 –1.5 –3 –6 –10 10 6 3 1.5 1 0.75 0.6 y –0.13 –0.22 –0.5 –2 –8 –8 –2 –0.5 –0.22 –0.13

y
y Scale:
x-axis: 1 cm represents 1 unit
10 y-axis: 2 cm represent 1 unit
y= 3
x x
–4 –3 –2 –1 0 1 2 3 4
8

–1
6 2
y=–
x2
–2
4

–3
2

–4
x
–5 –4 –3 –2 –1 0 1 2 3 4 5
–5
–2

–6
–4

–7
–6

–8
–8
Scale: (i) From the graph, when x = 1.5, y = –0.9.
x-axis: 1 cm represents 1 unit (ii) From the graph, when y = –3.2, x = 0.8 or –0.8.
–10
y-axis: 1 cm represents 1 unit

(i) From the graph, when x = 2.5, y = 1.2.


(ii) From the graph, when y = –1.2, x = –2.5.

178
Practise Now 4

x –2 –1.5 –1 –0.5 0 0.5 1 1.5 2

y 0.11 0.19 0.33 0.58 1.0 1.73 3.0 5.20 9.0

Scale: y
x-axis: 4 cm represent 1 unit
y-axis: 2 cm represent 1 unit y = 3x
9

x
–2 –1 0 1 2

(i) From the graph, when x = –1, y = 0.3.


(ii) From the graph, when y = 0.7, x = –0.3.

179
Practise Now 5 Practise Now 6
(a) When x = –1, (a) Distance
from P (km) Scale:
y = (–1)2 – 4(–1) = 5 x-axis: 2 cm represent 2 minutes
a=5 y-axis: 2 cm represent 2 km
When x = 3, 8 vertical
y = 32 – 4(3) = –3 change
b = –3 = 2.25 km
6
(b) horizontal change
y = 5 minutes
Scale: 4
x-axis: 2 cm represent 1 unit
y-axis: 2 cm represent 1 unit
5
2

4
0 2 4 6 8
2
y = x – 4x Time since leaving P (minutes)
3
(b) From the graph, the train takes approximately 3.5 minutes to travel
2
(c) The gradient of the tangent at the point 6 minutes after it left station
1
P gives the speed at that particular point. It is called the instantaneous
speed. A tangent is drawn to the curve at the point 6 minutes after
it has left station P.
x
–1 0 1 2 3 4 5 From the graph,
vertical change
Gradient =
–1 horizontal change
2.25 km
=
–2 5 minutes
2.25 km
vertical =
–3 5
change = 2.9 h
60
= 27 km/h
–4
The speed of the train 6 minutes after it has left station P is
horizontal change = 1.8 approximately 27 km/h.
(d)
(c) A tangent is drawn to the curve at the point where x = 2.8.
gradient of the curve increases.
From the graph,
During the last 4 minutes, the speed of the train decreases as the
vertical change
Gradient = gradient of the curve decreases.
horizontal change
2.9 Distance from
= P (km)
1.8
= 1.61 (to 3 s.f.)
(d) (ii) A line parallel to the x-axis at the minimum point of the curve
has a gradient equal to zero. From the graph and table, h = 2,
k = – 4.

Time since leaving


0 P(minutes)
4 8

180
Practise Now 7 (b)
v(m/s) Scale:
8 m/s x-axis: 2 cm represent 1 second
(i) Acceleration = y-axis: 2 cm represent 5 m/s
3s
2 20
= 2 m/s2
3
(ii) Speed (m/s) 15 vertical
2
change
v = 2t – 9t + 12 = 9 m/s
10
8
acceleration = 0 horizontal change
6 5 =1s

4
t(s)
A B C 0 1 2 3 4 5
2
(c) (i) From the graph, when v = 7,
Time(s) t = 0.7 or t = 3.8.
0 3 5 9
(ii) The acceleration is zero when the gradient of the curve is zero.
Total distance = area under graph From the graph, the acceleration is zero at t = 2.25.
= Area of (A + B + C) (iii) A tangent is drawn to the curve at the point t = 4.5.
1 1 From the graph,
= 3 8 + (2 + 8) + 4 8 vertical change
2 2 Gradient =
= 12 + 16 + 16 horizontal change
= 44 m 9
=
1
Total distance
Average speed = =9
Total time
44 This acceleration of the particle at t = 4.5 is 9 m/s2.
= (iv) From the graph, when v < 10, 0.25 < t < 4.2.
9
= 4.89 m/s (to 3 s.f.)
(iii) Deceleration in the last 3 seconds = deceleration in the last 4 seconds Practise Now 9
8–0
=
9–5 Parking Charges ($)
= 2 m/s2
Carpark Y
8
Practise Now 8
Carpark X
(a) When t = 2, 6
v = 2(2)2 – 9(2) + 12 = 2
a=2
4
When t = 5,
v = 2(5)2 – 9(5) + 12 = 17
b = 17 2

Duration
0 1 2 3 4 5 6 (h)

Carpark Y:

When the duration is 3 h, the parking charge is


0.025 × (3 × 60 – 12) = $4.20
When the duration is 6 h, the parking charge is
0.025 × (6 × 60 – 12) = $8.70
From the graph, when the duration is 2 h,
Carpark X charges $2.40 while Carpark Y charges $2.70.
Mr Wong should park in Carpark X.

181
Practise Now 10 (c) (i) From the graph, when x = 1.5, y = 3.5.
(ii) From the graph, when y = 12, x = 2.3.
(a) From the graph, the resting heart rate is 60 beats/minute.
(b)
2. (a) When x = –0.5,
y = 2(–0.5)2 + 0.5 = 2.75
10th minute to the 20th minute.
p = 2.75
120 – 60
Gradient = (b)
20 – 10
= 6 beats/minute2 Scale: y
x-axis: 4 cm represent 1 unit
time is 6 beats/minute2. y-axis: 1 cm represents 5 units 20
y = 2x3 + 3
(c) 15
last 20 minutes, we need to calculate the gradient of the line from 10
the 40th minute to the 60th minute. 5
140 – 120 x
Gradient = 0
60 – 40 –2 –1 1 2
–5
= 1 beat/minute2
The rate of decrease in his heart rate as he slows down in the last –10

20 minutes is 1 beat/minute2. –15

Exercise 7A (c) (i) From the graph, when x = –1.2, y = –0.5.


(ii) From the graph, when y = 14, x = 1.75.
1. (a) y = x3 4
3. (a) y =
x
x –3 –2 –1 0 1 2 3

y –27 –8 –1 0 1 8 27 1 1
x 1 2 3 4 5
4 2
(b) Scale:
y
x-axis: 2 cm represent 1 unit y 16 8 4 2 1.3 1 0.8
y-axis: 2 cm represent 5 units
27 (b)
25 y Scale:
x-axis: 2 cm represent 1 unit
16 y-axis: 1 cm represents 1 unit
20 y = x3
15
14
15
13
y= 4
12 x
10
11
10
5
9
8
x
–3 –2 –1 0 1 2 3 7
6
–5
5
4
–10
3
2
–15
1
x
0 1 2 3 4 5
–20

(c) (i) From the graph, when x = 3.6, y = 1.1.


–25 (ii) From the graph, when y = 1.5, x = 2.65.
–27

182
4. (a) When x = 3, 5. y = 3x – x3
10
y = 2 = 1.1 (to 1 d.p.)
3 x –3 –2 –1 0 1 2 3
a = 1.1 y 18 2 –2 0 2 –2 –18
When x = 5,
10 y
y = 2 = 0.4
5 Scale:
b = 0.4 20 x-axis: 2 cm represent 1 unit
(b) y-axis: 2 cm represent 5 units
y
Scale: 15
x-axis: 2 cm represent 1 unit y = 3x – x3
10 y-axis: 2 cm represent 1 unit
10
9
5
8
x
y = 102 –3 –2 –1 0 1 2 3
7 x
–5
6
–10
5
–15
4
–20
3
(i) From the graph, when x = 1.4, y = 1.5.
(ii) From the graph, when y = –6.6, x = 2.4.
2

x
0 1 2 3 4 5

(c) (i) From the graph, when x = 2.8, y = 1.3.


(ii) From the graph, when y = 4.4, x = 1.5.

183
6. y = x3 – 6x2 + 13x 2
7. y = – –1
x
x 0 1 2 3 4 5

y 0 8 10 12 20 40 1 1 1 1
x 1 1 2 2 3 3 4
2 2 2 2
y
Scale:
y –5 –3 –2.33 –2 –1.8 –1.67 –1.57 –1.5
x-axis: 2 cm represent 1 unit
40 y-axis: 1 cm represents 5 units
y

35
y = x3 – 6x2 + 13x x
0 1 2 3 4
30

25
–1

20

15
–2

10 y=–2 –1
x

5
–3

x
0 1 2 3 4 5

(a) (i) From the graph, when x = 1.5, y = 9.5.


(ii) From the graph, when x = 3.5, y = 15. –4
(iii) From the graph, when x = 4.45, y = 27.
(b) (i) From the graph, when y = 7, x = 0.8.
(ii) From the graph, when y = 15, x = 3.5. Scale:
x-axis: 4 cm represent 1 unit
(iii) From the graph, when y = 22, x = 4.15. –5 y-axis: 4 cm represent 1 unit

(i) From the graph, when x = 2.5, y = –1.8.


(ii) From the graph, when y = –1.6, x = 3.35.

184
8. (a) When x = 3, 3
9. y = 2 –
3 x2
y=3– =2
3
h=2 x 1 2 3 4 5 6
When x = 3, y –1 1.25 1.67 1.81 1.88 1.92
3
y=6– = 5.5
6 y
k = 5.5
(b)
y 2

y = 2 – 32
x
5

1
4
y=x– 3
x
3

x
0 1 2 3 4 5 6
2

1
Scale:
–1 x-axis: 2 cm represent 1 unit
x y-axis: 4 cm represent 1 unit
0 1 2 3 4 5 6

–1 (i) From the graph, when x = 1.5, y = 0.65.


(ii) From the graph, when y = 1.5, x = 2.45.
–2

–3

–4 Scale:
x-axis: 2 cm represent 1 unit
y-axis: 2 cm represent 1 unit
–5

(c) (i) From the graph, when x = 1.6, y = –0.3.


(ii) From the graph, when y = –2.5, x = 0.9.

185
2 11. y = x3 – 2x – 1
10. y = x +
x2
x –3 –2 –1 0 1 2 3
x 1 2 3 4 5 6
y –22 –5 0 –1 –2 3 20
y 3 2.5 3.2 4.1 5.1 6.1
y

y
20
Scale:
7 x-axis: 2 cm represent 1 unit
y-axis: 2 cm represent 1 unit 15
y = x3 – 2x – 1
6
10
5
5
y=x
4
y = x + 22 x
x –3 –2 –1 0 1 2 3
3
–5
2
–10
1
–15 Scale:
x x-axis: 2 cm represent 1 unit
0 1 2 3 4 5 6 y-axis: 2 cm represent 5 units
–20
(i) From the graph, when x = 5.4, y = 5.5.
(ii) From the graph, when y = 3, x = 1 or 2.75. (a) From the graph, the x-coordinates of the points of intersection
of the curve with the x-axis are –1, –0.6 and 1.6.
(b) (i) y = x

x –3 –2 –1 0 1 2 3

y –3 –2 –1 0 1 2 3

From the graph, the x-coordinates of the points at which


the line meets the curve are –1.5, –0.35, and 1.9.
(ii) Hence, the solutions of the equation are –1.5, –0.35 and 1.9.

186
12. (a) When x = 2.5,
1
y = 2.5 + – 1 = 1.7
2(2.5)
p = 1.7
(b) y

5 Scale:
x-axis: 4 cm represent 1 unit
y-axis: 4 cm represent 1 unit

y=x+ 1 –1
2x
2

x
0 1 2 3 4

1
(c) x+ =1
2x
1
x+ –1=0
2x
y =0
From the graph, in the range 0.1 ! x ! 4, there are no solutions.

187
1 2 8 1 2 8
13. y = x + –9 (ii) x + =x+4
4 x 4 x
1 8
x 0.5 1 2 3 4 5 6 7 x2 + –9 =x+4–9
4 x
y 7.1 –0.8 –4 –4.1 –3 –1.2 –1.3 4.4 1 8
x2 + –9 =x–5
4 x
y y =x–5
Scale:
x-axis: 2 cm represent 1 unit x 0.5 1 2 3 4 5 6 7
8
y-axis: 1 cm represents 1 unit
7 y – 4.5 –4 –3 –2 –1 0 1 2
6
5 A line y = x – 5 is drawn.
4 From the graph, the solution is x = 1.6 or 5.8.
y = 1 x2 + 8 – 9
3
4 x 1 2 8
(iii) x + 2x = 15 –
4 x
2 y=x–5
1 2 8 8 8
1 x + 2x + = 15 – +
4 x x x
x 1 2 8
0 1 2 3 4 5 6 7 x + 2x + = 15
–1 4 x
–2 1 2 8
x + 2x + – 2x = 15 – 2x
4 x
–3 y = –3
1 2 8
–4 x + = 15 – 2x
4 x
–5 1 2 8
x + – 9 = 15 – 2x – 9
–6 4 x
–7 y = 6 – 2x 1 2 8
x + – 9 = 6 – 2x
4 x
–8
y = 6 – 2x

(a) From the graph, x 0 1 2 3 4 5


The minimum value of y in the given range 0.5 ! x ! 7 is –4.2.
y 0 8 10 12 20 40
1 2 8
(b) (i) x + =6
4 x A line y = 6 – 2x is drawn.
1 2 8 From the graph, the solution is x = 0.6 or 4.3.
x + –9 =6–9
4 x
1 2 8
x + – 9 = –3
4 x
y = –3
A line y = –3 is drawn.
From the graph, the solution is x = 1.5 (to 1 d.p.) or 4.

188
Exercise 7B
1. (a) y = 4x

x –1 –0.5 0 0.5 1 1.5 2 2.5

y 0.25 0.5 1 2 4 8 16 32

(b)
y
Scale:
32 x-axis: 4 cm represent 1 unit
30 y-axis: 1 cm represents 2 units

28
26 y = 4x
24
22
20
18
16
14
12
10
8
6
4
2
x
–1 0 1 2 3

(c) (i) From the graph, when x = 1.8, y = 12.


(ii) From the graph, when y = 0.4, x = –0.65.

189
2. (a) y = 3(2x)

x –1 –0.5 0 0.5 1 1.5 2 2.5

y 1.5 2.1 3 4.2 6 8.5 12 17.0

(b)
y
Scale:
18 x-axis: 4 cm represent 1 unit
y-axis: 1 cm represents 1 unit
17
16
y = 3(2x)
15
14
13
12
11
10
9
8
7
6
5
4
3
2
1
x
–1 0 1 2 3

(c) (i) From the graph,


When x = 0.7, y = 4.9.
When x = 2.3, y = 14.8.
(ii) From the graph,
When y = 2.5, x = –0.25.
When y = 7.4, x = 1.30.

190
3. y = –2(3x)

x –2 –1.5 –1 0 1 1.5 2

y –0.2 –0.4 –0.7 –2 –6 –10.4 –18

x
–2 –1 0 1 2
–2
–4
–6
–8
–10
–12 y = –2(3x)
–14
Scale: –16
x-axis: 4 cm represent 1 unit
y-axis: 1 cm represents 2 units –18
–20

(i) From the graph, when x = 1.2, y = –7.4,


(ii) From the graph, when y = –6.7, x = 1.1.
4. (a) When x = –1,
y = 2 + 2–1 = 2.5
a = 2.5
When x = 2.5,
y = 2 + 22.5 = 7.7 (to 1 d.p.)
b = 7.7
(b) y
Scale:
10 x-axis: 4 cm represent 1 unit
y-axis: 2 cm represent 1 unit
y = 2 + 2x
9

x
–1 0 1 2 3

191
(c) (i) From the graph,
When x = –0.7, y = 2.6,
When x = 2.7, y = 8.5.
(ii) From the graph,
When y = 5.3, x = 1.725,
When y = 7.5, x = 2.45.
5. y = 3x

x –2 –1.5 –1 0 1 1.5 2

y 0.1 0.2 0.3 1 3 5.2 9


y
Scale:
x-axis: 4 cm represent 1 unit 10
y-axis: 1 cm represents 1 unit
9
y = 3x
8
7
6
5
4
3
2
1
x
–2 –1 0 1 2
–1
y = 1x – 1
–2 2 x

–3
–4
–5
–6
–7
–8
–9
–10

(a) From the graph, when y = 5.8, x = 1.6,


1 1
(b) y = x –
2 x

x –2 –1.5 –1 –0.5 –0.1 0.1 0.5 1 1.5 2

y –0.5 –0.08 0.5 1.75 9.95 –9.95 –1.75 –0.5 0.08 0.5

(i) From the graph, the coordinates of the point are (–1.15, 0.3).
1 1
(ii) 3x + – x=0
x 2
x 1 1
3 = x–
2 x
Hence, the solution of the equation is –1.15.

192
6. y = kax 8. (a) y = 12 + 10x – 3x2
y = 3ax
k=3 x –2 –1 0 1 2 3 4 5

y –20 –1 12 19 20 15 4 –13
7. (a) y = (x + 2)(4 – x)
y
x –2 –1 0 1 2 3 4
20
y 0 5 8 9 8 5 0 y = 12 + 10x – 3x2

15
(b) y
Scale: vertical
x-axis: 2 cm represent 1 unit 10 change
10
y-axis: 2 cm represent 1 unit =8

9 5 vertical
horizontal change
change = 0.8 = 17
8 x
–2 –1 0 1 2 3 4 5
y = (x + 2)(4 – x)

7 –5

horizontal
6 –10 change = 1.2

5 –15
vertical Scale:
change x-axis: 2 cm represent 1 unit
= 3.6 y-axis: 2 cm represent 5 units
4 –20

3 (b) A tangent is drawn to the curven at the point when x = 4.


horizontal From the graph,
change = 0.9
2 –17
Gradient = = –14 (to 2 s.f.)
1.2
(c) A tangent is drawn to the curve at the point where the curve
1
intersects the y-axis.
From the graph,
x 8
–2 –1 0 1 2 3 4 Gradient = = 10
0.8
(c) A tangent is drawn to the curve at the point where x = –1.
From the graph,
3.6
Gradient = =4
0.9
(d) (i) A line parallel to the x-axis the maximum point of the curve
has a gradient equal to zero.
(ii) From the graph and table, h = 1, k = 9.

193
1
9. y = 2x +
x2

x –2 –1 –0.5 –0.2 0.2 0.5 1 1.5 2 2.5 3

y 0.5 1.5 4.7 25.9 26.1 5.4 3 3.3 4.3 5.8 8.1

y
Scale:
30 x-axis: 4 cm represent 1 unit
y-axis: 1 cm represents 2 units
28
26
24 y = 2x + 12
x
22
20
18
16
14
12
10
8
6
4
2
x
–2 –1 0 1 2 3
–2
y=1–x
–4

(a) (i) y = 1 – x

x –2 0 2 3

y 3 1 –1 –2

1
(ii) 2x + –1+x =0
x2
1
2x + 2 = 1 – x
x
From the graph, the solution is –0.85.
1
(b) For all real values of x, 2x and 2 are always positive, hence y is always positive.
x
Hence the graph will not lie below the x-axis.

194
1
10. (a) y = 1 + Scale:
x x-axis: 2 cm represent 30 minutes
Distance (km)
y-axis: 1 cm represents 2 km
x 0.5 1 1.5 2 2.5 3
40
y 3 2 1.7 1.5 1.4 1.3

y
Scale:
x-axis: 4 cm represent 1 unit
3 y-axis: 4 cm represent 1 unit 30

2
20
y=1+ 1
x

10

x
0 1 2 3

Time
0 0900 0930 (h)
y = –x 1000 1030 1100 1130 1200
–1

(ii) Hence, from the graph, the time at which the cyclist reached his
(b) y = –x destination is 1153.
45
2. (i) Distance travelled between 0900 and 0945 = 20
x 0.5 1 1.5 60
= 15 km
y –0.5 –1 –1.5
(ii) Distance travelled after the rest = 30 – 15
(c) A line parallel to the line y = –x at a point of the curve has a = 15 km
gradient equal to –1. 15
Time taken for the distance travelled after the rest =
From the graph, the coordinates are (1, 2). 30
= 0.5 h
Exercise 7C Time at which he continued on this journey after rest = 1050

1. (i) Distance travelled between 0900 and 1030 = 15 1.5


= 22.5 km
Distance travelled after the rest = 40 – 22.5
= 17.5 km
Time taken for the distance travelled after the rest
17.5
=
20
= 0.875 h
= 52.5 minutes

195
Distance (km) Scale:
x-axis: 2 cm represent 30 minutes
y-axis: 1 cm represents 2 km
30

20

10

Time (h)
0 0900 0930 1000 1030 1100 1130

(iii) Hence, from the graph, the duration of his stop is 65 minutes.
3. (i) From the graph, the car is not moving between t = 1.5 and
t = 2.5.
Total distance
(ii)
Total time
60
=
2
= 30 km/h
(iii) Average speed of car for the whole journey
Total distance
=
Total time
120
=
3.5
= 34.3 km/h (to 3 s.f.)
60
(iv) Speed of car between t = 0 and t = 1.5 =
1.5
= 40 km/h
120 – 60
Speed of car between t = 2.5 and t = 3.5 =
1
= 60 km/h

196
Speed (km/h) Scale:
x-axis: 4 cm represent 1 hour
y-axis: 2 cm represent l0 km/h
60

50

40

30

20

10

Time (h)
0 1 2 3 4

10 m/s
4. (i) Acceleration =
2s
= 5 m/s2
(ii) Total distance = area under graph
1 1
= 2 10 + (3 10) + 1 10
2 2
= 10 + 30 + 5
= 45 m
Total distance
Average speed during the whole journey =
Total time
45
=
6
= 7.5 m/s
m/s
5. (i) Gradient of OA =
s
= m/s2
= Acceleration between t = 0 and t = 30
14 m/s
(ii) Acceleration between t = 0 and t = 30 =
30 s
7
= m/s2
15
Let the speed of the car when t = 15 be x m/s.
Acceleration between t = 0 and t = 15
= Acceleration between t = 0 and t = 30
7
=
15
x 7
=
15 15
x =7
The speed of the car when t = 15 is 7 m/s.

197
6. (i)
h (m) Scale:
x-axis: 2 cm represent 5 seconds
y-axis: 1 cm represents 5 metres

60
vertical
change = 23
50

horizontal change = 6.5


40

30

20

10

t (s)
0 5 10 15 20 25 30

(ii) From the graph, at t = 8,


23 m
Gradient = = 3.54 m/s (to 3 s.f.)
6.5 s
The gradient of the tangent at t = 8 represents the speed at that
particular point. It is called the instantaneous speed.
(iii) After 25 s, the worker will have descended 0.8 25 = 20 m.
Hence at t = 30 s, the worker is at the 20-metre level. The
distance-time curve of the worker is drawn onto the graph.
From the graph, the worker and the lift are at the same height
at t = 25.5.
7. (a)
Distance (km) Scale:
x-axis: 2 cm represent 1 minute
y-axis: 4 cm represent 1 km
3

vertical
change
= 0.75
Time
0 1 2 3 4 5 6 (minutes)
horizontal
change = 1.7
(b) (i) From the graph,
The approximate time taken to travel the first 1 km
= 2.3 minutes

198
1
(ii) From the graph, at t = 1 ,
2
0.75
Gradient = = 0.44 km/min (to 3 s.f.)
1.7
1
The gradient of the tangent at t = 1 represents the speed at that particular point.
2
(iii) From the graph,
Time taken to travel the last 1 km = 6 – 3.25
= 2.75 minutes
32
8. (a) Time taken for Ethan = = 1.6 h
20
32
Time taken for Michael = = 4.6 h (to 1 d.p)
7
Distance (km)

Scale:
32 x-axis: 2 cm represent 1 hour
y-axis: 1 cm represents 2 km
Ethan
Michael
28

24

20

16

12

Time (h)
0 1 2 3 4 5

(b) (i) From the graph,


Time taken for Ethan and Michael to pass each other = 1 h 11 min
(ii) From the graph,
Times when Ethan and Michael will be 5 km apart = 1 h and 1 h 22 min
1
9. (a) = 9 km
2
11
Time taken for Shirley to travel the remaining 11 km = = 1.375 h
8
20
Time taken for Kate to travel 20 km = = 2.9 h (to 1 d.p.)
7

199
Distance (km)
Scale:
x-axis: 6 cm represent 1 hour
20
y-axis: 1 cm represents 1 km

Kate Shirley

15

10

Time (h)
0 0900 1000 1100 1200

(b) (i) From the graph, the time at which Shirley and Kate meet is 1011.
(ii) From the graph, the distance away from Kate’s home when they meet is 8.3 km.

10.
10 000
= 500 cm3/s
20
Volume of each container = 10 000 cm3
For container B,
30 30 10
= 18 s
500
Volume of container B = 30 30 10 + 5 20 10 = 10 000 cm3
For container C,
1
10 000 – 32 10 40
2 = 7.2 s
500
1
10 000 – 16 10 40
2 = 13.6 s
500

200
Depth (d cm)

50

40
B
A C
30

20

10

Time
0 2 4 6 8 10 12 14 16 18 20 (t seconds)

11. (i) Given that the object slows down at a rate of 12 m/s2,
Deceleration = 12 m/s2
36 – 0
= 12
t–6
36 = 12(t – 6)
36 = 12t – 72
12t = 108
t =9
(ii) Given that the distance travelled when the object is slowing down is 54 m,
Total distance travelled = 36 6 + 54
= 270 m
270
Average speed for the whole journey =
9
= 30 m/s
30 – 0
12. (i)
20
= 1.5 m/s2
(ii) Given that the train decelerates at a rate of 0.75 m/s2,
Deceleration = 0.75 m/s2
30 – 0
= 0.75
t – 60
30 = 0.75(t – 60)
30 = 0.75t – 45
0.75t = 75
t = 100
The time taken for the whole journey is 100 s.

201
13. (a) When t = 3, 14. (i)
v = 32 – 7(3) + 16 = 4 v (m/s)
a=4 Scale:
x-axis: 1 cm represents 1 second
When t = 6, 45 y-axis: 2 cm represent 5 m/s
v = 62 – 7(6) + 16 = 10
b = 10 40
(b)

v (m/min) 35
Scale:
x-axis: 2 cm represent 1 minute
16 y-axis: 1 cm represents 1 m/min
30
vertical change
14 =9
25

12
20 horizontal change
= 1.8
v = t2 – 7t + 16
10
15

8
10
vertical
change = 5.5
6
vertical 5
change horizontal change = 2.2
4 = 3 m/min
t (s)
0 2 4 6 8 10 12
horizontal change
2 (ii) From the graph,
= 1 minute
Speed when t = 5 is 9.5 m/s
t (minute) Speed when t = 11 is 34 m/s
0 1 2 3 4 5 6
(iii) A tangent is drawn to the curve at t = 4.
(c) (i) From the graph, From the graph,
When v = 7, t = 1.7, 5.3. 5.5 m/s
Gradient = = 2.5 m/s2
(ii) From the graph, 2.2 s
The time at which the speed is a minimum is at 3.5 s. Acceleration at t = 4 = 2.5 m/s2
(iii) A tangent is drawn to the curve at t = 2. A tangent is drawn to the curve at t = 10.
From the graph, From the graph
–3 m/min 9 m/s
Gradient = = –3 m/min2 Gradient = = 5 m/s2
1 min 1.8 s
The gradient of the tangent at t = 2 represents the Acceleration at t = 10 = 5 m/s2
deceleration at that particular point. 15. (a) When t = 2,
(iv) From the graph, the time interval when the speed is not v = 3(2)2 – 17(2) + 30 = 8
more than 5 m/min is 2.4 < t < 4.6. h=8
When t = 4,
v = 3(4)2 – 17(4) + 30 = 10
k = 10

202
(b) v (m/s) Since the gradient of a speed-time graph gives the acceleration,
v 30
Scale: =
45 10 45
x-axis: 2 cm represent 1 hour
y-axis: 2 cm represent 5 km/h 30
v = 10
45
40
2
= 6 m/s
3
35 2
Hence the speed after 10 seconds is 6 m/s.
3
30 17. (a) From the graph,
Postage to mail a letter with a mass of 50 g = 55 cents
(b) From the graph,
25
For Company A,
Postage to mail a letter with a mass of 220 g = 280 cents
20
= $2.80
For Company B,
vertical
15 Postage to mail a letter with a mass of 220 g
change
= 12 = $1 + $0.01 (220 – 80)
10 = $2.40
Company B offers a lower postage.
horizontal change
18. (a) From the graph, the coach stopped between 1330 and 1400.
5 = 1.2
Duration of stop = 30 minutes
50 – 40
t (h) (b) Initial acceleration of the coach = = 20 km/h2
0 1 2 3 4 5 0.5
(c)
(c) (i) From the graph, the time at which the speed is a minimum Speed (km/h)
is at t = 2.8 s.
(ii) A tangent is drawn to the curve at t = 4.5. 50
From the graph, 40
12 km/h 30
Gradient = = 10 km/h2
1.2 h 20
The gradient of the tangent at t = 4.5 represents the
10
acceleration at that particular point.
Time (h)
(iii) From the graph, the time interval when the speed does not 0 1200 1300 1400 1500 1600 1700
exceed 10 km/h is 1.65 < t < 4.
Distance travelled
(d) From the graph, the value of t at which both objects have the
= area under the graph
same speed is 0.4.
1 1 1 1 1
16. Speed (m/s) = (40 + 50) + 50 + 50
2 2 2 2 2
1 1 1 1 1 1
+ 30 + 30 + (30 + 20) +
2 2 2 2 2 2
30
1 1 1 18
(20 + 50) + 50 + 50
2 2 2 60
= 22.5 + 25 + 12.5 + 7.5 + 15 + 12.5 + 17.5 + 25 + 7.5
= 145 km
The distance between Blue Town and Summer City is less
than 250 km.
v
(d) From the graph, the time when coach reached Summer City is
at 1648.
Time (s)
0 10 45

203
19. 20. (a)
rate, Scale:
v (cm/s)
x-axis: 2 cm represent 1 second
14 y-axis: 2 cm represent 2 cm/s
h (cm)

12 vertical
8 change
= 1.6
10 horizontal change
= 1.6
vertical
8 change
4
= 3.6

6
horizontal change
= 1.2
4
t (s)
0 48 60

2
For the second container,
h (cm)
t (s)
0 1 2 3 4 5 6 7

8 (b) (i) A tangent is drawn to the curve at t = 2.


From the graph,
3.6 cm/s
Gradient = = 3 cm/s2
1.2 s
Acceleration at t = 2 = 3 cm/s2
4
A tangent is drawn to the curve at t = 6
From the graph,
–1.6 cm/s
Gradient = = –1 cm/s2
1.6 s
t (s) Acceleration at t = 6 = –1 cm/s2
0 60
(ii) From the graph, the time interval when the speed is greater
For the third container, than 11 m/s is 3.3 < t < 6.7.
(c) v = at2 + bt + c
h (cm)
When t = 0, v = 0, c = 0.
When t = 1, v = 4.5,
8 4.5 = a + b
a = 4.5 – b —(1)
When t = 2, v = 8,
8 = a(2)2 + 2b
4a + 2b = 8
4
2a + b = 4 —(2)
Substitute (1) into (2):
2(4.5 – b) + b = 4
9 – 2b + b = 4
t (s) 9–b =4
0 60
b =5
a = 4.5 – 5
= –0.5
v = –0.5t2 + 5t
a = –0.5, b = 5, c = 0

204
8 (b) y Scale:
21. Between t = 0 and t = 2, speed = = 4 m/s
2 x-axis: 1 cm represents 1 unit
18 – 8 45 y-axis: 1 cm represents 5 units
Between t = 3 and t = 4, speed = = 10 m/s
1 40
Speed (m/s)
35
30
10 25
20
15
10
5
4 x
–3 –2 –1 0 1 2 3 4
–5
–10
Time (s) –15
0 1 2 3 4 5 6
–20

22. (a) v = 6 + 2t –25

v (m/s) –30

12
(c) (i) From the graph, when x = 1.8, y = –9.5.
(ii) From the graph, when y = 10, x = 3.1.
2. (a) y = x(x – 2)(x + 2)

x –3 –2 –1 0 1 2 3
6
y –15 0 3 0 –3 0 15

(b) y
Time (s)
0 1 2 3
15
(b) When t = 3, v = 6 + 2(3) = 12 m/s
12 – 6
(c) Acceleration = = 2 m/s2 10
3
a (m/s2)
5

x
–3 –2 –1 0 1 2 3
2
y = x(x – 2)(x + 2)
–5

Time (s)
0 1 2 3 –10
Scale:
x-axis: 2 cm represent 1 unit
–15 y-axis: 2 cm represent 5 units
Review Exercise 7
1. (a) y = x3 – 3x – 10 (c) (i) From the graph, when x = 1.4, y = –2.75.
(ii) From the graph, when y = 4.5, x = 2.4.
x –3 –2 –1 0 1 2 3 4
(iii) From the graph, the solutions to the equation
y –28 –12 –8 –10 –12 –8 8 42 x(x – 2)(x + 2) = 0 are –2, 0 and 2.

205
1
3. y = 1 – 2x –
x

x –4 –3.5 –3 –2 –1 –0.5 –0.25

y 9.3 8.3 7.3 5.5 4 4 5.5

y
Scale:
x-axis: 4 cm represent 1 unit
y-axis: 2 cm represent 1 unit
10

9
y = 1 – 2x – 1
x
8

x
–4 –3 –2 –1 0

(a) (i) From the graph, when x = –0.75, y = 3.85.


(ii) From the graph, when y = 4.5, x = –0.375 or –1.375.
(b) From the graph, the coordinates of the point on the curve where the tangent to the curve is a horizontal line are (–0.7, 3.8).

206
4. y = 3x – 2

x –1.5 –1 –0.5 0 1 1.5 2

y –1.8 –1.7 –1.4 –1 1 3.1 7

y
Scale:
x-axis: 4 cm represent 1 unit
y-axis: 2 cm represent 1 unit
7

y = 3x – 2
6

y = 2x
2

x
–2 –1 0 1 2

–1

–2

(i) 3x = 2
3x – 2 = 0
y =0
From the graph, when y = 0, x = 0.625.
(ii) y = 2x

x –1 0 1

y –2 0 –1

A tangent parallel to the line y = 2x is drawn to the curve.


From the graph, the coordinates of the point on the graph of y = 3x – 2 where the gradient of tangent is 2 are (0.55, –0.2).

207
3
5. y = x – 2 +
x

x 0.5 1 1.5 2 2.5 3 3.5 4

y 4.5 2 1.5 1.5 1.7 2 2.4 2.8

5 y=6–x

y=x–2+ 3
x
4

vertical
2 change
= 0.8

horizontal change
= 1.2

x
0 1 2 3 4

(i) From the graph, the minimum value of y = 1.45 where x = 1.75.
(ii) From the graph, the range of values of x for which y < 2.2 is 0.9 < x < 3.275.
(iii) A tangent is drawn to the curve at the point where x = 3.
From the graph,
0.8
Gradient = = 0.67
1.2
3
(iv) 2x + =8
x
3
2x + –x =8–x
x
3
x+ =8–x
x
3
x+ –2=8–x–2
x
3
x–2+ =6–x
x
y =6–x

208
x 1 2 3 4

y 5 4 3 2

From the graph, the value of x = 3.575.


20
6. (i) Distance travelled by coach from airport to hotel = 45
60
= 15 km
15
Time taken by coach from hotel to airport =
60
1
= h
4
Distance (km) Scale:
x-axis: 2 cm represent 10 minutes
y-axis: 1 cm represents 5 km
15

10

Time
0 10 20 30 40 50 60 (minutes)

Total distance
(ii) Average speed =
Total time
(15 + 15) km
=
65
km
60
= 27.7 km/h (to 3 s.f.)
7. (a) (i) From the graph, the time interval during which the vehicle stopped to unload goods
is 1 ! t ! 2.
60 – 40
(ii) Speed when t = 3 =
3.5 – 2
1
= 13 km/h
3
(iii) Maximum speed during journey = Speed between t = 0 and t = 1
40
=
1
= 40 km/h
Total distance
(iv) Average speed =
Total time
60 km
=
3.5 h
= 17.1 km/h (to 3 s.f.)

209
(b) Speed (m/s) 9. (i) Total distance travelled
= area under the graph
1 10 10 10 1
40 = 60 + 60 + 60 + 60 5+
2 60 60 60 2
5
60
60
= 5 + 10 + 10 + 5 + 2.5
= 32.5 km
13 1 (ii) Average speed =
Total distance
3
Total time
32.5
=
25
Time (h) 60
0 1 2 3.5
= 78 km/h
k
10. (i) I =
D2
4–0 When D = 20, I = 800,
8. (i) Acceleration =
2 k
800 =
= 2 m/s2 20 2
(ii) Greatest acceleration = acceleration from 4 s to 6 s k = 800 202
10 – 4 = 320 000
=
2 320 000
(ii) I =
= 3 m/s2 D2
(iii) Total distance moved
= area under the graph D 10 16 20 25 33 40
1 1 I 3200 1250 800 512 294 200
= 2 4+2 4+2 4+ 2 6
2 2
= 4+8+8+6
I (units)
= 26 m
Scale:
Total distance x-axis: 4 cm represent 10 cm
(iv) Average speed = 3500
Total time y-axis: 2 cm represent 250 units
26
= 3000
6
1
=4 m/s
4 2500

2000

1500

1000

500

D
0 10 20 30 40

(iii) From the graph, when D = 30 cm, I = 350 units.


(iv) I is inversely proportional to D2.

210
11. (a)
(70 – 20)°C
=
10 minutes
= 5°C/minute
(b) The fresh vegetables from the refrigerator caused the drop in
temperature between the 10th and 11th minute when they were
added into the soup.
(c)
(90 – 80)°C
=
5 minutes
= 2°C/minute

Challenge Yourself
Since the gradient of a speed-time graph gives the acceleration,

Between t = 0 and t = 2:

The object is undergoing an increasing acceleration, as such the gradient


of the speed-time graph increases at an increasing rate during this period.

Between t = 2 and t = 4:
Where the graph is below the x-axis, the object is undergoing decreasing
deceleration until it reaches zero acceleration. The gradient of the speed-
time graph decreases at a decreasing rate during this period.

Where the graph is above the x-axis, the object is undergoing increasing
acceleration. The gradient of the speed-time graph increases at an
increasing rate during this period. The gradient is steeper than that
between t = 0 and t = 2.

Between t = 4 and t = 6:

The object is undergoing constant deceleration, as such the gradient is


decreasing at a constant rate.
Speed (m/s)

Time (s)
0 2 4 6

Teachers should note that this is one possible speed-time graph for the
motion of the object. As long as the students are able to give the correct
shape for each time period of the speed-time graph, that is a possible
speed-time graph.

211
Revision Exercise B1 5. (i) 3x + 4y = 24
Where l crosses the x-axis at the point A, y = 0
1. 100% of cost price = $160
3x + 4y = 24
$160
1% of cost price = 3x + 4(0) = 24
100
3x = 24
$160
145% of cost price = 145 x =8
100
= $232 The coordinates of the point A are (8, 0).
The article was sold to the dealer at $232. Where l crosses the y-axis at the point B, x = 0
($266.80 – 232) 3x + 4y = 24
100% 3(0) + 4y = 24
$232
= 15% 4y = 24
2. Cost of 3 kg of Coffee A = 3 $9 y =6
= $27 The coordinates of the point B are (0, 6).
Cost of 1 kg of Coffee B = $13 (ii) Length of perpendicular from B to AC = 13 units
Total cost of 4 kg of mixture = $27 + $13 1
Area of !ABC = 13 6
= $40 2
$40 = 39 units2
Total cost of 100 g mixture = 100 0–6
4000 (iii) Gradient of BC = = 1.2
= $1 –5 – 0
$(1.25 – 1) (iv) AB = (8 – 0)2 + (0 – 6)2
100%
$1
= 25% = 8 2 + (–6)2
12 – 7 5 = 100
3. Gradient of AB = =
8–5 3 = 10 units
5 Let the length of the perpendicular from C to AB be x units.
Equation of AB is in the form y = x + c
3
Area of !ABC = 39 units2
Since (5, 7) lies on the line,
1
5 x 10 = 39
7 = (5) + c 2
3
5x = 39
25
7 = +c x = 7.8
3
The length of the perpendicular from C to AB is 7.8 units.
4
c =– Total distance
3 6. (i) Jun Wei’s average speed =
5 4 Total time
y = x– 40
3 3 =
3y = 5x – 4 3
1
4. (i) Length of perpendicular from C to AB = 5 units = 13 km/h
3
1
Area of !ABC = 4 5 (ii) From the graph, the time when Rui Feng and Jun Wei meet is
2
1030 and they are 20 km from the airport when they met.
= 10 units2
(iii) From the graph, Rui Feng took a rest between 1000 and 1100.
(ii), (iii)
Hence, the time interval during which Rui Feng took a rest is
y
1 h.
(iv) From the graph, when Jun Wei reaches the train station, Rui
D C P
6 Feng is 10 km away from the airport.
2
4 7. (i)
V–4
2 = 1.5
A
8
B
x V – 4 = 12
–2 0 2 4 6 8 V = 16
From graph, D(0, 6) and P(8, 6).

212
(ii) Acceleration between 8 s and t s = –1
0 – 16
= –1
t–8
–16 = –t + 8
t = 24
8. (a) When x = 1.5,
12
y= + 1.5 – 6 = 3.5
1.5
h = 3.5
When x = 7,
12
y= + 7 – 6 = 2.7 (to 1 d.p.)
7
k = 2.7
(b) y
Scale:
x-axis: 2 cm represent 1 unit
8 y-axis: 2 cm represent 1 unit

6 12
y= +x–6
x

y=4
4

vertical change
1 = 1 unit

horizontal change = 2 units


x
0 1 2 3 4 5 6 7 8

(c) (i) From the graph, when x = 2.3, y = 1.5.


(ii) From the graph, the minimum value of y = 0.9.
(d) A tangent is drawn to the curve at the point where x = 5.
From the graph,
1
Gradient = = 0.5
2
2
(e) x + 12 = 10x
12
x+ = 10
x
12
+ x – 6 = 10 – 6
x
y =4
From the graph, the solution of the equation x2 + 12 = 10x is 1.4.

213
Revision Exercise B2 Substitute (1) into (2):
(–y) – 2y = – 4
1. (i) 96% of cost price = $408
–3y = – 4
$408
1% of cost price = 1
96 y =1
3
$408
100% of cost price = 100 1
96 x = –1
3
= $425
1 1
The cost price of the vase is $425. The coordinates of the point M are –1 , 1 .
3 3
$(510 – 425)
(ii) Percentage gain = 100% 1
$425 (iii) Area of !PMO = PO h
2
= 20%
1 1
25 = 4 1
2. Commission for the month = $5264 2 3
100 2 2
= $1316 = 2 units
3
Total income for the month = $520 + $1316
5. (i) DC = 13 units
= $1836
3. 5x + 7y = 46 (0 – a) + (6 – 4)2 = 13
2

7y = –5x + 46 (–a)2 + 22 = 13
–5 x + 46 a2 + 4 = 13
y= a2 = 9
7
5 46 a = 3 (since a > 0)
y =– x+
7 7 The coordinates of the point C are (3, 4).
5 (ii) Let the coordinates of the point A be (0, y).
Gradient of line = –
7 2
Gradient of AB =
5 3
Given that the line passes through (2, –5) and the gradient = – ,
7 –1 – y 2
=
y = mx + c 3– 0 3
5 –1 – y 2
–5 = – (2) + c =
7 3 3
10 –1 – y = 2
–5 = – +c
7 y = –3
25 The coordinates of the point A are (0, –3).
c =–
7 1
(iii) Area of trapezium ABCD = (AD + BC) h
5 25 2
y =– x–
7 7 1
= (9 + 5) 3
7y = –5x – 25 2
5x + 7y + 25 = 0 = 21 units2
4. (i) x – 2y = – 4
(iv) AB = (3 – 0)2 + [–1 – (–3)]2
Where the line crosses the x-axis at the point P, y = 0
x – 2(0) = – 4 = 32 + 2 2
x = –4
= 13
The coordinates of the point P are (– 4, 0).
= 3.61 units (to 3 s.f.)
Where the line crosses the y-axis at the point Q, x = 0
2
0 – 2y = – 4 (v) Gradient of AB =
3
–2y = – 4 y-intercept = –3
y =2 2
The coordinates of the point Q are (0, 2). y= –3
3
(ii) For M to be equidistant from the coordinate axes, 3y = 2x – 9
Vertical distance of M from the x-axis = Vertical distance of M
from the y-axis
x = –y — (1)
x – 2y = – 4 — (2)

214
6. (a) Time taken by the coach to travel from Watertown to Sandcity 8. (i) When x = 6.5,
120 1
= y = (6.5)[15 – 2(6.5)] = 6.5
50 2
= 2.4 h p = 6.5
Time taken by the car to travel from Sandcity to Watertown (ii)
120 y Scale:
= x-axis: 2 cm represent 1 unit
80
y-axis: 1 cm represents 1 unit
= 1.5 h 16

Distance (km) Scale: 15


x-axis: 2 cm represent 30 minutes 14
y-axis: 1 cm represents 10 km
120 13 vertical
1 change
car coach 12 y = 11
2 = 5 units
100 11
10
80 9 horizontal
change
8
= 2 units
60 7
6
40 5 1
y = x(15 – 2x)
2
4
20 3
2
Time (h) 1
0 1100 1130 1200 1230 1300 1330 1400
x
(b) (i) From the graph, the time when the car meets the coach is 0 1 2 3 4 5 6 7
1233 and the distance from Watertown at this instant is
(iii) x(15 – 2x) = 23
77 km. 1 1
(ii) From the graph, the distance between the coach and the car x(15 – 2x) = (23)
2 2
at 1300 is 60 km. 1 1
7. (i) Let the total time taken for the journey be t s. x(15 – 2x) = 11
2 2
Deceleration = 0.6 m/s2 1
y = 11
27 – 0 2
= 0.6
t – 50 From the graph, the solutions of the equation x(15 – 2x) = 23
27 = 0.6(t – 50) are 2.15 and 5.35.
27 = 0.6t – 30 (iv) A tangent is drawn to the curve at the point (5, 12.5).
0.6t = 57 From the graph,
t = 95 s 5
Gradient = – = –2.5
= 1 minute 35 seconds 2
The total time taken for the journey is 1 minute 35 seconds. (v) From the graph, the maximum value of y = 14.1 where x = 3.75.
27 – 45 (vi) Hence, when the area is a maximum, the area is 14.1 cm2 and
(ii) = –0.9 m/s2
20 the length of one side of the rectangle is 3.75 cm.
Acceleration (m/s2) 14.1
Length of other side of the rectangle = 3.76 cm
3.75
The dimensions of the rectangle are 3.75 cm by 3.76 cm.
Time (s) The rectangle is a square.
20 50 95
Teachers may wish to highlight to the students that although
– 0.6 the dimensions differ by 0.01 cm, based on calculations instead
of using the graph, the rectangle is a square when the area is
– 0.9
a maximum.

215
Chapter 8 Further Trigonometry
TEACHING NOTES
Suggested Approach
Teachers may want to introduce this topic by asking students how measurements are obtained for towers and buildings in real
life. Teachers can mention that such measurements can be easily obtained using trigonometry before introducing Sine Rule and
Cosine Rule to them. This chapter exposes students to problems involving triangles which can be solved using trigonometric
ratios of acute and obtuse angles.

Section8.1: Sine and Cosine of Obtuse Angles


Teachers may wish to recap with the students the trigonometric ratios of an acute angle before introducing the
trigonometric ratios of the sine and cosine of obtuse angles. Before the students move on to solving simple
trigonometric equations, teachers can guide them along in discovering the relationship between trigonometric
ratios of acute and obtuse angles (see Investigation: Relationship between Trigonometric Ratios of Acute and
Obtuse Angles).

Section 8.2: Area of Triangle


1
In primary school, students are taught to find the area of a triangle by the formula, base height. Teachers
2
can build upon this and ask the students to derive a formula with what they have learnt so far, such that the area
of a triangle can still be calculated if the height of the triangle is not given. Some guidance will be needed while
the students work among themselves to derive the formula.

Section 8.3: Sine Rule


Teachers can start off with an activity to introduce the Sine Rule to the students (see Investigation: Sine Rule).

In using a calculator, it is important to remind students to check and see that the MODE is set as DEG. Also, the
examination requirements state that students are to give their answers correct to 3 significant figures and angles
in degrees correct to 1 decimal place. Therefore, students should have developed the habit of working with 4 or
5 significant figures and angles in degrees to 2 decimal places and give their final answers correct to the required
accuracy since Secondary 2.

Worked Example 8 provides an example of an ambiguous case of Sine Rule. Teachers should carefully go through
this example with the students and highlight to them that for ambiguous cases, two sets of solutions will be
obtained.

Section 8.4: Cosine Rule


Similar to Sine Rule, teachers can start off with an activity to introduce the Cosine Rule to the students (see
Investigation: Cosine Rule). Teachers can show students how the Cosine Rule is derived when the angle is an
acute angle, and then challenge the students’ thinking further by asking them to prove the Cosine Rule where
the angle is an obtuse angle (see Thinking Time on page 248 of the textbook).

Challenge Yourself
Students should recall the formula for finding the value of angles in a polygon to calculate some of the angles
needed to find the lengths of HK and GK and then to find the area of !FGK. Both Sine Rule and Cosine Rule,
and the formula for the area of triangle which they have learnt in this chapter, will have to applied when solving
this problem.

216
WORKED SOLUTIONS a b c
4. The 3 quantities are equal; or = = .
Investigation (Relationship between Trigonometric sin A sin B sin C
a b c
Ratios of Acute and Obtuse Angles) 5. = =
sin A sin B sin C
A 180° – A sin A sin (180° – A) cos A cos (180° – A) a b c
6. Yes. We can manipulate = = to obtain
sin A sin B sin C
0.866 –0.866
(a) 30° 150° 0.5 0.5 sin A sin B sin C
(to 3 s.f.) (to 3 s.f.) = = .
a b c
0.970 0.970 0.242 –0.242 7. The lengths of the sides of a triangle are proportional to the sine of
(b) 76° 104°
(to 3 s.f.) (to 3 s.f.) (to 3 s.f.) (to 3 s.f.)
the angles opposite the sides.
0.934 0.934 –0.358 0.358
(c) 111° 69°
(to 3 s.f.) (to 3 s.f.) (to 3 s.f.) (to 3 s.f.) Journal Writing (Page 242)
0.225 0.225 –0.974 0.974
(d) 167° 13° Sine rule can be used to solve a triangle when
(to 3 s.f.) (to 3 s.f.) (to 3 s.f.) (to 3 s.f.)
Case 1: two angles and a side are given,
Table 8.1
Case 2: two sides and an angle opposite a given side is given.
1. sin A = sin (180° – A)
2. cos A = –cos (180° – A) Investigation (Cosine Rule)
1. (b) The length of the triangle opposite vertex B is labelled b.
Thinking Time (Page 232) (c) The length of the triangle opposite vertex C is labelled c.
(a) Since 62 + 82 = 102, !B = 90° 2.
1
Area of "ABC = 6 8 = 24 cm2
2 3. The values in the 5th column are equal to the values in the 6th column.
1 The values in the 7th column are equal to the values in the 8th column.
s= (6 + 8 + 10) = 12
2 The values in the 9th column are equal to the values in the 10th column.
Area of "ABC = 12(12 – 6)(12 – 8)(12 – 10) 4. The values in the 5th column are equal to the values in the 6th column.
= 24 cm2 The values in the 7th column are equal to the values in the 8th column.
10 2 + 9 2 – 8 2 The values in the 9th column are equal to the values in the 10th column.
(b) !A = cos–1 = 49.46° (to 2 d.p.)
2(10)(9) 5. a2 = b2 + c2 – 2bc cos A
1 b2 = a2 + c2 – 2ac cos B
Area of "ABC = 10 9 sin 49.46°
2 c2 = a2 + b2 – 2ab cos C
= 34.2 cm2 (to 3 s.f.)
b2 + c2 – a2
1 6. cos A =
s = (8 + 9 + 10) = 13.5 2bc
2
a2 + c2 – b2
Area of "ABC = 13.5(13.5 – 8)(13.5 – 9)(13.5 – 10) cos B =
2ac
= 34.2 cm2 (to 3 s.f.) a2 + b2 – c2
7 + 5 2 – 32
2 cos C =
2ab
(c) !B = cos–1 = 21.79° (to 2 d.p.)
2(7)(5)
1 Thinking Time (Page 248)
Area of "ABC = 7 5 sin 21.79°
2
1. When !A is an obtuse angle,
= 6.50 cm2 (to 3 s.f.)
1 B
s= (5 + 3 + 7) = 7.5
2
a
Area of "ABC = 7.5(7.5 – 3)(7.5 – 5)(7.5 – 7) h c
= 6.50 cm2 (to 3 s.f.)
C
D x A b
Investigation (Sine Rule) In "BCD,
1. (b) The length of the triangle opposite vertex B is labelled b. a2 = h2 + (b + x)2 (Pythagoras’ Theorem)
(c) The length of the triangle opposite vertex C is labelled c. = h2 + b2 + 2bx + x2
2. = b2 + (h2 + x2) + 2bx — (1)
.
a b c
3. The 3 quantities are equal; or = = .
sin A sin B sin C

217
In !BAD, opp
(c) tan "BAC =
c2 = h2 + x2 (Pythagoras’ Theorem) — (2) adj
x BC
and cos A = – cos (180° – "BAD) = – , =
c AB
i.e. x = –c cos A — (3) 4
=
Substituting (2) and (3) into (1), 3
a2 = b2 + (h2 + x2) + 2bx 1
=1
= b2 + c2 – 2bc cos A (proven) 3
2. (a) If A = 90°, 2. (a) BC = [8 – (–4)]2 + (1 – 6)2
a2 = b2 = c2 – 2bc cos 90° = 12 2 + (–5)2
a2 = b2 + c2 – 2bc(0)
a2 = b2 + c2 = 169
(b) Yes, this is because cos 90° = 0. = 13 units
3. Pythagoras’ Theorem is a special case of the Cosine Rule. (b) (i) sin "HBC = sin (180° – "HBC)
= sin "ABC
Practise Now 1 opp
=
hyp
1. (a) sin 96° = sin (180° – 96°)
AC
= sin 84° =
BC
= 0.995 12
(b) cos 51° = –cos (180° – 51°) =
13
= –cos 129° (ii) cos "BCK = –cos (180° – "BCK)
= –(–0.629) = –cos "ACB
= 0.629 adj
=–
2. sin 8° – cos 140° = sin (180° – 8°) – [–cos (180° – 140°)] hyp
= sin 172° – (–cos 40°) BC
=–
= sin 172° + cos 40° AC
= 0.139 + 0.766 4
=–
= 0.905 5
opp
(ii) tan "ABC =
Practise Now 2 adj
AC
=
1. (a) sin "ACD = sin (180° – "ACD) AB
= sin "ACB 12
=
opp 5
=
hyp 2
=2
AB 5
=
AC
3 Practise Now 3
=
5 (a) Since sin x is positive, x can either be an acute angle or an obtuse
(b) cos "ACD = –cos (180° – "ACD)
angle.
= –cos " ACB
sin x = 0.415
adj
=– x = sin–1 0.415 = 24.5° (to 1 d.p.)
hyp
or 180° – 24.5° = 155.5° (to 1 d.p.)
BC x = 24.5° or 155.5°
=–
AC
(b) Since cos x is negative, x is an obtuse angle.
4
=– cos x = –0.234
5
x = cos–1 (–0.234) = 103.5° (to 1 d.p.)
(c) Since cos x is positive, x is an acute angle.
cos x = 0.104
x = cos–1 (0.104) = 84.0° (to 1 d.p.)

218
Practise Now 4 (iii) Using sine rule,
b a
A =
sin !B sin !A
b 12.5
=
24.8 m sin 58.3° sin 82.3°
12.5 sin 58.3°
b=
sin 82.3
49°
B C = 10.7 cm (to 3 s.f.)
31.8 m
AC = 10.7 cm
We have a = 31.8, b = 24.8 and C = 49°.
1 Practise Now 7
Area of !ABC = ab sin C
2
1 1. (i) Using sine rule,
= 31.8 24.8 sin 49° sin !R sin !Q
2 =
= 298 m2 r q
sin !R sin 42°
=
10.2 12
Practise Now 5
10.2 sin 42°
sin "R =
1. We have q = 2x, r = x and P = 104°. 12
1 = 0.5688 (to 4 s.f.)
Area of !PQR = qr sin P
2 "R = sin–1 0.5688 = 34.67° (to 2 d.p.)
1 or 180° – 34.67° = 145.33° (to 2 d.p.)
12.5 = 2x x sin 104°
2 Since r < q, "R < "Q, hence "R cannot be 145.33°.
= x2 sin 104° "R = 34.7° (to 1 d.p.)
12.5 (ii) "P = 180° – 42° – 34.67° (" sum of a !)
x2 =
sin 104° = 103.3° (to 1 d.p.)
12.5 (iii) Using sine rule,
x= sin 104° (since x is positive) p q
=
sin !P sin !Q
= 3.59 (to 3 s.f.)
p 12
1 =
2. Area of !XYZ = xz sin "XYZ sin 103.33° sin 42°
2
12 sin 103.33°
1 p=
12 = 6 5 sin "XYZ sin 42°
2
= 17.5 cm (to 3 s.f.)
12 = 15 sin "XYZ
QR = 17.5 cm
12
sin "XYZ = 2. (i) Using sine rule,
15
12 sin !B sin !A
=
"XYZ = sin–1 b a
15
sin !B sin 96.8°
= 53.1° (to 1 d.p.) =
12.4 15.6
12.4 sin 96.8°
Practise Now 6 sin "B =
15.6
(i) "A = 180° – 58.3° – 39.4° (" sum of a !) = 0.7893 (to 4 s.f.)
= 82.3° "B = sin–1 0.7893 = 52.12° (to 2 d.p.)
(ii) Using sine rule, or 180° – 52.12° = 127.88° (to 2 d.p.)
c a Since b < c, "B < "C, hence "B cannot be 127.88°.
=
sin !C sin !A "ABC = 52.1° (to 1 d.p.)
c 12.5 (ii) "BCA = 180° – 96.8° – 51.12° (" sum of a !)
=
sin 39.4° sin 82.3° = 31.1° (to 1 d.p.)
12.5 sin 39.4°
c=
sin 82.3°
= 8.01 cm (to 3 s.f.)
AB = 8.01 cm

219
(iii) Using sine rule, (ii) Using sine rule,
c a sin !QPR sin !PQR
= =
sin !C sin !A QR PR
c 15.6 sin !QPR sin 71°
= =
sin 31.08° sin 96.8° 15.9 16.05
15.6 sin 31.08° 15.9 sin 71°
c = sin !QPR =
sin 96.8° 16.05
= 8.11 cm (to 3 s.f.) = 0.9367 (to 4 s.f.)
AB = 8.11 cm !QPR = sin–1 0.9367 = 69.50° (to 2 d.p.)
or 180° – 69.50° = 110.5° (to 2 d.p.)
Practise Now 8 Since QR < PR, !P < !Q, hence !P cannot be 110.5°.
!QPR = 69.5° (to 1 d.p.)
Using sine rule,
(iii) !PRQ = 180° – 71° – 69.50°
sin !ACB sin 46°
= = 39.5° (to 1 d.p.)
9.8 7.1
9.8 sin 46°
sin !ACB = Practise Now 10
7.1
= 0.9929 (to 4 s.f.) The largest angle is the angle opposite the longest side, i.e. !QPR.
!ACB = sin–1 0.9929 = 83.17° (to 2 d.p.) Using cosine rule,
or 180° – 83.17° = 96.83° (to 2 d.p.) (Since c > b, !C < !B, q2 + r 2 – p2
i.e. !ACB > 46°, hence both answers are possible.) cos P =
2qr
When !ACB is 83.17°, !BAC = 180° – 46° – 83.17° 112 + 132 – 18 2
= 50.83° =
2 11 13
When !ACB is 96.83°, !BAC = 180° – 46° – 96.83°
17
= 37.17° =–
143
Case 1: when !ACB = 83.17° and !BAC = 50.83° 17
Using sine rule, !P = cos–1 –
143
a 9.8 = 96.8° (to 1 d.p.)
=
sin 50.83° sin 83.17° The largest angle is 96.8°.
9.8 sin 50.83°
a=
sin 83.17°
Exercise 8A
= 7.65 cm (to 3 s.f.)
ACB = 83.2°, !BAC = 50.8° and BC = 7.65 cm 1. (a) sin 110° = sin (180° – 110°)
Case 2: when !ACB = 96.82° and !BAC = 37.17° = sin 70°
Using sine rule, (b) sin 176° = sin (180° – 176°)
a 9.8 = sin 4°
=
sin 37.17° sin 96.83° (c) sin 98° = sin (180° – 98°)
9.8 sin 37.17° = sin 82°
a=
sin 96.83° (d) cos 99° = –cos (180° – 99°)
= 5.96 cm (to 3 s.f.) = –cos 81°
!ACB = 96.8°, !BAC = 37.2° and BC = 5.96 cm (e) cos 107° = –cos (180° – 107°)
= –cos 73°
Practise Now 9 (f) cos 175° = –cos (180° – 175°)
(i) Using cosine rule, = –cos 5°
q2 = p2 + r2 – 2pr cos Q 2. (a) sin 148° = sin (180° – 148°)
PR2 = 15.92 + 10.82 – 2 15.9 10.8 cos 71° = sin 32°
= 257.6 (to 4 s.f.) = 0.530
(b) cos 35° = –cos (180° – 35°)
PR = 257.6
= –cos 145°
= 16.1 cm (to 3 s.f.)
= –(–0.819)
= 0.819

220
3. (a) 2 cos 45° + 3 sin 135° = 2 cos 45° + 3 sin (180° – 135°) 6. (a) Since sin x is positive and it is an acute angle,
= 2 cos 45° + 3 sin 45° sin x = 0.52
= 2(0.707) + 3(0.707) x = sin–1 0.52 = 31.3° (to 1 d.p.)
= 1.414 + 2.121 x = 31.3°
= 3.535 (b) Since sin x is positive and it is an acute angle,
(b) 3 cos 135° + 4 sin 135° sin x = 0.75
= 3 –cos (180° – 135°) + 4 sin (180° – 135°) x = sin–1 0.75 = 48.6° (to 1 d.p.)
= –3 cos 45° + 4 sin 45° x = 48.6°
= –3(0.707) + 4(0.707) (c) Since sin x is positive and it is an acute angle,
= –2.121 + 2.828 sin x = 0.875
= 0.707 x = sin–1 0.875 = 61.0° (to 1 d.p.)
(c) cos 135° – 2 sin 45° = –cos (180° – 135°) – 2 sin 45° x = 61.0°
= –cos 45° – 2 sin 45° (d) Since sin x is positive and it is an acute angle,
= –0.707 – 2(0.707) sin x = 0.3456
= –0.707 – 1.414 x = sin–1 0.3456 = 20.2° (to 1 d.p.)
= –2.121 x = 20.2°
4. (a) sin !ABD = sin (180° – !ABD) 7. (a) Since sin x is positive and it is an acute angle,
= sin !CBD sin x = 0.52
CD x = sin–1 0.52 = 31.33° (to 2 d.p.)
=
BD 180° – 31.33° = 148.7° (to 1 d.p.)
8 x = 148.7°
=
10 (b) Since sin x is positive and it is an obtuse angle,
4 sin x = 0.75
=
5 x = sin–1 0.75 = 48.59° (to 2 d.p.)
(b) cos !DBA = –cos (180° – !DBA)
180° – 48.59° = 131.4° (to 1 d.p.)
= –cos !CBD
x = 131.4°
BC
=– (c) Since sin x is positive and it is an obtuse angle,
BD
sin x = 0.875
6
=– x = sin–1 0.875 = 61.04° (to 2 d.p.)
10
180° – 61.04° = 119.0° (to 1 d.p.)
3
=– x = 119.0°
5
CD 8 4 1 (d) Since sin x is positive and it is an obtuse angle,
(c) tan !CBD = = = =1 sin x = 0.3456
BC 6 3 3
5. (a) By Pythagoras’ Theorem, x = sin–1 0.3456 = 20.22° (to 2 d.p.)
PR2 = PQ2 + QR2 180° – 20.22° = 159.8° (to 1 d.p.)
412 = x2 + 402 x = 159.8°
x2 = 412 – 402 8. (a) Since cos x is positive and it is an acute angle,
= 81 cos x = 0.67
x = 81 (since x > 0) x = cos–1 0.67
= 47.9° (to 1 d.p.)
=9
(b) Since cos x is positive and it is an acute angle,
(b) (i) sin !PRS = sin (180° – !PRS)
cos x = 0.756
= sin !PRQ
x = cos–1 0.756
PQ
= = 40.9° (to 1 d.p.)
PR
(c) Since cos x is positive and it is an acute angle,
9
= cos x = 0.5
41
(ii) cos !PRS = –cos (180° – !PRS) x = cos–1 0.5
= –cos !PRQ = 60°
QR (d) Since cos x is positive and it is an acute angle,
=– cos x = 0.985
PR
40 x = cos–1 0.985
=–
41 = 9.9° (to 1 d.p.)
PQ 9
(iii) tan !PRQ = =
QR 40
221
9. (a) Since sin x is positive, x can either be an acute angle or an obtuse 11. y
angle, A(–2, 4)
sin x = 0.753 4
x = sin–1 0.753 = 48.9° (to 1 d.p.)
or 180° – 48.9° = 131.1° (to 1 d.p.) 2
x = 48.9° or 131.1° C(6, 1)
K(–2, 1) B(2, 1)
(b) Since sin x is positive, x can either be an acute angle or an obtuse x
angle, –2 0 2 4 6
sin x = 0.952
K is the point (–2, 1).
x = sin–1 0.952 = 72.2° (to 1 d.p.)
AK = 3 units, BK = 4 units
or 180° – 72.2° = 107.8° (to 1 d.p.)
By Pythagoras’ Theorem,
x = 72.2° or 107.8°
AB2 = AK2 + BK2
(c) Since sin x is positive, x can either be an acute angle or an obtuse
= 32 + 42
angle,
= 25
sin x = 0.4714
x = sin–1 0.4714 = 28.1° (to 1 d.p.) AB = 25
or 180° – 28.1° = 151.9° (to 1 d.p.) = 5 units
x = 28.1° or 151.9° (a) sin !ABC = sin (180° – !ABC)
(d) Since cos x is negative, x is an obtuse angle. = sin !ABK
cos x = –0.238 AK
=
x = cos–1 (–0.238) AB
= 103.8° (to 1 d.p.) 3
=
(e) Since cos x is negative, x is an obtuse angle. 5
cos x = –0.783 (b) cos !ABC = –cos (180° – !ABC)
x = cos–1 (–0.783) = –cos !ABK
= 141.5° (to 1 d.p.) BK
=–
AB
(f) Since cos x is negative, x is an obtuse angle.
4
cos x = 0.524 =–
5
x = cos–1 0.524
(c) CK = 8 units
= 58.4° (to 1 d.p.)
AK 3
10. (a) By Pythagoras’ Theorem, tan !ACB = =
CK 8
PR2 = PQ2 + QR2 12.
PR2 = 82 + 152 y
= 64 + 225
= 289 4
A(14, 2)
PR = 289 (since PR > 0) 2
x
= 17 –14 –12 –10 –8 –6 –4 –2 0 2 4 6 8 10 12 14
sin !PRS = sin (180° – !PRS) –2
= sin !PRQ C(–13, –3) –4 B(2, –3) H(14, –3)
PQ
=
PR H is the point (14, –3).
8 AH = 5 units, BH = 12 units
=
17 By Pythagoras’ Theorem,
(b) cos !SRP = –cos (180° – !SRP) AB2 = AH2 + BH2
= –cos !PRQ = 52 + 122
RQ = 169
=–
PR
AB = 169
15
=– = 13 units
17
PQ 8
(c) tan !PRQ = =
QR 15

222
(a) sin !ABC = sin (180° – !ABC) (d) !K = 180° – 105° – 28° (!sum of ")
= sin !ABH = 47°
AH 1
= Area of "JKL = sin K
AB 2
5 1
= = 13.35 6.5 sin 47°
13 2
(b) cos !ABC = –cos (180° – !ABC) = 31.7 m2 (to 3 s.f.)
= –cos !ABH (e) Area of MNOP = 2 Area of "PMN
BH 1
=– =2 PM MN sin !PMN
AB 2
12
=– 1
13 =2 5.3 5.8 sin 117°
2
(c) CH = 27 units
= 27.4 cm2 (to 3 s.f.)
AH 5
tan !ACB = = (f) Area of QRST = 2 Area of "TQR
CH 27
13. sin x° = sin 27° = 0.4540 (to 4 s.f.) 1
=2 TQ QR sin !TQR
Since sin x is positive, x can either be an acute angle or an obtuse 2
angle. 1
=2 8.5 8.5 sin 78°
x = 27° 2
or 180° – 27° = 153° = 70.7 m2 (to 3 s.f.)
x = 27° or 153° 1
2. Area of "ABC = AB AC sin !BAC
14. (a) Since sin x is positive, x can either be an acute angle or an obtuse 2
1
angle. = 22 15 sin 45°
2
sin (x + 10°) = 0.47
= 117 cm2 (to 3 s.f.)
x + 10° = sin–1 0.47 = 28.0° (to 1 d.p.)
1
x = 28.0° – 10° = 18.0° 3. Area of "PQR = qr sin !P
2
or x + 10° = 180° – 28.0° = 152.0° (to 1 d.p.)
1
x = 152.0° – 10° = 142.0° = 152 125 sin 72°
2
x = 18.0° or 142.0° = 9040 cm2 (to 3 s.f.)
(b) Since cos x is negative, x is an obtuse angle. 1
cos (x – 10°) = –0.56 4. (i) Area of "ABC = AB BC sin !ABC
2
x – 10° = cos–1 (–0.56) = 124.1° (to 1 d.p.) 1
= 32 43 sin 67°
x = 124.1° + 10° = 134.1° 2
= 633 cm2 (to 3 s.f.)
Exercise 8B (ii) Let the perpendicular distance from A to BC be h cm.
C
1
1. (a) Area of "ABC = bc sin A
2
1
= 9 8 sin 72°
2
43 cm
= 34.2 cm2 (to 3 s.f.)
1
(b) Area of "DEF = sin E h cm
2
1 67°
= 9 7 sin 111° A
32 cm
B
2
= 29.4 cm2 (to 3 s.f.) Area of "ABC = 633.3 cm2
(c) !H = 180° – 62° – 57° (!sum of ") 1
BC h = 633.3
= 61° 2
1 1
Area of "GHI = gi sin H 43 h = 633.3
2 2
1 21.5h = 633.3
= 10 9.5 sin 61°
2 h = 29.5 (to 3 s.f.)
= 41.5 m2 (to 3 s.f.) The perpendicular distance from A to BC is 29.5 cm.

223
1 1 1
5. Total area = 112 202 sin 30° + 202 197 sin 60.5° Area of "ABC = AB BC sin !ABD
2 2 2
= 5646 + 17 317 1
= 20 40 sin 60°
= 23 000 m2 (to 3 s.f.) 2
3.7 = 346 cm2 (to 3 s.f.)
6. (i) sin !ACD =
8.0 9.
3.7
!ACD = sin–1 7.8 cm 120°
8.0 8.6 cm
= 27.5° (to 1 d.p.)
7.8 cm
AC 8.6 cm
(ii) cos !BAC =
AB
8.0 1
cos 40.4° = Area of parallelogram = 4 7.8 8.6 sin 120°
AB 2
8.0 = 116 cm2 (to 3 s.f.)
AB =
cos 40.4°
10. Let the length of QR be x cm.
= 10.5 cm (to 3 s.f.)
QR = x cm
1
(iii) Area of "AED = AE AD sin !DAE 3QR = 4PR
2
3x = 4PR
1
= 4.1 3.7 sin 55.1° 3
2 PR = x cm
= 6.22 cm2 (to 3 s.f.) 4
Area of "PQR = 158 cm2
7. Area of "ABC = 97 cm2
1
1 PR QR sin PRQ = 158
AB AC sin !BAC = 97 2
2
1 3
1 x x sin 55° = 158
5x 4x sin 68° = 97 2 4
2
3
10 sin 68° x2 = 97 x2 sin 55° = 158
8
97
x2 = x2 3 sin 55° = 1264
10 sin 68°
1264
97 x2 =
x = (since x > 0) 3 sin 55°
10 sin 68°
1264
x= (since x > 0)
= 3.23 (to 3 s.f.) 3 sin 55°
8. (i) !ABD = 180° – !ACB – !BAC (! sum of "ABC)
= 22.7 (to 3 s.f.)
= 180° – 30° – 90°
The length of QR is 22.7 cm.
= 60°
11. Let the angle of one side of the rhombus be a°.
!BAD = 180° – !ABD – !ADB (!sum of "ABD)
= 180° – 60° – 90°
= 30°
15 cm
BD
(ii) sin !BAD =
AB
BD
sin 30° =
20 Area of rhombus = 40 cm2
BD = 20 sin 30° 1
2 15 15 sin a° = 40
= 10 cm 2
AB 225 sin a° = 40
(iii) sin !ACB =
BC 40
sin a° =
20 225
sin 30° =
BC 40
a° = sin–1
20 225
BC =
sin 30° = 10.2° (to 1 d.p.)
= 40 cm Angles of rhombus = 10.2° and 180° – 10.2° = 169.8°

224
12. (i) D Using sine rule,
f 6.25
=
C sin 79.3° sin 62°
6.25 sin 79.3°
=
sin 62°
7.5 cm
= 6.96 m (to 3 s.f.)
5.1 cm "F = 79.3°, d = 4.43 m, = 6.96 m
(c) "H = 180° – 188° – 24° (" sum of !)
34.4°
= 38°
A 3.2 cm B Using sine rule,
g 8
1 =
Area of !BCD = BC BD sin "CBD sin 118° sin 38°
2
8 sin 118°
1 g=
= 5.1 7.5 sin 34.4° sin 38°
2
= 11.5 mm (to 3 s.f.)
= 10.8 cm2 (to 3 s.f.)
Using sine rule,
(ii) Area of !ABD = 11.62 cm2
i 8
1 =
AB BD sin "ABD = 11.62 sin 24° sin 38°
2
8 sin 24°
1 i=
3.2 7.5 sin "ABD = 11.62 sin 38°
2
= 5.29 mm (to 3 s.f.)
12 sin "ABD = 11.26
"H = 38°, g = 11.5 mm, t = 5.29 mm
11.62
sin "ABD = 2.
12
= 0.9683 (to 4 s.f.) P

"ABD = sin–1 0.9683 = 75.5° (to 1 d.p.)


or "ABD = 180° – 75.5° = 104.5° (to 1 d.p.)
Given that "ABD is obtuse,
"ABD = 104.5°

47° 97°
Exercise 8C Q R
7 cm
1. (a) "C = 180° – 76° – 42° (" sum of !) "QPR = 180° – 97° – 47° (" sum of !)
= 62° = 36°
Using sine rule, Using sine rule,
b 7.4 PQ 7
= =
sin 76° sin 42° sin 97° sin 36°
7.4 sin 76° 7 sin 97°
b= PQ =
sin 42° sin 36°
= 10.7 cm (to 3 s.f.) = 11.8 cm (to 3 s.f.)
Using sine rule,
c 7.4
=
sin 62° sin 42°
7.4 sin 62°
c=
sin 42°
= 9.76 cm (to 3 s.f.)
"C = 62°, b = 10.7 cm, c = 9.76 cm
(b) "F = 180° – 62° – 38.7° (" sum of !)
= 79.3°
Using sine rule,
d 6.25
=
sin 38.7° sin 62°
6.25 sin 38.7°
d=
sin 62°
= 4.43 m (to 3 s.f.)

225
3. P Using sine rule,
c 17.4
75° =
sin 26.39° sin 98.0°
17.4 sin 26.39°
14 cm c=
sin 98.0°
= 7.81 m (3 s.f.)
!A = 55.6°, !C = 26.4°, c = 7.81 m
60°
Q R (c) Using sine rule,
sin !B sin !C
!R = 180° – 60° – 75° (!sum of ") =
b c
= 45° sin !B sin 35.0°
=
The longest side is the side opposite the largest angle, i.e. QR. 8.7 9.5
Using sine rule, 8.7 sin 35.0°
sin !B =
QR 14 9.5
=
sin 75° sin 60° = 0.5253 (to 4 s.f.)
14 sin 75° !B = sin–1 0.5253 = 31.69° (to 2 d.p.)
QR =
sin 60° Or 180° – 31.69° = 148.31° (to 2 d.p.)
= 15.6 cm (to 3 s.f.) Since b < c, then !B > !C. i.e. !B < 35.0°, hence !B cannot
4. (a) Using sine rule, be 148.31°.
sin !B sin !A !B = 31.7° (to 1 d.p.)
=
b a !A = 180° – 35.0° – 31.69°
sin !B sin 92.0° = 113.3° (to 1 d.p.)
=
6.93 15.3 Using sine rule,
6.93 sin 92.0°
sin !B = a 9.5
15.3 =
sin 113.3° sin 35.0°
= 0.4527 (to 4 s.f.)
9.5 sin 113.3°
!B = sin–1 0.4527 = 26.92° (to 2 d.p.) a=
sin 35.0°
Or 180° – 26.92° = 153.08° (to 2 d.p.) = 15.2 cm (to 3 s.f.)
Since a > b, then !A > !B, i.e. !B < 92.0°, hence !B cannot !A = 113.3°, !B = 31.7°, a = 15.2 cm
be 153.08°, 5. (i) Using sine rule,
!B = 26.9° (to 1 d.p.) 13.4 20.8
!C = 180° – 92.0° – 26.92° =
sin !R sin 101°
= 61.1° (to 1 d.p.) 13.4 sin 101°
sin !R =
Using sine rule, 20.8
c 15.3 = 0.6324 (to 4 s.f.)
=
sin 61.08° sin 92.0° !R = sin–1 39.23° = 140.77° (to 2 d.p.)
15.3 sin 61.08° Since PQ < RQ, then !R < !P, i.e. !R < 101°, hence !R
c=
sin 92.0° cannot be 140.77°.
= 13.4 cm (to 3 s.f.) !R = 39.2° (to 1 d.p.)
!B = 26.9°, !C = 61.1°, c = 13.4 cm (ii) !Q = 180° – 101° – 39.23°
(b) Using sine rule, = 39.8° (to 1 d.p.)
sin !A sin !B (iii) Using sine rule,
=
a b PR 20.8
sin !A sin 98.0° =
= sin 39.77° sin 101°
14.5 17.4 20.8 sin 39.77°
14.5 sin 98.0° PR =
sin !A = sin 101°
17.4 = 13.6 cm (to 3 s.f.)
= 0.8252 (to 4 s.f.)
!A = sin–1 0.8252 = 55.61° (to 2 d.p.)
Or 180° – 55.61° = 124.39° (to d.p.)
Since a < b, then !A < !B, i.e. !A < 98.0°, hence !A cannot
be 124.39°.
!A = 55.6° (to 1 d.p.)
!C = 180° – 98.0° – 55.61°
= 26.4° (to 1 d.p.)

226
6. (i) A (ii) !BCD = 180° – 68° (adj. !s on a str. line)
= 112°
Area of region
11.6 cm 1 1
= AC BC sin !ACB + BC CD sin !BCD
2 2
1 1
= 6.916 6 sin 68° + 6 7.5 sin 112°
2 2
91° = 19.24 + 20.86
B C = 40.1 m2 (to 3 s.f.)
7.4 cm
9. (i) !ADB = 180° – 30° – 80° (! sum of ")
Using sine rule, = 70°
sin !BAC sin 91° Using sine rule,
=
7.4 11.6 AB 5
7.4 sin 91° =
sin !BAC = sin 70° sin 30°
11.6 5 sin 70°
= 0.6378 (to 4 s.f.) AB =
sin 30°
!BAC = sin–1 0.6378 = 39.63° (to 2 d.p.) = 9.40 cm (to 3 s.f.)
Or 180° – 39.63° = 140.37° (to 2 d.p.) (ii) !BDC = 80° – 40° (ext. ! of ")
Since BC < AC, then !BAC < !ABC, i.e. !BAC < 91°, hence = 40°
!BAC cannot be 140.37°. Since !BDC = !BCD, "BDC is an isosceles triangle.
!BAC = 39.6° (to 1 d.p.) BC = BD = 5 cm
(ii) !ACB = 180° – 91° – 39.63° (iii) !CBD = 180° – 80° (adj. !s on a str. line)
= 49.4° (to 1 d.p.) = 100°
(iii) Using sine rule, Let the vertical distance between the mass and the horizontal
AB 11.6 bar be h cm.
=
sin 49.37° sin 91° Area of "ADC
11.6 sin 49.37°
AB = = Area of "ABD + Area of "BCD
sin 91°
1 1
= 8.80 cm (to 3 s.f.) = 9.397 5 sin 80° + 5 5 sin 100°
2 2
5.3
7. (i) sin 25° = = 23.14 + 12.31
AB
= 35.45 cm2 (to 4 s.f.)
5.3
AB = 1
sin 25° AC h = 35.45
2
= 12.5 m (to 3 s.f.)
1
5.3 (AB + BC) h = 35.45
(ii) cos !DBN = 2
7.1
1
5.3 (9.397 + 5) h = 35.45
!DBN = cos–1 2
7.1
7.1985h = 35.45
= 41.7° (to 1 d.p.)
h = 4.92 (to 3 s.f.)
(iii) !BCD = 180° – 103° – 46° (! sum of ")
The vertical distance between the mass and the horizontal
= 31°
bar is 4.92 cm.
Using sine rule,
PS 4.3
CD 7.1 10. (i) tan !PTS = =
= ST 5.7
sin 103° sin 31°
4.3
7.1sin 103° !PTS = tan–1
CD = 5.7
sin 31°
= 37.03° (to 2 d.p.)
= 13.4 m (to 3 s.f.)
!QSR = 180° – 90° – 74° (!s on a straight line)
8. (i) !BAC = 180° – 62° – 68° (! sum of ")
= 16°
= 50°
Since !PTS !QSR, QS is not parallel to PT.
Using sine rule,
AC 6
=
sin 62° sin 50°
6 sin 62°
AC =
sin 50°
= 6.92 m (to 3 s.f.)

227
PS 13. (a) Using sine rule,
(ii) cos !SPR =
PR sin !B sin 91°
=
4.3 7.5 8.5
cos 63° =
PR 7.5 sin 91°
sin !B =
4.3 8.5
PR =
cos 63° = 0.8822 (to 4 s.f.)
= 9.47 cm (to 3 s.f.) !B = sin–1 0.8822 = 61.91° (to 2 d.p.)
(iii) !PQS = 180° – 63° – 74° (! sum of ") Or 180° – 61.91° = 118.09° (to 2 d.p.)
= 43° Since b < a, then !B < !A, i.e. !B < 92°, hence !B cannot be
Using sine rule, 118.09°.
QS 4.3 !B = 61.9° (to 1 d.p.)
=
sin 63° sin 43° !C = 180° – 61.91° = 118.1° (to 1 d.p.)
4.3 sin 63° The triangle exists and it is not an ambiguous case.
QS =
sin 43°
(b) Using sine rule,
= 5.62 cm (to 3 s.f.)
sin !E sin 47°
QR =
11. cos 73° = 80 75
5.7
80 sin 47°
QR = 5.7 cos 73° sin !E =
75
= 1.667 km (to 4 s.f.) = 0.7801 (to 4 s.f.)
!QPS = 180° – 48° – 55° (! sum of ") !E = sin–1 0.7801 = 51.27° (to 2 d.p.)
= 77° Or 180° – 51.27° = 128.73° (to 2 d.p.) (Since d < e, then
PQ 5.7 !D < !E, i.e.
=
sin 55° sin 77°
!E > 47°, hence both
5.7 sin 55°
PQ = answers are possible.)
sin 77°
when !E = 51.3° (to 1 d.p.),
= 4.792 km (to 4 s.f.)
!F = 180° – 47° – 51.2° = 81.7° (to 1 d.p.)
Area of nature reserve
when !E = 128.7° (to 1 d.p.)
= Area of "QRS + Area of "PQS
!F = 180° – 47° – 128.73° = 4.3° (to 1 d.p.)
1 1
= 1.667 5.7 sin 73° + 5.7 4.792 sin 48° The triangle exists and it is an ambiguous case.
2 2
= 4.543 + 10.149 (c) Using sine rule,
= 14.7 km2 (to 3 s.f.) sin !H sin 58°
=
QR 37 37
12. (i) sin 27.6° = 37 sin 58°
5.7 sin !H =
QR = 5.7 sin 27.6 37
= 0.8480 (to 4 s.f.)
= 2.64 cm (to 3 s.f.)
!H = sin–1 0.8480 = 57.99° (to 2 d.p.)
3.2
(ii) cos !SPR = Or 180° – 57.99° = 122.01° (to 2 d.p.)
5.7
3.2 Since g = h, then !G = !H, i.e. !G = 58°, hence !H cannot
!SPR = cos–1 be 122.01°.
5.7
= 55.8° (to 1 d.p.) !H = 58.0° (to 1 d.p.)
(iii) Using sine rule, !I = 180° – 58° – 57.99° = 64.0° (to 1 d.p.)
sin !PST sin 64.2° The triangle exists and it is not an ambiguous case.
=
2.7 3.2 (d) Using sine rule,
2.7 sin 64.2° sin !J sin 39°
sin !PST = =
3.2 19 15
= 0.7596 (to 4 s.f.) 19 sin 39°
sin !J =
!PST = sin–1 0.7596 15
= 49.4° (to 1 d.p.) = 0.7971 (to 4 s.f.)
!J = sin–1 0.7971 = 52.85° (to 2 d.p.)
or 180° – 52.85° = 127.15° (to 2 d.p.) (Since k < j, then
!K < !J, i.e. !J > 39°,
hence both answers are
possible.)

228
when !J = 52.9° (to 1 d.p.), sin !LMN sin 42°
(c) = = 0.08579 (to 4 s.f.)
!L = 180° – 39° – 52.85° = 88.1° (to 1 d.p.) LN 7.8
when !J = 127.1° (to 1 d.p.), sin !LNM sin 57°
= = 0.1215 (to 4 s.f.)
!L = 180° – 39° – 127.15° = 13.9° (to 1 d.p.) LM 6.9
The triangle exists and it is an ambiguous case. sin !LMN sin !LNM
Since ,
(e) Using sine rule, LN LM
sin !N sin 43° It is not possible to construct "LMN.
= (d) !GKH = 180° – 90° – 60° (! sum of ")
80 67
80 sin 43° = 50°
sin !N =
67 sin !GHK sin 90°
= = 0.07813 (to 4 s.f.)
= 0.8143 (to 4 s.f.) GK 12.8
!N = sin–1 0.8143 = 54.52° (to 2 d.p.) sin !GKH sin 30°
= = 0.0.7813 (to 4 s.f.)
or 180° – 54.52° = 125.48° (to 2 d.p.) (Since o < n, then GH 6.4
!O < !N, i.e. sin !GHK sin !GKH
Since = ,
GK GH
!N > 43°, hence both
It is possible to construct "GHK.
answers are possible.)
15. A
when !N = 54.5° (to 1 d.p.),
!M = 180° – 43° – 54.52° = 82.5° (to 1 d.p.)
58°
when !N = 125.5° (to 1 d.p.),
!M = 180° – 43° – 125.48° = 11.5° (to 1 d.p.) 15.4 cm
The triangle exists and it is an ambiguous case.
(f) Using sine rule,
sin !P sin 52° B C
= 14.0 cm
19 25
19 sin 52° sin !ABC sin 58°
sin !P = =
25 15.4 14.0
= 0.5989 (to 4 s.f.) 15.4 sin 58°
sin !ABC =
!P = sin–1 0.5989 = 36.79° (to 2 d.p.) 14.0
Or 180° – 36.79° = 142.21° (to 2 d.p.) = 0.9329 (to 4 s.f.)
Since p < q, then !P < !Q, i.e. !P < 52°, hence !P cannot !ABC = sin–1 0.9329 = 68.89° (to 2 d.p.)
be 143.21°. Or 180° – 68.89° = 111.11° (to 2 d.p.)
!P = 36.8° (to 1 d.p.) Since BC < AC, then !BAC < !ABC, i.e. !ABC > 58°, hence both
!R = 180° – 52° – 36.79° = 91.2° (to 1 d.p.) answers are possible.
The triangle exists and it is not an ambiguous case. When !ABC = 68.9° (to 1 d.p.),
14. (a) !BAC = 180° – 90° – 35° (! sum of ") !ACB = 180° – 58° – 68.89° = 53.1° (to 1 d.p.)
= 55° Using sine rule,
sin !BAC sin 55° AB 14.0
= = 0.1024 (to 4 s.f.) =
BC 8 sin 53.11 sin 58°
sin !ACB sin 35° 14.0 sin 53.11°
= = 0.09560 (to 4 s.f.) AB =
AB 6 sin 58°
sin !BAC sin !ACB = 13.2 cm (to 3 s.f.)
Since , When !ABC = 111.1° (to 1 d.p.),
BC AB
It is not possible to construct "ABC. !ACB = 180° – 58° – 111.11° = 10.9° (to 1 d.p.)
sin !PQR sin 30° Using sine rule,
(b) = = 0.1
PR 5 AB 14.0
=
sin !PRQ sin 36.9° sin 10.89° sin 58°
= = 0.1
PQ 6 14.0 sin 10.89°
AB =
sin !PQR sin !PRQ sin 58°
Since = = 0.1,
PR PQ = 3.12 cm (to 3 s.f.)
It is possible to construct "PQR. !ABC = 68.9°, !ACB = 53.1°, AB = 13.2 cm
or !ABC = 111.1°, !ACB = 10.9°, AB = 3.12 cm

229
16. D C Exercise 8D
40°
77° 1. A
7

55° 48° 7 cm
A B

(i) Using sine rule,


60°
AB 7
= B 5 cm C
sin 77° sin 48°
7 sin 77° Using cosine rule,
AB =
sin 48° c2 = a2 + b2 – 2ab cos C
= 9.18 cm (to 3 s.f.) c2 = 52 + 72 – 2 5 7 cos 60°
AD c2 = 74 – 70 cos 60°
(ii) tan 40° =
7 = 39
AD = 7 tan 40°
c = 39
= 5.874 cm (to 4 s.f.)
= 6.24 cm (to 3 s.f.)
Map Actual 2. G
8 cm represents 1 km
1 7 cm
1 cm represents = 0.125 km
8
5.874 cm represents (5.874 0.125) km 30°
= 0.734 km (to 3 s.f.) I 9 cm H
The length represented by AD is 0.734 km.
Using cosine rule,
(iii) Map Actual h2 = g2 + i2 – 2gi cos H
8 cm represents 1 km = 92 + 72 – 2 9 7 cos 30°
1 = 130 – 126 cos 30°
1 cm represents = 0.125 km
8 = 20.88 (to 4 s.f.)
7 cm represents (7 0.125) km = 0.875 km
h = 20.88
1
Area of !ADC = 0.734 0.875 = 4.57 cm (to 3 s.f.)
2
3.
= 0.321 km2 (to 3 s.f.) M
4
17. (i) sin "B = sin "A
3
4 5.8 cm
= sin 35° 141.4°
3
= 0.7648 (to 4 s.f.) N 4.2 cm O

"B = sin–1 0.7648 = 49.89° (to 2 d.p.)


Using cosine rule,
or 180° – 49.89° = 130.11° (to 2 d.p.)
o2 = m2 + n2 – 2mn cos O
"B = 49.9° (to 1 d.p.), 130.1° (to 1 d.p.)
= 4.22 + 5.82 – 2 4.2 5.8 cos 141.4°
4
(ii) sin "B = sin "A = 51.28 – 48.72 cos 141.4°
3
= 89.36 (to 4 s.f.)
sin !B 4
= o = 89.36
sin !A 3
b a 5 = 9.45 cm (to 3 s.f.)
= =
sin !B sin !A sin 35°
5 sin !B 4 2
b = =5 =6 cm
sin !A 3 3
2
AC = 6 cm
3

230
4. X 6. P

7.8 cm 4.9 cm

9m 8m
Q 9.1 cm R

The largest angle is the angle opposite the longest side, i.e. !P.
Using cosine rule,
q2 + r 2 – p2
Y 7m Z cos P =
2qr
Using cosine rule, 4.9 2 + 7.8 2 – 9.12
=
x2 = y2 + z2 – 2yz cos X 2 4.9 7.8
72 = 82 + 92 – 2 8 9 cos X = 0.02669 (to 4 s.f.)
49 = 145 – 144 cos X !P = cos–1 0.02669
144 cos X = 96 = 88.5° (to 1 d.p.)
96 The largest angle is 88.5°.
cos X = 7. (i) !DBC = 180° – 125° ( adj. !s on a str. line)
144
96 = 55°
!X = cos–1
144 Since !CBD = !BCD,
= 48.2° (to 1 d.p.) BD = CD = 9 m (isos. ")
Using cosine rule, CD = 9 m
y2 = x2 + z2 – 2xz cos Y (ii) Using cosine rule,
82 = 72 + 92 – 2 7 9 cos Y AD2 = AB2 + BD2 – 2 AB BD cos !ABD
64 = 130 – 126 cos Y = 82 + 92 – 2 8 9 cos 125°
126 cos Y = 66 = 145 – 144 cos 125°
66 = 227.6 (to 4 s.f.)
cos Y =
126 AD = 227.6
66
!Y = cos–1 = 15.1 m (to 3 s.f.)
126
8. (i) !APB = 180° – 60° (adj. !s on a str. line)
= 58.4° (to 1 d.p.)
= 120°
!Z = 180° – 48.19° – 58.14° (! sum of ")
AB 5
= 73.4° (to 1 d.p.) =
sin 120° sin 45°
!X = 48.2°, !Y = 58.4° and !Z = 73.4° 5 sin 120°
5. A AB =
sin 45°
= 6.12 m (to 3 s.f.)
(ii) Using cosine rule,
6.7 cm
5.3 cm AC2 = AP2 + PC2 – 2 AP PC cos !APC
= 52 + 82 – 2 5 8 cos 60°
= 89 – 80 cos 60°
B 3.8 cm C = 49
AC = 49
The smallest angle is the angle opposite the shortest side, i.e. !A.
=7m
Using cosine rule,
9. (i) MC = MB = 2 cm
b2 + c2 – a2
cos A = !AMC = 180° – 120° (adj. !s on a str. line)
ab
= 60°
5.32 + 6.7 2 – 3.8 2
= Using cosine rule,
2 5.3 6.7
= 0.8243 (to 4 s.f.) AC2 = AM2 + MC2 – 2 AM MC cos !AMC
!A = cos–1 0.8243 = 42 + 22 – 2 4 2 cos 60°
= 34.5° (to 1 d.p.) = 20 – 16 cos 60°
The smallest angle is 34.5°. = 12
AC = 12
= 3.46 cm (to 3 s.f.)

231
(ii) Using cosine rule, 12. (i) Using cosine rule,
AB2 = AM2 + BM2 – 2 AM BM cos !AMB 2 2 + 3.5 2 – 4.5 2
cos !ADC =
= 42 + 2 – 2 4 2 cos 120° 2 2 3.5
= 20 – 16 cos 120° = –0.2857 (to 4 s.f.)
= 28 !ADC = cos–1 (–0.2857)
AB = 28 = 106.60° (to 2 d.p.)
!ADB = 180° – 106.60° (adj. !s on a str. line)
= 5.29 cm (to 3 s.f.)
= 73.4° (to 1 d.p.)
(iii) Using sine rule,
sin !ACB sin 60° 1
(ii) Area of "ACD = 2 3.5 sin 106.60°
= 2
4 3.464
4 sin 60° = 3.354 cm2 (to 4 s.f.)
sin !ACB = Let the shortest distance from A to CB be h cm.
3.464
= 1.000 (to 4 s.f.) 1
3.5 h = 3.354
!ACB = sin–1 1.000 = 90° 2
5 h = 1.92 (to 3 s.f.)
10. (i) tan !BAD = The shortest distance from A to CB is 1.92 cm.
12
5 (iii) !BAD = 180° – 50° – 73.40° (! sum of ")
!BAD = tan–1 = 56.60° (to 2 d.p.)
12
= 22.6° (to 1 d.p.) Using sine rule,
(ii) !APD = 180° – 50° – 22.62° (! sum of ") BD 2
=
= 107.38° (to 2 d.p.) sin 56.60° sin 50°
Using sine rule, 2 sin 56.60°
BD =
PD 12 sin 50°
= = 2.18 cm (to 3 s.f.)
sin 22.62° sin 107.38°
12 sin 22.62° 13. (i) Map Actual
PD = 2 cm represents 5 km
sin 107.38°
= 4.84 m (to 3 s.f.) 5
1 cm represents km = 2.5 km
(iii) !BCD = !BAD = 22.62° 2
Using cosine rule, 8 cm represents (2.5 8) km = 20 km
DQ2 = CD2 + CQ2 – 2 CD CQ cos !BCD XZ = 20 km
= 122 + 62 – 2 12 6 cos 22.62° (ii) Using cosine rule,
= 180 – 144 cos 22.62° 8 2 + 9 2 – 12 2
cos !YXZ =
= 47.08 (to 4 s.f.) 2 8 9
1
DQ = 47.08 =
144
= 6.86 m (to 3 s.f.) 1
11. (i) Using cosine rule, !YXZ = cos–1
144
a2 = 52 + 62 – 2 5 6 cos 92° = 89.6° (to 1 d.p.)
= 61 – 60 cos 92° 1
(iii) Area of "XYZ = 8 9 sin 89.60°
= 63.09 (to 4 s.f.) 2
a = 63.09 = 36.00 cm2 (to 4 s.f.)
= 7.94 (to 3 s.f.) Map Actual
(ii) Using cosine rule, 5
1 cm represents km = 2.5 km
7 2 + 5 2 – 7.9432 2
cos = 1 cm 2
represents (2.5 km)2 = 6.25 km2
2 7 5
= 0.1558 (to 4 s.f.) 36.00 cm2 represents (6.25 36.00) km2 = 225 km2
2
= cos–1 0.1558 Area of "XYZ = 225 km
= 81.0° (to 1 d.p.)

232
14. 16. A
A B

6 6 8
5 6

B 5 C 3 R
D 4.5 X 3 C
52 + 62 – 82 1
ABXD becomes a parallelogram. (i) cos !BCA = =–
2 5 6 20
CD = 3 cm, BX = 6 cm
(ii) cos !RCA = –cos (180° – !BCA)
32 + 5 2 – 6 2
cos !BCX = = cos !RCA
2 3 5
1
1 =– –
=– 20
15
1
1 =
!BCX = cos–1 – 20
15
AR2 = 32 + 62 – 2 3 6 cos !RCA
= 93.8° (1 d.p.) 1
BD2 = 52 + 7.52 – 2 5 7.5 cos !BCX AR2 = 45 – 36
20
1
BD2 = 81.25 – 75 – 216
15 AR =
5
BD = 86.25 = 6.57 cm (to 3 s.f.)
= 9.29 cm (to 3 s.f.) 17. In "ABC,
AD 3+ 2 5 8 2 + 132 – 14 2 37
15. (i) = = cos A = =
AB 3 3 2 8 13 208
AE 6+5 11
= = PQ2 = 52 + 62 – 2 5 6 cos A
AC 6 6
AD AE 37
PQ2 = 61 – 60
AB AC 208
ADE is not an enlargement of "ABC. 2617
6 2 + 32 – 3.5 2 32.75 131 PQ =
(ii) cos = = = 52
2 6 3 36 144 = 7.09 cm (to 3 s.f.)
131 112 + 5 2 – x 2
(iii) =
144 2 11 5 Review Exercise 8
131 146 – x 2
=
144 110 1. (a) By Pythagoras’ Theorem,
131 AC2 = 72 + 242
110 = 146 – x2
144 = 625
5 AC = 625
x2 = 146 – 100
72 = 25 cm
3307
x2 = AB 7
72 (b) (i) tan !ACB = =
BC 24
3307 (ii) cos !ACD = –cos (180° – !ACD)
x=
72 = –cos !ACB
= 6.78 (to 3 s.f.) BC
=–
AC
24
=–
25
(iii) sin !TAC = sin (180° – !TAC)
= sin !BAC
BC
=
AC
24
=
25

233
2. (a) By Pythagoras’ Theorem, (ii) sin !ABC = sin (180° – !ABC)
QR2 = 122 + 352 = sin !ABH
= 1369 AH
=
QR = 1369 AB
6
= 37 cm =
(b) (i) sin !PQT = sin (180° – !PQT) 45
= 0.8944 (to 4 s.f.)
= sin !RQT
(iii) CH = 8 units
RT
= AH 6 3
QR tan !ACB = = =
CH 8 4
12
=
37 4. (i) BC = [9 – (–3)]2 + [4 – (–1)]2
(ii) cos !PQT = –cos (180° – !PQT)
= 12 2 + 5 2
= –cos !RQT
QT = 169
=–
QR = 13 units
35
=– AC = [9 – ( 3)]2 + [4 – (–4)]2
37
RT QT = 12 2 + 8 2
(ii) tan !TQR + tan !TRQ = +
QT RT
12 35 = 208
= +
35 12 = 14.4 units (to 3 s.f.)
109 (ii)
=3
420
y
3.
y M(–3, 4) C(9, 4)
4
6
H(–3, 5) A(3, 5)
2
4

x
B(–3, 2) 0
2 –4 –2 2 4 6 8 10
B(–3, –1)
–2
x
–4 –2 0 2 4
–4
–2 A(–3, –4)

M is the point (–3, 4).


C(–3, –3)
–4 BM = 5 units, CM = 12 units
sin !ABC = sin (180° – !ABC)
= sin !CBM
H is the point (–3, 5).
CM
AH = 6 units, BH = 3 units =
BC
By Pythagoras’ Theorem,
12
AB2 = AH2 + BH2 =
13
= 62 + 32
cos !ABC = –cos (180° – !ABC)
= 45
= –cos !CBM
AB = 45 units BM
=–
(i) cos !ABC = –cos (180° – !ABC) BC
= –cos !ABH 5
=–
BH 13
=–
AB 1
(iii) Area of "ABC = 3 12
3 2
=–
45 = 18 units2
= –0.4472 (to 4 s.f.)

234
(iv) Let the length of the perpendicular from B to AC be h cm. SQ 16 + 35 51
(ii) tan "QPS = = =
Area of !ABC = 18 PQ 12 12
1 51
AC h = 18 "QPS = tan–1 = 76.76° (to 2 d.p.)
2 12
1 By cosine rule,
14.42 h = 8
2 PQ 2 + PR 2 – QR 2
cos "QPR =
7.21h = 18 2 PQ PR
h = 2.50 units (to 3 s.f.) 12 2 + 37 2 – 35 2
=
5. (a) Since sin x is positive, x can either be an acute angle or an obtuse 2 12 37
angle. = 0.3242 (to 4 s.f.)
sin x = 0.419 "QPR = cos–1 0.3243 = 71.08° (to 2 d.p.)
x = sin–1 0.419 = 24.8° (to 1 d.p.) "RPS = "QPS – "QPR
or 180° – 24.8° = 155.2° (to 1 d.p.) = 76.76° – 71.08°
x = 24.8° or 155.2° = 5.6° (to 1 d.p.)
(b) Since cos x is positive, x is an acute angle. 7. (i) sin "CBA = sin (180° – "CBA)
cos x = 0.932 = sin "CBD
x = cos–1 (0.932) 3
=
= 21.3° (to 1 d.p.) 4
(c) Since tan x is positive, x is an acute angle. Using sine rule,
tan x = 0.503 sin !ACB sin !CBA
=
x = tan–1 0.503 AB CA
= 26.7° (to 1 d.p.) 3
(d) Since cos x is negative, x is an obtuse angle. sin !ACB 4
=
cos x = –0.318 7 11
x = cos–1 (–0.318) 3
4
= 108.5° (to 1 d.p.) sin "ACB = 7
11
6. (a) PQ = 122 = 144
2
21
SQ2 = 352 = 1225 =
44
PR2 = 372 = 1369 21
PQ2 + SQ2 = 144 + 1225 (ii) "ACB = sin–1 = 28.51° (to 2 d.p.)
44
= 1369 3
"CBD = sin–1 = 48.59° (to 2 d.p.)
= PR2 4
Since PQ + SQ2 = PR2, then by the converse of Pythagoras’
2
"BAC = 48.59° – 28.51° = 20.1° (to 1 d.p.) (ext. " of !)
Theorem, !PQR is a right-angled triangle with "PQR = 90°. 1
(iii) Area of !ABC = 7 11 sin 20.08°
(b) (i) cos "PRS = –cos (180° – "PRS) 2
= –cos "PRQ = 13.2 cm2 (to 3 s.f.)
QR (iv) By cosine rule,
=–
PR BC2 = 72 + 11 – 2 7 11 cos 20.08°
35 = 25.36 (to 4 s.f.)
=–
37 BC = 25.36
sin "PRS = sin (180° – "PRS)
= 5.04 cm (to 3 s.f.)
= sin "PRQ
BC
PQ 8. (i) cos "CBQ =
= BQ
PR BC
12 cos 52° =
= 4.8
37 BC = 4.8 cos 52°
= 2.96 m (3 s.f.)
(ii) "ABP = 90° – 52° – 26° = 12°
AP
sin "ABP =
BP
AP
sin 12° =
5.7
AP = 5.7 sin 12°
= 1.19 m (to 3 s.f.)

235
1 10. (i) "ACB = 180° – 64° – 42° (" sum of !)
(iii) Area of !PBQ = 4.8 5.7 sin 26°
2 = 74°
= 6.00 m2 (to 3 s.f.) Using sine rule,
(iv) By cosine rule, AC AB
=
PQ2 = 4.82 + 5.72 – 2 4.8 5.7 cos 26° sin !ABC sin !ACB
= 6.348 (to 4 s.f.) AC 168 sin 64°
=
PQ = 6.348 sin 64° sin 74°
= 2.52 m (to 3 s.f.) 168 sin 64°
AC =
(v) Using sine rule, sin 74°
sin !BPQ sin !PBQ = 157 cm (to 3 s.f.)
=
BQ PQ (ii) By cosine rule,
sin !BPQ sin 26° AD 2 + CD 2 – AC 2
= cos "ADC =
4.8 2.519 2 AD CD

sin "BPQ = 4.8


sin 26° 210 2 + 192 2 – 157.12
=
2.519 2 210 192
= 0.8353 (to 4 s.f.) = 0.6980 (to 4 s.f.)
"BPQ = sin–1 0.8353 = 56.6° (to 1 d.p.) "ADC = cos–1 0.6980 = 45.7° (to 1 d.p.)
9. (i) "PQS = 180° – 55° – 38° (" sum of !) (iii) Area of ABCD
= 87° = Area of !ABC + Area of !ACD
Using sine rule, 1 1
QS PS = 1.68 1.571 sin 42° + 1.92 2.10 sin 45.73°
2 2
=
sin !QPS sin !PQS = 2.33 m2 (to 3 s.f.)
QS 460 11. (i) Using sine rule,
=
sin 55° sin 87° QS RS
460 sin 55° =
QS = sin !QRS sin !RQS
sin 87° QS 68
= 377 m (to 3 s.f.) =
sin 71° sin 62°
(ii) By cosine rule, 68 sin 71°
QS =
RS2 = 3252 + 377.32 – 2 325 377.3 cos 32° sin 62°
= 40 000.7 (to 6 s.f.) = 72.82 m (to 4 s.f.)
RS = 40 000.7 "QSR = 180° – 71° – 62° (" sum of !)
= 200 m (to 3 s.f.) = 47°
1 1
(iii) Area of !PQS = 460 377.3 sin 38° Area of !SQR = 68 72.82 sin 47°
2 2
= 53 430 m2 (to 4 s.f.) = 1810 m2 (to 3 s.f.)
Let the shortest distance between Q and PS be h m. (ii) By cosine rule,
Area of !PQS = 53 430 m2 PS2 = 72.822 + 982 – 2 72.82 98 cos 43°
1 = 4468 (to 4 s.f.)
460 h = 53 430
2 PS = 4468
230h = 53 430 = 66.8 m (to 3 s.f.)
h = 232 (to 3 s.f.) 1
12. (i) Area of the park = 320 470 sin 35°
The shortest distance between Q and PS is 232 m. 2
1 = 43 100 m2 (to 3 s.f.)
(iv) Area of !QRS = 325 377.3 sin 32°
2 (ii) By cosine rule,
= 32 490 m2 (to 4 s.f.) BC2 = 3202 + 4702 – 2 320 470 cos 35°
Area occupied by the park = 32 490 + 53 430 = 76 900 (to 4 s.f.)
= 85 900 m2 (to 3 s.f.) BC = 76 900
= 277 m (to 3 s.f.)

236
(iii) !PBC = 180° – 118° – 24° (! sum of ")
= 38°
Using sine rule,
PC BC
=
sin !PBC sin !BPC
PC 277.3
=
sin 38° sin 118°
277.3 sin 38°
PC =
sin 118°
= 193 m (to 3 s.f.)

Challenge Yourself
180°(5 – 2)
(a) !KAH = = 108°
5
HK = 8 2 + 8 2 – 2(8)(8) cos 108°
= 12.944 cm (to 5 s.f.)
= 12.9 cm (to 3 s.f.)
180°(8 – 2)
!GHA = = 135°
8
180° – 108°
!AHK = = 36°
2
!GHK = 135° + 36° = 171°

GK = 12.944 2 + 8 2 – 2(8)(12.944) cos 171°


= 20.883 cm (to 5 s.f.)
= 20.9 cm (to 3 s.f.)
sin !HGK sin 171°
(b) =
12.944 20.883
12.944 sin 171°
sin !HGK =
20.883
!HGK = 5.56° (to 2 d.p.)
!FGK = 135° – 5.56° = 129.44° (to 2 d.p.)
1
Area of "FGK = 20.883 8 sin 129.44°
2
= 64.5 cm2 (to 3 s.f.)

237
Chapter 9 Applications of Trigonometry
TEACHING NOTES
Suggested Approach
In Book 2, students have learnt how to use the clinometer to measure angles and find the heights of buildings, flagpoles, etc.
This chapter introduces students to angles of elevation and depression. Teachers can use various illustrations to help students
with their learning and understanding of angles of elevation and depression and how they may be used to solve simple problems.

Section 9.1: Angles of Elevation and Depression


Teachers may use the following illustration to show students the angle of elevation and the angle of depression.

monkey

! of depression

! of elevation

hunter

Using the board or a visualiser, draw a picture of a monkey up in a tree and a hunter with a gun, looking for
game. When the hunter spots the monkey in the tree, his gun, which was initially down, is now raised through
an angle towards the monkey. Teachers may illustrate this as the angle of elevation.

When the monkey notices that a hunter is pointing a gun at it, it looks down at an angle and teachers may illustrate
this as the angle of depression.

Section 9.2: Bearings


To introduce the concept of true bearing, teachers may revisit the general compass bearing that students would
have learnt in Geography. Thus true North is equivalent to 000° or 360°and East is equivalent to 090° while
North-West will be represented by 315°, etc. From this explanation, students could understand better how the
concept of bearing is used in the different disciplines.

Section 9.3: Three-Dimensional Problems


Teachers can start off with an activity to introduce three-dimensional solids so that students are better able to
visualise these solids when solving three-dimensional problems (see Investigation: Visualising 3D Solids).
Teachers should highlight to the students that the basic technique used in solving three-dimensional problems is
to reduce it to a problem in a plane. Students may encounter difficulties in this area, as such, much practice and
guidance will have to be given to them.

Challenge Yourself
For Questions 1 and 2, students should be able to visualise and reduce the three-dimensional problem to a problem
in a plane. Students should approach the problem by applying trigonometric ratios to find the unknown sides and
angles in triangles.

238
WORKED SOLUTIONS Practise Now 2
Investigation (Visualising 3D Solids) 1. (i) !ACB = 25° (alt. !s)
1. Yes. 165
tan 25° =
2. 90° for both angles. BC
165
4. Yes, NOB BC = = 354 m (to 3 s.f.)
tan 25°
right angle look smaller or larger than 90 .
The distance of the sports car from the point B is 354 m.
5. Method 1: Find a rectangle
165 – 75
(a) Yes, the plane EFGH is a rectangle because it is a face of the (ii) tan =
353.84
cuboid.
90
(b) Since !EFG is a right angle of the rectangle EFGH, then = tan–1
353.84
!EFG = 90 and "EFG is a right-angled triangle. = 14.3° (to 1 d.p.)
(c) "CGH lies on the plane DCGH. The plane DCGH is a rectangle The angle of depression of the sports car from the centre of
because it is a face of the cuboid. Since !CGH is a right angle the wheels is 14.3°.
of the rectangle DCGH, then !CGH = 90 and so "CGH is a 2. C
right-angled triangle.
Method 2: Find a normal to a plane
52 m
(a) Since the plane EFGH is horizontal and the line CG is vertical,
then CG is a normal to the plane EFGH. 36° 24°
G A B
(b) Yes, the line GE is a line on the plane EFGH.
(c) Since CG is a normal to the plane EFGH, and GE is a line on 52
tan 36° =
the plane EFGH, then !CGE = 90 because a normal to a plane GA
52
is perpendicular to every line on the plane. Thus "CGE is a GA =
tan 36°
right-angled triangle.
= 71.57 (to 4 s.f.)
(d) Since the plane DCGH is vertical and the line EH is horizontal,
52
then EH is a normal to the plane DCGH. Since the line HC lies tan 24° =
GA + AB
on the plane DCGH, then the normal EH is perpendicular to the
tan 24° (71.57 + AB) = 52
line HC, i.e. !CHE = 90 . Therefore, "CHE is a right-angled
52
triangle. AB = – 71.57 = 45.2 m (to 3 s.f.)
tan 24°
The distance between the two ships is 45.2 m.
Practise Now 1
1. Let x m be the distance of the point on the ground from the foot of Practise Now 3
1. N
43
tan 34° = N N
x B 4

x tan 34° = 43
A
43 40°
x= = 63.8 (to 3 s.f.)
tan 34° 3
30°
1

tower is 63.8 m. 40°


50 O
2. tan 48° =
LA 2

LA tan 48° = 50
(a) The bearing of A from O is equal to (90° – 40°).
50
LA = = 45.02 (to 4 s.f.) Bearing of A from O is 050°.
tan 48°
50 (b) The bearing of B from O 2, which
tan 38° = is (360° – 30°).
LB
LB tan 38° = 50 Bearing of B from O is 330°.
50 (c) The bearing of O from A 3, which
LB = = 64.00 (to 4 s.f.)
tan 38° is (270° – 40°).
Distance between boats A and B = 64.00 – 45.02 Bearing of O from A is 230°.
= 19.0 m (to 3 s.f.) (d) The bearing of O from B is given by the obtuse angle 4 , which
(180° – 30°).
Bearing of O from B is 150°.

239
2. N 250 sin 65°
AC =
sin 47°
= 310 m (to 3 s.f.)
O 1 2 Using sine rule,
39° 33° BC 250
N N =
sin 68° sin 47°
4
3
33° 250 sin 68°
39° BC =
P sin 47°
Q
= 317 m (to 3 s.f.)

(a) The bearing of P from O is equal to (90° + 33°). Practise Now 5


Bearing of P from O is 123°. (i) !ABC = 360° – 238° – 60° = 62° (!s at a point)
(b) The bearing of Q from O 2 , which Using cosine rule,
is (270° – 39°). AC2 = 352 + 552 – 2 35 55 cos 62°
Bearing of Q from O is 231°. = 4250 – 3850 cos 62°
(c) The bearing of O from P 3 , which = 2443 (to 4 s.f.)
is (270° + 33°).
AC = 2443
Bearing of O from P is 303°.
= 49.4 km (to 3 s.f.)
(d) The bearing of O from Q is given by the acute angle 4 , which
(ii) Using sine rule,
is (90° – 39°).
sin BAC sin 62°
Bearing of O from Q is 051°. =
55 49.42
55 sin 62°
Practise Now 4 sin BAC =
49.42
1. Since the bearing of R from Q is 118°, 55 sin 62°
!BAC = sin–1
!PQR = 180° – 118° = 62° 49.42
i.e. !PRQ = 180° – 62° – 44° (! sum of a ") = 79.31 (to 2 d.p.)
= 74° 120° + 79.31° = 199.3° (to 1 d.p.)
Using sine rule, Bearing of town C from town A is 199.3°.
PQ 150
=
sin 74° sin 44° Practise Now 6
150 sin 74°
PQ = (i) N
sin 44°
= 208 m (to 3 s.f.)
2. (i) N
118° N
D
3.4 km
A N
1
68°
115° N
C C
250 m X
254°
1 126°
B
B A 2.6 km
115°
D 1 = 180° – 118° – 60° (int. !s)
The bearing of B from C , which = 62°
1

is (360° – 115°). i.e. !BCD = 360° – 254° – 62° (!s at a point)


Bearing of B from C is 245°. = 44°
(ii) !ABC = 180° – 115° = 65° (adj. !s on a str. line) !CBD = 180° – 126° (adj. !s on a str. line)
i.e. !ACB = 180° – 65° – 68° (! sum of a ") = 54°
= 47° !BDC = 180° – 54° – 44° (! sum of a ")
Using sine rule, = 82°
AC 250 118° + 82° = 200°
= Bearing of B from D is 200°.
sin 65° sin 47°

240
(ii) Using sine rule, (iii) Q
BD 3.4
=
sin 44° sin 54° 8 cm
3.4 sin 44°
BD =
sin 54° D 15 cm C
= 2.92 km (to 3 s.f.)
(iii) Using cosine rule, "DCQ, !DCQ = 90°.
AD2 = 2.62 + 2.9192 – 2 2.6 2.919 cos 126° 8
tan !CDQ =
= 24.20 (to 4 s.f.) 15
8
AD = 24.20 !CDQ = tan–1
15
= 4.92 km (to 3 s.f.) = 28.1° (to 1 d.p.)
(iv) The shortest distance from B to AD is BX, where BX is perpendicular 2. (i) 18 cm
B
to AD. 48° C
Using sine rule,
sin !BAD sin 126°
=
2.919 4.919 18 cm
2.919 sin 126°
!BAD = sin–1
4.919
= 28.69° (to 2 d.p.)
In "ABX, Q
BX Using cosine rule,
sin !BAX =
2.6 CQ2 = 182 + 182 – 2 18 18 cos 48°
BX = 2.6 sin 28.69°
= 648 – 648 cos 48°
= 1.25 km (to 3 s.f.)
= 214.4 (to 4 s.f.)
CQ = 214.4
Practise Now 7
= 14.6 cm (to 3 s.f.)
1. (i) B 24 cm C (iii) Using Pythagoras’ Theorem,
15 cm AQ2 = AB2 + BQ2
= 242 + 182
A = 576 + 324
= 900
In "ABC, !ABC = 90°.
AQ = 900
24
tan !BAC = = 30 cm
15
24 CA = AQ = 30 cm
!BAC = tan–1
15 A
= 58.0° (to 1 d.p.)
(ii) Q
30 cm

8 cm 30 cm

A C C

Using Pythagoras’ Theorem, 14.64 cm


B
AC2 = AB2 + BC2
= 152 + 242 Using cosine rule,
= 225 + 576 30 2 + 30 2 – 14.64 2
cos !CAQ =
= 801 2 30 30
AC = 801 = 0.8809 (to 4 s.f.)
= 28.30 cm (to 4 s.f.) !CAQ = cos–1 0.8809
In "ACQ, !ACQ = 90°. = 28.3 (to 1 d.p.)
28.30
tan !AQC =
8
28.30
!AQC = tan–1
8
= 74.2° (to 1 d.p.)
241
Practise Now 8 (iii) K

1. (i) K
10 cm

B 22.63 cm D

In !BDK,
B 16 cm C "DBK = 90°
10
In !BCK, tan "BDK =
22.63
"CBK = 90°
10
BK = 16 – 6 = 10 cm "BDK = tan–1
22.63
10 = 23.8° (to 1 d.p.)
tan "BCK =
16 2. (i) P
10
"BCK = tan–1
16
= 32.0° (to 1 d.p.)
8 cm
(ii) S

B 12 cm D
In !ABP,
16 cm "BAP = 90°
8
tan "ABP =
12
8
D C
"ABP = tan–1
12
= 33.7° (to 1 d.p.)
16 cm
(ii) Q
A 16 cm B
In !ABD,
Using Pythagoras’ Theorem, 8 cm
DB2 = AD2 + AB2
= 162 + 162
= 256 + 256 B 6 cm C
= 512 In !BCQ,
DB = 512 "CBQ = 90°
= 22.63 cm (to 4 s.f.) 8
tan "BCQ =
In !BDS, "BDS = 90°. 6
16 8
tan "SBD = "BCQ = tan–1
22.63 6
16 = 53.1° (to 1 d.p.)
"SBD = tan–1
22.63
= 35.3° (to 1 d.p.)

242
(iii) R Practise Now 10
T
38°
42°
8m 48°

54 m

D C 38°
42° O B

6m A

A 12 m B In !ATO,
"AOT = 90°
In !ABC,
54
Using Pythagoras’ Theorem, sin 42° =
AT
AC2 = AB2 + BC2
54
= 122 + 62 AT =
sin 42°
= 144 + 36 = 80.70 m (to 4 s.f.)
= 180 In !BTO,
AC = 180 "BOT = 90°
= 13.42 cm (to 4 s.f.) 54
sin 38° =
In !ACR, BT
"ACR = 90° 54
BT =
8 sin 38°
tan "CAR = = 87.71 m (to 4 s.f.)
13.42
8 Using cosine rule,
"CAR = tan–1
13.42 AB2 = AT2 + BT2 – 2 AT BT cos "ATB
= 30.8° (to 1 d.p.) = 80.702 + 87.712 – 2 80.70 87.71 cos 48°
= 4733 (to 4 s.f.)
Practise Now 9 AB = 4733
(i) In !ABC, = 68.8 m (to 3 s.f.)
"ACB = 180° – 105° – 32° (" sum of a !) The distance between A and B is 68.8 m.
= 43°
Using sine rule, Exercise 9A
BC 120 1. Let h m be the height of the kite above Lixin’s hand.
=
sin 105° sin 43° h
120 sin 105° sin 58° =
BC = 140
sin 43° h = 140 sin 58°
= 170 m (to 3 s.f.)
= 119 m (to 3 s.f.)
(ii) T
The height of the kite above Lixin’s hand is 119 m.
25 m 2. Let d m be the distance between the two buildings.
Height difference between the two buildings = 120 – 85
B 170.0 m C = 35 m
In !BCT,
25
tan "TBC = 35 m
170.0
25
"TBC = tan–1 33.9°
170.0 dm
= 8.4° (to 1 d.p.)
35
The angle of elevation of T from B is 8.4°. tan 33.9° =
d
35
d=
tan 33.9°
= 52.1 m (to 3 s.f.)
The distance between the two buildings is 52.1 m.

243
3. Let x° be the angle of elevation of the top of the spire from the 7.
point R.
44
tan x° =
84
45 m
44
x° = tan–1
84
= 27.6 (to 1 d.p.)
42° 37°
The angle of elevation of the top of the spire from the point R
xm
is 27.6°.
ym
4. Let d
building. 45
tan 42° =
x
33° 45
x=
tan 42°
41 m = 49.98 (to 4 s.f.)
45
33° tan 37° =
y
dm
45
41 y=
tan 33° = tan 37°
d = 59.72 (to 4 s.f.)
41
d= Distance between the two points = 59.72 – 49.98
tan 33°
= 9.74 m (to 3 s.f.)
= 63.1 m (to 3 s.f.)
8. Let a m be the height of the castle and b m be the height of the
mountain.
is 63.1 m.
5. Let h m be the height of the cliff.

28.9°
am
hm

28.9°
65.7 m
h
tan 28.9° =
65.7
h = 65.7 tan 28.9° bm
= 36.3 m (to 3 s.f.) 60°
The height of the cliff is 36.3 m.
45°
6. Let x° be the angle of depression of the bird from the top of the
castle. 55 m
b
x° tan 45° =
55
218 m b = 55 tan 45°
= 55

85 m a + 55
tan 60° =
55
218
tan x° = a + 55 = 55 tan 60°
85
a = 55 tan 60° – 55
218
x° = tan–1 = 40.3 (to 3 s.f.)
85
The height of the castle is 40.3 m.
= 68.7 (to 1 d.p.)
The angle of depression of the bird from the top of the castle
is 68.7°.

244
9. Let the distance of Q from the bottom of the bridge by y m and the 12. A
37°
distance of P from the bottom of the bridge be z m. B
32°
5.5
tan 23° =
y
5.5
y=
tan 23°
= 12.96 (to 4 s.f.)
z = 12.96 – 5.1 = 7.86 37° 32°
5.5 C
80 m
tan x° =
7.86 BC
5.5 tan 32° =
x° = tan–1 80
7.86 BC = 80 tan 32°
= 35.0 (to 1 d.p.)
= 49.99 m (to 4 s.f.)
320
10. tan 58° = AC
BC tan 37° =
80
320
BC = AC = 80 tan 37°
tan 58°
= 60.28 m (to 4 s.f.)
= 200.0 m (to 4 s.f.)
AB = 60.28 – 49.99
320
tan 38° = = 10.3 m (to 3 s.f.)
AC
The height of the satellite dish is 10.3 m.
320
AC = 13. Let d m be the distance between the bottom of the building and point
tan 38°
= 410.0 m (to 4 s.f.) T on level ground.
x = 410.0 – 200.0 h
tan 26° =
= 200 (to 3 s.f.) d
h
11. D d= — (1)
tan 26°
h + 12.2
tan 35° =
88 m d
h + 12.2
d= — (2)
18° 23° tan 35°
A B C
Substitute (1) into (2):
?m
h h + 12.2
=
88 tan 26° tan 35°
tan 23° = h tan 35° = tan 26° (h + 12.2)
BC
88 h tan 35° = h tan 26° + 12.2 tan 26°
BC = h tan 35° – h tan 26° = 12.2 tan 26°
tan 23°
= 207.3 m (to 4 s.f.) h(tan 35° – tan 26°) = 12.2 tan 26°
88 12.2 tan 26°
tan 18° = h=
AC (tan 35° – tan 26°)
88 = 28.0 (to 3 s.f.)
AC =
tan 18°
= 270.8 m (to 4 s.f.)
AB = 270.8 – 207.3
= 63.5 m (to 3 s.f.)
The distance between the two boats is 63.5 m.

245
14. Let h m be the height of the cliff. 3. (a) Bearing of Q from P is 036°.
(b) The bearing of P from Q is (180°+ 36°).
A
56° Bearing of P from Q is 216°.
27 m
(c) The bearing of R from P is (36° + 37°).
B
49° Bearing of R from P is 073°.
(d) 36° + 37° = 73°
The bearing of P from R is (180° + 73°)
hm Bearing of P from R is 253°.
(e) 100° – 36° = 64°
The bearing of Q from R is (360° – 64°)
56° Bearing of Q from R is 296°.
C (f) 100° – 36° = 64°
49°
The bearing of R from Q is (180° – 64°)
h
(i) tan 49° = Bearing of R from Q is 116°.
CD
4. N
h = CD tan 49° — (1)
h + 27
tan 56° =
CD R
h + 27 = CD tan 56° N
h = CD tan 56° – 27 — (2)
Substitute (1) into (2):
Q
CD tan 49° = CD tan 56° – 27 320°
CD tan 56° – CD tan 49° = 27 72° 24 km
CD(tan 56° – tan 49°) = 27 P
27
CD =
tan 56° – tan 49°
= 81.3 m (to 3 s.f.) (a) !PQR = 320° – 180° – 72°
The distance between the base of the cliff and the guard house = 68°
is 81.3 m. !PRQ = 180° – 68° – 72° (! sum of a ")
(ii) h = 81.28 tan 49° = 40°
= 93.5 m (to 3 s.f.) Using sine rule,
The height of the cliff is 93.5 m. PR 24
=
sin 68° sin 40°
Exercise 9B 24 sin 68°
PR =
sin 40°
1. (a) Bearing of A from O is 033°. = 34.6 km (to 3 s.f.)
(b) The bearing of B from O is (90° + 28°). (b) Using sine rule,
Bearing of B from O is 118°. QR 24
=
(c) The bearing of C from O is (270° – 44°). sin 72° sin 40°
Bearing of C from O is 226°. 24 sin 72°
QR =
(d) The bearing of D from O is (360° – 39°). sin 40°
Bearing of D from O is 321°. = 35.5 km (to 3 s.f.)
2. (a) The bearing of A from P is equal to (90° – 35°).
Bearing of A from P is 055°.
(b) The bearing of B from P is (180° – 15°).
Bearing of B from P is 165°.
(c) The bearing of C from P is (270° + 47°).
Bearing of C from P is 317°.
(d) The bearing of P from A is (270° – 35°).
Bearing of P from A is 235°.
(e) The bearing of P from B is (360° – 15°).
Bearing of P from B is 345°.
(f) The bearing of P from C is (90° + 47°).
Bearing of P from C is 137°.

246
5. N (b) The bearing of Q from R is (48° – 45°).
Bearing of Q from R is 003°.
135° Or
P
The bearing of Q from R is (360° – 45° – 48°).
12 km Bearing of Q from R is 267°.
(c) !QPR = 180° – 55° – 48° (! sum of a ")
120°
Q = 77°
135° – 77° = 58°
180° – 58° = 122°
The bearing of P from Q is (360° – 122°).
Bearing of P from Q is 238°.
Or
R
The bearing of P from Q is (77° – 45°).
!RPQ = 180° – 135° (adj. !s on a str. line) Bearing of P from Q is 032°.
= 45° 8. N N
!PRQ = 180° – 120° – 45° (! sum of a ")
= 15° 122° 122°
A C A
Using sine rule, 32°
1 N N
PR 12 32°
= 86°
sin 120° sin 15° 2

12 sin 120° B B
86°
PR =
sin 15°
= 40.2 km (to 3 s.f.)
6. N
B C
N = 180° – 122° ( adj. !s on a str. line)
1
1
40° = 58°
40°
1 The bearing of C from B is (86° – 58°).
Bearing of C from B is 028°.
A C
Or
2 = 180° – 122° (alt. !s)

= 58°
D The bearing of C from B is (360° – 86° – 58°).
Bearing of C from B is 216°.
(a) = 90° – 40° = 50°
1 9. (a) B
The bearing of B from C is equal to (360° – 50°)
Bearing of B from C is 310°.
(b) Bearing of A from C is 270°.
(c) The bearing of D from C is (360° – 90° – 50°).
Bearing of D from C is 220°. 280 m
C
7. N
N
Q 50°
N
55° 135° T
P

135° !CBT = 50° (isos. ")


P
N !BCT = 180° – 50° – 50° (! sum of a ")
N
N = 80°
55° 48° R Using sine rule,
48°
Q CT 280
=
R sin 50° sin 80°
280 sin 50°
CT =
(a) 180° – 135° = 45° sin 80°
The bearing of P from R is (360° – 45°). = 218 m (to 3 s.f.)
Bearing of P from R is 315°.

247
(b) B Using cosine rule,
PR2 = 6002 + 9502 – 2(600)(950) cos 47°
= 1 262 500 – 1 140 000 cos 47°
= 485 000 ( to 4 s.f.)
280 m PR = 485 000 = 696 m (to 3 s.f.)
C (ii) Using sine rule,
sin !QPR sin 47°
50° =
600 696.4
600 sin 47°
T sin !QPR =
696.4
CT = 0.6301 (to 4 s.f.)
cos 50° =
280 !QPR = sin–1 0.6301
CT = 280 cos 50° = 39.06° (to 2 d.p.)
= 180 m (to 3 s.f.) The bearing of R from P is (78° – 39.06°).
(c) B Bearing of R from P is 038.9° (to 1 d.p.).
C
12. N

A
280 m

34 m 57 m
50°

T
280
cos 50° = D B
CT d 37°
280
CT = N
cos 50° 28 m
= 436 m (to 3 s.f.) 1

10. 1 = 180° – 128° (int. !s) C


= 52°
34
!PQR = 360° – 295° – 52° (!s at a point) (i) cos !BAD =
57
= 13° 34
Using cosine rule, !BAD = cos–1
57
PR2 = 25° + 302 – 2(25)(30) cos 13° = 53.38 (to 2 d.p.)
= 1525 – 1500 cos 13° The bearing of B from A is (180° – 53.38°).
= 63.44 (to 4 s.f.) Bearing of B from A is 126.6° (to 1 d.p.).
PR = 63.44 = 7.97 km (to 3 s.f.) (ii) Let d m be the shortest distance from C to BD.
The distance between P and R is 7.97 km. d
sin !CBD =
11. N N
28
d
R sin 37° =
28
600 m d = 28 sin 37°
1 = 16.9 m (to 3 s.f.)
Q (iii) By Pythagoras’ Theorem,
78°
950 m 305° DB2 = 572 – 342
P = 2093
(i) = 360° – 305° (!s at a point)
1
DB = 2093
= 55° = 45.75 m (to 4 s.f.)
!PQR = 180° – 78° – 55° (int. !s)
= 47°

248
Using cosine rule, 14. N
P
CD2 = 282 + 45.752 – 2(28)(45.75) cos 37°
= 831.0 (to 4 s.f.) Q
CD = 831.0 = 28.83 m (to 4 s.f.) N 20 km
16.85 24 km 30°
cos =
28.83
16.85
= cos–1 300°
R
28.83
= 54.24° (to 2 d.p.)
After 2 hours,
The bearing of D from C is (360° – 54.24°).
Distance travelled by P = 10 2 = 20 km
Bearing of D from C is 305.8° (to 1 d.p.).
Distance travelled by Q = 12 2 = 24 km
13. N
360° – 300° = 60° (!s at a point)
N !PRQ = 60° + 30° = 90°
B "PRQ is a right-angled triangle.
68°
370 m By Pythagoras’ Theorem,
A
144° PQ2 = PR2 + QR2
= 202 + 242
= 976
510 m
PQ = 976
= 31.2 km (to 3 s.f.)
20
tan !PQR =
C 24
20
(i) Using cosine rule, !PQR = tan–1
24
BC 2 = 370° + 5102 – 2(370)(510) cos 76°
= 39.81° (to 2 d.p.)
= 397 000 – 377 400 cos 76°
The bearing of P from Q is (180° – 60° – 39.81°).
= 305 700 (to 4 s.f.)
Bearing of P from Q is 080.2°.
BC = 305 700 = 553 m (to 3 s.f.) 15. N
(ii) Using sine rule,
sin !ACB sin 76° Q
=
370 552.9 35 km
370 sin 76° N
sin !ACB =
552.9
32°
–1 370 sin 76°
!ACB = sin 108°
552.9 P
= 40.5° (to 1 d.p.)
S
(iii) 180° – 68° = 112° (int. !s) 65 km
!ABC = 180° – 76° – 40.49° (! sum of a ") R
= 63.51° (to 2 d.p.)
The bearing of C from B is (360° – 112° – 63.51°). (a) (i) !QPR = 108° – 32° = 76°
Bearing of C from B is 184.5°. Using cosine rule,
(iv) Let the shortest distance from A to BC be h m. QR2 = 352 + 652 – 2 35 65 cos 76°
1 = 4349 (to 4 s.f.)
Area of "ABC = 370 510 sin 76° QR = 65.9 km (to 3 s.f.)
2
= 94 350 sin 76° (ii) Using sine rule,
1 sin !PQR sin 76°
552.9 h = 94 350 sin 76° =
2 65 65.95
h = 331 m (to 3 s.f.) 65 sin 76°
!PQR = sin–1
The shortest distance from A to BC is 331 m. 65.95
= 73.00° (to 2 d.p.)
180° – 32° = 148°
The bearing of R from Q is (360° – 148° – 73.00°).
Bearing of R from Q is 139.0° (to 1 d.p.).

249
(b) !PQS = 32° (alt. !s) 2. (i) H
!ABC = 180° – 76° – 32° (! sum of a ")
= 72°
PS 35
=
sin 32° sin 72°
35 sin 32°
PS = 12 cm
sin 72°
= 19.50 km
19.50
Time taken = = 0.65 h = 39 minutes
30
The time when the ship reaches S is 1009.
D 5 cm B
Exercise 9C By Pythagoras’ Theorem,
1. (i) By Pythagoras’ Theorem, BH2 = DH2 + DB2
AC2 = 62 + 72 = 122 + 52
= 85 = 169

AC = 85 BH = 169
= 9.22 cm (to 3 s.f.) = 13 cm
(ii) P (ii) D

11 cm 5 cm
3 cm

C A C 4 cm B
9.220 cm

By Pythagoras’ Theorem, 4
tan !BDC =
PC 2 = 112 – 9.2202 3
4
= 35.99 (to 4 s.f.) !BDC = tan–1
3
PC = 35.99 = 53.1° (to 1 d.p.)
= 6.00 cm (to 3 s.f.) 12
9.220 (iii) tan !HBD =
(iii) cos !PAC = 5
11 12
9.220 !HBD = tan–1
!PAC = cos–1 5
11 = 67.4° (to 1 d.p.)
= 33.1° (to 1 d.p.) 4
(iv) P
3. (i) tan !DGH =
6
4
!DGH = tan–1
6
= 33.7° (to 1 d.p.)
5.999 cm
(ii) By Pythagoras’ Theorem,
HF2 = GH2 + GF2
= 62 + 82
C 6 cm B = 100
5.999 HF = 100
tan !PBC =
6 = 10 cm
5.999
!PBC = tan–1
6
= 45.0° (to 1 d.p.)
The angle of elevation of P from B is 45.0°.

250
(iii) D O

4 cm
10.64 m

H 10 cm F

4
tan !DFH =
10 B 8.320 m P
4 –1
!DFH = tan 10.64
10 tan !OBP =
8.320
= 21.8° (to 1 d.p.)
10.64
The angle of elevation of D from F is 21.8°. !OBP = tan–1
8.320
4. (i) A = 52.0° (to 1 d.p.)
5. !AQB = 180° – 75° – 45° (! sum of a ")
= 60°
Using sine rule,
10 m
AQ 60
=
sin 75° sin 60°
60 sin 75°
AQ =
B 15 m C sin 60°
= 66.92 m (to 4 s.f.)
15
tan !BAC = PQ
10 tan 30° =
66.92
15
!BAC = tan–1 PQ = 38.6 m (to 3 s.f.)
10
The height of the tower is 38.6 m.
= 56.3° (to 1 d.p.)
6. (i) By Pythagoras’ Theorem,
(ii) A P
56.31° AC 2 = AB2 + BC 2
= 202 + 202
= 800

10 m AC = 800
= 28.28 cm (to 4 s.f.)
28.28
AM =
2
= 14.1 cm (to 2 s.f.)
B (ii) O
AP
cos 56.31° =
10
AP = 10 cos 56.31° 32 cm

= 5.55 m (to 3 s.f.)


(iii) By Pythagoras’ Theorem,
OP2 = AO2 – AP2 A 14.14 cm M
= 122 – 5.5472
By Pythagoras’ Theorem,
= 113.2 (to 4 s.f.)
OM 2 = AO2 – AM 2
OP = 113.2 = 322 – 14.142
= 10.6 cm (to 3 s.f.) = 824.1 (to 4 s.f.)
(iv) By Pythagoras’ Theorem,
OM = 824.1
BP2 = AB2 – AP2
= 28.7 cm (to 3 s.f.)
= 102 – 5.5472
14.14
= 69.23 (to 4 s.f.) (iii) cos !OAM =
32
BP = 69.23 14.14
!OAM = cos–1
= 8.320 cm (to 4 s.f.) 32
= 63.8° (to 1 d.p.)

251
7. (i) A 8. (i) By Pythagoras’ Theorem,
EG2 = EH2 + HG2
75 cm = 82 + 152
= 289
25° EG = 289
B C
= 17 cm
C
BC
cos 25° =
75
BC = 75 cos 25° 7 cm
= 67.97 cm (to 4 s.f.)
C
E 17 cm G
7
tan !CEG =
17
69.97 cm 7
!CEG = tan–1
17
= 22.4° (to 1 d.p.)
(ii) By Pythagoras’ Theorem,
A 90 cm B XF 2 = XB2 + BF 2
By Pythagoras’ Theorem, = 42 + 72
AC2 = AB2 + BC2 = 65
= 902 + 69.972 XF = 65
= 13 000 (to 4 s.f.) = 8.062 cm (to 4 s.f.)
AC = 13 000 X
= 113 cm (to 3 s.f.)
PC
(ii) sin 25° =
75
PC = 75 sin 25°
8.062 cm
= 31.70 cm (to 4 s.f.)

31.70 cm F

8 cm
A 113 cm C G

31.70 8
tan !PAC = tan !GXF =
113 8.062
31.70 8
!PAC = tan–1 !GXF = tan–1
113 8.062
= 15.7° (to 1 d.p.) = 44.8° (to 1 d.p.)
(iii) C

67.97 cm

A 90 cm B

67.97
tan !CAB =
90
67.97
!CAB = tan–1
90
= 37.1° (to 1 d.p.)

252
9. (i) By Pythagoras’ Theorem, 10. (i) P 1000 m Q
AC2 = AB2 + BC2
= 122 + 162
= 400 750 m

AC = 400
N
= 20 cm
P
R

5 cm
1000
tan !PRQ =
750
A 10 cm C 1000
!PRQ = tan–1
750
By Pythagoras’ Theorem,
= 53.1° (to 1 d.p.)
AP2 = 52 + 102
Bearing of Q from R is 053.1°.
= 125
(ii) T
AP = 125
= 11.18 cm (to 4 s.f.)
P

11.18 cm 11.18 cm

3.5°
A 20 cm C P 1000 m Q
Using cosine rule, TP
tan 3.5° =
11.18 2 + 20 2 – 11.18 2 1000
cos !PAC =
2 11.18 20 TP = 1000 tan 3.5°
= 0.8945 (to 4 s.f.) = 61.2 m (to 3 s.f.)
!PAC = cos–1 0.8945 The height of the mast is 61.2 m.
= 26.6° (to 1 d.p.) (iii) T
(ii) P

61.16 m
11.18 cm 11.18 cm

P 750 m R
A 12 cm B
61.16
Using cosine rule, tan =
750
11.18 2 + 12 2 – 11.18 2
cos !PAB = 61.16
2 11.18 12 = tan–1
750
= 0.5367 (to 4 s.f.)
= 4.7° (to 1 d.p.)
!PAB = cos–1 0.5367
The angle of elevation of T from R is 4.7°.
= 57.5° (to 1 d.p.)

253
11. (a) C 80
(i) tan !ABC =
60
80
!ABC = tan–1
70 m 60
= 53.1° (to 1 d.p.)
50 m N N
Bearing of C from B is 053.1°.
(ii) Using cosine rule,
30 m 80 2 + 90 2 – 150 2
O B cos !DAC =
2 80 90
= –0.5556 (to 4 s.f.)
28 m
!DAC = cos–1 (–0.5556)
= 123.75° (to 2 d.p.)
A 123.75° – 90° = 33.75°
28 The bearing of D from A is (360° – 33.75°).
(i) tan !OBA = Bearing of D from A is 326.3°
30
28 (b) T
!OBA = tan–1 8.6°
30
= 43.03° (to 2 d.p.)
The bearing of A from B is (270° – 43.03°).
Bearing of A from B is 227°.
(ii) Using cosine rule,
50 2 + 30 2 – 70 2 8.6°
cos !COB = C A
2 50 30 80 m
= – 0.5 TA
!COB = cos–1 (– 0.5) tan 8.6° =
80
= 120° TA = 80 tan 8.6°
(iii) 120° – 90° = 30° = 12.1 m (to 3 s.f.)
The bearing of C from O is (360° – 30°). The height of the mast is 12.1 m.
Bearing of C from O is 330°.
T
(b) T

12.10 m

29°
O 30 m B
D A
TB 90 m
tan 29° =
30 12.10
tan !TDA =
TB = 30 tan 29° 90
= 16.6 m (to 3 s.f.) 12.10
!TDA = tan–1
90
12. (a) = 7.7° (to 1 d.p.)
The angle of depression of D from the top of the mast is 7.7°.
D 13. (a) (i) !ACB = 180° – 94° – 47° (! sum of a ")
= 39°
150 m
90 m Using sine rule,
N
BC 240
80 m =
A C sin 94° sin 39°
240 sin 94°
60 m BC =
sin 39°
= 380 m (to 3 s.f.)
B 1
(ii) Area of "ABC = 240 380.4 sin 47°
2
= 33 400 m2 (to 3 s.f.)

254
(iii) Let the shortest distance from A to BC be h m. P
Area of !ABC = 33 400 m2
1
BC h = 33 400
2
1
380.4 h = 33 400 10 m
2
h = 176 (to 3 s.f.)
The shortest distance from A to BC is 176 m.
(b)
Y 3m R

24 m By Pythagoras’ Theorem,
PR2 = PY 2 – RY 2
A = 102 – 32
175.6 m = 91
24 PR = 91
tan =
175.6 = 9.539 cm (to 4 s.f.)
24
= tan–1 P
175.6
= 7.8° (to 1 d.p.)
The greatest angle of depression of the boat from the top of
the tree is 7.8° 9.359 cm
8m
14. (i) P

Z R
10 m
8
sin "PRZ =
9.359
8
"PRZ = sin–1
9.359
Z 6m Y = 57.0° (to 1 d.p.)
6 15. (i) By Pythagoras’ Theorem,
cos "PYZ =
10 AC2 = 102 + 82
6 = 164
"PYZ = cos–1
10 AC = 164
= 53.1° (to 1 d.p.)
= 12.81 cm (to 4 s.f.)
(ii) By Pythagoras’ Theorem,
By Pythagoras’ Theorem,
PZ 2 = PY 2 – ZY 2
AB2 = 82 + 62
= 102 – 62
= 100
= 64
AB = 100
PZ = 64
= 10 cm
= 8 cm
By Pythagoras’ Theorem,
BC2 = 102 + 62
= 136
BC = 136
= 11.66 cm (to 4 s.f.)
Using cosine rule,
10 2 + 11.66 2 – 12.812
cos "ABC =
2 10 11.66
= 0.3081 (to 4 s.f.)
"ABC = cos–1 (0.3081)
= 72.0° (to 1 d.p.)

255
(ii) Area of surface to be coated with varnish 17. (i) C
1
= 10 11.66 sin 72.02° 95 m
2
= 55.5 cm2 (to 3 s.f.)
D
16. Let h m be the height of the tower and d m be the distance OB. 58 m
72 m
T N N
48°

A B
36 m
hm
!CAB = 90° – 48° = 42°
Using sine rule,
40°
O B sin!ACB sin 42°
dm =
36 58
h
tan 40° = 36 sin 42°
d sin !ACB =
58
h
d= — (1) = 0.4153 (to 4 s.f.)
tan 40°
!ACB = sin–1 0.4153
C
= 24.54° (to 2 d.p.)
!ABC = 180° – 42° – 24.54° (! sum of a ")
= 113.46°
70 m The bearing of C from B is (113.46° – 90°).
Bearing of C from B is 023.5° (to 1 d.p.).
(ii) Let the shortest distance from B to DC be h m.
O dm B
Using cosine rule,
By Pythagoras’ Theorem, 58 2 + 72 2 – 95 2
cos !CBD =
OC2 = d 2 + 702 2 58 72
= –0.05711 (to 4 s.f.)
OC = d 2 + 70 2 m
!CBD = cos–1 (–0.05711)
= 93.27° (to 1 d.p.)
hm 1
Area of "ABC = 72 58 sin 93.27°
2
25° = 2085 m2 (to 4 s.f.)
O 2 C
d + 70
2
m 1
h DC d = 2085
2
tan 25° = d 2 + 70 2 1
95 d = 2085
2
h = tan 25° d 2 + 70 2 — (2) d = 43.89 m (to 4 s.f.)
Substitute (1) into (2):
2
h
h = tan 25° + 70 2 35 m
tan 40°
2
h h P B
= + 70 2 43.89 m
tan 25° tan 40°
2 2 35
h h tan =
= + 702 43.89
tan 25° tan 40°
35
4.5989h2 = 1.420h2 + 4900 = tan–1
43.89
3.1789h2 = 4900
= 38.6° (to 1 d.p.)
h2 = 1541
The angle of depression of P from the top of the tower is
h = 1541 38.6°.
= 39.3 m (to 3 s.f.)
39.26
d =
tan 40°
= 46.8 m (to 3 s.f.)
The height of the tower is 39.3 m and the distance OB is 46.8 m.

256
Review Exercise 9 3. C

hm 10°
1. Let d m be the distance between the castle and point A and h m be
the height of the castle. 12°
O dm A 200 m B
C
h
tan 12° =
d
hm h
d= — (1)
tan 12°
47° 29° h
O dm A 35 m B tan 10° =
d + 200
tan 10° (d + 200) = h — (2)
h
tan 47° = Substitute (1) into (2):
d
h h
d= — (1) tan 10° + 200 = h
tan 47° tan 12°
h h tan 10°
tan 29° = + 200 tan 10°= h
d + 35 tan 12°
tan 29° (d + 35) = h — (2) 0.8296h + 200 tan 10° = h
Substitute (1) into (2): 0.1704h = 200 tan 10°
h h = 207 m (to 3 s.f.)
tan 29° + 35 = h
tan 47° The height of the lighthouse is 207 m.
h tan 29° AT
+ 35 tan 29° = h 4. (i) sin 40° =
tan 47° 23
0.5169h + 35 tan 29° = h AT = 23 sin 40°
0.4831h = 35 tan 29° = 14.8 cm (to 3 s.f.)
h = 40.2 m (to 3 s.f.) 14.78
(ii) tan !ABT =
14
The height of the castle is 40.2 m.
14.78
2. 24° !ABT = tan–1
14
C
32° = 46.6° (to 1 d.p.)
The angle of elevation of T from B is 46.6°.
hm AC
(iii) cos 40° =
23
32° 24° AC = 23 cos 40°
O dm A 80 m B = 17.62 cm (to 4 s.f.)
h By Pythagoras’ Theorem,
tan 32° = BC2 = AB2 + AC2
d
h = 142 + 17.622
d= — (1)
tan 32° = 506.5 (to 4 s.f.)
h BC = 506.5
tan 24° =
d + 80 = 22.5 cm (to 3 s.f.)
tan 24° (d + 80) = h — (2)
12
Substitute (1) into (2): 5. (i) tan 35° =
AB
h 12
tan 24° + 80 = h AB =
tan 32° tan 35°
h tan 24° = 17.1 m (to 3 s.f.)
+ 80 tan 24° = h
tan 32° (ii) By Pythagoras’ Theorem,
0.7125h + 80 tan 24° = h BC 2 = AC 2 – AB 2
0.2875h = 80 tan 24° = 24 – 17.142
h = 124 m (to 3 s.f.) = 282.2 (to 4 s.f.)
The height of the cliff is 124 m. BC = 282.2
= 16.8 m (to 3 s.f.)

257
12 !CAB = 225° – 135° = 90°
(iii) tan !BCT =
16.80 180° – 135° = 45° (int. !s)
12 !ABC = 360° – 250° – 45° (!s at a point)
!BCT = tan–1
16.80 = 65°
= 35.5° (to 1 d.p.) AB
The angle of depression of C from T is 35.5°. cos 65° =
50
6. (i) By Pythagoras’ Theorem, AB = 50 cos 65° = 21.13 m (to 4 s.f.)
AC 2 = AB2 + BC 2 AT
tan 35° =
= 122 + 162 21.13
= 400 AT = 21.13 tan 35°
AC = 400 = 14.8 m (to 3 s.f.)
= 20 m AC
sin 65° =
1 50
AX = 20 = 10 m AC = 50 sin 65° = 45.32 (to 4 s.f.)
2
By Pythagoras’ Theorem, 14.80
tan !ACT =
VX 2 = AV 2 – AX 2 45.32
= 262 – 102 14.80
!ACT = tan–1 = 18.1° (to 1 d.p.)
45.32
= 576
The height of of the tower is 14.8 m and the angle of elevation
VX = 576 of T from C is 18.1°.
= 24 m 8. (a) (i) 180° – 125° = 55° (int. !s)
(ii) V !ABC = 90° – 55° = 35°
Using cosine rule,
AC 2 = 4602 + 4302 – 2 460 430 cos 35°
26 m 26 m = 72 440 (to 4 s.f.)
AC = 72 440 = 269 m (to 3 s.f.)
(ii) Using sine rule,
A 20 m C
sin !ACB sin 35°
=
Using cosine rule, 430 269
26 2 + 26 2 – 20 2 430 sin 35°
cos !AVC = !ACB = sin–1
2 26 26 269
= 0.7041 (to 4 s.f.) = 66.5° (to 1 d.p.)
!AVC = cos–1 0.7041 (iii) Using sine rule,
= 45.2° (to 1 d.p.) sin !CAB sin 66.47°
=
(iii) By Pythagoras’ Theorem, 460 430
VP 2 = VX 2 + XP 2 460 sin 66.47°
sin !CAB =
= 242 + 62 430
= 612 460 sin 66.47°
!CAB = sin–1
430
VP = 612
= 78.76° (to 2 d.p.)
= 24.7 m (to 3 s.f.)
The bearing of C from A is (90° – 78.76°).
The length of the crack is 24.7 m.
Bearing of C from A is 011.2°.
7. N
1
(iv) Area of "ABC = 460 269.1 sin 66.47°
2
N
= 56 700 m2 (to 3 s.f.)
(b) Let the distance of the hot air balloon above C be h m.
A 135°

225°

B hm
250°
5.2°
50 m B 460 m C

258
h 10. (i) Using cosine rule,
tan 5.2° =
460 PS2 = 4.62 + 5.22 – 2 4.6 5.2 cos 134°
h = 460 tan 5.2° = 81.43 (to 4 s.f.)
= 41.86 m (to 4 s.f.)
PS = 81.43 = 9.02 km (to 3 s.f.)
(ii) N

Q 122° N

41.86 m
1 4.6 km
R
134° S
5.2 km
A 269.1 m C 52°

41.86 P
tan =
269.1 1= 180° – 52° (int. !s)
41.86 = 128°
= tan–1
269.1 The bearing of S from R is (360° – 134° – 128°).
= 8.8° (to 1 d.p.) Bearing of S from R is 098°.
The angle of elevation of the hot air balloon from A is 8.8°. (iii) !PQR = 180° – 122° (adj. !s on a str. line)
9. (a) (i) Using cosine rule, = 58°
650 2 + 520 2 – 900 2 !PRQ = 180° – 58° – 52° (! sum of a ")
cos !CAD =
2 650 520 = 70°
= – 0.1732 (to 4 s.f.) Using sine rule,
!CAD = cos–1 (– 0.1732) PQ 5.2
= 100.0° (to 1 d.p.) =
sin 70° sin 58°
(ii) The bearing of C from A is (226° – 100.0°). 5.2 sin 70°
PQ =
Bearing of C from A is 126.0°. sin 58°
(iii) !BAC = 226° – 90° – 100.0° = 36.0° = 5.76 km (to 3 s.f.)
Using sine rule, (iv) Let the shortest distance from R to PS be d km.
BC 520 1
= Area of "PRS = 5.2 4.6 sin 134°
sin 36.0° sin 54° 2
520 sin 36.0° = 8.603 km2 (to 4 s.f.)
BC =
sin 54° 1
9.024 d = 8.603
= 378 m (to 3 s.f.) 2
(iv) Let the shortest distance from A to CD be d m. d = 1.907 (to 4 s.f.)
1
Area of "ACD = 520 650 sin 100.0°
2
= 166 400 m2 (to 4 s.f.)
1 75 m
900 d = 166 400
2
d = 370 m (to 3 s.f.) 1.907 km R
The shortest distance from A to CD is 370 m.
(b) 0.075
tan =
1.907
0.075
= tan–1
80 m 1.907
= 2.3° (to 1 d.p.)
The greatest angle elevation of the top of the building as
369.8 m A
Huixian walks along PS is 2.3°.
80
tan =
369.8
80
= tan–1
369.8
= 12.2° (to 1 d.p.)
The angle of elevation of the top of the tower from P is 12.2°.

259
(v) Let the point where the policeman is standing be X.

X
R

52° 5.2 km

!XRP = 52° (isos. ")


!PXR = 180° – 52° – 52° (! sum of a ")
= 76°
Using sine rule,
PX 5.2
=
sin 52° sin 76°
5.2 sin 52°
PX =
sin 76°
= 4.22 km (to 3 s.f.)
The distance between the policeman and P is 4.22 km.

Challenge Yourself
1. In "BCP,
PC
sin =
BP
PC = BP sin
In "ABP,
BP
cos =
AP
BP
AP =
cos
In "ACP,
PC BP sin
sin = =
AP BP
cos
sin = sin cos
2. In "ABC,
BC
tan =
h
BC = h tan
In "TBC,
BC
cos =
TB
BC h tan
TB = =
cos cos

260
Chapter 10 Arc Length, Sector Area and Radian Measure
TEACHING NOTES
Suggested Approach
In this chapter, students will be introduced to circles and how to calculate the arc length and area of the sector of a circle.
Teachers may begin the chapter by asking students to identify the different parts of a circle using real-life examples of arcs,
sectors and segments of a circle.

Students are expected to know how to apply the formulas of trigonometry and Pythagoras’ Theorem when solving problems
involving the arc length, sector area and radian measure.

Section 10.1: Length of Arc


Teachers may begin the chapter by showing students a circle with centre O and highlight to students the minor arc,
major arc, minor sector, major sector, minor segment and major segment of a circle. Once students are familiar
with these terms and are able to identify the parts of a circle, teachers can proceed to guide students on how to
derive the formula for the length of an arc of a circle (see Investigation: Arc Length).

Section 10.2: Area of Sector


Teachers can ask students to discover the formula for the area of sector on their own (see Investigation: Area of
Sector). Teachers should take note that some students may need some guidance when finding the area of a shaded
region involving sectors of circles. Teachers can suggest to students that when tackling such questions, they may
need to draw additional lines in the figures given to help them better visualise and work out the solutions.

Section 10.3: Radian Measure


So far, students have been using the measurement of 360° to denote the angle for one complete revolution. In this
section, students will learn how to use the measurement of 2 to denote the angle for one complete revolution
(see Investigation: Visualise the Size of an Angle of 1 radian and Class Discussion: Estimate the Size of Angles
in Radians).

Since the students have learnt how to find the arc length and area of sector in the previous sections, teachers may
ask students to find the arc length and area of a sector, giving their answers both in degrees and radians, and ask
them if the answers that they got for both are the same.

Teachers should remind students that when questions require the answers to be given in radians, students have
to first set the calculator to the ‘radian’ mode.

Section 10.4: Arc Length and Area of Sector using Radian Measure
In this section, students will solve problems involving trigonometry and Pythagoras’ Theorem. Students need
to have grasped these concepts which they have learnt in Books 2 and 3 well in order to solve such problems.
Teachers may guide the students along as they practise as many problems as possible.

Challenge Yourself
For Question 1, students need to make an observation from the perimeters of each figure. For Question 2,
students need to use Pythagoras’ Theorem to find the value of r and then be able to solve the remaining parts
of the question. For Questions 3 and 4, students will need to add additional lines to each of the figures in both
questions so that they can better visualise and find the respective areas of the shaded region.

261
WORKED SOLUTIONS Practise Now 2
Investigation (Arc Length) 228°
1. (i) Length of major arc AYB = 2 25
3. The third last column and the last column are equivalent. 360°
4. The third last column and the last column are equivalent. = 99.5 cm (to 3 s.f.)
x° (ii) Perimeter of minor sector = length of arc AXB + OA + OB
5. Arc length = circumference, where x° is the angle subtended
360° 360° – 228°
= 2 25 + 25 + 25
by the arc at the centre of the circle of radius r. 360°
= 108 cm (to 3 s.f.)
Investigation (Area of Sector) 2. Perimeter of shaded region = length of major arc AOB + OA + OB
360° – 150°
3. The third last column and the last column are equivalent. = 2 9+9+9
360°
4. The third last column and the last column are equivalent.
x° 21
5. Area of a sector of a circle = area of the circle, where x° is = + 18 cm
360° 2
the angle subtended by the arc at the centre of the circle of radius r. 3. Length of major arc PXQ = 36 cm
360° – 50°
2 r = 36
Investigation (Visualise the Size of an Angle of 1 radian) 360°
5.411r = 36
5
4. = = 1 radian r = 6.65
5
15
6. (a) (i) Size of the angle 3 radians Practise Now 3
5
30 1. !OQR = 180° – 90° – 36.9° (! sum of a ")
(ii) Size of the angle 6 radians
5 = 53.1°
(b) The approximate size of !AOB is 60°. RQ
(c) Hence, an angle of 1 radian is approximately equal to 60°. tan 36.9° =
8
2 (5) RQ = 8 tan 36.9°
7. (a) = = 2 radians
5 = 6.007 m (to 4 s.f.)
(b) 2 radians = 360° 36.9°
360° Length of arc RP = 2 8
1 radian = 360°
2 = 1.64 m
180° By Pythagoras’ Theorem,
=
OQ2 = OR2 + RQ2
= 82 + 6.0072
Class Discussion (Estimate the Size of Angles in Radians)
= 100.1 (to 4 s.f.)
(i) Figure (c) OQ = 100.1
(ii) Figure (a)
= 10.00 m (to 4 s.f.)
(iii) Figure (f)
PQ = OQ – OP
(iv) Figure (d)
= 10.00 – 8
(v) Figure (b)
= 2.00 m
(vi) Figure (e)
Perimeter of shaded region PQR = RQ + PQ + length of arc RP
= 6.007 + 2.00 + 1.64
Practise Now 1 = 13.2 m
40 80°
External radius of ring, R = 2. Perimeter of sector = 2 10 + 10 + 10
2 360°
= 20 mm = 34.0 cm (to 3 s.f.)
33
Internal radius of ring, r =
2 Practise Now 4
= 16.5 mm
(i) In "OQR,
Area of ring = R2 – r2
RQ
= (20)2 – (16.5)2 cos 36° =
35
= (202 – 16.52) RQ = 35 cos 36°
= 401 mm2 (to 3 s.f.) = 28.32 cm (to 4 s.f.)
PQ = 2RQ = 2(28.32) = 56.6 cm (to 3 s.f.)

262
(ii) !ROQ = 180° – 90° – 36° (! sum of a ") (ii) Using Pythagoras’ Theorem,
= 54° DT 2 = 342 – 42
!POQ = 2 !ROQ = 2(54°) = 108° DT = 34 2 – 4 2
108°
Length of arc PAQ = 2 35 = 1140
360°
= 33.76 m (to 4 s.f.)
= 65.97 cm (to 4 s.f.)
!DAT = 180° – 90° – 6.76° (! sum of a ")
1
Length of arc PBQ = 56.63 = 83.24°
2
= 88.95 cm (to 4 s.f.) Area of shaded region
Perimeter of shaded region = 65.97 + 88.95 = Area of trapezium ABCD – area of sector APB – area of sector DPC
= 15.5 cm (to 3 s.f.) 1 83.24° 96.76°
= (15 + 19)(33.76) – 192 – 152
2 360° 360°
= 122 m2 (to 3 s.f.)
Practise Now 5
(i) Since the length of the minor arc AQB is 33 cm, Practise Now 7
!AOB
2 15= 33 (a) Since radians = 180°,
360°
!AOB 33 180°
= rad =
360° 30 15 15
33 = 12°
!AOB = 360° = 126.1° (to 1 d.p.) (b) Since radians = 180°,
30
(ii) !AOB = 360° – 126.05° (!s at a point) 3 3 180°
rad =
= 233.95° 2 2
233.95° = 270°
Area of major sector OAPB = 152 (c) Since radians = 180°,
360°
= 459 cm2 (to 3 s.f.) 180°
1 radian =

Practise Now 6 180°


3.04 radians = 3.04
(i) Draw a line DT such that T lies on AB and DT is perpendicular to = 174.2° (to 1 d.p.)
AB. (d) Since radians = 180°,
A 180°
P 1 radian =
T D
180°
19 m 8 radians = 8
15 m
X Y = 458.4° (to 1 d.p.)
B C

AT = AB – TB Practise Now 8
= 19 – 15 (a) Since 180° = radians,
=4m
AD = AP + PD 36° = 36°
180°
= 19 + 15 = 0.628 rad (to 3 s.f.)
= 34 m (b) Since 180° = radians,
In "ATD,
288° = 288°
AT 4 180°
sin !ADT = =
AD 34 = 5.03 rad (to 3 s.f.)
4 (c) Since 180° = radians,
!ADT = sin–1 = 6.76° (to 2 d.p.)
34
!ADC = 6.76° + 90° = 96.8° (to 1 d.p.) 197.5° = 197.5°
180°
= 3.45 rad (to 3 s.f.)
(d) Since 180° = radians,

400° = 400°
180°
= 6.98 rad (to 3 s.f.)

263
Practise Now 9 AB = AC – BC
= 8.527 – 3.227
(a) Press sin 0 . 6 5 = and the display shows
= 5.30 cm (to 3 s.f.)
0.605 186 405,
i.e. sin 0.65 = 0.605 (to 3 s.f.) Practise Now 13
(b) Press cos 0 . 2 3 5 = and the display shows
7
0.972 514 341, !AOB = 2 –
4
i.e. cos 0.235 = 0.973 (to 3 s.f.)
(c) Press tan 1 . 2 3 = and the display shows = rad
4
2.819 815 734,
Length of minor arc APB = 6
i.e. tan 1.23 = 2.82 (to 3 s.f.) 4
= 4.71 cm (to 3 s.f.)
Practise Now 10
–1
Practise Now 14
(a) For sin x = 0.87, press sin 0 . 8 7 = to get
1.055 202 321, 1. (i) Given that the perimeter = 33 m,
i.e. when sin x = 0.87, r + r + s = 33
x = 1.06 rad (to 3 s.f.) 12 + 12 + s = 33
(b) For cos x = 0.347, press cos–1 0 . 3 4 7 = to get 24 + s = 33
1.216 425 878, s = 33 – 24
i.e., when cos x = 0.347, =9m
x = 1.22 rad (to 3 s.f.) Using s = r ,
(c) For tan x = 0.88, press tan–1 0 . 8 8 = to get 9 = 12
0.721 654 85, 9
= = 0.75 rad
12
i.e. when tan x = 0.88,
!AOB = 0.75 rad
x = 0.722 rad (to 3 s.f.)
1
(ii) Area of sector OAB = 122 0.75
2
Practise Now 11
= 54 m2
QR 2. Given that the area = 30.25 cm2,
(i) tan 0.63 =
9.6 1
r2 = 30.25
QR = 9.6 tan 0.63 2
= 7.00 cm (to 3 s.f.) 1
552 = 30.25
9.6 2
(ii) cos 0.63 =
PR 15.125 = 30.25
9.6 = 2 rad
PR =
cos 0.63 Length of arc = r
= 11.9 cm (to 3 s.f.) = 5.5 2
= 11 cm
Practise Now 12 Perimeter of sector = r + r + s
8.3 = 5.5 + 5.5 + 11
(i) sin !KAC = = 22 cm
11.9
8.3
!KAC = sin–1
11.9 Practise Now 15
= 0.772 rad (to 3 s.f.)
OP
8.3 (i) cos 0.98 =
(ii) tan 1.2 = 19
BC OP = 19 cos 0.98
8.3
BC = = 10.6 m (to 3 s.f.)
tan 1.2
AP
= 3.23 cm (to 3 s.f.) (ii) sin 0.98 =
19
8.3
(iii) tan 0.7719 = AP = 19 sin 0.98
AC
= 15.8 m (to 3 s.f.)
8.3
AC =
tan 0.7719
= 8.527 cm (to 4 s.f.)

264
1 5 2
(iii) Area of sector OAB = 192 0.98 (b) Area of circle, r2 = 779 m
2 8
= 176.89 m2 5
779
1 2
r = 8
Area of !OAP = 10.6 15.78
2
= 83.634 m2 5
779
Area of shaded region = 176.89 – 83.634 r= 8 m
= 93.4 m2 (to 3 s.f.)
5
779
Practise Now 16 Diameter of circle = 2 8

(i) Length of arc AB = 12 1.08 = 31.5 m (to 3 s.f.)


= 12.96 m (c) Area of circle, r = 3850 cm2
2

1 3850
(ii) Area of sector OAB = 122 1.08 r2 =
2
= 77.76 m2
3850
OK = 12 – 5.5 = 6.5 cm r= cm
1
Area of !OAK = 12 6.5 sin 1.08 3850
2 Diameter of circle = 2
= 34.40 cm2 (to 4 s.f.)
= 70.0 cm (to 3 s.f.)
Area of shaded region = 77.76 – 34.40
15
= 43.4 m2 (to 3 s.f.) 2. (a) External radius of ring, R =
2
= 7.5 cm
Practise Now 17
13
Internal radius of ring, r =
(i) Using cosine rule, 2
AC2 = 4.82 + 4.82 – 2 4.8 4.8 cos 2.4 = 6.5 cm
= 80.06 (to 4 s.f.) Area of ring = R2 – r2
= (7.5)2 – (6.5)2
AC = 80.06
= (7.52 – 6.52)
= 8.95 cm (to 3 s.f.)
= 44.0 cm2 (to 3 s.f.)
(ii) "AOC = (2 – 2.4) rad
1.2
Length of major arc ABC = (4.8)(2 – 2.4) = (9.6 – 11.5) cm (b) External radius of ring, R =
2
1
(iii) Area of major sector ABC = 4.82 (2 – 2.4) = 0.6 m
2
0.9
= 44.73 cm2 (to 4 s.f.) Internal radius of ring, r =
2
1
Area of !AOC = 4.8 4.8 sin 2.4 = 0.45 m
2
Area of ring = R2 – r2
= 7.781 cm2 (to 4 s.f.)
= (0.6)2 – (0.45)2
Area of major segment ABC = 44.73 + 7.781
= (0.62 – 0.452)
= 52.5 cm2 (to 3 s.f.)
= 0.495 m2 (to 3 s.f.)
1
Exercise 10A 3. (a) (i) Area of big semicircle = 142
2
1. (a) Area of circle, r2 = 616 mm2 = 98 cm2
616 Area of two small semicircles
r2 = 2
1 14
=2
616 2 2
r= mm
= 49 cm2
616 Area of shaded region = 98 – 49
Diameter of circle = 2 = 49
= 28.0 mm (to 3 s.f.) = 154 cm2 (to 3 s.f.)

265
(ii) Length of arc of big semicircle 4. (a) Area of shaded region
1 = Area of square – area of 4 circles
= 28 2
2 56
= 562 – 4
= 14 cm 4
Length of arc of small semicircles = 3136 – 784
1 = 673 cm2 (to 3 s.f.)
=2 14
2 (b) Area of unshaded region
= 14 cm = 4 area of semicircles
2
Perimeter of shaded region 1 3.5
=4
= 14 + 14 2 2
= 28 = 6.125 cm2
= 88.0 cm (to 3 s.f.) Area of shaded region
(b) (i) Area of square = 142 = Area of circle – area of unshaded region
= 196 cm2 = 3.52 – 6.125
2
1 14 = 19.2 cm2 (to 3 s.f.)
Area of 4 quadrants = 4
4 2 (c) Area of shaded region
= 49 cm2 = Area of big circle – area of small circle
2
Area of shaded region = 196 – 49 14
= 142 –
= 42.1 cm2 2
(ii) Perimeter of shaded region = (142 – 72)
= 4 arc length of quadrant = 462 cm2 (to 3 s.f.)
1 (d) Area of middle shaded region
=4 14 2
4 1 14
= 48 14 – 2
= 44.0 cm (to 3 s.f.) 2 2
1 = (672 – 49 ) cm2
(c) (i) Area of shaded region = area of circle
2 Area of shaded region
2
1 1 48
= 282 =2 + (672 – 49 )
2 2 2
= 1230 cm2 (to 3 s.f.) = 576 + (672 – 49 )
(ii) Arc length of big semicircle = 2330 cm2 (to 3 s.f.)
1 5. (a) Diameter of outer circular edge of grass
= (28 2)
2 = 12 2 + 2 + 2
= 28 cm = 28 m
Arc length of small semicircle Circumference of outer circular edge of grass
1 = 28
= 28
2 = 28 m
= 14 cm 2
28
Perimeter of shaded region (b) Area of land in between = – (12)2
2
= 28 + 14 + 14 = (142 – 122)
= 176 cm (to 3 s.f.) = 52 m2
(d) (i) Area of square = 102 6. (a) Area of shaded region
= 100 cm2 = Area of rectangle + area of semicircle
Area of shaded region 1 4
2

= Area of square – area of 4 quadrants =5 4+


2 2
1 = 20 + 2
= 100 – 4 3.52
4 = 26 m2 (to 2 s.f.)
= 100 – 12.25 (b) Total length painted in black
= 61.5 cm2 (to 3 s.f.) = Circumference of circle + 5 + 4 + 5
(ii) Perimeter of shaded region = 4 + 14
1 = 27 m (to 2 s.f.)
=4 (3.5 2) + 4 (10 – 3.5 2)
4 82°
= 7 + 12 7. (a) Length of arc AXB = 2 8
360°
= 34.0 cm (to 3 s.f.) = 11.4 cm (to 3 s.f.)

266
134° (b) Since the length of arc is 19.5 m,
(b) Length of arc AXB = 2 14
360°
2 14 = 19.5
= 32.7 cm (to 3 s.f.) 360°
214° 0.2443 = 19.5
(c) Length of arc AXB = 2 17
360° = 80° (to the nearest degree)
= 63.5 cm (to 3 s.f.) (c) Since the length of arc is 64.2 m,
(d) !AOB = 360° – 46° = 314°
2 14 = 64.2
314° 360°
Length of arc AXB = 2 9.8
360° 0.2443 = 64.2
= 53.7 cm (to 3 s.f.) = 263° (to the nearest degree)
76° (d) Since the length of arc is 84.6,
8. (a) (i) Length of minor arc AXB = 2 9
360°
= 11.9 cm (to 3 s.f.) 2 14 = 84.6
360°
(ii) !AOB = 360° – 76° = 284° 0.2443 = 84.6
Perimeter of major sector OAYB = 346° (to the nearest degree)
= length of major arc AYB + OA + OB 45°
11. Distance travelled by the tip of the hour hand = 2 1.5
284° 360°
= 2 9+9+9
360° = 1.18 m (to 3 s.f.)
= 62.6 cm (to 3 s.f.) 12. Since the length of wire is 32 cm,
112°
(b) (i) Length of minor arc AXB = 2 16 2 6 + 6 + 6 = 32
360° 360°
= 31.3 cm (to 3 s.f.) 0.1047 + 12 = 32
(ii) !AOB = 360° – 112° = 248° 0.1047 = 20
Perimeter of major sector OAYB = 191.0° (to 1 d.p.)
= length of major arc AYB + OA + OB 13. (a) Since the perimeter of minor sector is 77.91 cm,
248° 148°
= 2 16 + 16 + 16 2 r + r + r = 77.91
360° 360°
= 101 cm (to 3 s.f.) 2.583r + 2r = 77.91
(c) (i) !AOB = 360° – 215° = 145° 4.583r = 77.91
145° r = 17.0 cm (to 3 s.f.)
Length of minor arc AXB = 2 17.6
360° (b) !AOB = 360° – 44° = 316°
= 44.5 cm (to 3 s.f.) Since the perimeter of major sector is 278.1 cm,
(ii) Perimeter of major sector OAYB 316°
= length of major arc AYB + OA + OB 2 r + r + r = 278.1
360°
215° 5.515r + 2r = 278.1
= 2 17.6 + 17.6 + 17.6
360° 7.515r = 278.1
= 101 cm (to 3 s.f.) r = 37.0 cm (to 3 s.f.)
9. (a) Since the length of minor arc is 26.53 cm, 60°
95° 14. Perimeter of arc AOB = 2 8
2 r = 26.53 360°
360° 8
1.658r = 26.53 = cm
3
r = 16.0 cm (to 3 s.f.) 60°
Perimeter of arc POQ = 2 17
(b) Since the length of major arc is 104.6 cm, 360°
214° 17
2 r = 104.6 = cm
360° 3
3.735r = 104.6 AP = BQ = 17 – 8 = 9 cm
r = 28.0 cm (to 3 s.f.) Perimeter of shaded region = arc AB + arc PQ + AP + BQ
10. (a) Since the length of arc is 12 m, 8 17
= + +9+9
3 3
2 14 = 12
360° 25
= 18 + cm
0.2443 = 12 3
= 49° (to the nearest degree)

267
!AOB 115.59°
15. (i) Length of minor arc AOB = 2 r cm (ii) Length of arc = 2 13
360° 360°
Circumference of circle = (2 r) cm = 26.23 cm (to 4 s.f.)
7 Perimeter of shaded region = 26.23 + 22
Since the length of the minor arc is of the circumference = 48.2 cm (to 3 s.f.)
24
of the circle, 19. (i) AP = 16 cm
!AOB 7 OP = OP = 9 cm
2 r = 2 r
360° 24 Using cosine rule,
!AOB 7 9 2 + 16 2 – 9 2
= cos !APB =
360° 24 2 9 16
7 = 0.8889 (to 4 s.f.)
!AOB = 360°
24 !APB = cos–1 0.8889
= 105° = 27.3° (to 1 d.p.)
14 (ii) !PAO = !APB = 27.27°
(ii) Radius of circle = = 7 cm
2 !AOB = 2 27.27° (ext. ! of ")
105° = 54.4° (to 1 d.p.)
Length of minor arc = 2 7
360° 54.54° 2
= 12.8 cm (to 3 s.f.) (iii) Length of arc ABD = 2 9
360°
14 = 17.13 cm (to 4 s.f.)
16. (i) tan !POA =
7.5 27.27° 2
14 Legnth of arc ACD = 2 16
!POA = tan–1 360°
7.5 = 15.23 cm (to 4 s.f.)
= 61.8° (to 1 d.p.)
Perimeter of shaded region = 17.13 + 15.23
61.82°
(ii) Length of minor arc OAB = 2 7.5 = 32.4 cm (to 3 s.f.)
360°
20. Let the radius of the circle be r cm.
= 8.092 cm (to 4 s.f.)
!OBA = !OAB = 30°
By Pythagoras’ Theorem,
!AOB = 180° – 30° – 30° (! sum of a ")
OP2 = 7.52 + 142
= 120°
= 252.25
Using cosine rule,
OP = 252.25 15
2

= 15.88 cm (to 4 s.f.) 13 = r2 + r2 – 2 r r cos 120°


2
BP = OP – OB = 15.88 – 7.5 = 8.38 cm 168.75 = 2r2 + r2
Perimeter of shaded region = arc OAB + BP + AP 168.75 = 3r2
= 8.092 + 8.38 + 14 r2 = 56.25
= 30.5 cm (to 3 s.f.) r = 7.5
138° 360° – 120°
17. (i) !ROQ = = 69° Length of arc = 2 7.5
2 360°
QR = 31.42 cm (to 4 s.f.)
tan 69° =
26 15
QR = 26 tan 69° Perimeter of shaded region = 3 + 31.42
2
= 67.7 cm (to 3 s.f.) = 44.4 cm (to 3 s.f.)
138° OP
(ii) Length of minor arc OPQ = 2 26 21. sin 36° =
360° 14
= 62.62 cm (to 4 s.f.) OP = 14 sin 36°
Perimeter of shaded region = arc OPQ + RP + QR = 8.229 cm (to 4 s.f.)
= 62.62 + 67.73 + 67.73 !POR = 36° + 90° (ext. ! of ")
= 198 cm (to 3 s.f.) = 126°
18. (i) Using cosine rule, 126°
Length of arc PR = 2 8.229
132 + 132 – 22 2 360°
cos !AOB =
2 13 13 = 18.10 cm (to 4 s.f.)
= – 0.4320 (to 4 s.f.) OR = OP = 8.229 cm
!AOB = cos–1 (– 0.4320)
= 115.6° (to 1 d.p.)

268
TR = 14 + 8.229 = 22.229 cm (d) Arc length = 220 cm
36° 150°
Length of arc QR = 2 22.229 2 r = 220
360° 360°
= 13.97 cm (to 4 s.f.) 5
r = 220
TP 6
cos 36° =
14 Radius, r = 84.0 cm (to 3 s.f.)
TP = 14 cos 36° 150°
Area = 84.032
= 11.33 cm (to 4 s.f.) 360°
PQ = 22.229 – 11.33 = 9240 cm2 (to 3 s.f.)
= 10.899 cm Perimeter = 220 + 84.03 + 84.03
Perimeter of shaded region = arc PR + arc QR + PQ = 388 cm (to 3 s.f.)
= 18.10 + 13.13.97 + 10.899 (e) Arc length = 55 m
= 43.0 cm (to 3 s.f.) 2 14 = 55
360°
Exercise 10B 7
= 55
90
72° 55 90
1. (a) Arc length = 2 7 Angle at centre, =
360° 7
= 8.80 cm (to 3 s.f.) = 225.1° (to 1 d.p.)
72° 225.09°
Area = 72 Area = 142
360° 360°
= 30.8 cm2 (to 3 s.f.) = 385 m2 (to 3 s.f.)
Perimeter = 8.796 + 7 + 7 Perimeter = 55 + 14 + 14
= 22.8 cm (to 3 s.f.) = 83 m
(b) Perimeter = 136 (f) Area = 154 cm2
s + 35 + 35 = 136 75°
r2 = 154
s + 70 = 136 360°
Arc length, s = 66 mm 5
r2 = 154
24
2 35 = 66 mm
360° 739.2
r2 =
7
= 66
36 r = 15.3 cm (to 3 s.f.)
66 36 75°
Angle at centre, = Arc length = 2 15.34
7 360°
= 108.0° (to 1 d.p.) = 20.1 cm (to 3 s.f.)
108.04° Perimeter = 20.08 + 15.34 + 15.34
Area = 352
360° = 50.8 cm (to 3 s.f.)
= 1150 mm2 (to 3 s.f.) 30°
2. (a) (i) Perimeter = 2 7+7+7
(c) Area = 1848 mm2 360°
270° = 17.7 cm (to 3 s.f.)
r2 = 1848
360° 30°
(ii) Area = 72
3 360°
r2 = 1848
4 = 12.8 cm2 (to 3 s.f.)
2464 360° – 340°
r2 = (b) (i) Perimeter = 2 3.5 + 3.5 + 3.5
360°
Radius, r = 28.0 mm (to 3 s.f.) = 8.22 cm (to 3 s.f.)
270° 20°
Arc length = 2 28.00 (ii) Area = 3.52
360° 360°
= 132 mm (to 3 s.f.) = 2.14 cm2 (to 3 s.f.)
Peterimeter = 131.9 + 28.00 + 28.00 140°
(c) (i) Perimeter = 2 6+6+6
= 188 mm (to 3 s.f.) 360°
= 26.7 cm (to 3 s.f.)
140°
(ii) Area = 62
360°
= 44.0 cm2 (to 3 s.f.)

269
3. Circumference of circle = 88 cm 198°
(d) Area of sector OPSQ = 35.002
2 r = 88 360°
r = 14.01 cm (to 4 s.f.) = 2120 cm2 (to 3 s.f.)
60° 198°
(a) Length of arc ACB = 2 14.01 Legnth of arc = 2 35.00
360° 360°
= 14.7 cm (to 3 s.f.) = 121 cm (to 3 s.f.)
60° 5. (a) Area of minor sector = 114 cm2
Area of sector OACB = 14.012
360° 150°
r2 = 114
= 103 cm2 (to 3 s.f.) 360°
99° 5
(b) Length of arc ACB = 2 14.01 r2 = 114
360° 12
= 24.2 cm (to 3 s.f.) 273.6
r2 =
99°
Area of sector OACB = 14.012
360° r = 9.33 cm (to 3 s.f.)
= 169 cm2 (to 3 s.f.) (b) Area of major sector = 369 cm2
126° 360° – 66°
(c) Length of arc ACB = 2 14.01 r2 = 369
360° 360°
= 30.8 cm (to 3 s.f.) 49
r2 = 369
126° 60
Area of sector OACB = 14.012
360° 22 140
r2 =
= 216 cm2 (to 3 s.f.)
216° r = 12.0 cm (to 3 s.f.)
(d) Length of arc ACB = 2 14.01
360° 18
6. Radius of circle = = 9 cm
= 52.8 cm (to 3 s.f.) 2
216° (a) Area of sector = 42.6 cm2
Area of sector OACB = 14.012
360°
92 = 42.6
= 370 cm2 (to 3 s.f.) 360°
4. Area of circle = 3850 cm 2
9
= 42.6
r2 = 3850 40
3850 = 60.3° (to 1 d.p.)
r2 = (b) Area of sector = 117.2 cm2
r = 35.00 cm (to 4 s.f.) 92 = 117.2
36° 360°
(a) Area of sector OPSQ = 35.002 9
360° = 117.2
= 385 cm2 (to 3 s.f.) 40
36° = 165.8° (to 1 d.p.)
Length of arc PSQ = 2 35.00 (c) Area of sector = 214.5 cm2
360°
= 22.0 cm (to 3 s.f.) 92 = 214.5
84° 360°
(b) Area of sector OPSQ = 35.002 9
360° = 214.5
= 898 cm2 (to 3 s.f.) 40
84° = 303.5° (to 1 d.p.)
Length of arc PSQ = 2 35.00 (d) Area of sector = 18.9 cm2
360°
= 51.3 cm (to 3 s.f.) 92 = 18.9
108° 360°
(c) Area of sector OPSQ = 35.002 9
360° = 18.9
= 1150 cm2 (to 3 s.f.) 40
108° = 26.7° (to 1 d.p.)
Length arc PSQ = 2 35.00
360°
= 66.0 cm (to 3 s.f.)

270
45° 5
7. (i) Length of arc AB = 2 10 8. (i) Since the shaded area POQ is of the area of the whole
360° 18
circle,
= 7.854 cm (to 4 s.f.)
45° !POQ 5
Length of arc CD = 2 20 r2 = r2
360° 360° 18
= 15.71 cm (to 4 s.f.) !POQ 5
=
AD = BC = 20 – 10 = 10 cm 360° 18
Perimeter of shaded region = 7.854 + 15.71 + 10 + 10 5
!POQ = 360°
= 43.6 cm (to 3 s.f.) 18
45° = 100°
Area of sector OAB = 102 (ii) Area of shaded sector = 385 cm2
360°
= 39.27 cm2 (to 4 s.f.) 100°
r2 = 385
45° 360°
Area of sector ODC = 202 5
360° r2 = 385
= 157.1 cm2 (to 4 s.f.) 18
Area of shaded region = 157.1 – 39.27 1386
r2 =
= 118 cm2 (to 3 s.f.)
r = 21.00 cm (to 4 s.f.)
120°
(ii) Length of arc AB = 2 5 Diameter of circle = 21.00 2
360°
= 10.47 cm (to 4 s.f.) = 42.0 cm (to 3 s.f.)
120° 9. Perimeter = 38 cm
Length of arc CD = 2 8 Arc length + 12 + 12 = 38
360°
= 16.76 cm (to 4 s.f.) Arc length = 38 – 12 – 12
AD = BC = 8 – 5 = 3 cm = 14 cm
Perimeter of shaded region = 10.47 + 16.76 + 3 + 3 2 12 = 14
= 33.2 cm (to 3 s.f.) 360°
120° = 14
Area of sector OAB = 52 15
360°
= 26.18 cm2 (to 4 s.f.) = 66.85° (to 2 d.p.)
120° 66.85°
Area of sector ODC = 82 Area of paper used = 122
360° 360°
= 67.02 cm2 (to 4 s.f.) = 84.0 cm2 (to 3 s.f.)
Area of shaded region = 67.02 – 26.18 10. (i) Draw a line BT such that T lies on AP and BT is perpendicular
= 40.8 cm2 (to 3 s.f.) to AP.
160°
(iii) Length of arc AB = 2 35
360°
= 97.74 cm (to 4 s.f.)
A
160° R
Length of arc CD = 2 49 T B
360° 11 cm 7 cm
= 136.8 cm (to 4 s.f.)
M N
AD = BC = 49 – 35 = 14 cm P Q

Perimeter of shaded region = 97.74 + 136.8 + 14 + 14 AT = AP – TP


= 263 cm (to 3 s.f.) = 11 – 7
160° = 4 cm
Area of sector OAB = 352
360° AB = AR + RB
= 1710 cm2 (to 4 s.f.) = 11 + 7
160° = 18 cm
Area of sector ODC = 492
360° In "ATB,
= 3352 cm2 (to 4 s.f.) AT 4
cos !BAT = =
Area of shaded region = 3352 – 1710 AB 18
= 1640 cm2 (to 3 s.f.) 4
!BAT = cos–1 = 77.2° (to 1 d.p.)
18
!PAB = 77.2°

271
(ii) Using Pythagoras’ Theorem, (ii) !AOQ = 180° – 132° = 48°
BT 2 = 182 – 42 Using sine rule,
BT = 18 2 – 4 2 AQ 16
=
sin 48° sin 66°
= 308
16 sin 48°
= 17.55 cm (to 4 s.f.) AQ =
sin 66°
!ABT = 180° – 90° – 77.16° (! sum of a ") = 13.0 cm (to 3 s.f.)
= 12.84° 132°
!ABQ = 90° + 12.84° = 102.84° (iii) Length of arc QB = 2 16
360°
Area of shaded region = 36.86 cm (to 4 s.f.)
= Area of trapezium ABPQ – area of sector APR 66°
Length of arc PYB = 2 32
– area of sector RBQ 360°
1 77.16° 102.84° = 36.86 cm (to 4 s.f.)
= (11 + 7)(17.55) – 112 – 72
2 360° 360° PQ = 32 – 13.02 = 18.98 cm
= 32.5 cm2 (to 3 s.f.) Perimeter of shaded region = 36.86 + 36.86 + 18.98
11. Draw a line YT such that T lies on XC and YT is perpendicular to = 92.7 cm (to 3 s.f.)
XC. 132°
(iv) Area of sector BOQ = 162
360°
= 294.9 cm2 (to 4 s.f.)
4p cm 66°
X Area of sector APYB = 322
p cm 360°
T Y = 589.8 cm2 (to 4 s.f.)
1
Area of "AOQ = 16 13.02 sin 66°
A C D B 2
XT = XC – TC = 95.15 cm2 (to 4 s.f.)
= 4p – p Area of shaded region = 589.8 – 294.9 – 95.15
= 3p cm = 200 cm2 (to 3 s.f.)
XY = 4p + p 13. (i) A
= 5p cm
In "XTY, B

XT 3p 3
cos !CXY = = = 12 cm
XY 5p 5
–1 3
!CXY = cos = 53.13° (to 2 d.p.)
5
Using Pythagoras’ Theorem, C D O
YT 2 = XY 2 – XT 2
Since B is the midpoint of arc AC,
= (5p)2 – (3p)2
!BOC = 45°
= 25p2 – 9p2
OB = OA = 12 cm
= 16p2
BD
sin 45° =
YT = 16p 2 = 4p cm 12
!XYT = 180° – 90° – 53.13° (! sum of a ") BD = 12 sin 45°
= 36.87° = 8.49 cm (to 3 s.f.)
!XYD = 90° + 36.87° = 126.87° OD
(ii) cos 45° =
Area of enclosed region 12
1 53.13° 126.87° OD = 12 cos 45°
= (4p + p)(4p) – (4p)2 – p2
2 360° 360° = 8.485 cm (to 4 s.f.)
= 10p2 – 7.418p2 – 1.107p2 CD = 12 – 8.485 = 3.515 cm
= 1.47p2 cm2 (to 3 s.f.) 45°
Length of arc CB = 2 12
12. (i) Since OQ = OA = 16 cm, 360°
!OQA = !OAQ = 66° = 9.425 cm (to 4 s.f.)
!BOQ = 2 66° (ext. ! of ") Perimeter of shaded region = 8.485 + 3.515 + 9.425
= 132° = 21.4 cm (to 3 s.f.)

272
45° (b) Since 180° = radians,
(iii) Area of sector OBC = 122
360° 18° = 18°
= 56.55 cm2 (to 4 s.f.) 180°
1 = rad
Area of !BDO = 8.485 8.485
2 10
= 36.00 cm2 (to 4 s.f.) (c) Since 180° = radians,
Area of shaded region = 56.55 – 36.00 75° = 75°
= 20.5 cm2 (to 3 s.f.) 180°
5
= rad
Exercise 10C 12
(d) Since 180° = radians,
1. (a) Since radians = 180°,
225° = 225°
5 5 180° 180°
rad =
6 6 5
= rad
= 150° 4
(b) Since radians = 180°, 4. (a) Press sin 0 . 8 = and the display shows
180°
rad = 0717 356 09,
7 7
i.e. sin 0.8 = 0.717 (to 3 s.f.)
= 25.7° (to 1 d.p.)
(b) Press cos 0 . 5 4 3 = and the display shows
(c) Since radians = 180°,
180° 0.856 162 416,
1 radian =
i.e. cos 0.543 = 0.856 (to 3 s.f.)
180° (c) Press tan 1 . 5 = and the display shows
3.2 radians = 3.2
14.101 419 95,
= 183.3° (to 1 d.p.) i.e. tan 1.5 = 14.1 (to 3 s.f.)
(d) Since radians = 180°, (d) Press sin ( ÷ 8 ) = and the display shows
180°
1 radian = 0.382 683 432,
180° i.e. sin = 0.383 (to 3 s.f.)
2.56 radians = 2.56 8
(e) Press cos 0 . 4 5 = and the display shows
= 146.7° (to 1 d.p.)
2. (a) Since 180° = radians, 0.156 434 465,
i.e. cos 0.45 = 0.156 (to 3 s.f.)
37.4° = 37.4°
180° (f) Press tan ( 2 ÷ 5 ) = and the display
= 0.653 rad (to 3 s.f.)
shows 3.077 683 537,
(b) Since 180° = radians,
2
i.e. tan = 3.08 (to 3 s.f.)
78.9° = 78.9° 5
180° 5. (a) For sin x = 0.74, press sin
–1
0 . 7 4 = to get
= 1.38 rad (to 3 s.f.)
0.833 070 358,
(c) Since 180° = radians,
i.e. when sin x = 0.74,
142° = 142° x = 0.833 rad (to 3 s.f.)
180°
= 2.48 rad (to 3 s.f.) (b) For cos x = 0.17, press cos–1 0 . 1 7 = to get

(d) Since 180° = radians, 1.399 966 658,


i.e. when cos x = 0.17,
308° = 308° x = 1.40 rad (to 3 s.f.)
180°
= 5.38 rad (to 3 s.f.) (c) For tan x = 0.48, press tan–1 0 . 4 8 = to get
3. (a) Since 180° = radians, 0.447 519 975,
15° = 15° i.e. when tan x = 0.48,
180° x = 0.448 rad (to 3 s.f.)
= rad (d) For sin x = 0.147, press sin–1 0 . 1 4 7 = to
12
get 0.147 534 635,
i.e. when sin x = 0.147,
x = 0.148 rad (to 3 s.f.)

273
(e) For cos x = 0.769, press cos–1 0 . 7 6 9 = to QR
(iii) tan !QSR =
get 0.693 520 986, SR
i.e. when cos x = 0.769, QR
tan 0.85 =
x = 0.694 rad (to 3 s.f.) 7.4
QR = 7.4 tan 0.85
(f) For tan x = 1.256, press tan–1 1 . 2 5 6 = to
= 8.424 m (to 4 s.f.)
get 0.898 390 01,
Using Pythagoras’ Theorem,
i.e. when tan x = 1.256,
PR2 = PS2 – SR2
x = 0.898 rad (to 3 s.f.)
= 13.92 – 7.42
BC
6. (i) tan 0.93 = = 138.45
8.7
BC = 8.7 tan 0.93 PR = 138.45
= 11.7 cm (to 3 s.f.) = 11.77 m (to 4 s.f.)
8.7 PQ = 11.77 – 8.424
(ii) cos 0.93 = = 3.34 m (to 3 s.f.)
AC
8.7 10. Let h m be the height of the tree and d m be the distance between
AC =
cos 0.93 points A and Q.
= 14.6 cm (to 3 s.f.) AT
tan !AQT =
12.7 AQ
7. (i) tan 1.08 =
AB h
tan 1.3 =
12.7 d
AB =
tan 1.08 h
d = — (1)
= 6.79 m (to 3 s.f.) tan 1.3
The height of the tree is 6.79 m. AT
tan !APT =
12.7 AP
(ii) sin 1.08 = h
AC tan 0.72 =
12.7 d + 11.9
AC = h = tan 0.72 (d + 11.9) — (2)
sin 1.08
= 14.4 m (to 3 s.f.) Substitute (1) into (2):
PQ h
8. (i) cos 0.98 = h = tan 0.72 + 11.9
16.8 tan 1.3
PQ = 16.8 cos 0.98 tan 0.72
h= h + 11.9 tan 0.72
= 9.36 cm (to 3 s.f.) tan 1.3
The length of cake to be sprinkled with chocolate rice is tan 0.72
h– h = 11.9 tan 0,72
9.36 cm. tan 1.3
0.7565h = 11.9 tan 0.72
QR
(ii) sin 0.98 = h = 13.8 (to 3 s.f.)
16.8
QR = 16.8 sin 0.98 The height of the tree is 13.8 m.
= 14.0 cm (to 3 s.f.)
The length of cake to be sprinkled with icing sugar is Exercise 10D
14.0 cm. 1. (a) Arc length = 6 1.6 = 9.6 cm
SR 7.4 (b) Arc length = 14 0.25 = 3.5 cm
9. (i) cos !PSR = =
PS 13.9 (c) Arc length = 25 1.75 = 43.75 m
7.4 3
!PSR = cos–1 (d) Arc length = 12 = 9 mm
13.9 4
= 1.009 rad (to 4 s.f.) 1
!PSQ = 1.009 – 0.85 2. (a) Area of sector = 82 2.2 = 70.4 cm2
2
= 0.159 rad (to 3 s.f.) 1
(b) Area of sector = 172 0.46 = 66.47 cm2
SR 2
(ii) cos !QSR =
SQ 1 1
(c) Area of sector = 332 = 108.9 m2
7.4 2 5
cos 0.85 =
SQ
7.4
SQ =
cos 0.85
= 11.2 m (to 3 s.f.)

274
1 (f) Arc length = 6 mm
(d) Area of sector = 942 0.6 = 26508 mm2
2 r =6
3. (i) Perimeter = 50 cm, 6
r = — (1)
r + r + r = 50
16 + 16 + 16 = 50 Area of sector = 27 mm2
16 + 32 = 50 1 2
r = 27 — (2)
16 = 50 – 32 2
16 = 18 Substitute (1) into (2):
2
1 1 6
=1 = 27
8 2
1 1 36
!AOB = 1 rad 2
= 27
8 2
1 1 18
(ii) Area of sector OAB = 162 1 = 27
2 8
= 144 cm2 27 = 18
4. (a) Arc length = 4 1.25 = 5 cm 2
=
1 3
Area of sector = 42 1.25 2
2 Angle at centre = rad
= 10 cm2 3
(b) Arc length = 9 cm 6
r= =9
6 =9 2
3
= 1.5
Radius = 9 mm
Angle at centre = 1.5 rad
5. (i) Arc length = 14 3.8 = 53.2 cm
1
Area of sector = 62 1.5 Perimeter of major sector OAB = 14 + 14 + 53.2
2
= 27 cm2 = 81.2 cm
(c) Arc length = 9.6 m 1
(ii) Area of minor sector OAB = 142 (2 – 3.8)
r 0.8 = 9.6 2
= 243 cm2
r = 12 2
6. Area of sector = 60 cm
Radius = 12 m
1
1 102 = 60
Area of sector = 122 0.8 2
2
50 = 60
= 57.6 m2
= 1.2
(d) Area of sector = 60 m2
Perimeter of sector = 10 1.2 + 10 + 10
1
r2 1.2 = 60 = 32 cm
2
0.6r2 = 60 7. Area of sector = 729 cm2
r2 = 100 1
182 = 729
r = 10 2
162 = 729
Radius = 10 m
= 4.5
Arc length = 10 1.2
Perimeter of sector = 18 4.5 + 18 + 18
= 12 m
= 117 cm
(e) Area of sector = 64 mm2
1
1 8. (i) Area of minor sector AOB = 52 1.8
82 = 64 2
2
= 22.5 cm2
32 = 64
(ii) !AOB = (2 – 1.8) rad
=2
(iii) Length of major arc ACB = 5 (2 – 1.8)
Angle at centre = 2 rad
= (10 – 9) cm
Arc length = 8 2
= 16 mm

275
9. (i) Perimeter of sector = 18 cm 12. (i) Given that !AOB is an equilateral triangle,
r + r + r = 18
"AOB = "OBA = "OAB =
r + 2r = 18 — (1) 3
Area of sector = 8 cm2 =
1 2 3
r =8 — (2)
2 (ii) Length of arc APB = 12 = 12.6 cm (to 3 s.f.)
(ii) From (2): 3
1 2 (iii) Area of sector segment APB
r =8 = Area of sector APBO – Area of !ABO
2
r2 = 16 1 1
= 122 – 12 12 sin
16 2 3 2 3
= 2 — (3) = 13.04 cm2 (to 4 s.f.)
r
Substitute (3) into (1): Area of shaded region
16 = Area of semicircle ABQ – Area of segment APB
r 2 + 2r = 18
r 1
= 62 – 13.04
16 2
+ 2r = 18
r = 43.5 cm2 (to 3 s.f.)
16 + 2r2 = 18r 13. (i) Length of arc AB = 15 1.2 = 18 cm
2
2r – 18r + 16 = 0 (ii) 2OK = 3BK
r2 – 9r + 8 = 0 OK 3
=
(r – 1)(r – 8) = 0 BK 2
r =1 or r =8 3 3
OK = OB = 15 = 9 cm
16 16 5 5
= 2 or = 2 Area of shaded region = Area of sector OAB – Area of !AKO
1 8
1 1 1
= 16 (rejected) = = 152 1.2 – 15 9 sin 1.2
4 2 2
1 = 72.1 cm2 (to 3 s.f.)
r = 8, =
4 14. (i) Length of arc AB = 8 0.8 = 6.4 cm
AC OP 3
10. (i) sin 1.05 = (ii) =
18 PA 2
AC = 18 sin 1.05 3 3
OP = OA = 8 = 4.8 cm
= 15.6 cm (to 3 s.f.) 5 5
1 OQ 3
(ii) Area of sector AOB = 182 1.05 =
2 QB 4
= 170.1 cm2 3 3
OQ = OB = 8 = 3.429 cm (to 4 s.f.)
1 7 7
Area of !AOC = 18 15.61 sin – – 1.05 Area of shaded region
2 2
= 69.90 cm2 (to 4 s.f.) = Area of sector OAB – Area of !OPQ
Area of shaded region = 170.1 – 69.90 1 1
= 82 0.8 – 4.8 3.429 sin 0.8
= 100 cm2 (to 3 s.f.) 2 2
11. Arc length = 31 cm = 19.7 cm2 (to 3 s.f.)
25 = 31 15. (i) Using cosine rule,
= 1.24 82 + 82 – 72
cos "AOB =
1 2 8 8
Area of sector AOB = 252 1.24 = 0.6172 (to 4 s.f.)
2
= 387.5 cm2 "AOB = cos–1 0.6172
OC = 0.906 rad (to 3 s.f.)
cos 1.24 = 2
25 Max. no. of slices that can be obtained = 6
OC = 25 cos 1.24 0.906
= 8.120 cm (to 4 s.f.) (ii) Area of shaded segment
1 = Area of sector OAB – Area of !OPQ
Area of !AOC = 8.120 25 sin 1.24 1 1
2 = 82 0.9056 – 8 8 sin 0.9056
= 96.00 cm2 (to 4 s.f.) 2 2
Area of shaded region = 387.5 – 96.00 = 3.80 cm2 (to 3 s.f.)
= 292 cm2 (to 3 s.f.)

276
1 (ii) Area of shaded segment
16. (i) Area of !ABC = 40 80 sin
2 6 = Area of sector POQ – Area of !POQ
= 800 cm2 1 1
= 62 2.094 – 6 6 sin 2.094
(ii) Area of shaded region = Area of !ABC – Area of sector BXY 2 2
2
1 = 22.1 cm (to 3 s.f.)
= 800 – 102
2 6 1
20. Surface area of cone = 812 2
= 774 cm2 (to 3 s.f.) 2
17. (i) Major arc length = 24 m = 6561 cm2
6 = 24 r 81 = 6561 cm2
=4 r = 25.78 cm (to 4 s.f.)
"AOB = 4 rad Using Pythagoras’ Theorem,
(ii) Using cosine rule, h2 = 812 – 25.782
AB2 = 62 + 62 – 2 6 6 cos 4 = 5.896 (to 4 s.f.)
= 119.1 (to 4 s.f.) h = 76.8 cm (to 3 s.f.)
AB = 10.9 m (to 3 s.f.) 21. (i) Draw a line QC such that C lies on AP and CQ is perpendicular
(iii) Area of major segment = Area of sector APB + Area of !AOB to AP.
1 1
= 62 4 + 6 6 sin (2 – 4)
2 2
2
= 85.6 cm (to 3 s.f.)
P
18. (i) R
C Q
5 cm 3 cm

O T X
A B
15 cm
22 cm B
A PC = PA – CA
X
h cm =5–3
= 2 cm
PQ = PR + RQ
Using Pythagoras’ Theorem,
=5+3
OX 2 = 152 – 112
= 8 cm
= 104
In !PCQ,
OX = 104
PC 2
= 10.20 cm (to 4 s.f.) cos "CPQ = =
PQ 8
h = 15 – 10.20 –1 2
= 4.80 cm (to 3 s.f.) "CPQ = cos = 1.32 rad (to 3 s.f.)
8
(ii) Using cosine rule, "APQ = 1.32 rad
15 2 + 15 2 – 22 2 = 1.32
cos "AOB =
2 15 15 (ii) Using Pythagoras’ Theorem,
= –0.07556 (to 4 s.f.) CQ2 = 82 – 22
"AOB = cos–1 (–0.07556)
CQ = 8 2 – 2 2
= 1.656 rad (to 4 s.f.)
Area of cross section = 60
= Area of sector AOB – Area of !AOB = 7.746 cm (to 4 s.f.)
1 1 "PQC = – – 1.318 (" sum of a !)
= 152 1.646 – 15 15 sin 1.646 2
2 2
= 73.0 cm2 (to 3 s.f.) = 0.2527 rad (to 4 s.f.)
3 "PQB = + 0.2527 = 1.823 rad (to 4 s.f.)
19. (i) cos = 2
6
3 Area of shaded region
= cos–1 = Area of trapezium ABPQ – area of sector APR
6
= 1.047 (to 4 s.f.) – area of sector RBQ
Obtuse "POQ = 1.047 2 1 1 1
= (3 + 5)(7.746) – 52 1.318 – 32 1.823
= 2.09 rad (to 3 s.f.) 2 2 2
= 6.30 cm2 (to 3 s.f.)

277
OP 8 1
22. (i) tan = = 25. (i) Length of arc PQR = 2 15
2 PT 14 4
8 = 7.5 cm
= tan–1 = 0.5191 (to 4 s.f.)
2 14 PQ 4
=
= 0.5191 2 QR 3
= 1.04 (to 3 s.f.) 3
QR = 7.5
7
(ii) !POT = – – 0.5191 (!sum of a ") 45
2 = cm
= 1.052 rad (to 4 s.f.) 14
!POQ = 1.052 2 45
15 =
= 2.103 rad (to 4 s.f.) 14
3
1 =
Area of sector POQ = 82 2.103 14
2
45 2
= 67.31 cm2 (to 4 s.f.) (ii) !POQ = – = rad
2 14 7
1
Area of "POT = 14 8 Area of shaded region
2
= Area of quadrant – Area of sector OQR – Area of "OPQ
= 56 cm2
1 1 3 1 2
Area of shaded region = 2 56 – 67.31 = 152 – 152 – 15 15 sin
4 2 14 2 7
= 44.7 cm2 (to 3 s.f.)
= 164 cm2 (to 3 s.f.)
AB 10
23. (i) cos !HAB = =
AH 12.5
10
Review Exercise 10
!HAB = cos–1 = 0.6435 rad (to 4 s.f.)
12.5 360° – 120°
1. (i) Length of arc BPA = 2 12
!HAK = – 0.6435 – 0.6435 360°
2 = 5.03 cm (to 3 s.f.)
= 0.284 rad (to 3 s.f.) 240°
(ii) Area of sector OBPA = 122
(ii) Area of "ABH = Area of "ADK 360°
1 = 302 cm2 (to 3 s.f.)
= 10 12.5 sin 0.6435
2 1
2. (i) Area of minor sector POQ = 82 1.9
= 37.50 cm2 (to 4 s.f.) 2
1 = 60.8 cm2
Area of sector AKH = 12.52 0.2838
2 (ii) !POQ = (2 – 1.9) rad
= 22.17 cm2 (to 4 s.f.) (iii) Length of major arc PRQ = 8 (2 – 1.9)
Area of shaded region = 10 10 – 22.07 – 2 37.50 = (16 – 15.2) cm
= 2.83 cm2 (to 3 s.f.) 3. (i)
24. (i) Length of arc PSQ = 12 1.2 = 14.4 cm
1 1 Q
(ii) Area of segment PSQ = 122 1.2 – 12 12 sin 1.2
2 2 P S
2 O
= 19.3 cm (to 3 s.f.)
R
(iii) Using cosine rule,
PQ2 = 122 + 122 – 2 12 12 cos 1.2
= 183.6 (to 4 s.f.) In "POQ,
PQ = 13.6 cm (to 3 s.f.)
– 1.2 !SPQ = rad
(iv) !OPQ = (! sum of an isos. ") 2
2
= 0.9708 rad (to 4 s.f.) !PQO = rad (isos. ")
2
!QPR = – 0.9708 (! sum of an isos. ") Using cosine rule,
2 82 + 82 – 72
= 0.6 rad cos !QOR =
2 8 8
1 = 0.6172 (to 4 s.f.)
Area of "PQR = 13.55 14.4 sin 0.6
2 !QOR = cos–1 0.6172
= 55.09 cm2 (to 4 s.f.) = 0.9056 rad (to 4 s.f.)
Area of shaded region = 55.09 – 19.29
= 35.8 cm2 (to 3 s.f.)

278
!QOR = 2 – 0.9056 (iii) OC = 30 – 12 = 18 cm
= 5.378 rad (to 4 s.f.) 1
Area of "BOC = 12 18 sin 1.15
5.378 2
!POQ = = 2.689 rad (to 4 s.f.)
2 = 98.6 cm2 (to 3 s.f.)
2 – 2.689 (iv) Using cosine rule,
!SPQ = = 1.797 rad (to 4 s.f.)
2 BC2 = 122 + 182 – 2 12 18 cos 1.15
!QPR = 0.2265 2 = 0.453 rad (to 3 s.f.) = 291.5 (to 4 s.f.)
= 0.453 BC = 17.1 cm (to 3 s.f.)
(ii) Area of minor segment 8. (i) Using sine rule,
= Area of sector OQR – Area of "OQR OC 12
=
1 1 sin 0.6 sin 1.2
= 82 0.9056 – 8 8 sin 0.9056
2 2 12 sin 0.6
OC = = 7.270 (to 4 s.f.)
= 3.80 cm2 (to 3 s.f.) sin 1.2
4. (i) Using cosine rule, BC = 12 – 7.270 = 4.73 cm (to 3 s.f.)
10 2 + 10 2 – 14 2 !AOC = – 1.2 – 0.6 = ( – 1.8) rad
cos !AOB =
2 10 10 Using sine rule,
= 0.02 AC 12
=
!AOB = cos–1 0.02 sin ( – 1.8) sin 1.2
= 1.55 rad (to 3 s.f.) 12 sin ( – 1.8)
AC = = 12.5 cm (to 3 s.f.)
(ii) Cross-sectional area of cake sliced off sin 1.2
= Area of sector AOB – Area of "AOB (ii) Area of shaded region
1 1 = Area of sector AOB – Area of "AOC
= 102 1.551 – 10 10 sin 1.551
2 2 1 1
= 122 ( – 1.8) – 12 12.54 sin 0.6
= 27.6 cm2 (to 3 s.f.) 2 2
2
5. Let the radius of the circle be r cm. = 54.1 cm (to 3 s.f.)
Area of circle = r2 cm2 9. Q R
Area of segment = Area of sector POQ – Area of "POQ
1 1
= r2 2.2 – r r sin 2.2
2 2
= 0.6958r2 cm2 (to 4 s.f.) P S
O
0.6958r 2 2r cm
Percentage = 100%
r2
Since lengths of arcs PQ = QR = RS,
= 22% (to the nearest integer)
6. (i) !AOB = (2 – 5.8) rad !POQ = !QOR = !ROS = rad
3
1
Area of "OAB = 152 sin (2 – 5.8) Using cosine rule,
2
= 52.3 cm2 (to 3 s.f.) QR2 = r2 + r2 – 2 r r cos
3
(ii) Area of unshaded region 2 2
= 2r – r
= Area of sector AOB – Area of "AOB
= r2
1
= 152 (2 – 5.8) – 52.3 QR = r cm
2
= 2.06 cm2 (to 3 s.f.) RS2 = r2 + r2 – 2 r r cos
3
2.06 2 2
P(dart lands on unshaded area) = = 2r – r
15 2
= r2
= 2.91 10–3 (to 3 s.f.)
RS = r cm
7. (i) !AOB = ( – 1.15) rad (adj. !s on a str. line)
!QRS = !QRO + !ORS
Length of arc AB = 12 ( – 1.15)
= 23.9 cm (o 3 s.f.) = +
3 3
(ii) Area of shaded segment
2
= Area of sector AOB – Area of "AOB = rad
3
1 1
= 122 ( – 1.15) – 12 12 sin ( – 1.15) 1 2
2 2 Area of shaded region = r r sin
2
2 3
= 77.7 cm (to 3 s.f.)
= 0.433r2 cm2 (to 3 s.f.)

279
10. G Challenge Yourself
45° 45°
1. (a) Perimeter = arc AB + AB
H F
14
= + 14
2
= (7 + 14) cm
14 14
A
(b) Perimeter = + + 14
B C D E 2 2
20 cm
= (7 + 14) cm
!HGF = 45° + 45° = 90° 14 14 14
(c) Perimeter = + + + 14
!GEC = 180° – 90° – 45° (! sum of a ") 6 6 6
= (7 + 14) cm
= 45°
14 14 14 14
Radius of quadrant = 10 cm (d) Perimeter = + + + + 14
8 8 8 8
EF = 10 cm
= (7 + 14) cm
DE
cos !DEF = All the perimeters are equal.
EF
In general, it does not matter how many identical semicircles are on
DE
cos 45° = the line AB, all the perimeters are equal.
10
DE = (10 cos 45°) cm 2. (i) By Pythagoras’ Theorem,
1 AP2 = PB2 + AB2
Area of "DEF = 10 10 cos 45° sin 45° (12 + r)2 = (12 – r)2 + 122
2
= 25 cm2 144 + 24r + r2 = r2 – 24r + 144 + 144
1 144 + 24r + r2 = r2 – 24r + 288
Total area of shaded region = 2 10 2 – 25
4 24r = –24r + 144
= 2[25 – 25] 48r = 144
= 50( – 1) cm2 r =3
11. (a) (i) Using cosine rule, (ii) AP = 12 + 3 = 15 cm
PQ2 = 152 + 152 – 2 15 15 cos 1.25 PB = 12 – 3 = 9 cm
= 308.1 (to 4 s.f.) Using cosine rule,
PQ = 17.6 cm (to 3 s.f.) 15 2 + 12 2 – 9 2
cos !PAB =
1 2 15 12
(ii) Area of minor sector APQ = 152 1.25
2 = 0.8
= 141 cm2 (to 3 s.f.) !PAB = cos–1 0.8
1 = 0.644 rad (to 3 s.f.)
(iii) Area of "PAQ = 15 15 sin 1.25
2 !PAC = 0.644 rad
= 107 cm2 (to 3 s.f.) (iii) !PCA = !PAC = 0.6435 rad
(b) Using cosine rule, !APC = – 0.6435 – 0.6435 (! sum of a ")
12 2 + 12 2 – 17.55 2 = 1.855 rad
cos !PBQ =
2 12 12 1
= – 0.06979 (to 4 s.f.) Area of minor sector RPS = 32 1.855
2
!PBQ = cos–1 (– 0.06979) = 8.346 cm2 (to 4 s.f.)
= 1.641 rad (to 4 s.f.) 1
Area of minor sector ARB = 122 0.6435
1 2
Area of minor sector BPQ = 122 1.641
2 = 46.33 cm2 (to 4 s.f.)
= 118.1 cm2 (to 4 s.f.) Area of minor sector BCS = Area of minor sector ARB
1 = 46.33 cm2
Area of "PBQ = 12 12 sin 1.641
2 1
= 71.82 cm2 (to 4 s.f.) Area of "APC = 15 15 sin 1.855
2
Area of shaded region = (140.6 – 106.8) + (118.1 – 71.82) = 108.0 cm2 (to 4 s.f.)
= 80.0 cm2 (to 3 s.f.) Area of shaded region = 108.0 – 46.33 – 46.33 – 8.346
= 6.99 cm2 (to 3 s.f.)

280
3. (i) O (i) OT = (48 – r) cm
PT = r cm
"TOP = 30°
10
r
sin 30° =
48 – r
48 – r = 2r
P T Q
3r = 48
5
r = 16
R S
(ii) "OTP = 90° – 30° = 60°
PC = 2 5 = 10 cm 1
Area of !OTP = 16 (48 – 16) sin 60°
!OPQ is an equilateral triangle. 2
3
"POQ = rad = 256
3 2
2 = 128 3 cm2
"RPT = – = rad
3 3 60°
2 10 Area of sector STP = 162
Length of arc RT = 5 = cm 360°
3 3 128
= cm2
Length of arc RS = 15 = 5 cm 3
3 128
10 Ara of shaded region = 2 128 3 –
Perimeter of shaded region = 2 +5 3
3 = 175 cm2 (to 3 s.f.)
35
= cm
3
1
(ii) Area of sector ORS = 152
2 3
1
= 37 cm2
2
1
Area of !OPQ = 10 10 sin
2 3
2
= 25 3 cm
1 2
Area of sector RPT = 52
2 3
1 2
=8 cm
3
1 1
Area of shaded region = 37 – 25 3 – 2 8
2 3
= 22.1 cm2 (to 3 s.f.)
4. A

O S T
R
r

281
Revision Exercise C1 9
sin a° =
1. Since sin x is positive, x can either be an acute angle or an obtuse 15.592
9
angle. a = sin–1
15.592
sin x = 0.345
= 35.3 (to 3 s.f.)
x = sin–1 0.345 = 0.352 (to 3 s.f.)
CD
or – 0.3522 = 2.79 (to 3 s.f.) 5. (i) sin 40° =
8
x = 0.352 or 2.79
CD = 8 sin 40°
2. (i) Since the length of arc is 15.2 cm,
= 5.14 cm (to 3 s.f.)
8 = 15.2
(ii) !PQD = 180° – 20° – 40° = 120°
= 1.9
Using cosine rule,
!AOB = 1.9 rad
PD2 = 42 + 82 – 2 4 8 cos 120°
1
(ii) Area of sector = 82 1.9 = 112
2
= 60.8 cm2 PD = 112 = 10.58 cm (to 4 s.f.)
AP In "PBQ,
3. (i) cos 0.6 =
16 BP
cos 70° =
AP = 16 cos 0.6 4
= 13.2 cm (to 3 s.f.) BP = 4 cos 70°
1 = 1.368 cm (to 4 s.f.)
(ii) Area of "PAB = 13.21 16 sin 0.6
2 AP = 5.142 – 1.368 = 3.774 cm
= 59.7 cm2 (to 3 s.f.) 3.774
cos !APD =
(iii) !POB = 1.2 rad 10.58
Area of shaded region = Area of sector POB – Area of "POB 3.774
!APD = cos–1
1 1 10.58
= 82 1.2 – 8 8 sin 1.2
2 2 = 69.1° (to 1 d.p.)
2
= 8.57 cm (to 3 s.f.) (iii) By Pythagoras’ Theorem,
4. H AD2 = PD2 – AP2
= 10.582 – 3.7742
9
= 97.69 (to 4 s.f.)
Q a° AD = 97.69
P R
56°
= 9.89 cm (to 3 s.f.)
6. (i) Using cosine rule,
12 62 + 62 – 92
cos !BAD =
2 6 6
32°
= –0.125
!BAD = cos–1 (–0.125)
M
= 97.2° (to 1 d.p.)
In "QRM, 1
QR (ii) Area of "ABD = 6 6 sin 97.18°
tan 32° = 2
12 = 17.9 cm2 (to 3 s.f.)
QR = 12 tan 32° 1
= 7.498 cm (to 4 s.f.) (iii) Area of "BCD = 9 12 sin 60°
2
In "PQM, = 46.77 cm2 (to 4 s.f.)
12 Area of quadrilateral ABCD = 17.86 + 46.77
tan 56° =
PQ = 64.6 cm2 (to 3 s.f.)
12 (iv) Using sine rule,
PQ =
tan 56° sin !ABD sin 97.18°
= 8.094 cm (to 4 s.f.) =
6 9
PR = 7.498 + 8.094 = 15.592 cm
–1 sin 97.18°
!ABD = sin
9
= 41.41° (to 2 d.p.)
!ABC = 60° + 41.41°
= 101.4° (to 1 d.p.)

282
(v) Using cosine rule, (iii) P
DC2 = 92 + 122 – 2 9 12 cos 60°
= 117
10.39 cm
DC = 117
= 10.8 cm (to 3 s.f.)
D B
7. (i) Using sine rule,
By Pythagoras’ Theorem,
sin sin 42°
= BD2 = AB2 + AD2
12 10
12 sin 42° = 62 + 82
sin = = 100
10
= 0.8030 (to 4 s.f.) BD = 10 cm
! = sin–1 0.8030 = 53.42° (to 2 d.p.) 10.39
tan !PDB =
or 180° – 53.42° = 126.58° (to 2 d.p.) 10
!ACB = 53.42° and !ADB = 126.58° 10.39
!PDB = tan–1
10
!BAC = 180° – 53.42° – 42° (! sum of a ")
= 46.1° (to 1 d.p.)
= 84.58°
9. T
The bearing of C from A is (90° – 84.58°).
Bearing of C from A is 005.4°.
(ii) !BAD = 180° – 42° – 126.58° (! sum of a ")
= 11.42° 20 cm

The bearing of D from A is (90° – 11.42°).


54.7° 25.4°
Bearing of D from A is 078.6°. B O A
(iii) !CAD = 84.58° – 11.42° = 73.16°
20
Using cosine rule, tan 54.7° =
OB
CD2 = 102 + 102 – 2 10 10 cos 73.16°
20
= 142.1 (to 4 s.f.) OB =
tan 54.7°
CD = 142.1 = 14.16 m (to 4 s.f.)
= 11.9 km (to 3 s.f.) 20
tan 25.4° =
6 OA
8. (i) cos !PAB = 20
12 OA =
6 tan 25.4°
!PAB = cos–1 = 42.12 m (to 4 s.f.)
12
= 60° AB = 14.16 + 42.12
(ii) By Pythagoras’ Theorem, = 56.3 m (to 3 s.f.)
PB2 = AP2 – AB2
= 122 – 62
= 108
PB = 108
= 10.4 cm (to 3 s.f.)

283
Revision Exercise C2 (ii) By Pythagoras’ Theorem,
AC2 = AB2 + BC2
1. Since cos x is negative, x is an obtuse angle.
= 8.3912 + 102
cos x = –0.5
= 170.4 (to 4 s.f.)
x = cos–1 (–0.5) = 2.09 (to 3 s.f.)
2. (a) sin 140° = sin (180° – 140°) AC = 170.4
= sin 40° = 13.1 cm (to 3 s.f.)
= 0.643 8.391
(iii) tan !AMB =
(b) cos 165° = –cos (180° – 165°) 5
= –cos 15° 8.391
!AMB = tan–1
= –0.966 5
3. = 59.2° (to 1 d.p.)
A B (iv) !BAM = 180° – 90° – 59.21° (! sum of a ")
= 30.79°
!BAC = 180° – 90° – 40° (! sum of a ")
= 50°
F C !CAM = 50° – 30.79° = 19.2° (to 1 d.p.)
6. (i) Using cosine rule,
AC2 = 42 + 52 – 2 4 5 cos 120°
= 61
E D AC = 61
60° 1 = 7.81 cm (to 3 s.f.)
Area of half a petal = 42 – 42 sin 60°
360° 2 (ii) Using sine rule,
= 8.378 – 6.928 sin !DAC sin 120°
= 1.450 cm2 (to 4 s.f.) =
5 7.810
Total shaded area = 12(1.450) 5 sin 120°
!DAC = sin–1
= 17.4 cm2 (to 3 s.f.) 7.810
4. 180° – 50° – 70° = 60° (! sum of a ") = 33.7° (to 1 d.p.)
Using sine rule, (iii) Using sine rule,
x 8 sin !ABC sin 60°
= =
sin 60° sin 70° 7.810 10
8 sin 60° 7.810 sin 60°
x= !ABC = sin–1
sin 70° 10
= 7.37 (to 3 s.f.) = 42.6° (to 1 d.p.)
Using sine rule, (iv) !BAC = 180° – 60° – 42.56° (! sum of a ")
y 8 = 77.44°
=
sin 50° sin 70° !BAD = 33.67° + 77.44°
8 sin 50° = 111.11°
y=
sin 70° Using cosine rule,
= 6.52 (to 3 s.f.) BD2 = 42 + 102 – 2 4 10 cos 111.11°
AB = 144.8 (to 4 s.f.)
5. (i) tan 40° =
10
BD = 144.8
AB = 10 tan 40°
= 8.391 cm (to 4 s.f.) = 12.0 cm (to 3 s.f.)
By Pythagoras’ Theorem, (v) Area of quadrilateral ABCD
AM 2 = AB2 + BM 2 = Area of "ADC + Area of "ABC
= 8.3912 + 52 1 1
= 4 5 sin 120° + 10 78.10 sin 77.44°
= 95.41 (to 4 s.f.) 2 2
= 46.8 cm3 (to 3 s.f.)
AM = 95.41
= 9.77 cm (to 3 s.f.)

284
(5 – 2) 180° 3.562
7. (a) (i) !ABC = = 108° (iii) cos !QVS =
5 10.62
The bearing of C from B is (180° – 108°). 3.562
!QVS = cos–1
Bearing of C from B is 072°. 10.62
(ii) !BAE = 108° = 70.4° (to 1 d.p.)
Bearing of E from A is 108°. 3.562
(iv) tan !VRQ =
(iii) The bearing of D from E is (108° – 72°). 6
Bearing of D from E is 036°. 3.562
!VRQ = tan–1
6
(b) Using cosine rule,
= 30.7° (to 1 d.p.)
AC2 = 8002 + 8002 – 2 800 800 cos 108°
The angle of elevation of V from R is 30.7°.
= 1 676 000 (to 4 s.f.)
Area of ABCDE
1 1
= 800 800 sin 108° + 1 676 000 sin 36°
2 2
= 1.101 106 cm3 (to 4 s.f.)
1
8. (a) (i) MQ = QR = 3 cm
2
By Pythagoras’ Theorem,
PM2 = 62 – 32 = 27
PM = 27 = 5.196 cm (to 4 s.f.)
Since PQ = QR = PR, "PQR is an equilateral triangle.
3
cos 30° =
PX
3
PX = = 3.464 cm (to 4 s.f.)
cos 30°
By Pythagoras’ Theorem,
OX2 = 82 – 3.4642 = 52.00 (to 4 s.f.)
OX = 52.00 = 7.21 cm (to 3 s.f.)
7.211
(ii) sin !OPX =
8
7.211
!OPX = sin–1
8
= 64.3° (to 1 d.p.)
(b) MX = 5.196 – 3.464
= 1.732 cm
7.211
tan !OMX =
1.732
7.211
!OMX = tan–1
1.732
= 76.5° (to 1 d.p.)
VQ
9. (i) tan 24° =
8
VQ = 8 tan 24°
= 3.56 m (to 3 s.f.)
(ii) By Pythagoras’ Theorem,
SQ2 = 62 + 82 = 100
SQ = 100 = 10 m
By Pythagoras’ Theorem,
VS2 = 102 + 3.5622 = 112.7 (to 4 s.f.)
VS = 112.7 = 10.6 m (to 3 s.f.)

285
Chapter 11 Congruence and Similarity Tests
TEACHING NOTES
Suggested Approach
Teachers may want to introduce this topic by asking students to recall what they have learnt in Book 2 on congruent and similar
triangles. Then, teachers may get students to investigate whether all the conditions are necessary to prove whether two triangles
are congruent or similar.

Section 11.1: Congruence Tests

Teachers may wish to recap with the students, that for congruent triangles, all the corresponding lengths and
angles are equal.

For each of the 4 congruent tests that are covered in this section, teachers should ask the students to construct a
triangle in as many ways as possible and see what conclusion they can make by comparing these triangles (see
Investigation: SSS Congruence Test, Investigation: SAS Congruence Test, Investigation: AAS Congruence Test
and RHS Congruence Test).

Teachers should teach students how to match the vertices of two triangles correctly, even if the two triangles are
not congruent. Once students have learnt all the 4 congruent tests, they can learn to identify pairs of congruent
triangles and prove the congruency (see Class Discussion: Consolidation for Congruence Tests).

Teachers should highlight to students that the 4 congruent tests covered in this chapter are not the only congruence
tests.

Section 11.2: Similarity Tests

Teachers may wish to recap with the students, that for similar triangles, all the corresponding lengths and angles
are proportional and equal, respectively.

For each of the 3 similarity tests that are covered in this section, teachers should ask the students to construct a
triangle in as many ways as possible and see what conclusion they can make by comparing these triangles (see
Investigation: AA Similarity Test, Investigation: SSS Similarity Test and Investigation: SAS Similarity Test).

Once students have learnt all the 4 congruent tests and 3 similarity tests, teachers may ask students to compare the
congruent tests with the similarity tests (see Thinking Time on page 366). Teachers should highlight to students
that the 3 similarity tests covered in this chapter are not the only similarity tests.

Section 11.3: Applications of Congruent and Similar Triangles

Now that students have learnt the congruent and similarity tests, they can apply the concepts to solve problems
in mathematics and in real life. For Worked Example 10, teachers should recap with students the properties of
a perpendicular bisector and an angle bisector before going through the problem.

Challenge Yourself

For Question 1, let the height of !PST from P to ST be h units and use similar triangles to solve the problem.
Students need to manipulate algebra properly, otherwise they may end up with a long and tedious working.
For Question 2, students should identify a pair of similar triangles and let QU be x cm and VS be y cm. Then
they can formulate a pair of simultaneous equations involving x and y and solve for x and y to find the length of
QU. For Question 3, students should identify two pairs of similar triangles first.

286
WORKED SOLUTIONS (d) W W
Investigation (SSS Congruence Test) X Z
Y Y
5. From this investigation, we can conclude that if the 3 sides of a WX = WZ (given)
triangle are equal to the 3 corresponding sides of another triangle, WYX = WYZ = 90°
then the two triangles are congruent. WY = WY (common side)
!WXY WZY (SAS)
Investigation (SAS Congruence Test) (e) A C
5. From part 1 of this investigation, if two sides and the included angle B D
of a triangle are given, then only a unique triangle can be constructed. C A
9. From part 2 of this investigation, if two sides and an angle which AB = CD (given)
is not the included angle of a triangle are given, then there is more BAC = DCA (corr. "s, AB // DC)
than one way to construct the triangle. AC = CA (common side)
!ABC !CDA (SAS)
Investigation (AAS Congruence Test) (f) E G
F H
3. From part 1 of this investigation, if two angles and the side of the
G E
triangle between the two angles are given, then only a unique triangle
EF = GH
can be constructed.
EF = HE
6. From part 2 of this investigation, if two angles and the side of the
EG = GE (common side)
triangle that is not between the two angles are given, then only a
!EFG !GHE (SSS)
unique triangle can be constructed.
(g) I K
7.
J L
value of the third angle in the triangle.
K I
IJ = KL (given)
Investigation (RHS Congruence Test)
JK = LI (given)
3. From the investigation, if the hypotenuse and one side of a right- IK = KI (common side)
angled triangle are given, then only a unique triangle can be !IJK !KLI (SSS)
constructed. (h) M O
N P
Class Discussion (Consolidation for Congruence Tests) O M
ON = MP (given)
(a) A A
MON = OMP (corr. "s, ON // PM)
B D
OM = MO (common side)
C C
!MNO !OPM (SAS)
AB = AD (given)
BC = DC (given)
Investigation (AA Similarity Test)
AC = AC (common side)
!ABC ADC (SSS) 2. "ACB = 180° – 50° – 30° = 100°
(b) D D "XZY = 180° – 50° – 30° = 100°
E G Yes, "ACB = "XZY
F F AB BC AC
3. Yes, = =
DEF = DGF (given) XY YZ XZ
DFE = DFG = 90° 4. Yes, the two triangles are similar.
DF = DF (common side) 5. Yes, these given conditions are enough to prove that the two triangles
!DEF !DGF (AAS) are similar.
(c) P P
Q S Thinking Time (Page 360)
R R 1. If two angles of a triangle are given, the third unknown angle is a
QR = SR (given) unique angle of the triangle. As such, the AAA Similarity Test is
PRQ = PRS = 90° not necessary.
PR = PR (common side) 2. Yes, two congruent triangles satisfy the AA Similarity Test.
!PQR !PSR (RHS) Congruence is a special case of similarity.

287
Investigation (SSS Similarity Test) Practise Now 1
DE 2 1 1. A E
3. = =
PQ 4 2 B F
EF 3 1 C D
= =
QR 6 2 AB = EF = 5 m
DF 4 1 BC = FD = 11 m
= =
PR 8 2 AC = ED (given)
DE EF DF
Yes, = = "ABC "ABC (SSS)
PQ QR PR
2. W W
4. !EDF = !QPR
X Z
!DEF = !PQR
Y Y
!DFE = !PRQ
WX = WZ (given)
5. Yes, the two triangles are similar.
XY = ZY (given)
6. Yes, these given conditions are enough to prove that the two triangles
WY = WY
are similar.
"WXY "WZY (SSS)

Thinking Time (Page 364)


Worked Example 2
For both the SSS Congruence Test and the SSS Similarity test, the 3 ratios
P G
of the corresponding sides of two triangles must be equal. However, for
Q H
the SSS Congruence Test, the ratio of the corresponding sides of the two
R F
triangles must be equal to 1.
PQ = GH = 9 mm
QPR = HGF = 40°
Investigation (SAS Similarity Test)
PR = GF = 12 mm
PQ 4.5 "PQR "GHF (SAS)
3. = = 1.5
AB 3
QR 7.5 Practise Now 2
= = 1.5
BC 5
PQ QR 1. P S
Yes, = Q P
AB BC
PR PQ QR R T
4. Yes, = =
AC AB BC PQ = SP (given)
5. !BAC = !QPR and !ACB = !PRQ PQR = SPT (given)
6. Yes, the two triangles are similar. QR = PT (given)
7. Yes, these given conditions are enough to prove that the two triangles "PQR "SPT (SAS)
are similar. 2. A X
B Y
Thinking Time (Page 366) C Z
ABC = XYZ = 36°
1. For both the SAS Congruence Test and the SAS Similarity test, the
BC = YZ = 12 cm
2 ratios of the corresponding sides of two triangles must be equal
However, AB is not equal to XY.
and the pair of included angles must also be equal. However, for
"ABC is not congruent to "XYZ.
the SAS Congruence Test, the ratio of the corresponding sides of
the two triangles must be equal to 1.
Practise Now 3
2. Since the given conditions for the AA Similarity Test is enough,
there is no need for AAS Similarity Test. 1. (i) A C
3. Yes. For RHS Similarity Test, if the ratio of the hypotenuse and one O O
side of a right-angled triangle is equal to the ratio of the hypotenuse B D
and one side of another right-angled triangle, then the two triangles AB = CD (given)
are similar. However, this test is not included in the syllabus. OAB = ODD = 25° (alt. !s)
OA = OC
"AOB "COD (SAS)
(ii) Since "AOB "COD, then all corresponding angles are equal.
BDC = ABO = 25°

288
2. (i) P R P S
Q S Q U
S Q R T
PQ = RS (given) PQR = SUT = 90°
PQS = RSQ (alt. !s) PR = ST = 5 cm
SQ = QS (common side) QR = TU = 3 cm
"PQS "RSQ (SAS) " PQR "SUT (RHS)
(ii) Since "PQS "RSQ, then all the corresponding sides and
angles are equal. Practise Now 5
QR = PS = 7 cm
(a) A E
QPS = SRQ = 140°
B D
C C
Worked Example 4
ACB = ECD = 90°
In "DEF, EFD = 180° – 80° – 30° (! sum of a ") AB = ED (given)
= 70° BC = DC (given)
A F "ABC "EDC (RHS)
B E (b) X Z
C D W Y
ABC = FED = 80° Z X
BAC = EFD = 70° XWZ = ZYX = 90°
BC = ED = 10 mm WZ = YX (given)
"ABC "FED (AAS) XZ = ZX (same side)
" XWZ "ZYX (RHS)
Practise Now 4
Worked Example 6
(a) V Z
W Y ACB = ABC (base !s of isos. ")
X X = 70°
VWX = ZYX (alt. !s) 180° – 40°
YXZ = YZX = (base !s of isos. ")
WXV = YXZ (vert. opp. !s) 2
WX = YX (given) = 70°
"VWX "ZYX (AAS) A Y
(b) A D B X
B C C Z
C B ABC = YXZ = 70°
BAC = CDB = 35° ACB = YZX = 70°
ACB = DBC (given) "ABC is similar to "YXZ (2 pairs of corr. !s equal).
BC = CB (same side)
"ABC "DCB (AAS) Practise Now 6
1. (a) ABC = 180° – 60° – 45° (! sum of a ")
Worked Example 5 = 75°
By Pythagoras’ Theorem, YXZ = 180° – 60° – 75° (! sum of a ")
= 45°
ST = TU 2 + SU 2
A Y
= 32 + 4 2 B Z
C X
= 25
BAC = ZYX = 60°
= 5 cm ACB = YXZ = 45°
"ABC is similar to "YZX (2 pairs of corr. !s equal).

289
(b) DEF = 180° – 90° – 30° (! sum of ") Worked Example 7
= 60°
A T
QPR = 180° – 90° – 50° (! sum of a ")
B U
= 40°
C S
D P
AB 7.5
E Q = = 2.5
TU 3
F R AC 10
Since there are no corresponding pairs of angles that are equal, = = 2.5
TS 4
the two triangles are not similar. BC 15
(c) R V = = 2.5
US 6
S U "ABC is similar to "TUS (3 ratios of corr. !s equal).
T T
STR = UTV (vert. opp. !s) Practise Now 7
RST = VUT (alt. !s)
(a) A Z
"RST is similar to "VUT (2 pairs of corr. !s equal).
B Y
(d) K K
C X
L N
AB 5 2
M P = =
ZY 7.5 3
LKM = NKP ( common angle)
AC 6 2
KLM = KNP (corr. !s, LM // NP) = =
ZX 9 3
"KLM is similar to "KNP (2 pairs of corr. !s equal).
BC 8 2
2. (i) A A = =
YX 12 3
B D "ABC is similar to "ZYX (3 ratios of corr. !s equal).
C E (b) P U
BAC = DAE (common angle) Q S
ABC = ADE (corr. !s, BC // DE) R T
"ABC is similar to "ADE (2 pairs of corr. !s equal). PQ 5 1
(ii) Since "ABC is similar to "ADE, = =3
US 1.5 3
AB BC AC PR 3
= = = =3
AD DE AE UT 1
7 8 QR 6
= = =3
DE 12 ST 2
8DE = 84 Since the 3 ratios of corresponding angles are not equal, the two
DE = 10.5 cm triangles are not similar.
6 8
=
AD 12 Worked Example 8
8AD = 72
AD = 9 cm D D
BD = 9 – 6 = 3 cm E G
AB 6 F H
(iii) = =2 EDF = GDH (common angle)
AD 3
AC 8 8 DE 1
= = =2 =
CE 12 – 8 4 DG 2
AB AC DF 1
= =
BD CE DH 2
DE DF
=
DG DH
"DEF is similar to "DGH (2 ratios of corr. sides and included
! equal).

290
Practise Now 8 2. A X
X B
(a) J N
P Q
K M
AXP = XBQ (corr. !s, XP // BC)
L L
APX = XQB (corr. !s, XQ // AC)
JLK = NLM (vert. opp !s)
"AXP is similar to "XBQ (2 pairs of corr. !s equal).
JL 3 5
= = Since "AXP and "XBQ are similar, then
NL 5.4 9
BQ XB
KL 4 5 =
= = XP AX
ML 7.2 9
BQ 4
JL KL i.e. =
= 15 3
NL ML
4
"JKL is similar to "NML (2 ratios of corr. sides and included BQ = 15
3
! equal).
= 20 cm
(b) A Z
A X
B Y
B B
C X
C Q
ACB = ZXY = 75°
ABC = XBQ (common angle)
AC 15
= = 1.5 ACB = XQB (corr. !s, XP // BC)
ZX 10
"ABC is similar to "XBQ (2 pairs of corr. !s equal)
BC 10
= =2 Since "ABC and "XBQ are similar, then
YX 5
Since the 2 ratios of corresponding sides are not equal, the two AC AB
=
triangles are not similar. XQ XB
AC 7
i.e. =
16 4
Practise Now 9
7
AC = 16
1. (i) A E 4
B C = 28 cm
C G
AC = EG (given) Practise Now 10
ACB = EGC (corr. !s, BC // EG)
P P
BC = CG (given)
A B
"ABC "ECG (SAS)
Q Q
(ii) A F
AP = BP
C F
AQ = BQ
D D
PQ = PQ (common side)
ADC = EDF (vert. opp. !s)
"PAQ and "PBQ are congruent (SSS Congruence Test).
Since "ABC and "ECG are congruent,
APQ = BPQ
then BAC = CEG,
i.e. APR = BPR
i.e. DAC = DEF.
Since AP = BP and PR is a common side, "PAR and "PBR are congruent
"ACD is similar to "EFD (2 pairs of corr. !s equal).
(SAS Congruence Test).
(iii) Since AC = EF = 15 cm, then EF = 15 – 9 = 6 cm
AR = RB
Since "ACD and "EFD are similar, then
ARP = BRP
DF EF
= 180°
DC AC = (adj. !s on a str. line)
2
DF 6
i.e. = = 90°
5 15
PQ is the perpendicular bisector of AB.
6
DF = 5
15
= 2 cm

291
Practise Now 11 (b) D Z
E Y
1. A A
F X
B D
DE = ZY = 3 m
C E
DEF = ZYX = 70°
BC AC
= EF = YX = 5 m
DE AE
!DEF !ZYX (SAS)
BC 10
i.e. = (c) L W
11.2 2
10 M V
BC = 11.2 N U
2
= 56 m LMN = WVU = 30°
2. C A LNM = WUV = 70°
D B MN = VU = 7 cm
E E !NML !UVW (AAS)
AB BE (d) G U
=
CD DE H T
AB 18 I S
i.e. =
1.4 2.1 GHI = UTS = 90°
18 GI = US = 13 mm
AB = 1.4
2.1 HI = TS = 5 mm
= 12 m !IHG !STU (RHS)
3. (a) A E
Exercise 11A B D
1. (a) Comparing triangle (ii) and triangle (vii), C F
The 3 sides of triangle (ii) are equal to the 3 corresponding sides The 3 sides of !ABC are not equal to the 3 corresponding sides
of triangle (vii). of !EDF.
The two triangles are congruent (SSS). !ABC is not congruent to !EDF.
(b) Comparing triangle (iii) and triangle (v), (b) X Q
The 2 sides and the included angle of triangle (iii) are equal to Y R
the 2 corresponding sides and the corresponding included angle Z P
of triangle (v). XZP = QPR = 40°
The two triangles are congruent (SAS). YZ = RP = 6 mm
(c) Comparing triangle (i) and triangle (ix), XZ is not equal to QP.
The 2 angles and 1 side of triangle (i) are equal to the 2 !XYZ is not congruent to !QRP.
corresponding angles and the corresponding side of triangle (ix). (c) G U
The two triangles are congruent (AAS). H T
(d) Comparing triangle (vi) and triangle (viii), I S
The hypotenuse and 1 side of triangle (vi) are equal to the HGI = 180° – 75° – 40° (" sum of a !)
hypotenuse and 1 side of triangle (viii). = 65°
The two triangles are congruent (RHS). TSU = 180° – 55° – 40° (" sum of a !)
= 85°
2. (a) A P HI = ST = 5 cm
B Q GHI = UTS = 40°
B R GIH is not equal to UST and HGI is not equal to TUS.
AB = PQ (given) !GHI is not congruent to !UTS.
BC = QR = 8 cm
AC = PR = 6 cm
!ABC !PQR (SSS)

292
(d) M P (g) A C
N Q B B
O R D D
By Pythagoras’ Theorem, AD = CD (given)
PQ = 12 2 – 5 2 AB = CB (given)
= 10.91 cm (to 4 s.f.) BD = BD (common side)
MNO = PQR = 90° !ABD !CBD (SSS)
NO = QR = 5 cm (h) A C
MN is not equal to PQ and OM is not equal to RP. B D
!MNO is not congruent to !PQR. C A
4. (a) A C BC = DA (given)
B B AC = CA (common side)
D D ABC = CDA = 90°
AB = CB (given) !ABC !CDA (RHS)
AD = CD (given) 5. (i) R V
BD = BD (common side) S U
!ABD !CBD (SSS) T T
(b) A C RT = VT (given)
B D ST = UT (given)
D B RTS = VTU (vert. opp. ")
AB = CD (given) !RST !VUT (SAS)
AD = CB (given) (ii) Since !RST !VUT,
BD = DB (common side) UV = SR = 4 cm
!ABD !CDB (SSS) (iii) Since !RST !VUT,
(c) A E UVT = SRT = 80°
B D (iv) RS is parallel to UV.
C C 6. (i) J G
AC = EC (given) I H
CB = CD (given) H I
ACB = ECD (vert. opp. "s) JI = GH (given)
!ABC !EDC (SAS) JH = GI (given)
(d) A C IH = HI (common side)
B D !JIH !GHI (SSS)
C A (ii) Since !JIH !GHI,
BC = DA (given) IGH = HJI = 60°
AC = CA (common side) GHI = 180° – 60° – 40° (" sum of a !)
BCA = DCA (alt. "s) = 80°
!ABC !CDA (SAS) 7. (a) A C
(e) A C B D
D D C A
E B BC = DA (given)
AE = CB (given) AC = CA (common side)
AED = CBD (given) BCA = DAC (alt. "s)
EAD = BCD (given) !ABC !CDA (SAS)
!ADE !CDB (AAS) (b) E G
(f) B E F H
C F G E
D D GF = EH (given)
BC = EF (given) EG = GE (common side)
CBD = FED (given) EFG = GHE = 90°
BDC = EDF (vert. opp. "s) !EFG !GHE (RHS)
!BCD !EFD (AAS)

293
(c) I K (b) Comparing triangle (ii) and triangle (vi),
J L 12
=2
K I 6
IJ = KL (given) 8
=2
JK = LI (given) 4
IK = KI (common side) 7.8
=2
!IJK !KLE (SSS) 3.9
The 3 ratios of the corresponding sides of triangle (ii) and triangle
(d) M O
(vi) are equal.
N P
The two triangles are similar (3 ratios of corr. sides equal).
O M
(c) Comparing triangle (iv) and triangle (viii),
MO = OM (common side)
24
MNO = OPM = 90° =4
6
MON = OMP (alt. "s)
18
!MNO !OPM (AAS) =4
4.5
(e) Q S
The ratios of the corresponding sides of triangle (iv) and triangle
R T
(viii) are equal and the pair of included angles are also equal.
S Q
The two triangles are similar (2 ratios of corr. sides and
QS = SQ (common side)
included " equal).
RQS = TSQ (alt. "s)
2. (a) STU = 180° – 70° – 50° ("sum of a !)
QSR = SQT (alt. "s)
= 60°
!QRS !STQ (AAS)
A S
(f) U Q
B T
V X
C U
W U
BAC = TSU = 70°
VW = XU (given)
ABC = STU = 60°
UV = QX (given)
!ABC is similar to !STU (2 pairs of corr. "s equal)
UVW = QXU = 90°
(b) X N
!UVW !QXU (RHS)
Y M
8. A C
Z L
B D
XY 24
C A = =3
NM 8
AB = CD
XZ 21
BC = DA = =3
NL 7
AC = CA (common side) YZ 15
!ABC !CDA (SSS) = =3
ML 5
BAC = DCA (alt. "s) !XYZ is similar to !NML (3 ratios of corr. sides equal)
ACB = CAD (alt. "s) (c) D G
AC = CA (common side) E I
!ABC !CDA (AAS) F H
BC = DA DEF = GIH = 90°
AC = CA (common side) DE 6 2
BAC = DCA (alt. "s) = =
GI 9 3
!ABC !CDA (SAS) EF 4 2
= =
IH 6 3
Exercise 11B DE EF
=
1. (a) Comparing triangle (i) and triangle (iii), GI IH
The 2 angles of triangle (i) are equal to the 2 corresponding !DEF is similar to !GIH (2 ratios of corr. sides and included
angles of triangle (iii). " equal).
The two triangles are similar (2 pairs of corr. "s equal).
Comparing triangle (v) and triangle (vii),
The 2 angles of triangle (v) are equal to the 2 corresponding
angles of triangle (vii).
The two triangles are similar (2 pairs of corr. "s equal).

294
3. (a) In the smaller triangle, (d) U U
180° – 60° – 60° = 60° V X
In the larger triangle, W Y
180° – 50° VUW = XUY (common angle)
= 65°
2 UV 8 8 2
= = =
Two angles of the smaller triangle are not equal to the two UX 8+4 12 3
corresponding angles of the larger triangle. UW 10 10 2
= = =
The two triangles are not similar. UY 10 + 5 15 3
22.5 "UVW is similar to "UXY (2 ratios of corr. sides and included
(b) = 0.5
45 ! equal).
15 5. (a) A A
= 0.5
30 B D
8 C E
= 0.5333 (to 4 s.f.)
15 ACB = AED (corr. !s, BC // DE)
The 3 ratios of corresponding sides of both triangles are not BAC = DAC (common angle)
equal. "ABC is similar to "ADE (2 pairs of corr. !s equal).
The two triangles are not similar. Since "ABC is similar to "ADE, then
(c) Included angle = 110° BC AC
10 =
= 0.4 DE AE
25 x 12
7 i.e. =
= 0.4667 (to 4 s.f.) 12 12 + 4
15 12
The ratios of the corresponding sides of both triangles are not x = 12
16
equal. =9
The two triangles are not similar. AB AC
=
4. (a) A E AD AE
B D y 12
i.e. =
C C y+6 16
ABC = EDC (alt. !s) y 3
=
ACB = ECD (vert. opp. !s) y+6 4
"ABC is similar to "EDC (2 pairs of corr. !s equal). 4y = 3(y + 6)
(b) I I 4y = 3y + 8
J F y = 18
H G (b) A E
IJH = IFG (corr. !s, JH // FG) B D
JIH = FIG (common angle) C C
"IJH is similar to "IFG (2 pairs of corr. !s equal). BAC = DEC (corr. !s, AB // DE)
(c) P T ACB = ECD (common angle)
Q S "ABC is similar to "EDC (2 pairs of corr. !s equal).
R R Since "ABC is similar to "EDC, then
PRQ = TRS (vert. opp. !s) AC BC
=
PR 3 EC DC
= = 0.5 x 6
TR 6 i.e. =
QR 4 4 5
= = 0.5 6
SR 8 x = 4
"PQR is similar to "TSR (2 ratios of corr. sides and included 5
= 4.8
! equal).
DE DC
=
BA BC
y 5
i.e. =
9 6
5
y = 9
6
= 7.5

295
(c) A A y 8
i.e. =
B E y+3 8 + 3.2
C D y 8
=
ABC = AED (given) y+3 11.2
BAC = EAD (common angle) 11.2y = (8y + 3)
!ABC is similar to !AED (2 pairs of corr. "s equal). 11.2y = 8y + 24
Since !ABC is similar to !AED, then 3.2y = 24
DE AE y = 7.5
= 6. (i) A D
CB AB
x 6+3 B C
i.e. =
4 3 D B
9 AB 5 1
x = 4 = =
3 DC 15 3
= 12 BD 6 1
= =
AB AC CB 18 3
=
AE AD AD 2 1
= =
3 6 DB 6 3
i.e. =
9 3+ y !ABD is similar to !DCB (3 ratios of corr. sides equal).
3(3 + y) = 54 (ii) Since !ACD is similar to !AFG, then
3 + y = 18 DAB = CDB = 110.5°
y = 15 3
7. XY = 18 cm
(d) A A 5
B E XY = 30 cm
C F 2
UY = 30 = 12 cm
ABC = AEF (corr. "s, BC // EF) 5
BAC = EAF (common angle) !YUV is similar to !YXZ (2 pairs of corr. "s equal).
!ABC is similar to !AEF (2 pairs of corr. "s equal). Since !YUV is similar to !YXZ, then
Since !ABC is similar to !AEF, then XZ XY
=
UV UY
AC AB
= 18 + WZ 30
AF AE i.e. =
18 12
8 10
i.e. = 18 + WZ 5
8+x 10 + 4 =
18 2
8 10
= 18 + WZ = 45
8+x 14
8 5 WZ = 27 cm
= 8.
8+x 7
B
56 = 40 + 5x
5x = 16
x = 3.2
A A O
T
9 cm
C F
D G P I
5 cm
ACD = AFG (corr. "s, CD // FG)
CAD = FAG (common angle) !POI is similar to !OBT.
!ACD is similar to !AFG (2 pairs of corr. "s equal). OI = 9 – 5 = 4 cm
Since !ACD is similar to !AFG, then BT OT
=
AD AC OI PI
= BT 9
AG AF i.e. =
4 5
9
BT = 4
5
= 7.2 cm
Length side of square BLUE = 9 + 7.2
= 16.2 cm

296
9. (i) B D (ii) !QPR is similar to !TUR (2 pairs of corr. "s equal).
A B PQ QR
=
C C TU RT
ABC = BDC = 90° PQ 12
= =2
BC = DBC (common angle) TU 6
!BAC is similar to !DBC (2 pairs of corr. "s equal). The ratio PQ : TU is 2 : 1.
B D 11. (i) Since A coincides with T when the triangle is folded, ANT is a
A A straight line.
C B Hence MN is perpendicular to AT.
ABC = ADB = 90° (ii) A M
BAC = DAB (common angle) R N
!BAC is similar to !DAB (2 pairs of corr. "s equal). T T
Hence !BAC, !DBC and !DAB are similar. ART = MNT = 90°
(ii) By Pythagoras’ Theorem, ATR = MTN (common angle)
!ART is similar to !MNT (2 pairs of corr. "s equal).
AB = 32 + 4 2
(iii) By Pythagoras’ Theorem,
= 25
AT = 6 2 + 8 2
=5m
Since !BAC, !DBC and !DAB are similar, then = 100
BC AB = 10 cm
=
DB AD 1
NT = 10 = 5 cm
BC 5 2
i.e. =
4 3 Since !ART is similar to !MNT, then
5 MN NT
BC = 4 =
3 AR RT
2 MN 5
=6 m i.e. =
3 6 8
By Pythagoras’ Theorem, 5
MN = 6
2 8
2
CD = 6 – 42 = 3.75 cm
3
12. A J
256 J A
=
9 D E
1 Since AO = JO and EO = DO,
=5 m
3 JD = AE
10. (i) !QTU is similar to !QRS (2 pairs of corr. "s equal). Since AO = JO, EO = DO and JOE = AOE (vert opp. "s),
QT QU
= AD = JE
QR QS
AJ = JA (same side)
12 + RT 8+4
i.e. = !AJD !JAE (SSS)
12 8
12 + RT 3
= Exercise 11C
12 2
2(12 + RT) = 36 1. A A
24 + 2RT = 36 B B
2RT = 12 C C
RT = 6 cm AC = A C (given)
!QPU is similar to !SRU (2 pairs of corr. "s equal). BC = B C (given)
PU QU ACB = A CB (vert. opp. "s)
=
RU SU !ABC !A B C (SAS)
PR + 6 8+4 Hence, AB = B A .
i.e. =
6 4
PR + 6
=3
6
PR + 6 = 18
PR = 12 cm

297
2. A B 7. P Q
O O
M M
O OP = OQ (given)
OM = OM (common side)
PMQ = 90°
PM = QM
B "POM and "QOM are congruent (SSS Congruence Test).
A
Hence OM is the angle bisector of AOB.
A A
O O
Review Exercise 11
B B
AOB = A OB (vert. opp. !s) 1. (a) A R
Since AA and BB are hinged halfway at O, B P
OA = OA and OB = OB C Q
"AOB "A OB (SAS) AB = RP = 5 m
Hence, AB = A B . AC = RQ = 7 m
3. "SOR is similar to "POQ. BC = PQ = 6 m
SR OR "ABC "RPQ (SSS)
=
PQ OQ (b) A P
SR 30 B Q
i.e. =
4 15 C R
30 AB = PQ = 12 cm
SR = 4
15 AC = PR = 8.9 cm
=8m BAC = QPR = 80°
Hence the height of the tree is 8 m. "ABC "PQR (SAS)
4. P Q (c) A P
O O B Q
C C C R
PO = QO (given) BC = QR = 9 cm
OC = OC (common side) ABC = PQR = 30°
OCP = OCQ = 90° AB is not equal to PQ.
"POC and "QOC are congruent (RHS Congruence Test). "ABC and "PQR are not congruent.
Hence OC is the angle bisector of AOB. (d) PQR = 180° – 75° – 45° (! sum of a ")
5. A A = 60°
B B A P
C C B Q
AC = AC (common side) C R
BCA = B CA (same line of vision) AB = PQ = 65 mm = 6.5 cm
BC = B C BC = QR = 89 mm = 8.9 cm
"AOB "A OB (SAS) ABC = PQR = 60°
Hence AB = AB . "ABC "PQR (SAS)
6. Given that the image of the candle is 3 times the length of the candle, 2. JLK = 180° – 55° – 50° (! sum of a ")
AC 1 = 75°
=
DE 3
D J
"ABC is similar to "DBE.
E L
BD DE
= F K
BA AC
DEF = JLK = 75°
x 3
i.e. = DFE = JKL = 50°
15 1
x = 45 EF = LK = 3 cm
"DEF "JLK (AAS)

298
3. (a) A D (f) F F
B E H I
C C G J
AB = DE (given) GF = JF (given)
ABC = DEC (given) GFH = JFI (vert. opp. "s)
BAC = EDC (given) HGF = JJF (corr. "s, GH // IJI)
!ABC !DEC (AAS) !FHG !FIJ (AAS)
ACB = DCE !FH !FIJ (AAS)
BC = EC FHG = FIJ
AC = DC FH = FI
(b) F F GH = JI
G I 4. (a) ABC = 180° – 75° – 40° (" sum of a !)
H J = 65°
GH = IJ (given) QPR = 180° – 65° – 40° (" sum of a !)
GHF = IJF (given) = 75°
GFH = IFJ (vert. opp. "s) A P
!FGH !FIJ (SAS) B Q
FGH = FIJ C R
FG = FI ABC = PQR = 65°
FH = FJ BAC = QPR = 75°
(c) K M BCA = QRP = 40°
L N !ABC is similar to !PQR (3 pairs of corr. "s equal).
N L (b) ABC = 180° – 120° – 25° (" sum of a !)
KN = ML (given) = 35°
LN = LN (same side) QRP = 180° – 120° – 45° (" sum of a !)
LKN = NML = 90° = 15°
!KLN !MNL (RHS) A Q
KLN = MNL B P
KNL = MLM C R
KL = MN ABC = QPR = 120°
(d) S R BAC is not equal to PQR and BCA is not equal to PRQ.
Q P !ABC is not similar to !QPR.
P Q (c) A P
QP = PR (same side) B Q
QSP = PRQ (given) C R
SPQ = RQP (given) AB 2 1
= =
!SQP !RPQ (AAS) PQ 4 2
SQP = RPQ BC 6 1
= =
SQ = RP QR 12 2
SP = RQ AC 5 1
= =
(e) E E PR 10 2
!ABC is similar to !PQR (3 ratios of corr. sides equal).
B C
(d) A P
F D
B Q
BE = CE (given)
C R
BEF = CED (vert. opp. "s)
AB 1
BFE = CDE (given) =
PQ 2
!EBF !ECD (AAS)
BC 4 1
EBF = ECD = =
QR 8 2
BF = CD
AC 3.5 7
EF = ED = =
PR 7.5 15
Since the 3 ratios of corresponding sides are not equal,
!ABC is not similar to !PQR.

299
(e) A P 8. (a) !ABC is similar to !ADE (2 pairs of corr. "s equal).
B R AE AD
=
C Q AC AB
BAC = QPR = 70° 7.4 + a 5+4
i.e. =
AB 6 7.4 5
= =3 7.4 + a 9
PR 2 =
AC 5 5 7.4 5
= = 5(7.4 + a) = 66.6
PQ 3 3
Since the 2 ratios of corresponding sides are not equal, 37 + 5a = 66.6
!ABC is not similar to !PRQ. 5a = 29.6
(f) A P a = 5.92
B Q (b) !ABC is similar to !EDC (2 pairs of corr. "s equal).
C R EC CD
=
ABC = PQR = 90° AC CB
AC 9 b 11
= =2 i.e. =
PR 4.5 10 7
BC 7 7b = 110
= =2 5
QR 3.5 b = 15
!ABC is similar to !PQR (2 ratios of corr. sides and included 7
" equal). ED CD
=
5. (i) O O AB CB
c 11
A B i.e. =
8 7
D C
7c = 88
AO = BO (given)
4
DO = CO (given) c = 12
7
AOD = BOC (vert. opp. "s)
(c) !PXQ is similar to !PAR (2 pairs of corr. "s equal).
!OAD !OBC (SAS)
PR PA
(ii) Since !OAD !OBC, =
PQ PX
then OAD = OBC and ODA = OCA. 6+d 9+4
6. (i) P S i.e. =
6 9
Q R 54 + 9d = 78
R Q 9d = 24
PQ = SR (given) 2
d =2
QR = RQ (common side) 3
PQR = SRQ (corr. "s, PQ // RS) !PQY is similar to !PRB (2 pairs of corr. "s equal).
!PQR !SRQ (SAS) PY PQ
=
(ii) Since !PQR !SRQ, PB PR
then QS = PR = 5 cm e 6
i.e. =
QPR = QSR = 50° e+3 6+2
2
7. P Q 3
2
O O 8 e = 6e + 18
3
C C 2
OP = OQ (given) 2 e = 18
3
OPC = OQC = 90° 3
e =6
OC = OC (common side) 4
!POC !QOC (RHS) 9. (i) P R
Since !POC !QOC, Q Q
POC = OC S P
Hence OC is the angle bisector of AOB. PQS = RQP (common angle)
QSP = QPR = 90°
!PQS is similar to !RQP (2 pairs of corr. "s equal).

300
(ii) Since !PQS and !RQP are similar, then (ii) Since !PLQ and !RLN are similar, then
QS QP LR LN
= =
QP QR LP LQ
QS 8 LR 12 + 4
i.e. = i.e. =
8 10 4 8
10QS = 64 8LR = 64
QS = 6.4 cm LR = 8 cm
10. (a) (i) B A (b) (i) N N
C C Q M
D E R S
CBD = CAE (corr. "s, BD // AE) NRQ = NSM (corr."s, SP // QR)
BCD = ACE (common angle) QNR = MNS (common angle)
!BCD is similar to !ACE (2 pairs of corr. "s equal). !NQR is similar to !NMS (2 pairs of corr. "s equal).
B G (ii) Since !NQR and !NMS are similar, then
C F MS NM
=
D E QR NQ
Since CBD = CAE, CBD = FGE (corr. "s, AC // FG) MS 12
i.e. =
BCD = GFE (corr. "s, BC // FC) 18 24
!BCD is similar to !GFE (2 pairs of corr. "s equal). MS 1
=
(ii) Since !BCD and !ACE are similar, then 18 2
MS = 9 cm
BD BC
= (c) !PLM is similar to !RLQ (2 pairs of corr. "s equal).
AE AC
BD 6 !PQM is similar to !SNM (2 pairs of corr. "s equal).
i.e. = !PQM is similar to !RNQ (2 pairs of corr. "s equal).
16 10 + 6
16BD = 96 12. (i) S S
BD = 6 cm T T
(b) BH = 18 + 6 = 24 cm R P
AG = BH = 24 cm SR = SP = 9 cm
EG = 24 – 16 = 8 cm ST = ST (given)
Since !BCD and !GFE are similar, then STR = STP = 90°
FG EG !STR !STP (RHS)
=
CB DB (ii) R R
FG 8 T P
i.e. =
6 6 U Q
FG = 8 cm RTU = RPQ (corr. "s, TU // PQ)
FH = 8 + 10 = 18 cm TRU = PRQ (common angle)
(c) A H !RTU is similar to !RPQ (2 pairs of corr. "s equal).
C F Since !STR !STP, TP = RT.
E D Since !RTU and !RPQ are similar, then
ACD = HFD (corr. "s, AC // FH) RQ RP
=
AEC = HDF (corr. "s, DH // AE) RU RT
!ACE is similar to !HFD (2 pairs of corr. "s equal). 7 + UQ 2
i.e. =
11. (a) (i) P R 7 1
L L 7 + UQ = 14
Q N UQ = 7 cm
QPL = NRL (corr. "s, PQ // RN) PQ RP
=
PLQ = RLN (vert. opp. "s) TU RT
!PLQ is similar to !RLN (2 pairs of corr. "s equal). PQ 2
i.e. =
5 1
PQ = 10 cm

301
13. (i) C P 2. Given PQ = QR = RS, !PQU, !VRU and !VST are similar (AA
A A Similarity Test).
N N Let QU = x cm and VS = y cm.
AN = AN (common side) Then RU = (5 – x) cm and VR = (5 – y) cm.
CAN = PAN (given) PQ VS
=
Since CP is a straight line, QU ST
ANC = ANP = 90° 5 y
=
!CAN !PAN (AAS) x 1
(ii) C C xy = 5 — (1)
M T PQ VR
=
N P QU RU
MCN = TCP (common angle) 5 5–y
=
Since !CAN !PAN, x 5–x
CN = PN x(5 – y) = 5(5 – x)
CN 1 5x – xy = 25 – 5x
= 10x – xy = 25 — (2)
CP 2
Since M is the midpoint of CT, Substitute (1) into (2),
CM 1 10x – 5 = 25
=
CT 2 10x = 30
!CMN is similar to !CTP (2 ratios of corr. sides and included x =0
" equal). QU = 3 cm
Since !CMN is similar to !CTP, CNM = CPT and MN is 3. It is given that AB = AC, CB = CE and BD = BE.
parallel to TA. Since CB = CE (given), then DE = 9 – 5 = 4 cm.
Hence MTAN is a trapezium. !CBE and !BDE are similar (AA Similarity Test since both
triangles are isosceles and "BEC = "DEB).
Challenge Yourself BE DE
=
BC DB
1. The following solution gives the shortest working. If the students do
BE 4
not manipulate algebra properly, this can lead to a long and tedious = since BD = BE (given)
9 BE
working. 2
BE = 36
Let the height of !PST from P to ST be h . BE = 6 cm (since BE > 0)
Using similar triangles, !ABC is also similar to !BDE (AA Similarity Test since both
h a triangles are isosceles and "ABC = "CBE).
=
h+h b AC CE
i.e. bh = a(h + h ) =
BC BE
bh – ah = ah — (1) AC 9
Area of trapezium QRST =
9 6
= area of !PST – area of !PQR 3
AC = 9
1 1 2
= b(h + h ) – ah
2 2 = 13.5 cm
1
= (bh – ah + bh)
2
1
= (ah + bh) (Substitute (1) into the equation)
2
1
= (a + b)h
2

302
Chapter 12 Area and Volume of Similar Figures and Solids
TEACHING NOTES
Suggested Approach
Teachers may bring some real-life samples of similar solids, i.e. shampoo and body bath bottles in original sizes and travel
sizes, and ask students whether these samples are similar and draw a relationship between their length, height, area, volume
and mass. Teachers may also highlight that scale drawings of structures and models are representations of the actual size of the
structures. With this, teachers may ask students to give real-life examples of such scale drawings and models.

Section 12.1: Area of Similar Figures


Teachers may wish to recap with the students what they have learnt in Book 2 and previously in Chapter 11 on
similar triangles and similarity tests before guiding them on how to find the relationship between the length of a
figure and its area (see Investigation: Areas of Similar Figures). For problems involving area of similar figures,
teachers can highlight to the students that it would be easier to write the unknown first to help in subsequent
algebraic manipulation.

Section 12.2: Volume of Similar Solids


Now that students have learnt how to find the area of similar figures, teachers may guide students on how to find
the relationship between the length of a solid and its volume, the height of a solid and its volume and the mass
of a solid and its volume (see Investigation: Volume and Mass of Similar Solids). Teachers may highlight to
students that the similarity ratio formula does have some real-life implications and ask them what some of these
implications are (see Thinking Time on page 402).

Challenge Yourself
For Question 1, students will have to find the area of !BAT and square BLUR in terms of x and y first before
finding the ratio of the areas. For Question 2, formulate a pair of simultaneous equations involving the lengths
of HM and MB to find the length of ST. For Question 3, let the length of the sides of the right-angled triangle be
a1, b1 and c1 corresponding to A1, A2 and A3 respectively then use Pythagoras’ Theorem.

303
WORKED SOLUTIONS Thinking Time (Page 402)
Investigation (Areas of Similar Figures) A1 l1
2

1. Given that the two cones are similar,


1. Since all the corresponding angles in the three squares are 90o and all A2 l2
the ratios of the corresponding sides of the three squares are equal, 2. Consider a giant human 20 m tall and a man of height 2 m, the
the three squares are similar. giant human is 10 times taller than the man. Using the similarity
2
2.
2

2 by both his leg bones. If the cross-sectional area of each of


the man’s leg bones is 10 cm2, 4 kg of mass is supported by
3. (a)
(b) 1 cm2 of the man’s leg bone. For each of the giant’s leg bones, 40 kg
2 of mass will be supported by 1 cm2 of the giant’s leg bone. As such, it
A2 l2
4. is not possible for a human to be a giant with a height of about 20 m
A1 l1
because the giant’s leg bones will not be able to support his mass.
5.
Teachers may ask students to search online for the Jolly Green Giant
6. (a) to learn more about some real-life implications of the similarity ratio
(b) formulae.
2
A2 l2
7. Journal Writing (Page 402)
A1 l1
These 5 Merlions in Singapore which are recognised by the Singapore
Investigation (Volume and Mass of Similar Solids) Tourism Board are:
1. Since all the ratios of the corresponding sides of the three cubes are 1.
equal, the three cubes are similar. 2. 2-m tall cub statue at Merlion Park
2. 3 3. Island
3 4. 3-m tall statue at Tourism Court
3 5. 3-m tall statue placed on Mount Faber’s Faber Point
3. (a) Assuming that all the 5 different Merlions are geometrically similar,
(b) (a) The ratio of the height is:
3
V2 l2
4.
V1 l1
r2 h2 (b) (i) The ratio of the total surface area is:
5. k
r1 h1 2
: 22 2
: 3 2 : 32
6. (a) V2 r22 h2
(kr1)2 (kh1)
k2r21 kh1
k ( r21 h1)
3
(ii) The ratio of the volume of material used is:
(b) V2 k3V1 3
: 23 3
: 33 : 33
3 3
V2 h2 r2
7. k3
V1 h1 r1
m2 V2
8.
m1 V1
Practise Now 1
2
A2 l2
(a)
A1 l1
2
A2 7
32 4
A2 49
32 16
49
A2 32
16
2

304
2 (ii) Since BC is parallel to QR, !ABC and !AQR are similar.
A1 l1
(b) 2
A2 l2 Area of !ABC AB
2
A1 2.5 Area of !AQR AQ
2
72 6 Area of !ABC 5
A1 25 21 6
72 144 25
Area of !ABC 21
25 36
A1 2
144
2

Practise Now 4
Practise Now 2 3
V2 l2
1.
Since BC is parallel to DE, !ABC and !ADE are similar. V1 l1
2 3
AB Area of !ABC V2 10
AD Area of !ADE 16.2 6
2
8.4 49 V2 1000
8.4 + BD 100 16.2 216
1000
8.4 49 V2
216
8.4 + BD 100 3

8.4 7 3
V1 r1
8.4 + BD 10 2.
V2 r2
BD
3
BD 2 r1
BD 16 1
1
r31
Practise Now 3 8
1
A r1 3
8

Practise Now 5
h cm
1. Let m1, V1 and h1 be the mass, volume and height of the smaller prism
5 cm respectively, and m2, V2 and h2 be the mass, volume and height of
the larger prism respectively.
C m2 V2
B
1 cm m1 V1
R
3
3 cm h2
P 1 cm Q
h1
(i) Notice that !APQ and !AQR have a common height corresponding 3
m2 8
to the bases PQ and QR respectively. 80 5
Let the common height be h cm. 3
8
1 m2
PQ h 5
Area of !APQ 2
Area of !AQR 1
QR h
2
PQ
QR
Area of !APQ 1
21 3
1
Area of !APQ 21
3
2

305
2. Let m1, V1 and h1 be the mass, volume and height of the smaller statue A1 0.4
2

respectively, and m2, V2 and h2 be the mass, volume and height of (b)
0.6 0.2
the larger statue respectively. A1
m2 V2 0.6
m1 V1 A1
3 2
h2
2
h1 A2 6
(c)
3 125 15
m2 200
A2 4
3 20
3 125 25
200 4
m2 3 A2 125
20 25
2

The mass of the statue made by Michael is 3000 kg. 2


A1 12
(d)
48 8
Practise Now 6
A1 9
(i) Let V1 and h1 be the volume and height of the smaller pyramid respectively, 48 4
and V2 and h2 be the volume and height of the larger pyramid 9
A1
respectively. 4
3 2
V1 h1
2
V2 h2 A2 6
(e)
h1
3 12 4
1
6 27 A2 9
1 h13 12 4
6 19 683 A2 12
2
h31
2
h31 (f)
A2 3p
h1 24 6p
The depth of the vegetable oil is 14.9 cm. A2 1
(ii) The top surface of the vegetable oil and that of the container are squares. 24 4
Let l1 and l2 be the length of the smaller square and the bigger square 1
A2 24
respectively. 4
2
l1 h1 14.86
l2 h2 27 2. Let r1 and A1 be the radius and area of the smaller circle respectively,
2
A1 14.86 and r2 and A2 be the radius and area of the larger circle respectively.
2
A2 27 A1 l1
1 A2 l2
3.30 2
4
The ratio of the area of the top surface of the vegetable oil to the
7
area of the top surface of the container is 1 : 3.30.
16
Exercise 12A 49
2
A2 2 3. (i) Since ST is parallel to QR, !PST and !PQR are similar.
1. (a)
64 8 2
Area of !PQR PR
A2 1
Area of !PST PT
64 16
2
1 Area of !PQR 10
A2
16 24 6
2
Area of !PQR 100
24 36
100
Area of !PQR 24
36
2
cm2
3
(ii) Area of SQRT !PQR – Area of !PST 7. 1 cm2 10–4 m2
2 3250 cm2 10–4
– 24 2
3
2
2 Actual land area 1.5
cm2
3 0.325 0.03
2
a 24 Actual land area
4. (a)
3 6 0.325
a2 0.325
9 2

a2 9 8. Since HG is parallel to QR, !PHG and !PQR are similar.


2
Area of !PHG HG
a
Area of !PQR QR
2
b 90 2
(b) p
5 10
p+q
b2 p2
25
( p + q)2
b2 25 9. (i)
2
b Surface area of larger cone 18
(ii)
2 124 12
c 240
(c) Surface area of larger cone 9
5 15
124 4
c2 9
25 124
4
c2 25 2

10. Since MN is parallel to YZ, !XMN and !XYZ are similar.


c 2
2 XM Area of !XMN
d 12
(d) XY Area of !XYZ
6 27
2
d2 12 6 14
36 27 6 + MY 14 + 22
12 36 14
d2
27 MY 2 + 12 MY + 36 36
36 7
d MY 2 + 12 MY + 36 18
2
Area of smaller hexagon 8
5. MY2 MY
200 10 2
MY MY
Area of smaller hexagon 64
200 100 –84 ± 84 2 – 4(7)(–393)
MY
64 2(7)
200
100 –84 ± 18 060
2

2
14
Area of !CAE 4
6.
9 3 (rejected since MY > 0)
Area of !CAE 16 The length of MY
9 9
16
Area of !CAE 9
9
2

Area of ABDE !CAE – Area of !CBD

2
11. Since MN is parallel to QR, !PMN and !PQR are similar. Area of !PQR 6
MN
2
Area of !PMN 40 4
QR Area of !PQR 6
Area of !PQR 40
2
4
MN 9 2

QR 9 + 16 14. (i) Since AB is parallel to QD, !ABR and !QDR are similar.
2
MN 9 AB AR
QR 25 QD QR
AB 4
MN 9
7 3+ 4
QR 25
4
3 AB
7
5
The ratio MN : QR is 3 : 5. Since AC is parallel to QP, !RAC and !RQP are similar.
12. Since BA is parallel to DE, !ABC and !EDC are similar. AC AR
BC
2
Area of !ABC QP QR
DC Area of !EDC AC 4
2 7+4 3+ 4
BC 25
4
BC + 4.5 64 AC 11
7
BC 25 2
cm
BC + 4.5 64 7
5 2
BC –4
8 7
BC BC 2
cm
3BC 7
BC (ii) Notice that !ARB and !BRC have a common height AR
13. (i) Since XY is parallel to RS, !PXY and !PRS are similar. corresponding to the bases AB and BC respectively.
1
Area of !PRS RS
2 AB AR
Area of !ARB 2
Area of!PXY XY Area of !BRC 1
2
BC AR
Area of !PRS 4 2
10 2 AB
Area of !PRS 10 BC
2 4
2
(ii) 2
P 7
7
4
The ratio of the area of !ARB to that of !BRC
h cm
Area of !BRC 4
(iii)
2 cm Area of !ARB 7
X Y
2
Area of !ARB AR
Area of !QRD QR
S
Q R 4 cm 2
4
Notice that !PQR and !PRS have a common height 7
corresponding to the bases QS and RS respectively. 16
Let the common height be h cm. 49
1
QR h
Area of !PQR 2
Area of !PRS 1
RS h
2
QR
RS
Area of !ARB 16 16 Exercise 12B
Area of ABDQ 49 – 16 33 3
V1 12
Area of !BRC Area of !BRC Area of !ARB 1. (a)
72 6
Area of ABDQ Area of !ARB Area of ABDQ V2
3
4 16
3
7 33 V2 6
(b)
64 48 4
231 V2 27
The ratio of the area of !BRC to that of ABDQ 48 8
15. (i) Since PQ is parallel to AC, !PBQ and !ABC are similar. 27
V2
Area of !ABC BC
2 8
3
Area of !PBQ BQ
3
V1 7.5
Area of !ABC
2
10 (c)
12 2.5
8 4
V1 12
100 3
Area of !ABC
16 3
2 V1 4
(d)
(ii) Notice that !PQC and !PBQ have a common height 2464 16
V1 1
corresponding to the bases QC and BQ respectively.
2464 64
Let the common height be h cm.
1
1 V1
QC h 64
Area of !PQC 2 3

Area of !PBQ 1
BQ h 3
2 V2 32
(e)
Area of !PQC QC 3.2 64
BQ V2 1
Area of !PBQ
3.2 8
Area of !PQC 10 – 4
1
8 4 V2 3.2
8
6 3
Area of !PQC
4 3 3
2 2. (a)

(iii) Notice that !ABC and !AQC have a common height


corresponding to the bases BC and QC respectively. 3
(b) : 123
Let the common height be h cm.
1
QC h
Area of !AQC 2 3 3
1 (c)
Area of !ABC BC h
2
Area of !AQC QC
Area of !ABC BC Capacity of Junior glass 6
3

Area of !AQC 10 – 4 3.
540 9
50 10 Capacity of Junior glass 8
6
Area of !AQC 50 540 27
10 8
2 540
27
3

3
a 48
4. (a)
2 6
a3

3
a 64

309
b
3 6. Let d1, V1 and m1 be the diameter, volume and mass of the larger
54
(b) sphere respectively and d2, V2 and m2 be the diameter, volume and
6 16
b3 54 mass of the smaller sphere respectively.
216 16 m1 V1
54 m2 V2
b3 3
16 d1
d2
3 3
b 729 640 d1
270 d2
3
c 270 d1 3
640
(c)
7 10 d2 270
4
c3 270
3
343 10
The ratio of the diameters is 4 : 3.
270
c3 343 7. Let h1, V1 and m1 be the height, volume and mass of the larger bottle
10
respectively and h2, V2 and m2 be the height, volume and mass of the
3 smaller bottle respectively.
c 9261
m1 V1
3
m2 V2
d 20 3
(d) h1
15 540
d3 20 h2
3
3375 540 750 h1
20 280 15
d3
540 750 h13
280 3375
3
d 125 750
h31
280
5. (i) Let h1 and A1 be the height and the area of the base of the smaller
3
cone respectively and h2 and A2 be the height and the area of the h1 9040
base of the larger cone respectively.

h1
2
A1 8. (i) Let h1 and A1 be the height and surface area of the smaller candy
h2 A2 cane respectively and h2 and A2 be the height and surface area
2 of the larger candy respectively.
h1 9
2
h2 16 A1 h1
A2 h2
h1 9
2
h2 16 4
h1 3 7
h2 4 16
The ratio of the heights of the cones is 3 : 4. 49
(ii) Let V1 and V2 be the volume of the smaller cone and the larger The ratio of the total surface areas of the candy canes is
volume respectively.
3
V1 3
448 4
V1 27
448 64
27
V1
64

3
.

310
(ii) Let m1 and V1 be the mass and volume of the smaller candy cane (ii) Let V1 and V2 be the capacity of the smaller train and the larger
respectively and m2 and V2 be the mass and volume of the larger train respectively.
candy respectively. V1 l1
3

10 V1 V2 l2
m2 V2 0.85 40
3
3
4 V2 1000
7 0.85
10–5
10 64 V2
m2 343 10–5V2
m2 V2
m2
12. Let l1, A1 and m1 be the length, base area and mass of the smaller
9. Let r1 and V1 be the radius and volume of the larger sphere respectively plastic box respectively and l2, A2 and m2 be the length, base area
and r2 and V2 be the radius and volume of the smaller sphere and mass of the larger plastic box respectively.
respectively. m1 l1
3

3
r1 V1 m2 l2
r2 V2 3
4.29 l1
3
r1 4 8.58 l2
3 1
l1 4.29
r13 3
l2 8.58
27
3
r31 0.5
2
r1 A1 l1
A2 l2
10. Let h1, V1 and m1 be the height, volume and mass of the smaller glass A1 3
0.5 )2
h2, V2 and m2 be the height, volume and 12.94
A1 3
0.5 )2 12.94
m1 V1
m2 V2 . 2

3
h1 13. (i) Let d1, A1 and V1 be the depth, top surface area and volume of
h2 the water respectively and d2, A2 and V2 be the depth, top surface
m1 4
3 area and volume of the container respectively.
3
500 6 Given that the volume of water, V1
1
m1 64 Base area
3
500 216
1
64
m1 500 3
216
336 3
Depth d1
28

11. (i) Let l1 and m1 be the length and mass of the smaller train 2
d1 A1
respectively and l2 and m2 be the length and mass of the larger (ii)
d2 A2
train respectively.
3 28
m1 l1
63
m2 l2
4
3
m1 40 9
72 000 1000 d1 4
m1 d2
10–5 9
72 000
m1 2
3
The ratio of the depth of the water to the height of the container
is 2 : 3.

311
V1 d1
3 Review Exercise 12
(iii)
V2 d2 1. (a) 2
: 52
3
336 2
2
V2 3 (b) : 92
336 8
V2 27
2
V2 (c) : 32
V
The capacity of the container is 1134 cm3. 2. Let l1 and A1 be the perimeter and the area of the smaller triangle
14. (i) Let d1 and V1 be the depth and volume of mercury respectively respectively and l2 and A2 be the perimeter and the area of the larger
and d2 and V 2 be the depth and volume of the container triangle respectively.
2
respectively. A1 294 49
d1
3
V1 A2 336 64
d2 V2
3 3. (i) Let l1 and A1 be the length and the area of the smaller pentagon
d1 8
respectively and l2 and A2 be the length and the area of the larger
15 27
pentagon respectively.
d1 8 A1 l1
2
3
15 27 A2 l2
2
d1 15 2
3 l1
2l1
The depth of the mercury is 10 cm. 1
2
A d1 4
(ii) 1
A2 d2 A2 A1
A1 10
2 When the length of a regular pentagon is doubled, its area
45 15 increases 4 times.
100 (ii) Let l1 and A1 be the length and the area of the smaller pentagon
A1 45 respectively and l2 and A2 be the length and the area of the larger
225
pentagon respectively.
2
The area of the mercury that is exposed to the air is 20 cm2. A1 l1
1 A2 l2
(iii) 45 15
3 2
3
25 l1
A2 2l1
15. Let h1, V1 and m1 be the height, volume and mass of the smaller clay
25 1
model respectively and h2, V2 and m2 be the height, volume and mass
A2 4
of the larger clay model respectively.
A2
m1 V1
The area of the enlarged pentagon is 100 cm2.
m2 V2
(iii) Let l1 and A1 be the length and the area of the smaller polygon
3
h1 respectively and l2 and A2 be the length and the area of the larger
h2 polygon respectively.
3 2
x + 0.3 20 A1 l1
x2 30 A2 l2
x + 0.3 8 2
A1 l1
x2 27
A2 3l1
x x2
x2 x A1 1
A2 9
27 ± (–27)2 – 4(8)(–8.1)
x A2 A1
2(8) When the length of a regular n-sided polygon is tripled, its
27 ± 988.2 area increases 9 times.
16

312
4. Let r1 and V1 be the base radius and volume of the smaller cylinder 7. Let r1 and A1 be the radius and the area of the smaller sector
respectively and r2 and V2 be the base radius and volume of the larger respectively and r2 and A2 be the radius and the area of the larger
cylinder respectively. sector respectively.
3 2
V1 r1 A1 r1
V2 r2 A2 r2
3 2
8 r1 3
27 r2 5
9
r1 8 25
3
r2 27 Area of shaded region 25 – 9 16
2 Area of sector RUN 25 25
3 The ratio of the area of the shaded region to that of the area of
The ratio of the base radii of the cylinders is 2 : 3. sector RUN
5. (i) Let h1 and V1 be the height and volume of the smaller jug 8. (i) Let l1 and A1 be the perimeter and surface area of the larger pond
respectively and h2 and V2 be the height and volume of the larger respectively and l2 and A2 be the perimeter and surface area of
jug respectively. the smaller pond respectively.
3
V1 h1 A1 l1
2

V2 h2 A2 l2
3
27 h1 3l2
2

64 h2 l2
h1 27 9
3
h2 64 1
3 The ratio of the total surface areas of the ponds is 9 : 1.
4 (ii) Let V1 and V2 be the capacity of the larger and the smaller pond
The ratio of the heights of the jugs is 3 : 4. respectively.
3
(ii) Let A1 and A2 be the total surface area of the smaller jug and the 10 800 3l2
larger jug respectively. V2 l2
A1 h1
2 10 800
A2 h2 V2
V2
2
A1 3 V2
A2 4 The amount of water contained in the smaller pond is 400 l.
9 9. (a) S Q
16 L L
P M
6. Let h1, V1 and m1 be the height, volume and mass of the larger marble SPL QML (alt. !s)
statue respectively and h2, V2 and m2 be the height, volume and mass PLS MLQ (vert. opp. !s)
of the smaller marble statue respectively. "SLP is similar to "QLM (2 pairs of corr. !s equal).
m1 V1 (b) (i) Since M is the midpoint of QR,
m2 V2 QM 1
3
h1 SP 2
2
h2 Area of !QLM 1 1
12 40
3 Area of !SLP 2 4
m2 320 LQ 1
(ii)
1 LS 2
512 LS LS
m2 QS LQ + LS
2
2 +1
2
3

313
2
(iii) !PLS and !PQS have a common height corresponding to Area of !PQR 5 25
the bases LS and QS respectively. (ii)
Area of !PML 2 4
Let the common height be h cm. Area of !PQR 25
(iii)
1 Area of !PML 4
LS h
Area of !PLS 2 LS 2 Area of !PQR 25
Area of !PQS 1 QS 3 6 4
QS h
2 25
10. (a) !LMN is similar to !LXY (2 pairs of corr. s equal). Area of !PQR
4
2
XY 2
MN 5 Area of LMRQ
2
Area of !LXY 2 4 2

Area of !LMN 5 25 XP 1
Area of quadrilateral XMNY 25 – 4 21 12.
PQ 2
Area of !LMN 25 25 QY 1
(b) (i) N X PQ 3
O O XP : PQ : QY
M Y
NMO XYO (alt. "s) !QZX and !QYZ have a common height corresponding to the bases
NOM XOY (vert. opp. "s) XQ and QY respectively.
!NOM is similar to !XOY (2 pairs of corr. "s equal). Let the common height be d cm.
YO XY 2 1
(ii) QY d
OM NM 5 Area of !QYZ 2 QY 2
(iii) !XOY and !XOM have a common height corresponding Area of !QZX 1 XQ 9
XQ d
to the bases OY and OM respectively. 2
Let the common height be h cm. !QRX and !QZX have a common height corresponding to the bases
1 RZ and XZ respectively.
OY h
Area of!XOY 2 YO 2 Let the common height be h cm.
Area of!XOM 1 OM 5 1
OM h RZ h
2 Area of !QRZ 2 RZ 3
!XOM and !MON have a common height corresponding Area of !QZX 1 XZ 5
XZ h
to the bases OX and OM respectively. 2
Let the common height be d cm. Area of !QRZ Area of !QRZ Area of !QZX
1 Area of !QYZ Area of !QZX Area of !QYZ
OX d
Area of !XOM 2 YO 2
3 9
Area of!MON 1 OM 5
OM d 5 2
2
27
Area of !XOY : Area of !XOM : Area of !MON
10
2 : 5
The ratio of the area of !QRZ to that of !QYZ
2 : 5
1
4 : 10 : 25 13. (i) 22
3
11. (i) P P 3
(to 3 s.f.)
Q M 3
R L Volume of wooden portion 6
(ii)
RPQ LPN (common angle) Volume of paper weight 8
PQ 11 + 4 5 Volume of wooden portion 27
PM 6 2 33.51 64
PR 10 5 27
33.51
PL 4 2 64
3
!PQR is similar to !PML (2 ratios of corr. sides and (to 3 s.f.)
included " equal). (iii)
3

0.9

314
VA a
3
1 (iii) Let HM a and MB b.
14. (i) Since !TSA is similar to !BSH,
VA+ B 2a 8
VA 1 1 AT ST
VB 8 –1 7 BH BS
The ratio of the volume of A to that of B a ST
3 a+b 18 + 32
VA+ B 2a 8
(ii) a ST
VA+ B+C 3a 27 — (1)
a+b 50
VB 7 7 Since !BME is similar to !BHT,
VC 27 – 8 19 BM BE
The ratio of the volume of B to that of C BH BT
3a
3
27 b 18
VA+ B+C — (2)
(iii) a+b 50 + ST
VA+ B+C + D 4a 64
VA+ B+C 27 27 a b ST 18
VD 64 – 27 37 a+b a+b 50 50 + ST
The ratio of the volume of A, B and C to that of D a+b ST (50 + ST ) + 18(50)
a+b 50(50 + ST )
Challenge Yourself 50ST + ST 2 + 900
50(50 + ST )
1. Let BT x and BR y.
ST ST ST 2
1
Area of !BAT AB BT ST 2
2
1 ST 1600
2x x
2 ST > 0)
x2 (iv) Area of !BME : Area of !BHT
Area of square BLUR y y BE 2 : BT 2
y2 2
: 902
y 1 : 25
.
x 3. Let the length of the sides of the right-angled triangle be a1, b1 and
!LAU is similar to !BAT. c1 corresponding to A1, A2 and A3 respectively.
AB TB By Pythagoras’ Theorem,
AL UL a21 b21 c21
2x x
2x – y y A1 : A2 : A3
2xy x2 – xy
a21 b21 c21
x(3y – 2x
Hence A1 A2 A3.
y 2
x
x 3
The ratio of the area of the square BLUR to the area of !BAT is
4 : 9.
2. (i) B B
M H
E T
BME BHT (corr. "s)
MBE HBT (common angle)
!BME is similar to !BHT (2 pairs of corr. s equal).
(ii) !EAS is similar to !THS.
!TSA is similar to !BSH.

315
Chapter 13 Geometrical Properties of Circles
TEACHING NOTES
Suggested Approach
Teachers may begin the topic by asking students how the centre of a circle can be determined, or how a circle can be constructed
given only three points that pass through the circle. With this, teachers may introduce both the symmetric and angle properties of
circles and build up on past knowledge, i.e. Pythagoras’ Theorem, Basic Geometry, Triangles learnt by students when covering
this chapter.

Section 13.1: Symmetric Properties of Circles


Teachers may recall with students what the perpendicular bisector of a chord is before introducing Circle
Symmetric Property 1 through exploring three conditions and using a cut-out piece of circle to better visualise
and understand the symmetric properties of circles (see Investigation: Circle Symmetric Property 1).

For Circle Symmetric Property 2, teachers may ask students what equal chords are and the properties of equal
chords of a circle (see Investigation: Circle Symmetric Property 2).

For Circle Symmetric Property 3 and Circle Symmetric Property 4, these two properties involve the radius of a
circle and tangent to a circle (see Investigation: Circle Symmetric Property 3 and Investigation: Circle Symmetric
Property 4). It is important for students to know and identify a secant and a tangent.

Section 13.2: Angle Properties of Circles


Now that students have learnt the symmetric properties of circles, teachers may guide students on angle properties
of circles. It is important for students to recognise which angle is subtended by which arc by looking at the shape
of the arc (see Class Discussion: Identifying Angles at the Centre and at the Circumference) before they explore
the relationship between the angle at the centre of a circle and the angle at the circumference subtended by the
same arc (see Investigation: Circle Angle Property 1 and Thinking Time on page 434).

For Circle Angle Property 2, teachers can ask students what the angle at the circumference subtended by the
arc is when the angle at the centre of the circle is 180o (see Investigation: Circle Angle Property 2 and Thinking
Time on page 438).

Teachers should guide students along on identifying angles in the same or opposite segments of a circle before
teaching Circle Angle Property 3 (see Class Discussion: Angles in Same or Opposite Segments, Investigation:
Circle Angle Property 3 and Investigation: Circle Angle Property 4).

Teachers should note that students may have difficulty in recognising the properties of circles when the diagrams
are not drawn and given in an obvious manner. As such, teachers may get students to observe the diagram from a
different orientation or to add extra lines to the diagram so that they can better visualise and identify the properties
of circles.

316
WORKED SOLUTIONS Class Discussion (Application of Circle Symmetric
Investigation (Circle Symmetric Property 1) Property 1)

1. Conditions A and B 1. Construct the perpendicular bisector of AB.


2. (a) AM = MB Construct the perpendicular bisector of BC.
(b) M is the midpoint of AB. The point of intersection of the two perpendicular bisectors will be
3. Conditions A and C the centre O of the circle passing through the three points A, B and
4. !AMO = !BMO = 90° C.
5. It has an equal distance from all of the points on its circumference. Construct the circle with centre O and radius OA.
6. (a) Conditions B and C
(b) Yes
B

Thinking Time (Page 414)


1. C C

l A

O
O

A M B

A B
C C 2. B
M M
CM = CM (common side) 1m 0.5 m
Balcony
AC = BC A C
3m
AMC = BMC = 90°
"ACM = "BCM (SAS).
Hence AM = MB and the line l bisects the chord AB.
2. C
6m
Living Room
l

A B 4m
M
Label the balcony ABC.
A B Draw a line segment AC of length 6 cm.
C C Construct the perpendicular bisector of AC.
M M Measure 2 cm from the midpoint of AC along the perpendicular
CM = CM (common side) bisector to get the point B.
AM = MB Construct the perpendicular bisector of AB (or BC).
AC = BC The point of intersection of the two perpendicular bisectors will be
"ACM "BCM (SSS). the centre O of the circlepassing through the three points A, B and
Hence AMC = BMC = 90° and the line l is perpendicular to the chord C.
AB. Construct the arc ABC of circle with centre O and radius OA (or ON
or OC).

317
Investigation (Circle Symmetric Property 2) Investigation (Circle Angle Property 1)
2. The distance of both chords from centre O is equal. 2.
!AOB 60o 90o 120o 145o 250o 320o
3. equidistant
5. The lengths of both chords are equal. !APB 30o 45o 60o 72.5o 125o 160o
6. equal
!AOB
2 2 2 2 2 2
!APB
Investigation (Circle Symmetric Property 3)
3. !AOB = 2!APB
3. (a) The secant is perpendicular to the radius.
4. twice
(b) It is a right-angle.
4. perpendicular
Thinking Time (Page 434)
Investigation (Circle Symmetric Property 4) Case 3
P
2. (a) AP = BP
(b) !OPA = !OPB a
3. (a) equal
(b) bisects O
4. O O
A B
P P B
OA = OB (radii of circle) A
OAP = OBP = 90° Let !APB = a.
OP = OP (common side) Since OP = OB (radii),
"OAP "OBP (SAS) "BOP is an isosceles triangle.
Hence, tangents from an external point are equal in length and the Then !OBP = !OPB = a
line from the centre of a circle to an external point bisects the angle !AOB = !OBP + !OPB (ext. ! of ")
between the two tangents. = 2a
= 2 !APB (proven)
Class Discussion (Identifying Angles at the Centre and at Case 4
the Circumference)

Q P

(d) B
A O
(a) (c) P
O O (b)
(c) (a) C
B
B A
(b)
A (d) Let !OPB = a.
P Q Since OB = OP (radii),
"BOP is an isosceles triangle.
P
Then !OBP = !OPB = a
!BOC = !OBP + !OPB (ext. ! of ")
(b) = 2a
(c)
O = 2 !OBP
Let !OPA = b.
(a)
Since OA = OP (radii),
"AOP is an isosceles triangle.
A (d) B Then !OAP = !OPA = b
Q !AOC = !OPA + !OAP (ext. ! of ")
= 2b
= 2 !OPA

318
Since !BOC = 2 !OBP and !AOC = 2 !OPA, 4. !AOB = 2 !APB ( at centre = 2! at circumference)
!APB = 2 !OAB (proven) = 2 !y
Since OA = OB (radii of circle),
Investigation (Circle Angle Property 2) 180° – 2!y
!OAB =
2
2. (a) !APB = 90°
= 90° – !y
(b) Semicircle
Since !OAT = 90° (tangent radius),
3. 90°
!OAB = 90° – !x
4. !AOB = 180°
!x = !y
1
!APB = 180° (! at centre = 2 ! at circumference)
2
Thinking Time (Page 446)
= 90°
(a)
Thinking Time (Page 438)
The diameter of a circle subtends a right angle to any point on the circle.
The converse is also true, whereby a right angle on the circle must cut
y
PQ and RS
the centre where they intersect. x

Class Discussion (Angles in Same or Opposite Segments) (b)


!x and !y are angles in the same segment (formed by the chord BE). y
!w and !z are angles in opposite segments (formed by the chord BF).

!w and !y are not angles in opposite segments because the two segments
in which the two angles lie are not formed by the same chord, i.e. the
two segments are just diffferent segments. x

Investigation (Circle Angle Property 3) (c)


2. !APB = !AQB
3. equal
4. !AOB = 2 !APB (! at centre = 2 ! at circumference)
!AOB = 2 !AQB (! at centre = 2 ! at circumference) y

!APB = !AQB x

Investigation (Circle Angle Property 4) (d)


2. !APB + !AQB = 180o
3. 180o
4. !AOB = 2 !APB (! at centre = 2! at circumference) x
!AOB = 2 !AQB (! at centre = 2! at circumference)
y
Since !AOB !AOB = 360° (!s at a point), then 2 !APB
+ 2 !AQB = 360°.
!APB + !AQB = 180°, i.e. !APB and AQB are supplementary.
For the special case where both !APB and !AQB are in a
semicircle, then !APB = !AQB = 90o (rt. ! in semicircle)
and so !APB + !AQB = 180°. Class Discussion (Proof of Tangent-Chord Theorem)
!ABS = 90° (rt. ! in a semicircle)
Investigation (Circle Angle Property 5) !SAQ = 90° (tangent radius)
2. !x = !y !ASB = !ACB (!s in same segment)
3. equal !ASB + !SAB = 90° (! sum of a ")
!SAB + !BAQ = 90 (tangent radius)
!ASB = !BAQ
!BAQ = !ACB (since !ASB = !ACB)

319
Practise Now 1 OK bisects PQ (perpendicular bisector of chord).
13
1. ON bisects XY (perpendicular bisector of chord). PK = QK = = 6.5 cm
2
26
XN = NY = = 13 cm Consider !OKP.
2
By Pythagoras’ Theorem,
Consider !ONX.
OK2 = OP2 – PK2
O
= 72 – 6.52
= 6.75
2
3 cm OX = 18.75 + 6.75
= 25.5
X Y
13 cm N 13 cm OX = 25.5 (since length OX > 0)
By Pythagoras’ Theorem, = 5.05 cm (to 3 s.f.)
OX2 = ON2 + XN2
= 32 + 132 Practise Now 2
= 178
There are two possible cases about the positions of the two chords AB
Since OP = OX (radii of circle),
and XY (equal chords).
then OP2 = OX2 = 178
Let AB = 10 cm and XY = 30 cm.
Consider !OMP.
By Pythagoras’ Theorem, 5 cm 5 cm
OP2 = OM2 + MP2 A N B
OX2 = 82 + MP2 (since OX = OP, radii of circle)
178 = 64 + MP2
20 cm
MP2 = 178 – 64
= 114 O
i.e. MP = 114 (since length MP > 0) 20 cm
OM bisects PQ. (perpendicular bisector of chord) 15 cm 15 cm
PQ = 2 MP X M Y

=2 114
Case 1: The chords are on opposite sides of the centre O.
= 21.35 cm (to 2 d.p.)
In !AON,
2. P By Pythagoras’ Theorem,
ON2 = 202 – 52
= 375
ON = 375
O = 19.36 cm (to 4 s.f.)
K
In !YOM,
By Pythagoras’ Theorem,
OM2 = 202 – 152
X
A H B = 175
Q OM = 175
OH bisects AB (perpendicular bisector of chord).
= 13.23 cm (to 4 s.f.)
11
AH = BH = = 5.5 cm Distance between the chords = MN
2
= NO + OM
Consider !OAH.
= 19.36 + 13.23
By Pythagoras’ Theorem,
= 32.6 cm (to 3 s.f.)
OH2 = OA2 – AH2
= 72 – 5.52
= 18.75

320
(iii) Area bounded = Area of "OAB – Area of sector OAC
1 64.01°
= 8 3.9 – 3.92
2 360°
= 7.10 cm2 (to 3 s.f.)
O

20 cm 20 cm Practise Now 4
15 cm 1. (i) !OBP = 90° (tangent radius)
X M Y !OPB = 180° – 90° – 62° (! sum of a ")
A N B = 28°
5 cm (ii) !AOP = !BOP = 62°
Case 2: The chords are on the same side of the centre O. Since OC = OA (radii of circle),
Distance between the chords = MN !OAC = !OCA
= ON – OM 180° – 62°
= (base !s of isos. ")
= 19.36 – 13.23 2
= 6.13 cm (to 3 s.f.) = 59°
The distance between the chords can either be 6.13 cm or 32.6 cm. BP
(iii) tan 62° =
14
BP = 14 tan 62°
Practise Now 3
= 26.3 cm (to 3 s.f.)
1. (i) !OAP = 90° (tangent radius) (iii) Area of quadrilateral APBO
In "OAP, = Area of "PBO + Area of "PAO
4.5 1
tan !OPA = =2 14 26.33
10.5 2
4.5 = 369 cm2 (to 3 s.f.)
!OPA = tan–1
10.5 2. !PQO = 90° (tangent radius)
= 23.2° (to 1 d.p.) !PQT = !PTQ
(ii) By Pythagoras’ Theorem, 180° – 64°
OP2 = OA2 + AP2 = (base !s of isos. ")
2
= 4.52 + 10.52 = 58°
= 130.5 !SQT = 90° – 58°
OP = 130.5 (since length OP > 0) = 32°
= 11.4 cm (to 3 s.f.)
1 Practise Now 5
(iii) Area of "OPA = 4.5 10.5
2 1. (i) !SOR = 2 !SPR (! at centre = 2 ! at circumference)
= 23.625 cm2 = 2 28°
2. (i) !OAB = 90° (tangent radius) = 56°
In "OAB, (ii) !POR = 180° + 56° = 236°
By Pythagoras’ Theorem, 1
!PQR = !POR (! at centre = 2 ! at circumference)
OB2 = OA2 2
(5 + x)2 = x2 + 82 1
= 236°
x2 + 10x + 25 = x2 + 64 2
10x = 39 = 118°
x = 3.9 2. Since OA = OB (radii of circle),
8 !OAB = !OBA = 35° (base !s of isos. ")
(ii) tan !AOB = !AOB = 180° – 35° – 35° (! sum of ")
3.9
8 = 110°
!AOB = tan–1
3.9 x = 110°
= 64.0° (to 1 d.p.) 1
!ACB = !AOB (! at centre = 2 ! at circumference)
2
1
= 110°
2
= 55°
y = 55°

321
3. !ABC = 180° – 73° (adj. !s on a str. line) (iv) !PAD = 21° + 64° (ext. ! of ")
= 107° = 85°
!AOC = 2 !ABC (! at centre = 2 ! at circumference) 2. !BCD = 180° – 68° = 112° (!s in opp. segments)
= 2 107° Since BC = CD,
= 214° 180° – 112°
!BDC = !CBD = = 34° (base !s of isos. ")
Obtuse !AOC = 360° – 214° (!s at a point) 2
= 146° !BAC = !BDC = 34° (!s in same segment)

Practise Now 6 Practise Now 9

(i) !POR = 180° – 50° (adj. !s on a str. line) 1. Let the centre of the circle be O.
= 130° !OAT = 90° (tangent radius)
Since OP = OR (radii of circle), !CAT = 33° (!s in alt. segment)
180° – 130° !OAC = !OCA (base !s of isos. "OAC)
!OPR = (base !s of isos. ")
2 = 90° – 33°
= 25° = 57°
(ii) !OPQ = 50° (corr. !s, PR // OR) !ATC = 57° – 33° (ext. ! of ")
!QPR = 50° – 25° = 25° = 24°
!QOR = 2 !QPR (! at centre = 2 ! at circumference) 2. (i) Since "ABC is an isos ",
= 2 25° !ABC = 180° – 2 41°
= 50° = 98°
(iii) !PQO = !QOR = 50° (alt. !s) !CAT = 98° (!s in alt. segment)
!PXQ = 180° – 50° – 25° (! sum of ") !DAT = 46° (!s in alt. segment)
= 105° ! CAD = 98° – 46°
= 52°
Practise Now 7 (ii) !ATC = 180° – 98° – 46° (! sum of ")
= 36°
1. (i) !CDX = !CAB (!s in same segment)
= 44°
Practise Now 10
(ii) !ABX = !DCX (!s in same segment)
= 25° !AOC = 2 114° (! at centre = 2 ! at circumference)
(iii) !CXB = 25° + 44° (ext. ! of ") = 228°
= 69° !AOC = 360° – 228° (!s at a point)
2. x = 2 25° (! at centre = 2 ! at circumference) = 132°
= 50° !ABC = 180° – 132° (!s in opp. segments)
y = 25° (!s in same segment) = 48°
3. !APB = 90° (rt. ! in semicircle)
!PRB = 45° (!s in same segment) Practise Now 11
!APR = 25° + 45° (ext. ! of ")
(i) A P
= 70°
X X
!BPR = 90° – 70°
Q B
= 20°
AXQ = PXB = 90°
QAX = BPX (!s in same segment)
Practise Now 8
AQX = PBX (!s in same segment)
1. (i) !BAD = 180° – 21° – x° (! sum of a ") !AXQ is similar to !PXB (3 pairs of corr. !s equal).
= (159 – x)° AX XQ
(ii) =
(ii) !BCD = 180° – 31° – x° (! sum of a ") PX XB
= (149 – x)° 5 10.5
=
(iii) !BAD + !BCD = 180° (!s in opp. segments) 3.4 XB
(159 – x)° + (149 – x)° = 180° 5XB = 35.7
308° – 2x° = 180° XB = 7.14 cm
2x° = 128°
x = 64

322
Exercise 13A 3.

1. (a) By Pythagoras’ Theorem,


a2 = 132 – 52
= 144 O
a = 144 rm
5m
= 12 (since length a > 0)
5 A 24 m B
cos b° =
13
5 By Pythagoras’ Theorem,
b = cos–1
13 r2 = 52 + 122
= 67.4 (to 1 d.p.) = 169
(b) By Pythagoras’ Theorem,
r = 169 (since length r > 0)
c2 = 172 – 132
= 120 = 13
The radius of the circle is 13 m.
c = 120
4.
= 11.0 (to 3 s.f.) (since length a > 0)
8
cos d° =
17
8 O
d = cos–1
17 8.5 cm
= 61.9 (to 1 d.p.) 5 cm
(c) Let the radius of the circle be r mm.
dm
By Pythagoras’ Theorem,
r2 = 52 + 62
By Pythagoras’ Theorem,
= 61
2
By Pythagoras’ Theorem, d
= 8.52 – 52
e2 = r2 – 52 2
= 61 – 25 d2
= 47.25
4
= 36
d = 189 (since length d > 0)
c = 36
= 13.7 (to 3 s.f.)
= 6 (since length a > 0)
The length of the chord is 13.7 cm.
6
tan f ° = 5. (a) a = 12
5
6 b = 90
f = tan–1 22
5 (b) x = = 11
= 50.2 (to 1 d.p.) 2
2. y = 90
6.
O

3 cm
O
17 cm P
x cm 4.8 cm

By Pythagoras’ Theorem,
A 16 cm B
OP2 = 32 + 4.82
By Pythagoras’ Theorem, = 32.04
x2 = 172 – 82 OP = 32.04 (since length OP > 0)
= 225 = 5.660 (to 4 s.f.)
x = 225 (since length x > 0)
= 15
The perpendicular distance from O to AB is 15 cm.

323
4.8 Case 1: The chords are on opposite sides of the centre O.
t an =
3 In "AON,
= tan–1 1.6 By Pythagoras’ Theorem,
= 57.99° (to 2 d.p.) ON 2 = 52 – 32
!POQ = 2 57.99° = 115.98° = 16
Area of cross section of pipe ON = 16
= Area of sector POQ – Area of "POQ
= 4 cm
115.98° 1
= 5.6602 – 9.6 3 In "YOM,
360° 2
2 By Pythagoras’ Theorem,
= 18.0 cm (to 3 s.f.)
OM 2 = 52 – 42
7.
=9
OM = 9
= 3 cm
O
Distance between the chords = MN
= NO + OM
8 cm 8 cm =4+3
X N Y = 7 cm
2 cm
P
Let the radius of the circle be r cm.
By Pythagoras’ Theorem,
OX2 = ON2 + XN2
O
r2 = (r – 2)2 + 82
r2 = r2 – 4r + 4 + 64 5 cm 5 cm

4r = 68 4 cm
r = 17 X M Y
The radius of the circle is 17 cm. A N B
8. (i) By Pythagoras’ Theorem, 3 cm
OC2 = 92 – 72 Case 2: The chords are on the same side of the centre O.
= 32 Distance between the chords = MN
OC = 32 (since length OC > 0) = ON – OM
=4–3
= 5.66 cm (to 3 s.f.)
= 1 cm
(ii) By Pythagoras’ Theorem,
The distance between the chords can either be 1 cm or 7 cm.
OE2 = 5.6572 + (6 + 7)2
10.
= 201.0 (to 4 s.f.)
OE = 201.0 (since length OE > 0)
= 14.2 cm (to 3 s.f.)
9. There are two possible cases about the positions of the two chords O
AB and XY (equal chords).
Let AB = 6 cm and XY = 8 cm.

3 cm 3 cm
M X N 3 cm
A N B P Y Q

Let the radius of the circle be r cm and the length of OX be x cm.


5 cm
Given PQ = 7 cm and MN = 14 cm,
O

5 cm

4 cm 4 cm
X M Y

324
In !OMX, 2. (i) "OAT = 90o (tangent radius)
By Pythagoras’ Theorem, "ATB = 180° – 90° – 64° (" sum of a !)
OM 2 = OX 2 + MX 2 = 26°
r2 = x2 + 72 (ii) OA = OB (radii of circle)
r2 = x2 + 49 — (1) 64°
"OAB = "OBA = = 32° (ext. " of !)
In !OPY, 2
By Pythagoras’ Theorem, "TAB = 90° + 32°
OP 2 = OY 2 + PY 2 = 122°
r2 = (x + 3)2 + 3.52 3. "AOB = 180° – x – 90° (" sum of a !)
r2 = x2 + 6x + 9 + 12.25 = 90° – x
r2 = x2 + 6x + 21.25 — (2) OB = OC (radii of circle)
Substitute (1) into (2): 90° – x x
"OCD = "OBD = = 45° – (ext. " of !)
x2 + 49 = x2 + 6x + 21.25 2 2
6x = 27.75 x
"COD = 180° – 90° – 45° – (" sum of a !)
x = 4.625 2
x
r = 4.625 2 + 7 2 = 180° – 90° – 45° +
2
= 8.39 (to 3 s.f.) x
= 45° +
The radius of the circle is 8.39 cm. 2
4. (a) a = 49
11.
b = 14
(b) "OAP = "OBP = 90o (tangent radius)
c° = 180° – 32° – 90° = 58° (" sum of a !)
O c = 58
d = 15
17 cm
9 cm (c) "OAP = "OBP = 90o (tangent radius)
OA = OB (radii of circle)
X H Y
112°
"POA = = 56°
2
9
cos "HOX = e° = 180° – 56° – 90° = 34° (" sum of a !)
17
e = 34
9
"HOX = cos–1 f
17 tan 56° =
10
= 58.03° (to 2 d.p.)
f = 10 tan 56°
"XOY = 2 58.03° = 115.98°
= 14.8 (to 3 s.f.)
By Pythagoras’ Theorem,
(d) "OAP = "OBP = 90o (tangent radius)
XH 2 = 172 – 92
OA = OB (radii of circle)
= 208
h° = 90° – 35° = 55°
XH = 208 (since length XH > 0) h = 55
= 14.42 cm (to 4 s.f.) g° = 90° – 55° = 35°
XY = 2 14.42 = 28.84 cm g = 35
Perimeter of minor segment = XY + minor arc XY (e) "OBP = 90o (tangent radius)
115.98° By Pythagoras’ Theorem,
= 28.84 + 2 17
360° (5 + i)2 = 52 + 122
= 63.3 cm (to 3 s.f.) 2
i + 10i + 25 = 169
i2 + 10i – 144 = 0
Exercise 13B (i – 8)(i + 18) = 0
1. "OPB = 90o (tangent radius) i = 8 (since length i > 0)
OA = OP (radii of circle) 12
tan j ° =
"OAP = "OPA = 33° (base "s of isos. !) 5
"PBA =180° – (33 + 90)° – 33° (" sum of a !) 12
j = tan–1
= 24° 5
= 67.4 (to 1 d.p.)

325
(f) !OBP = 90o (tangent radius) Obtuse angle between PQ and tangent at P
By Pythagoras’ Theorem, = 90° + 48°
(k + 7)2 = k2 + 152 = 138°
k2 + 14k + 49 = k2 + 225 8.
14k = 176
k = 12.6 (to 3 s.f.)
15
tan l° =
12.57 P 10.6 cm O
15
l = tan–1
12.57
5.6 cm
= 50.0 (to 1 d.p.)
5. (i) !OAT = 90o (tangent radius) N
OA = OB (radii of circle)
!OAB = !OBA = 46°
!BAT = 90° – 46° = 44° (base !s of isos. ")
(ii) !CAT = 180° – 69° (adj. !s on a str. line) !ONP = 90o (tangent radius)
= 111° By Pythagoras’ Theorem,
!PTC = 180° – 44° – 111° (! sum of a ") PN 2 = 10.62 – 5.62
= 25° = 81
6. (i) Let the radius of the circle be r cm.
PN = 81 (since length PN > 0)
!OAC = 90o (tangent radius)
By Pythagoras’ Theorem, = 9 cm
(r + 12)2 = r2 + 182 9.
r + 24r + 144 = r2 + 324
2

24r = 180
r = 7.5
9.1 m O
The radius of the circle is 7.5 cm. T
18
(ii) tan !AOB =
7.5 8.4 m
18
!AOB = tan–1 S
7.5
= 67.4° (to 1 d.p.)
(iii) Area of shaded of region
= Area of "OAC – Area of sector OAB
1 67.4° !OST = 90o (tangent radius)
= 18 7.5 – 7.52
2 360° By Pythagoras’ Theorem,
= 34.4 cm2 (to 3 s.f.) OS2 = 9.12 – 8.42
7. = 12.25
OS = 12.25 (since length OS > 0)
= 3.5 m
Diameter of circle = 2 3.5
O
=7m
84°
10. !ABO = !ACO = 90° (tangent radius)
!AOC = 180° – 122° (adj. !s on a str. line)
P Q
= 58°
!CAO = 180° – 90° – 58° (! sum of a ")
= 32°
!BAC = 2 32°
= 64°
OP = OQ (radii of circle)
!OPQ = !OQP
180° – 84°
= (base !s of isos. ")
2
= 48°

326
11. (e) 360° – 110° = 250° (!s at a point)
1
e° = 250° (! at centre = 2 ! at circumference)
2
A = 125°
e = 125
51° (f) !O = 2 40° (! at centre = 2 ! at circumference)
T O
= 80°
180° – 80°
f° = (base !s of isos. ")
2
B
= 50°
f = 50
(g) g° = ! at circumference (alt. !s)
!OAT = !OBT = 90° (tangent radius) 1
= 70° (! at centre = 2 ! at circumference)
!OAB = 180° – 90° – 51° (!sum of a ") 2
= 35°
= 39°
g = 35
!BAT = 90° – 39°
(h) ! at circumference
= 51°
1
12. = 98° (! at centre = 2 ! at circumference)
2
= 49°
180° – 98°
= 41° (base !s of isos. ")
2
O h° = 180° – 41° – 41° – 21° – 49° (! sum of a ")
= 28°
25.5 cm h = 28
12 cm
2. (a) a° = 180° – 40° – 90° (rt. ! in semicircle)
H P K = 50°
a = 50
180° – 90°
!OPK = 90° (tangent radius) (b) b° = (rt. ! in semicircle)
2
By Pythagoras’ Theorem, = 45°
PK 2 = 25.52 – 122 b = 45
= 506.25 (c) c° = 90° – 60° (rt. ! in semicircle)
PK = 506.25 (since length PK > 0) = 30°
= 22.5 cm c = 30
HK = 2 22.5 (d) d° = 90° – 30° (rt. ! in semicircle)
= 45 cm = 60°
d = 60
Exercise 13C 3. (a) x° = 50° (!s in same segment, alt. !s)
1. (a) a° = 2 40° (! at centre = 2 ! at circumference) x = 50
= 80° (b) y° = 180° – 100° – 68° (!s in same segment, ! sum of a ")
a = 80 = 12°
1 y = 12
(b) b° = 60° (! at centre = 2 ! at circumference)
2 4. !PTQ = !PSQ = 20° (!s in same segment)
= 30° !PQT = 180° – 100° – 20° (! sum of a ")
b = 30 = 60°
(c) c° = 40° (!s in the same segment) 5. !ADC = 180° – 65° (adj. !s on a str. line)
c = 40 = 115°
1 !ABC = 180° – 115° (!s in opp. segments)
(d) d° = 230° (! at centre = 2 ! at circumference)
2 = 65°
= 115°
d = 115°

327
6. (a) 180° – 110° = 70° (!s in opp. segments) Obtuse !BOC = 360° – 218° = 142°
x° = 180° – 70° – 70° (! sum of a ") !OBC = !OCB
= 40° 180° – 142°
= (base !s of isos. ")
x = 40 2
(b) y° + 54° + y° + 54° = 180° (!s in opp. segments) = 19°
2y° + 108° = 180° !OBA = 19° + 43° = 62°
2y° = 72° (ii) !OCA = 19° + 28°
y° = 36° = 47°
y = 36 12. (i) !PWS = 90° (rt. ! in semicircle)
(c) 41° + 27° + 65° + x° = 180° (!s in same segment, ! sum of a ") !PWR = 90° – 26°
x° + 133° = 180° = 64°
x° = 47° (ii) OW = OS (radii of circle)
x = 47 !OSW = 26°
(d) 40° + 90° + (180 – y)° = 180° !SPW = 180° – 90° – 26° (! sum of a ")
(!s in same segment, rt. ! in semicircle) = 64°
y° = 130° 13.
y = 130
7. (i) !BAD = 180° – 80° – 30° (! sum of a ")
= 70° 20°
(ii) !ADC = 180° – 80° (!s in opp. segments)
O
= 100° x°
!XCD = 100° – 30° (ext. ! of ")
= 70° x°
8. !PQR = 90° (rt. ! in semicircle)
!PRS + !PTS = 180° (!s in opp. segments)
!PQR + !PRS + !PTS = 90° + 180° x° = 180° – 20° – 90° (rt. ! in semicircle, ! sum of a ")
= 270° = 70°
9. (a) x° = 40° (!s in alt. segment) x = 70
x = 40° 14. In "ABD,
(b) y = 180° – 60° – 70° (!s in alt. segment, ! sum of ") !BAD = 90o
= 50 If !BAD is a right angle in a semicircle, then BD is the diameter of
y = 50 the circle.
(c) x = 180° – 44° – 59° (!s in alt. segment, ! sum of ") In "BCD,
= 77° !BCD = 90° (rt. ! in semicircle)
x = 77 By Pythagoras’ Theorem,
(d) y = 180° – 81° – 43° (!s in alt. segment, ! sum of ") BD2 = BC2 + CD2
= 56° = 62 + 82
y = 56 = 100
10. !AOC = 360° – 144° = 216°
BD = 100 = 10 cm
1
!ADC = 216° (! at centre = 2 ! at circumference) Area of circle = 52
2
= 108° = 78.5 cm2 (to 3 s.f.)
!APD = 180° – 145° (adj. !s on a str. line) 15. !BXQ = 24° (!s in same segment)
= 35° !AQB = 90° (rt. ! in semicircle)
!BAD = 180° – 108° – 35° (! sum of a ") !BQP = 90°
= 37° !BQX = 180° – 24° – 35° – 90° (! sum of a ")
11. (i) !BAC = 180° – 43° – 28° (! sum of a ") = 31°
= 109° 16. !ACB = 54° (!s in same segment)
!BOC = 2 109° (! at centre = 2 ! at circumference) !BCP = 180° – 58° – 54° (adj. !s on a str. line)
= 218° = 68°
!APD = 180° – 80° – 68° (! sum of a ")
= 32°

328
17. !ADC = 90° (rt. ! in semicircle) 22. (i) !PAX = 26° (!s in same segment)
!DCE = !DEC !YAQ = 26° (vert. opp. !s)
180° – 90° !YBQ = 26° (!s in same segment)
= (base !s of isos. ")
2 26° + 58° = 23° + 26° + !AQB (ext. ! of ")
= 45° !AQB = 35°
!BCD = 180° – 45° (adj. !s on a str. line) (ii) !YQA = 23° (!s in same segment)
= 135° !AYQ = 180° – 23° – 26° (! sum of a ")
!BAD = 180° – 135° (!s in opp. segments) = 131°
= 45° 23. (a) A A
18. In "ABC, B C
!ACB = 90° (rt. ! in semicircle) O D
!ABC = 180° – 90° – 35° (! sum of a ") BAO = CAD (common angle)
= 55° OBA = DCA = 90° (rt. ! in semicircle)
!ADC = 180° – 55° (!s in opp. segments) AOB = ADC
= 125° "ABO is similar to "ACD (2 pairs of corr. !s equal).
19. (i) !ABD = 45° (!s in alt. segment) (b) (i) OA = radius of larger circle
1 =2 4
!ABC = 130° (! at centre = 2 ! at circumference)
2 = 8 cm
= 65° OC = radius of larger circle
!CBD = 65° – 45° = 8 cm
= 20° OA OB
(ii) !OAT = 90° (tangent radius) (ii) =
DA CD
130° 8 4.5
!AOT = =
2 16 CD
= 65° 8CD = 72
!OTA = 180° – 65° – 45° (! sum of ") CD = 9 cm
= 25 24. (i) P P
!ATC = 2 25° A B
= 50° D Q
20. D APQ = BPQ (common angle)
ADP = BQP (corr. !s, BQ // AD)
G PAD = PBQ (corr. !s, BQ // AD)
"PAD is similar to "PBQ (3 pairs of corr. !s equal).
(ii) P P
35° C A
E F B D
H
CPB = APD (common angle)
CPB = 180° – ADC (!s in opp. segments)
(i) !GHF = 90° (rt. ! in semicircle) ADP = 180° – ADC (adj. !s on a str. line)
!GHE = 90° CBP = ADP
!EDG = 180° – 90° (!s in opp. segments) "PCB is similar to "PAD (2 pairs of corr. !s equal).
= 90° 25. !ACD = 180° – 90° – 18° (! sum of a ")
(ii) !DEF = 180° – 90° – 35° (! sum of a ") = 72°
= 55° !AEB = 72° (!s in same segment)
21. !ROQ = 2 110° (! at centre = 2 ! at circumference) !ABE = 90° (rt. ! in semicircle)
= 220° !BAE = 180° – 90° – 72° (! sum of a ")
Obtuse !ROQ = 360° – 220° = 18°
= 140°
!QPS = 180° – 140° (!s in opp. segments)
= 40°

329
70° 1
26. !CRQ = !QBC = = 35° (!s in same segment) y° = 33° (! at centre = 2 ! at circumference)
2 2
50° = 16.5°
!CRP = !PAC = = 25° (!s in same segment)
2 x = 28.5, y = 16.5
!R = 35° + 25° = 60° (d) x° = 64° (!s in alt. segment)
50° 180° – 64° = 116° (adj. !s on a str. line)
!PQB = !PAB = = 25° (!s in same segment)
2 y° = 180° – 116° – 26° (! sum of ")
60° = 38°
!RQB = !RCB = = 30° (!s in same segment)
2 x = 64, y = 38
!Q = 25° + 30° = 55° (e) !OAP = 90° (tangent radius)
!P = 180° – 60° – 55° (! sum of a ") !AOP = 2 32° (! at centre = 2 ! at circumference)
= 65° = 64°
27. (i) A C x° = 180° – 90° – 64° (! sum of ")
C B = 26°
K K y° = 180° – 32° (adj. !s on a str. line)
AKC = CKB = 90° = 148°
ACB = 90° (rt. ! in semicircle) x = 26, y = 148
CAK = 180° – 90° – CBK (! sum of ") (f) x° = 90° (rt. ! in semicircle)
BCK = 180° – 90° – CBK (! sum of ") y° = 180° – (180° – 90° – 28°) (! sum of ", !s in opp.
CAK = BCK = 118° segments)
"ACK is similar to "CBK (2 pairs of corr. !s equal). x = 90, y = 118
CK AK 2. (a) !OAP = 90° (tangent radius)
(ii) =
BK CK x° = 180° – 90° – 20° (! sum of ")
10 12
i.e. = = 70°
BK 10
180° – 70°
12BK = 100 = 55° (base !s of isos. ")
2
1
BK = 8 cm y° = 90° – 55° = 35°
3
x = 70, y = 35
1 1
AB = 12 + 8 = 20 cm (b) !AOP = 180° – 90° – 22° = 68° (! sum of ", tangent radius)
3 3
1 1 68°
Radius of circle = 20 x° = (base !s of isos. ")
2 3 2
1 = 34°
= 10 cm y° = 34° + 22° = 56°
6
x = 34, y = 56
(c) !OAP = !OBP = 90° (tangent radius)
Review Exercise 13 x° + 48° + 90° + 90° = 360° (! sum of quadrilateral)
1. (a) x° = 25° (!s in alt. segment) x° + 228° = 360°
y° = 90° – 65° (rt. ! in semicircle) x° = 132°
= 25° !AOB = 360° – 132°
x = 25, y = 25 = 228°
(b) !OAP = 90° (tangent radius) 1
y° = 228° (! at centre = 2 ! at circumference)
90° – 24° – 32° = 34° 2
x° = 34° (!s in same segment) = 114°
y° = 90° + 24° (rt. ! in semicircle, !s in same segment) x = 132, y = 114
= 114° (d) !OAP = !OBP = 90° (tangent radius)
x = 34, y = 114 !AOB = 360° – 150°
(c) !OAP = 90° (tangent radius) = 210°
!TOA = 180° – 90° – 33° (! sum of ") 1
x° = 210° (! at centre = 2 ! at circumference)
= 57° 2
57° = 105°
x° = (ext. ! of ", base !s of isos. ") y° + 150° + 90° + 90° = 360°
2
= 28.5° y° + 330° = 360°
y° = 30°
x = 105, y = 30

330
(e) !OAP = !OBP = 90° (tangent radius) 1
(c) x° = 230° (! at centre = 2 ! at circumference)
!AOB = 360° – 90° – 90° – 50° (! sum of quadrilateral) 2
= 130° = 115°
y° + y° + 130° = 180° (! sum of ") !O = 360° – 230° = 130°
2y° = 50° y° + y + 115° + 130° = 360°
y° = 25° 2y° + 245 = 360°
In "OBP, 2y° = 115
3 y = 57.5°
tan 25° =
x x = 115, y = 57.5
3 (d)
x=
tan 25°
= 6.43 (to 3 s.f.)
x = 6.43, y = 25
(f)
O
B
10°
x° x°
15°

O 72° P

C

! at circumference = 15° + 10° = 25°
A x° = 2 25° (!at centre = 2 ! at circumference)
= 50°
180° – 72° x = 50
!PAB = (base !s of isos. ")
2 (e) x° = 90° – 18° (rt. ! in semicircle)
= 54° = 72°
x° = 54° (alt. !s) y° = 180° – 80° – 72° (! sum of ")
!OAP = !OBP = 90° (tangent radius) = 28°
!BOA = 360° – 72° – 90° – 90° (! sum of quadrilateral) x = 72, y = 28
= 108° (f) x° = 180° – 90° – 42° (rt. ! in semicircle, ! sum of ")
1 = 48°
!BCA = 108° (! at centre = 2 ! at circumference)
2 y° = 48° – 26° (ext. ! of ")
= 54° = 22°
y° = 180° – 54° – 54° = 72° (! sum of ") x = 48, y = 22
x = 54, y = 72 4. (a) x° = 180° – 90° – 49° (rt. ! in semicircle, !s in same segment)
1 = 41°
3. (a) x° = 124° (! at centre = 2 ! at circumference)
2 x = 41
= 62° (b) y° = 108° – 78° = 30° (ext. ! of ")
y° = 180° – 62° (!s in opp. segments) 180° – 108° = 72° (supplementary !s)
= 118° x° = 180° – 72° – 30° (!s in opp. segments)
x = 62, y = 118 = 78°
(b) x° = 2 58° (! at centre = 2 ! at circumference) x = 78, y = 30
= 116° (c) x° = 180° – 72° (!s in opp. segments)
180° – 116° = 108°
= 32° (base !s of isos. ")
2 y° = 180° – 36° (!s in opp. segments)
y° = 180° – 12° – 32° – 32° – 58° (! sum of ")
= 144°
= 46°
x = 108, y = 144
x = 116, y = 46

331
(d) (h)

O

x° 4x°

66° x°
x° + x° + y° = 180° (! sum of isos. ")
2x° + y° = 180°
180° – 66° = 114° (adj. !s on a str. line) 180° – 4 x°
= (90 – 2x)° (base ! of isos. ")
180° – 114° = 66° (!s in opp. segments) 2
x° = 180° – 90° – 66° (! sum of ") x° + 4x° + x° + (90 – 2x)° = 180° (!s in opp. segments)
= 24° 4x° + 90° = 180°
y° = 66° – 24° (alt. !s) 4x° = 90°
= 42° x° = 22.5°
x = 24, y = 42 2 22.5° + y° = 180°
(e) y° = 180° – 45°
x° = 135°
x = 22.5, y = 135

5. !PSR + x° + y° = 180° (! sum of a ")
88°
!PSR + !PQR = 180° (!s in opp. segments)
!PSR + !PQR = !PSR + x° + y°
x° y° 30° !PQR = x° + y°
6. !BEF = 180° – 40° (adj. !s on a str. line)
x° + y° = 88° = 140°
x° + (180° – y°) + 30° = 180° (! sum of ") !BAF = 180° – 140° (!s in opp. segments)
y° = x° + 30° = 40°
x° + x° + 30° = 88° !AFB = 90° (rt. ! in semicircle)
2x° = 58° !ABF = 180° – 90° – 40° (! sum of a ")
x = 29° = 50°
y° = 29° + 30° = 59° !ABE = 40° + 30° = 70°
x = 29, y = 59 !EBF = 70° – 50°
(f) y° = 180° – 90° (!s in opp. segments) = 20°
= 90° 7. (i) !AOC = 180° – 2 66° (! sum of a ")
x° = 180° – 90° – 48° (! sum of ") = 48°
= 42° 1
!CQA = 48° (! at centre = 2 ! at circumference)
x = 42, y = 90 2
(g) 180° – 92° – 58° = 30° (adj. !s on a str. line) = 24°
y° = 180° – 105° – 30° (!s in same segment, adj. !s on a str. line) (ii) !AOQ = 66° + 32° = 98° (ext. ! of ")
= 45° 1
!QCA = 98° (! at centre = 2 ! at circumference)
x° = y° + 58° (ext. ! of ") 2
= 49°
= 45° + 58°
= 103°
x = 103, y = 45

332
8. A (ii) r2 = 132 + OH2
x° = 132 + 42
= 185
r = 185
B 95° O
= 13.6 (to 3 s.f.)
y° The radius of the circle is 13.6 cm.
130°
E 12. T
C 138°
D S

!CBE = 180° – 138° (!s in opp. segments) R

= 42°
!ABE = 95° – 42° = 53° 46° A
x° = 180° – 53° – 53° (! sum of an isos. ")
P
= 74°
!BED = 180° – 130° (!s in opp. segments)
= 50°
y° = 50° + 53° = 103°
x = 74, y = 103
9. (i) Let !ADE = !DCA = x° (i) !APT = 90° (tangent radius)
!ACB = 90° (rt. ! in semicircle) !PSA = 90° (rt. ! in semicircle)
= !ADB (!s in same segment) !SPA = 90° – 46° = 44°
!DAE = 180° – 90° – x° (! sum of a ") !PAS = 180° – 90° – 44° (! sum of a ")
= (90 – x)° = 46°
!FEB = (90 – x)° + x° (ii) !PRS = 180° – 46° (!s in opp. segments)
= 90° = 134°
(ii) !EFC = 360° – 90° – 90° – 70° 13.
= 110° L
10. (i) "CEB is similar to "CFO (2 pairs of corr. !s equal).
N
OF = OB = 2 cm (radii of circle)
BE BC O 58° P
=
OF OC
BE 2
i.e. =
2 4 M
1
BE = 2
2 !OLP = !OMP = 90° (tangent radius)
= 1 cm !LOM = 360° – 90° – 90° – 58° (! sum of quadrilateral)
(ii) !BOF = !CBE = y° = 122°
1 1
!FAO = y° (base ! isos. ") !LNM = 122° (! at centre = 2 ! at circumference)
2 2
11. (i) Let the radius of the circle be r cm. = 61°
By Pythagoras’ Theorem, or
OA2 = AH2 + OH2
r2 = 132 + OH2 — (1)
Pythagoras’ Theorem,
OP2 = PK 2 + OK 2
r2 = 112 + (4 + OH)2 — (2)
Substitute (1) into (2),
132 + OH2 = 112 + (4 + OH)2
169 + OH2 = 121 + OH2 + 8OH + 16
169 = 137 + 8OH
8OH = 32
OH = 4 cm

333
Challenge Yourself
L
For the camera which scans an angle of 45°,
360°
No. of cameras needed = =4
N 2 45°
O 58° P 3 more video cameras must be installed so that they will cover the entire
hall. The 4 cameras can be placed at any point on the 4 different quadrants.

45° 45°
!OLP = !OMP = 90° (tangent radius)
!LOM = 360° – 90° – 90° – 58° (! sum of quadrilateral)
= 122°
!LOM = 360° – 122° = 238°
1 45° 45°
!LNM = 238° (! at centre = 2 ! circumference)
2 C
= 119°
1
14. (i) !ABC = 156° (! at centre = 2 ! circumference) (a) For the camera which scans an angle of 35°,
2
360°
= 78° No. of cameras needed = 5
2 35°
!ADC = 180° – 78° (!s in opp. segments)
(b) For the camera which scans an angle of 60°,
= 102°
360°
!PDC = 180° – 102 (adj. !s on a str. line) No. of cameras needed = =3
2 60°
= 78°
(c) For the camera which scans an angle of 90°,
(ii) !PQC = 180° – 78° (!s in opp. segments)
360°
= 102° No. of cameras needed = =2
2 90°
15. (i) !APR = 180° – 64° – 54° (! sum of a ") (d) For the camera which scans an angle of 100°,
= 62° 360°
(ii) !BCD = 180° – 64° !s in opp. segments) No. of cameras needed = 2
2 100°
= 116°
!DCQ = 180° – 62° (!s in opp. segments)
= 118°
!BCQ = 360° – 118° – 116° (!s at a pt.)
= 126°
16. Since CA is the diameter of the circle,
!ABC = 90° (rt. ! in semicircle)
!ABE = 180° – 90° (adj. !s on a str. line)
= 90°
!ADE = 180° – !ABE (!s in opp. segments)
= 180° – 90°
= 90°
!ADE is a right angle.

334
Revision Exercise D1 Area of !ASQ 1
2
1
(iii) = =
1. P P Area of !ABC 3 9
Q R 1
Area of "ASQ = 36
9
S T
= 4 cm2
PQ = PR (given)
Area of quadrilateral ASQR = 2 4
Since PST = PTS, PTR (adj. !s on a straight line).
= 8 cm2
PQS = PRT (base !s of isos. ")
3. (i) The scale used is 45 cm : 30 m, i.e. 45 cm : 3000 cm which is
"PQS "PRT (ASA)
3 : 200.
2. (a) A Q 2
S R Area of model 3
(ii) =
Q A Actual area 200
810 9
AQS = ARQ (opp. !s of a parallelogram) =
Actual area 40 000
AQS = QAR (alt. !s, AR // SC)
810 40 000
AQ = QA (common side) Actual area = cm2
9
"ASQ is congruent to "QRA (ASA)
= 3 600 000 cm2
(b) A A
= 360 m2
S B 3
Volume of model 3
Q C (iii) =
Actual volume 200
ASQ = ABC (corr. !s, SQ // BC)
162 27
SAQ = BAC (common angle) =
Actual volume 8 000 000
"ASQ is similar to "ABC (2 pairs of corr. !s equal). 162 8 000 000
(c) Since "ABC is similar to "ADE, then Actual volume = cm3
27
AS SQ = 48 000 000 cm3
=
AB BC = 48 m3
2 SQ 4. (i) OB = OC (radii of circle)
i.e. =
6 15
!BOC = 180° – 18° – 18° (! sum of isos. ")
6SQ = 30
= 144°
SQ = 5 cm
1
(d) B P !BAC = 144° (! at centre = 2 ! at circumference)
2
C C = 72°
A Q 180° – 48°
BCA = PCQ (common angle) (ii) !ABP = (base !s of isos. ")
2
ABC = QPC (corr. !s, AB // QP) = 66°
"BCA is similar to "PCQ (2 pairs of corr. !s equal). !OBP = 90° (tangent radius)
R P !ABC = (90° – 66°) + 18° = 42°
A C 5. (i) !BAE = 180° – 26° – 26° (! sum of isos. ")
Q Q = 128°
RQA = PQC (vert. opp. !s) (ii) !AEB = 26°
ARQ = CPQ (alt. !s, AR // PC) !BED = 118° – 26° = 92°
"RAQ is similar to "PCQ (2 pairs of corr. !s equal). !BCD = 180° – 92° (!s in opp. segments)
2
Area of !PCQ 10 4 = 88°
(e) (i) = =
Area of !ABC 15 9 6. !ABF = 90° (tangent radius)
4 v = 180 – 90° – 20° (! sum of ")
Area "PCQ = 36
9 = 70°
= 16 cm2 !BDA = 90° (rt. ! in semicircle)
1 !ABD = 180 – 90° – 20° (! sum of ")
(ii) Area of "BPQ = "PCQ
2 = 70°
1
= 16 u = 90° – 70° = 20°
2
In "ABF,
= 8 cm2
90° + 20° + w° + 40° = 180° (! sum of ")
150° + w° = 180°
w = 30°

335
x = !ABD = 70°
y = 180° – 30° – 20° (!s in opp. segments)
= 130°
z = 180° – 2(20° + 30°) (! sum of isos. ")
= 80°
u = 20°, v = 70°, w = 30°, x = 70°, y = 130° and z = 80°
7. (i) !RSQ = !RPQ = b (!s in same segment)
!RQS = !RSQ = b (base !s of isos. "SRQ)
(ii) !PRS = b – a (ext. ! of ")
(iii) !SPR = b
!SPQ = b + b = 2b
!PST = 2b – a
(iv) !QRS = 180° – 2b (! sum of ")
(v) !PSQ = 180° – b – (2b – a)
= 180° + a – 3b
8. (i) By Pythagoras’ Theorem,
l2 = 52 + 122
= 169
l = 169
= 13 cm
Total surface area = 52 + 5 13
2
= 283 cm (to 3 s.f.)
(ii) Given that it takes 1 hour 40 minutes to burn the candle
completely,
1
Volume of candle burnt after 12 minutes
2
12.5 1
= 52 12
100 3
1
= 12 cm3
2
Let the radius of the candle burnt off be r cm.
12
Height of candle burnt off = r cm
5
1 12 1
r2 r = 12
3 5 2
4 3 1
r = 12
5 2
r3 = 15.625
3
r= 15.625
= 2.5
12
Height of candle burnt off = 2.5
5
= 6 cm
h = 12 – 6 = 6
(iii) By Pythagoras’ Theorem,
l2 = 2.52 + 62
= 42.45
l = 15.625
= 6.5 cm
Curved surface area of frustum = 5 13 – 2.5 6.5
= 48.75 cm2
Surface area of frustrum = 52 2.52 + 48.75
2
= 251 cm (to 3 s.f.)

336
Revision Exercise D2 (ii) Let V1 be the volume of the smaller cup and V2 be the volume
1. (i) A B of the larger cup.
3
P Q V1 h1
=
C A V2 h2
PA = QB (given) 3
2400 3
PC = QA =
V2 8
APC = BQA = 60°
!APC !BQA (ASA) 2400 27
=
(ii) C A V2 512
27V2 = 1 228 800
R P
1
B C V2 = 45 511 cm3
9
CR = AP = 4 cm
1
BR = CP = 12 cm The volume of the larger cup is 45 511cm3.
9
CRB = APC = 60°
4. (i) !BCX is similar to !ZYX (2 pairs of corr. "s equal).
!CRB !APC BQA (ASA)
XY YZ
Since RCB = PAC = QBA = RBC = PCA = QAB, =
XC CB
BAC = ABC = ACB = 60°. XY 15
!ABC is an equilateral triangle. i.e. =
4.8 8
2. (i) !ABX is similar to !PQX (2 pairs of corr. "s equal). 8XY = 72
PX PQ XY = 9 cm
=
AX AB !ABC is similar to !AYZ (2 pairs of corr. "s equal).
PX 7.2 AZ YZ
i.e. = =
3.6 4 AC BC
4PX = 25.92 8.5 + CZ 15
PX = 6.48 cm i.e. =
8.5 8
QX PQ 1
= 68 + 8CZ = 127
BX AB 2
QX 7.2 1
i.e. = 8CZ = 59
4.2 4 2
4QX = 30.24 7
CZ = 7 cm
QX = 7.56 cm 16
2
Area of !ABX 4 25 Area of !ABC 8
2
64
(ii) = = (ii) = =
Area of !PQX 7.2 81 Area of !AYZ 15 225
The ratio of the area of !ABX to that of !PQX is 25 : 81.
Area of !AYZ 225 225
3. (i) Let h1 and A1 be the height and the surface area of the smaller = =
Area of trapezium BCZY 225 – 64 161
cup respectively and h2 and A2 be the height and the surface area
The ratio of the area of !AYZ to that of the area of trapezium
of the larger cup respectively.
BCZY is 225 : 161.
A1 9
= 5. (i) "OBE = 90° (tangent radius)
A2 64
2
"OBA = 90° – 50° = 40°
h1 9 "AOB = 180° – 40° – 40° (" sum of isos. !)
=
h2 64 = 100°
h1 9 1
= (ii) "ACB = 100° (" at centre = 2 " at circumference)
h2 64 2
3 = 50°
=
8
25 3
=
h2 8
3h2 = 200
2
h2 = 66 cm
3
2
The height of the larger cup is 66 cm.
3

337
6. !OAP = !OBP = 90° (tangent radius) (ii) Let A1 be the surface area of the pyramid of water excluding
!AOB = 360° – 56° – 90° – 90° = 124° the base and A2 be the surface area of the pyramid containing
56° the water excluding the base.
!OPB = !OPA = = 28° 2
2 A1 h1
2
1 1
14 = = =
tan 28° = A2 h2 2 4
PB 1
14 Area in contact with water 1
PB = = 4 =
tan 28° Area not in contact with water 1 3
1–
= 26.33 cm (to 4 s.f.) 4
The ratio is 1 : 3.
Area of shaded region = Area of AOBP – Area of sector AOB
1 124°
=2 26.33 14 – 142
2 360°
= 157 cm2 (to 3 s.f.)
7. (i) !ABC = 180° – 56° – 56° (! sum of isos. ")
= 68°
!ACB = 10° – 42° – 68° (! sum of ")
= 70°
180° – 70°
(ii) !CQR = (base !s of isos. ")
2
= 55°
!PQR = 180° – 55° – 56° (adj. !s on a str. line)
= 69°
180° – 42°
(iii) !ARP = (base !s of isos. ")
2
= 69°
!PRQ = 180° – 55° – 69° (adj. !s on a str. line)
= 56°
!RPQ = 180° – 56° – 69° (! sum of ")
= 55°
8. (i) Let h1, t1 and V1
amount and the volume of water respectively after 30 s; and h2,
t2 and V2

3
V1 h1 t1
= =
V2 h2 t2
3
h1 0.5
=
18 4
1
=
8
h 1
= 3
18 8
1
=
2
2h = 18
h = 9 cm

338
Problems in Real-World Contexts (iii) Method 1
Equation of the graph representing the dome:
1. (a) From Fig. (b), we can state the coordinates of two more points
y = ax2 + bx + c
on the dome since we know that the dome is 82.5 m tall and
33 2 33
310 m wide. y=– x + x
9610 31
The coordinates of two more points are the maximum point
Method 2
(155, 82.5) and the other minimum point (310, 0).
Since the coordinates where the curve cuts the x-axis
(b) y
are known, we can write the equation in the form
100 (155, 82.5) y = a(x – h)(x – k).
80 i.e. y = ax(x – 310)
60
40
Since the curve passes through the point (155, 82.5),
20 (310, 0) 82.5 = 155a(155 – 310)
x
0 (0, 0) 50 100 150 200 250 300 350 33
= –155a
62
(c) (i) y = ax2 + bx + c 33
At (0, 0), a=–
9610
(0) = a(0)2 + b(0) Equation of the graph representing the dome:
c =0 —(1) 33
y=– x(x – 310)
At (155, 82.5), 9610
82.5 = a(155)2 + b(155) (since c = 0) Method 3
82.5 = 1552a + 155b —(2) Since the coordinates of the maximum point of the curve
At (310, 0), are known, we can make use of ‘completing the square’ to
0 = a(310)2 + b(310) (since c = 0)
0 = 3102a + 310b —(3) i.e y = a(x – 155)2 + 82.5
(ii) From (2), Since the curve passes through (0, 0),
155b = 82.5 – 1552a 0 = a(0 – 155)2 + 82.5
82.5 – 155 2 a 24 025a = –82.5
b=
155 33
a=–
33 9610
= – 155a —(4)
62 Equation of the graph representing the dome:
Substitute (4) into (3), 33
y=– (x – 155)2 + 82.5
33 9610
3102a + 31 – 155a = 0
62 (d) One possibility is to centre the y-axis at the maximum height
33 of the dome. We will then get the following three coordinates,
310 310a + – 155a = 0
62 (–155, 0), (0, 82.5) and (155, 0).
33 2. (a) Cost of 6-month road tax for a 1400 cc car
310a + – 155a = 0
62 = [250 + 0.375(1400 – 1000)] 0.782
33 = S$312.80
155a + =0
62 (b) Cost of 6-month road tax for a 3000 cc car
33
155a = – = [475 + 0.75(3000 – 1600)] 0.782
62
= S$1192.55
33
a =– Road tax payable per annum
9610
= 2 1192.55
33
Substitute a = – into (4), = S$2385.10
9610
33 33
b= – 155 –
62 9610
33
=
31
33 33
a=– ,b= and c = 0
9610 31

339
(c)
Engine capacity (cc) 0 600 1000 1600 3000 3500

Road tax ($) 156.40 156.40 195.50 371.45 1192.55 1583.55

Road tax ($)


Scale:
1600 x-axis: 2 cm represent 400 cc
y-axis: 2 cm represent $200

1400

1200

1000

800

600

400

200

0 400 800 1200 1600 2000 2400 2800 3200 3600 4000
Engine capacity (cc)

3. (i) Speeding Car Police Car


Distance travelled Total distance Distance travelled Total distance
Time,
during the travelled after during the travelled after
nth minute th th
n minute (km) n minutes (km) n minute (km) n minutes (km)

1 1
1 110 = 1.83 (to 3 s.f.) 1.83 95 = 1.58 (to 3 s.f.) 1.58
60 60

1 1
2 145 = 2.42 (to 3 s.f.) 1.83 + 2.42 = 4.25 140 = 2.33 (to 3 s.f.) 1.58 + 2.33 = 3.91
60 60

1 1
3 160 = 2.67 (to 3 s.f.) 4.25 + 2.67 = 6.92 185 = 3.08 (to 3 s.f.) 3.91 + 3.08 = 6.99
60 60

Distance (km)

6
Speeding car

4
Police car

Time
0 1 2 3 (minutes)

340
From the distance-time graph, the police car overtook the Linear function
speeding car between the 2.5th and the 3rd second. As such, From the spreadsheet, the equation of the trendline is
the police car will be able to overtake the speeding car and y = 56.262x + 94.893.
arrest the driver during the high-speed chase. Using the equation of the linear trendline,
(ii) Amount of debt owed at the end of 3 years
i.e. both cars did not stop along the way and that the motions = 56.262(36) + 94.893
of both the police car and the speeding car are travelling = $2120.33 (to the nearest cent)
along the same path. Exponential function
4. (a) (i) Polynomial function From the spreadsheet, the equation of the trendline is
From the spreadsheet, the equation of the trendline is y = 133.83e0.2057x.
y = 0.0037x3 + 0.5184x2 + 52.479x + 99.999. Using the equation of the exponential trendline,
Using the equation of the trendline, Amount of debt owed at the end of 3 years
Amount of debt owed at the end of 1 year = 133.83e0.2057(36)
= 0.0037(12)3 + 0.5184(12)2 + 52.479(12) + 99.999 = $220 085.27 (to the nearest cent)
= $810.79 (to the nearest cent) (b) (i) D at the end of 1 year = 100(1.02)12 + 2550(1.02)12 – 2550
Quadratic function = $810.84 (to the nearest cent)
From the spreadsheet, the equation of the trendline is (ii) D at the end of 3 years = 100(1.02)36 + 2550(1.02)36 – 2550
y = 0.557x2 + 52.363x + 100.09. = $2855.70 (to the nearest cent)
Using the equation of the trendline, (c) The polynomial trendline provides a better model for the
Amount of debt owed at the end of 1 year estimation of values for the different periods of time as the
= 0.557(12)2 + 52.363(12) + 100.09
= $808.65 (to the nearest cent) trendlines.
Linear function Moreover, comparing the values obtained in (a) with those in (b),
From the spreadsheet, the equation of the trendline is we observe that the estimated values for D using the polynomial
y = 56.262x + 94.893. trendline are the closest to those values computed in (b).
Using the equation of the linear trendline, 5. Assume that the quality of the watermelons is the same irrespective
Amount of debt owed at the end of 1 year of their sizes and that all the watermelons are spherical in shape.
= 56.262(12) + 94.893 For the small watermelon of r = 12 cm,
= $770.04 (to the nearest cent) 4 3 4
Volume of the watermelon = r = (12)3 = 2304 cm3
Exponential function 3 3
From the spreadsheet, the equation of the trendline is Amount of watermelon that can be bought per dollar
y = 133.83e0.2057x. 2304
=
Using the equation of the exponential trendline, 4.10
Amount of debt owed at the end of 1 year = 1765 cm3 (to the nearest cm3)
= 133.83e0.2057(12) $1 can buy approximately 1765 cm3 of watermelon.
= $1579.67 (to the nearest cent) For the medium watermelon of r = 14 cm,
(ii) Polynomial function 4 2
Volume of the watermelon = (14)3 = 3658 cm3
From the spreadsheet, the equation of the trendline is 3 3
Amount of watermelon that can be bought per dollar
y = 0.0037x3 + 0.5184x2 + 52.479x + 99.999.
2
Using the equation of the trendline, 3858
= 3
Amount of debt owed at the end of 3 years 5.80
= 0.0037(36)3 + 0.5184(36)2 + 52.479(36) + 99.999 = 1982 cm3 (to the nearest cm3)
= $2833.72 (to the nearest cent) $1 can buy approximately 1982 cm3 of watermelon.
Quadratic function For the small watermelon of r = 16 cm,
From the spreadsheet, the equation of the trendline is 4 1
Volume of the watermelon = (16)3 = 5461 cm3
y = 0.557x2 + 52.363x + 100.09. 3 3
Using the equation of the trendline, Amount of watermelon that can be bought per dollar
Amount of debt owed at the end of 3 years 1
5461
= 0.557(36)2 + 52.363(36) + 100.09 = 3
7.60
= $2707.03
= 2256 cm3 (to the nearest cm3)
$1 can buy approximately 2256 cm3 of watermelon.
The large watermelon is the best buy.

341
6. (i) Take any two points on the circumference of the circle and draw a (ii)
line passing through the points. Then construct the perpendicular
bisector of this line segment. Repeat for another two points. The
point of intersection of the two perpendicular bisectors gives the
centre of the circle. Hence, we are able to measure the radius to P
obtain the diameter.
(ii)
3.66
O M

3.66

Q 26 m

H A K B
8m

Radius of circle in scale drawing = 2 cm The optimal position for the winger to shoot at the goal will be at
Radius of plate = 2 4 the point A, which is the point of tangency to the circle passing
= 8 cm through P, Q and A, and with radius equal to OA. In this case,
Diameter of plate = 8 2 the point O is such that
= 16 cm OA = OP = 34 – 8 = 26 m
(iii) Area of original circular plate = (8)2 1
!PAQ = !AOQ
= 201 cm2 (to 3 s.f.) 2
7. (i) For the penalty shootout, the ball is placed 11 m from the centre = !POM (! at centre = 2 ! at circumference)
of the goal post. 3.66
= sin–1 = 8.1° (to 1 d.p.)
P 26
(iii) The size of the angle at the circumference of the circle, centre
O and radius 26 m, will be equal to !PAQ.
3.66 (ii),
11 1
O !PAQ = !POQ (! at centre = 2 ! at circumference)
2

Q
The widest angle for shooting is given by !POQ . P
An arc PQ with centre O and radius OP is drawn.
3.66
!POQ = 2 tan –1 3.66
11
O
= 36.8° (to 1 d.p.) O M

3.66

Q 26 m

H A K B
8m

342
However, if we consider a circle, centre O and radius O P, that (ii) Stamp duty
passes through the point H, = 1% $180 000 + 2% $180 000 + 3%
1 1 ($500 000 – $180 000 – $180 000)
!PHQ = !PO Q < !POQ = !PAQ
2 2 = 3% $180 000 + 3% $140 000
The angle for shooting is the largest when the circle passes = $5400 + $4200
through P and Q and HAB is a tangent to the circle at A. = $9600
8. (i) Total reliefs = $3000 + $2000 + $4000 + $23 000 + $1500 Yes, the answer obtained is the same as in (i).
= $33 500 (iii) x.
Chargeable income = $115 000 – $33 500 Using the formula in (i),
= $81 5000 Stamp duty = 3% $x – $5400
Tax Using the formula in (ii),
First $80 000 : $3350 Stamp duty
$81 500 = 1% $180 000 + 2% $180 000 + 3% $(x – 360 000)
Next $1500 at 11.5% : $172.50
= $5400 + 3% $x – 3% $360 000
Income tax payable = $3350 + $172.50 = $5400 + 3% $x – $10 800
= $3522.50 = 3% $x – $5400
(ii) Total reliefs Hence, both formulae can be used to calculate the stamp duty
= $3000 + $5000 + 15% $156 000 + 20% $156 000 + that Mr Lee has to pay.
$31 200 (iv)
= $93 800 below this amount, the formula in (i) will not work because
Chargeable income = $156 000 – $93 000 from (iii), the stamp duty for the formula in (ii) is only equal to
= $62 200 3% $x – $5400 if $(x – 360 000) is greater than or equal to
0. If the selling price is less than $360 000 but more than $180
Tax
000, the way in which stamp duty is calculated will be different.
First $40 000 : $550
$62 200 (v) Replace the 3% in both formulae with 4% for x 360 000.
Next $22 000 at 7% : $1554 Using the formula in (i),
Income tax payable = $550 + $1554 Stamp duty = 4% $x – $5400
= $2104 Using the formula in (ii),
(iii) This is because people who earn very little should not be required Stamp duty
= 1% $180 000 + 2% $180 000 + 4% $(x – 360 000)
On the other hand, people who earn more should be subjected = $5400 + 3% $x – $14 400
to paying a higher amount of income tax as they have more = 4% $x – $9000
disposable income. Hence, it is just a coincidence that the formula in (i) contains
(iv) This depends on the income tax systems of other countries. In
some countries, people whose incomes fall within the lowest formula in (ii).
income bracket are taxed. Although this system may seem fair, 10.
as everyone is required to pay income tax, this may not be ideal
Guiding Questions
1. Draw a regular pentagon ABCDE with AB = 4 cm. Join AP, BP,
Teachers may wish to discuss whether this type of income tax BQ, CQ, CR, DR, DS, ES, ET and AT to form the 2-dimensional
star.
9. (i) Stamp duty = 3% $500 000 – $5400 Angle subtended at the centre of regular pentagon by each side
= $15 000 – $5400 360°
= $9600 is = 72°
5
Let AB and OP intersect at K, i.e. AK = 2 cm and !AOK = 36°.

343
S R 1
Area of !HAP = 4.536 10.91 sin 40.19°
2
= 15.96 cm2 (to 4 s.f.)
Area of template in Fig. (b) 15.96
D = 63.84 cm2 (4 s.f.)

H
C AH 0.5 AP 0.5
E
= 2 4.536 0.5 + 8 0.5
O = 8.536 cm2 (to 4 s.f.)
K Area of template in Fig. (b)
T 8 cm A 4 cm B 8 cm Q = 72.37 cm2 (to 4 s.f.)
Total area of surface required for a 3-dimensional star, including
72.37
= 361.9 cm2 (to 4 s.f.)
Area of a sheet of A4-sized paper = 21 29.7
= 623.7 cm2
P Estimated number of templates in Fig. (b) obtainable from a sheet
AK of A4-sized paper
sin 36° =
OA 623.7
=
2 72.37
=
OA = 8 (round down to the nearest integer)
2
OA =
sin 36°
= 3.402 cm (to 4 s.f.)
BK
tan 36° =
OK
2
=
OK
2
OK =
tan 36°
= 2.752 cm (to 4 s.f.)
Using Pythagoras’ Theorem,
KP2 + BK2 = BP2
KP2 + 22 = 82
KP2 = 60
KP = 60
= 7.745 cm (to 4 s.f.)
OP = OK+ KP
= 2.752 + 7.745 3.
= 10.49 cm (to 4 s.f.) 4.
As H is 3 cm above O,
AH = 32 + 3.402 2
= 4.536 cm (to 4 s.f.)
PH = 10.49 2 + 32
= 10.91 cm (to 4 s.f.)
2. Consider !HAP.
Using cosine rule,
82 = 4.5362 + 10.912 – 2(4.536)(10.91) cos "AHP
cos "AHP = 0.7638 (to 4 s.f.)
"AHP = 40.19° (to 2 d.p.)

344

You might also like